You are on page 1of 374

STUDIES

IN

DEDUCTIVE
^
Manual
for

LOGIC
Sftiibmts.

W.

STANLEY
LL.D.

JEVONS,
(Land.),
F.R.S.

(Edinb.), M.A.

Bonbon
M

ACM

ILL

AN
1880.

AND

CO.

The

Right of

Translation

and

Reproduction is

Resei~ved.

LONDON :

R.

CLAY,

SONS, TAYLOR,

AND

BREAD

STREET HILL.

StacK
Annex

Ti 053

PREFACE.

IN

these preparing

'

Studies

'

I have tried to carry forward in

the chief purpose purpose


In the
was

of my

Elementary Lessons

which Logic,

the

in Logic. promotion of practical training those Lessons


I said in

prefaceto

1870: "The
present
cise exer-

relations of
as

and propositions

the forms of argument


as

a precise subjectof

instruction and

an vigorous

of

as thought,

the

of geometrical or properties figures


Yet every

the rules of

Algebra.
total

is made schoolboy
never

to learn

mathematical

problems which
which reasoning
...

he will

employ in

after

and is left in life,


and forms

of ignorance

those

simpleprinciples
of thoughts

of

will
own

enter

into the

every hour. found

In my

classes I have

constantly

that the

working and
of arguments

solution of and

logical questions,

the examination

the detection of fallacies, than

is
is

not

less

and practicable
of

useful exercise of mind the

the

performance
a

calculations and class." which


to

solution of

problemsin
The

mathematical
use seems

considerable

has that The

been

made
an

of the
tional educa-

Elementary Lessons
want

show

theymeet
now

of the present

day.

time has

perhaps

viii

PREFACE.

arrived

when

facilities for be offered back of


to

more

thorough
and
been

course

of

logical

trainingmay
For
a

teachers there have

students.

long

time

published

books
not
a

containing
few
In works
recent

abundance

mathematical

exercises,and
such

exist which years the

consist teachers and the

of exclusively
of

exercises. of

other

branches
of

science,
been

such

as

Chemistry
with

Theory
of

Heat,
and

have

furnished

similar collections
can

problems
about The learner
no use

numerical
of such

examples.
exercises education instead
lecture and

There when is
to

be
can

no

doubt be had.

the

value

they
throw

great
into
to

point
an

in

the mind
state.

of the It is of

active,
to
a

of
or

passive
read the
a

listen

to

lesson
and the

unless

the

mind

appropriates
it. The

digests
of

ideas

principles put answering only, means

before

working
is the active

problems
not

and

of definite

questions
this

best, if
exercise

almost

the

of

ensuring
at to

of

thought.

It is

possible that pushed

Cambridge
an

mathematical the

gymnastics have
of the

been

extreme,

study

principlesand
in the
most

philosophy of
race

Mathematics the

being

almost

forgotten
of time. the

to

solve

greatest
in the
of

possible
shortest

number

difficult there
can

problems
be of
no manner

possible
that from

But

doubt
up
to

the in

simple
the

addition

sums

the

schoolboy
and the
must

problems
of

Calculus real

of

Variations
of

Theory
consist

the Probability, student kernel be in

study
his

Mathematics

in the the

cracking
of

o'wn

nuts, and

gaining

for himself So it
must

understanding.
Students of

Logic.

Logic

must

have

PREFACE.

ix

logical nuts
the value of

to

crack.

Opinions

may
any

differ,indeed,
form. That

as

to

logical training in
from the I the

value

is

twofold, arisingboth
powers and from

general trainingof
of that

the mental
processes

command maintain need


the
not

reasoning
in

eventuallyacquired.
when

both

ways

Logic,
with the is

properly taught,
and in
to

fear

comparison
of view

Mathematics,

second

point
of

Logic
Many

decidedly superior
students

the

sciences

quantity.

acquire

wonderful

tial in integratingdifferenfacility other


an

equations,and
who settle will
never

cracking
to

hard

mathematical

nuts,

need in the

solve

equation again, after they


chambers
of
or

down

conveyancer's
the

the

vicar's

parsonage.
and of
rest

With

ordinary

forms

logical inference
deal for

logical combination
of

they will ceaselessly


for the

the and

their

lives ; yet
of

knowledge

of

the

forms
to

principles
nature. not

reasoning they generally

trust

the

light of
I

do

deny

that of

mind

of

first-rate

abilityhas
is often

siderable con-

command

natural
course

logic,which
of mathematical
as

greatly
But

improved
I have
an

by

severe

study.
a

had

abundant
of

both opportunities, the

teacher minds

and of
been

examiner,

estimating
and

of logicalfacility

various astonished break is very when


'

training
at

capacity,
in which

and
even

have

often

the way
a

well-trained
A

students
man

down

before
at

simple logical problem.


to

who

ready
he

begins integration
such
a

hesitate
as

and the

flounder

is asked

simple question

following:
any,

If all

trianglesare

what plane figures,

information, if

PREFACE.

does
not

this

propositiongive
As
to

us

concerning thingswhich

are

triangles?'

untrained
most

thinkers, they seldom


distinct
one

discriminate De
a

between has

the

widely
more

assertions.

Morgan

remarked asked
B what

in

than from

place l
A

that

beginner, when
'

follows
course.'

'Every
fact

is such

B,'
a

answers

Every
is

is A
true

of
in

The

that it

converse

often

geometry,
tends

although strongly
to

cannot

be the

inferred student.

by

pure

logic,
all

mystify
must

Although logical if

mathematical correct,
often

reasoning
yet
the
as

necessarily be
of

it be
are

conditions

quantitative reasoning
the
of
reasoner

such them

actually
the

to

mislead

who in

confuses

with
A

ditions con-

argumentation

ordinary life.
be

mathematical

education if indeed There


was

in short,to requires, it should


never a

corrected
a

and

completed,

not

be

preceded, by

logicaleducation.
than De

greater teacher
his earliest essay latest

of mathematics
on

Morgan

; but
to

from

the

Study

of Mathematics
upon

his
of
was

very

he always writings,
as

insisted

the

need This

logical as
the

well

purely
his
tract

mathematical of

ing. train-

purpose

of

1839, entitled,

First

Notions

of Logic preparatory
Essays
excellent
on

to the

Study of Geometry,
1836,
of vol. ii.

The
1

Schoolmaster:
note.
'

Practical
'

Education,
the Method

p.

20,

This

essay in
"

On

Teaching of
tion, Educa-

Geometry
No. in De

was

originally printed 1833,


vol. vi. pp. earlier in

the

Quarterly Journal
Similar and the views
are

XI.

237
On the

251.

put forth
matics, MatheUseful Memoir

Morgan's published

work,

the

Study
for

Difficulties of
Diffusion Fourth of

1831 by
xiv.

Society
also

Knowledge.
on

See

chapter
p. 4,

See

De

Morgan's

the

Syllogism,

in

the

Cambridge

Philosophical Transactions

for

1860.

PREFACE.

xi

subsequently reprinted as
Logic.
Method A

the

first

chapter
valuable
above.

of

the

Formal

like

idea

inspired

his

essays

On

the

of Teaching Geometry, quoted indeed, Sylvester,


in his

Professor upon

most

curious in

tractate

the Laws

of

Verse
of

(p. 19),has

called
"

question
to

the
me

nut-bearing
absurd
to

powers

logic, saying :
there
a

It

seems

suppose

that
bears

exists

in

the

science
to

of pure

logic anything developable


and

that

resemblance euristic
a

the

infinitely
of matical mathe-

interminable
To

processes

science."

such

remark

this

volume

is it is

perhaps
stated

the
I

best
have my

possible
had materials size. should

answer,

especially when

that

great
so

in difficulty
as

selecting and
them into
a

compressing
volume
Professor the
to

to

get

of

moderate

If

any

person
to

who

thinks

with of

Sylvester
as

object
concrete

the

greater

part

problems
the end
of

dealing with
book, where
to

logic,let

him

look

this

he

will find that of


law

the

closely
the

printed Logical
combinations in
any way of

Index

the

forms

governing
pages,

only

three the

terms,
almost forms. of of

fills four

without

including
those

various infinitely
He will also

logical
that

equivalentsof
a

distinct index

learn

similarly complete
the

the

forms

of

logical law
would is
scope

governing
fill
a

combinations

only

five

logical terms
there

library of 65,536
here for
'

volumes.

Surely

enough

euristic

processes.'

An

anxious this

and

difficult task consisted

which in

I had

to

encounter

in
or

compiling

book

choosing

the

system

xii

PREFACE.

systems

of

logicalnotation
When the
once

and the

method convenient

which but

were

to

be

expounded.

tyrannical
each of

uniformity of
writer
on

Aristotelian

logic was
to

overthrown,
a new

the
But

science it is

proceeded
to

invent

set

symbols.
letters of De of

impossible
Thomson,

employ
the
'

alike

the

Greek
'

Archbishop
the

mysterious spiculse
dots formulae of

Morgan,

cumbrous the

strokes, wedges, and


intricate renewed
I felt

Sir W.
of of

Hamilton,
After eminent discard
of De

and
a

mathematical

Boole. these
to

careful

study

of

the in

writings
the
first

logicians
the

compelled

place
methods
I do way,

diverse
Few

and
or none

complicated
admire
more

notative
than

Morgan.

the the

and, extraordinary ingenuity,fertility,


accuracy
of

in

certain

De

Morgan's
to

logical writings. My writings


be and
to

general indebtedness, both


own

those
cannot

his

unrivalled
I

oral

teaching,
moreover,
too

knowledged. acsufficiently

have,
works

drawn

many
to

particular specified.
into

hints

from

his
to

numerous

be

Nevertheless,
this book the
was

import
add
a

his needless have


De

'mysterious spiculae'
stumbling-block.
of his six

to

over, More-

question would
.to

arisen,which Morgan
created

various

systems

adopt

for

equally

important concurrent
of

the syllogistic systems,1 which


not

initial letters into in fact

the

names

of
'

he
!'
'

threw characteristically
?

the anagrams,

Rue

True

No

'

There

was

These

systems

were

the

Relative, Undecided,
See A

Exemplar, of

cal, Numeri-

Onymatic,
pp. 202,

and

Transposed.

Budget

Paradoxes,

3.

PREFACE.

xiii

an

unfortunate

want

of

power

of

generalisation in logical questions

De

Morgan;
their
very

his

mind

could
but he

dissect could
a

into
of

atoms,

not

put

the
As

atoms

thought together again


Sir antagonist,
W.

into

real

system.
De

his

great
was

Hamilton,

remarked,

Morgan

wanting
It W.

in

'

Architectonic

Power.'
to

seems

equally impossible, however,


own

adopt

Sir

Hamilton's
notation
in

logical symbols. brieflydescribed


He also

His in

chief the

method

of

has

been

Elementary
or templated con-

Lessons

Logic (p. 189).


other

constructed
as

systems

of

notation,

stated
In
no

in

his
case

Lectures

on

Logic (vol.iv., pp.


seem

464-476).
so

do

these

notations
one

to

be

good
Bentham.

as

the
And

earlier
after

and
a

simpler
laborious

of

Mr.

George

rendered re-investigation,
of

indispensable by
this almost

the been

composition
forced Hamilton
he
to

various

parts
that

of

book,
every
or

have
case

the

conviction
from was, arch

in

where

differed He
the

contemporaries
as

predecessors
put
the

blundered. into
of

his the

admirers

said, to

keystone
in

of

Aristotelic
Power' I

syllogism; but,
fear
we

spite
that

his
his

'Architectonic arch
of

must

allow 151-4,
With

has this

collapsed.

(See

pp.

129-133,

and the

157-8,

book.)
of
to

logical innovations
While he
appears of

Dr.

Thomson the

the credit
of

case

is different.

enjoy

of

an

independent

discovery
to

the
and

Quantification

the
of

Predicate, prior
the
same

any

public
De

explicit statement
or

by

Hamilton,

Morgan,

Boole,

but

xiv

PREFACE.

posteriorto
he

the

neglected
the

work

of of

Mr.

George Bentham,
nor

did

not

commit
with

blunders crowds the

Hamilton,

overlay
He

his work
most

useless

of short-hand ancient

symbols.
notation
77

aptly completed

scholastic

of
w

propositions(A, E, I, O)
denote the
new

by adding U, Y,
from the
I

and

to

forms

derived
at

Quantification of
same

the and his

showing Predicate, carefully


CD

time therefore

that

vj

are

practicalnonentities.
for

have

used

notation
necessary.

and quantified propositions

syllogisms where

Boole's all
as

great works

are

of in

course

the foundation

of almost
own

subsequent
I

progress

formal

logic. My
moulded dress
to

views,
of his.

long

since

are explicitly stated,1

out

Believing,however,
he his threw his

that

the is

mathematical
not

into which
and that

discoveries

proper
are

them,

quasi-mathematical processes
than

vastly more
of
course

cated complipreferred

they

need

have

been,

I have

my

simpler
power

version. and is

Students

who

wish
must

to to

comprehend
the

Boole's

Boole's

methods

go

original
or

writings.
summary
can

It

really impossible adequate


to construct

that

any

abstract

give an
made

idea
a

of the wonderful

efforts

which calculus
has

Boole of

general mathematical
of

inference.
a

Dr.
new

Macfarlane,
version
but any

Edinburgh,
system
unable
as

lately published
the
to

of
I

Boole's
am

under

title

Algebra of Logic,
that he has made

yet

discover

improvement

on

Boole.
1

Pure

Logic, 1864,

p.

3, "c.

PREFACE.

xv

The

writings

of

M.

Delbceuf

on

Algorithmic Logic,
and
since

first printed in the Revue

Philosophiquefor 1876,
were

reprinted,are
of what

very

but interesting, done in this

written

in

ignorance
and

had

been

country

by

Boole

others.

Quite recentlyMr.
the

Hugh

MacColl,

B.A., has

published

in in

Proceedings of
several

the London

Mathematical
a

Society,and
of

Mind,

papers
arose

upon
out

Calculus
an

Equivalent
in

Statements,
Educational both from
as

which
Times.

of

earlier

article

the

His

Calculus and
from Mr.

differs in several that described

points
in this

that

of Boole

book

Equational Logic.
of
:

MacColl
my Even A
=

rejects equations
AB

in favour him in
not

implications; thus
or

becomes

with differ

B,

implies

B.

his

letter-terms

meaning

from Thus

mine, since his letters denote


A
:

propositions,
statement

things.
the

B
or

asserts

that

the
A

A is

implies
also

statement

B,

that whenever
to

is true, B there is

true.

It in

is these

difficult

believe
;

that

any

advantage

innovations

certainly,in

preferring

implicationsto equations,Mr.
of the

MacColl

ignores the necessity


of
me

equation
His

for

the

application
seem

the
to

Principle
tend

of

Substitution.

proposals
back

to

towards

throwing Formal
In
one

Logic

into its ante-Boolian


no

confusion.

.point his
an

notation,
accent
as

doubt,

is very

elegant,
A' is

namely, in using
the the there

sign

of
can

negation.
be of

negative of
aid

; and
to

as

this
of

accent

applied

with

of brackets sometimes

terms

any

degree
in

complexity,
it. Thus

may

be

convenience

using

xvi

PREFACE.

(A
D' the

B)'
. . .

A'B' ;
.

(ABCD

...)'=

A' the

B'

C'

I shall
accent

occasionallytake (see
of p.

of using liberty

in
In

this the

way
case

199),

but

it is

not

often

needed.

single negative terms,


Italic

I find

perimentally ex-

that

De

Morgan's only
far
more

negatives are

the best.

The
A

Italic than is

is

not

from clearlydistinguished with


one

A',

but

it is written
run

pen-stroke less, importance.


he
The

which

in the
of

long

is
use

matter

of
a

student,

course,

can

A' for

whenever

finds

it

convenient.
The of investigations logical
because Mr. A.

J. Ellis,F.R.S., require
to, if
not to

notice,
identical

they
own.

are

closelyanalogous
I
am

nearly
him for
Not

with, my
to

much

indebted with his

assistingme only
has he

become
me

acquainted
with
an

views.

supplied

with unpublished reprint, Educational


of

additions, of his articles in the


has

Times, but
two

he

allowed

me

access

to

the

manuscripts
to

elaborate
and

memoirs which
Some
are

which
now

he

presented
in

the

Royal
of be

Society,
the found
No. p.

preserved
of these
the

the

archives

Society.
in
the 134, 497.
tributions con-

account

will investigations

Proceedings of
vol. In
xx.

Royal Society for April 1872,


and November
Mr.
to

p.

307,

1873,
remarks
"

vol. The

xxi.

the

former
are

place
believed

Ellis be this

above

original entirely
but direction,

....

Jevons

first led

my
us

thoughts
is

in

all resemblance
blance resem-

between

The entirelysuperficial."

question of
be

thus decide
; but

raised
in

by

Mr.
to

Ellis avoid

must

left

to

others

to

order

ipossible misapprehension,

PREFACE.

xvii

must

say, Mr.

that

however

different
seems

in
to

symbolic
me

sion, expresin

Ellis's
my

logicalsystem
own.

identical
of the

principlewith

The

developments
in the

binational ComTimes

Method,

as

described

Educational

(June,July,and
I had Mr.

August, 1872),are
in

the substantially papers


of

same

as

previously published
Ellis
also upon the

several

and

books.

employs
the

card of
a

diagrams

combinations

arranged
form
The

ledges

black-board,which
as

cally practiin

Logical Abacus,
in which
Mr.

described
am

by

me

1869.

only point
assistance

conscious has and

of

having
to

received
necessary of I of

from of

Ellis

regard
the

the

presence total have

combinations
as

significance

their
may
not

disappearance

proving
upon
so

contradiction. the

insisted sufficiently but


the

importance
as

this matter;

fact

is that
I

long

ago

1864

(see pp. 181,

192, of

of this
a

book)

pointed out
the

the

complete
as

disappearance
the criterion

letter-term

from in the
same

combinations

of

contradiction and
the

conditions

governing

logical combinations,
stated in the

principleis explicitly
i. p. 133; book
I
new

of Principles
In the

Science

(1874,vol.
of this

edition,p. 116).
more

latter part

have

fully developed
often

the
as

theory
it does

of

the

relation

of propositions,
of be

turning
This

upon

this criterion found in


my
to

contradiction. the
natural
;

theory will,I think, be


of ideas received above
stated
some

development
but I may

earlier
from
to

essays
Mr.

have The

hints

Ellis's

writings.
Mr.

remarks
as

apply only
treat

such

portionsof

Ellis's Memoirs

of b

logical

xviii

PREFACE.

combination

and

inference
in
at

other

portions
and

in

which

he

investigates sequence
"c.,
The
are

space
all.
now

time,

probability,

not

in

question

Logical Index, although


been
in my

printed
1871

for

the

first

time, has

possessionsince
i. pp. 157,

(seePrinciples
new

of Science, ist
pp. 137, 141,

edition,vol.

162;

edition,
I
over

"c.) ;

but

it is

only by degrees
which
terms

that

have
all

appreciated

the

wonderful

power

it

gives
and

logicalquestions involving three recentlythat


in the form
Mr. for Venn

only ;
it

it is be

quite

it has of
a

occurred and

to

me

how

might

printed

compact

convenient
in On of the

table.

has
a

published
"

PhilosophicalMagazine Diagrammatic
and
and

July, 1880,

paper

the

Mechanical
An

Representation
article also be
on
"

Propositions
"

ings." Reasonthe
same

Symbolic Reasoning
in
Mind

by
same

author The

will of

found

for

the

month.

text

this book

having
before
not

been
Mr.

completed
Venn's

and

placed

in the
were

hands printer's

ingenious papers
illustrate
or

published,

it has

been

possible to

to

criticise his views.


I

may
at

mention

that

M.

Louis had view

Liard, Professor

of

sophy Philoand

Bordeaux,
the

who

previously explained
of

criticised

substitutional

Logic

in
277,

the

Revue

Philosophique (Mars, 1877,


since

torn,

iii., p.
concise

has "rc.), of

published
recent

very

good though
in

account

the the

principal
title,Les

logical writings

England,

under

Logidens anglais contemporains (Paris:

Germer

Bailliere, 1878).

PREFACE.

xix

These and rule

'

Studies

'

consist in great part of


many

Questions logical
As the
a

Problems
I have

gathered from
indicated the

quarters.
letters

general
or

by

initial

source

authorship of
List of

questionswhen
on

clearly known
I

(see the
not

References
out

p.

xxiii); but
in
not
a

have

always

carried
have

this rule, and

few

cases

the

questions
are

been

printed

several A

times

already, and
fraction have

of

doubtful

authorship.
and

large remaining
are

of

the vised de-

questions

problems

new, As

and shown
have

been

for specially
name

this book. 'few the

by
been

the

author's borrowed
Dean

appended,
the

questions
Very
Rev.

from

work

of

Daniel
Notes
an on

Bagot,
the

of

Dromore,

entitled

Explanatory
. . .

Principal
337

Chapters of Murray's Logic Questions


have also
to

with
A
a

Appendix of

Correspond.
drawn late Sir
from

few

excellent

illustrations
on

been the
copy
a

privatelyprinted tract
Scourfield, M.P.,
his
to
me

Logic by
annotated the
In struck and author

J.

H.

own

having
years

been before

kindly presented
his have death. been
more

by

few

forming this compilation I

than

ever

by the fact that the larger part of logicaldifficulties


do
not turn

sophisms
upon

upon

questions of

formal

logic
to

but

the
or

relations which actual

certain of

assertions bear
the that
assertor
'

the the
are

presumed
hearer. honourable

knowledge
X remarks
one

and

If the

person it

All

lawyers
is the

men,'
of

is

question

what

pure

logical
effect
on

force the

this

proposition,as
of

measured concerned
b
2

by

its and

combinations

the

terms

xx

PREFACE.

their

negatives.
why
he

It

is

quite

another
he

matter

what
Y
to

means

by by

it ;

asserts

it ; what

expects
take
as

understand of X.

it ; and Under

what certain

actuallydoes

the

meaning
a

circumstances of
a

assertions

convey
at

meaning
times.
who
man

the
If
a

direct
man

opposite
is taken
"

what
fit and
not

they
the

convey

other
man

with

first medical of
be

arrives

says, the
; but

You

must

think will
not

putting
put

the

under pump the


man

pump,"
if the of

the

man

under

the about the


to

identically same
of It
an

assertion and that

is made
angry

centre to

interest

excited is evident deductive

mob,

goes
a

the pump. of

there

ought

exist

science
to

applied

logic, partly
of

sponding corre-

the force

ancient
of

doctrines
as

rhetoric, in

which their
A

the

popular

arguments
should be

distinguished from

purely logical force questions


and
answers

carefully analysed.
in

few

given

this

book

may

perhaps
119,

belong, properly speaking, to


140-1,

rhetorical found
It

logic (see pp.


practicable to
be evident

"c.), but
subject
in

I have this book.

not

it

pursue that
a

the

should

thorough
assertions

comprehension
must

of any

the

purely logical aspect


attempt
to

of

precede
rhetorical the

successful
I may

vestigat insome

their future
A
was

aspect

possibly at

time further that

attack

problems
which

of rhetorical

logic.
my

question
the

forced

itself upon

notice in ductive in-

of

of practicability As Mr. H. S.

including
Foxwell
are
even

exercises

Inductive

Logic.
exercises

suggested,
more

and

problems
character.

needed

than

those

of

deductive

But,

on

consideration

xxii

PREFACE.

Compound
with Table.
of any In

Division, but
the
every

on

no

account

to

trouble

selves them-

obsolete

formula

of
a

the

Multiplication
command

point of view, then,


is the
more

thorough

deductive

processes
to master

necessary

for starting-point

attempt

difficult and

apparently more

important
In I have form

processes

of

reasoning.
the didactic
of

the

composition
tried
of the
answers

of

parts of
my

this

book,
into

experiment
to

throwing
or

remarks
cases

the

assumed,
I
cannot treatment

in

many
to

actual,
any book

examination
in which

questions.
this mode it
seems

call

mind been

of
to

has

previously
to

adopted,
clear
of that
a

but

lend

itself very

readily

the

exposition

of

knotty points and

difficulties.

In

spite

much
to

popular clamour

against examinations,
complete written
not

I maintain
answer

give

clear, concise, and


or

to

definite

question
but be also

problem
the best

is
test

only
of

the

best

exercise

of

mind,
can

and ability

training

which The

generally applied.
contains
I

Frontispiece

rough copied

facsimiles
from

of

ancient
MS. of

logical diagrams
Aristotle's

which in

the

fine
at

Organon

the
a

Ambrosian visit
to

Library

Milan
I
was

(L. 93, Superior). During


much

Italy in 1874,
multitudes in
the

surprised and

interested
to

by

the

of curious

diagrammatic
of of the these
were

exercises

be

found
of

logical MSS.
abundance
cises exer-

great

public

libraries that in the

Italy.

The

diagrams
very

shows

rudimentary logical
country
science

popular
dawn of

where, began

and

at

the
I

time

when,

the

modern

to

break.

PREFACE.

xxiii

estimated
at

that

single

MS.
de

in

the

Biblioteca
cum

Communale Comment. least the

Perugia
55.

(Aristotelis
Chart.

Interpretation*
contained

A.

Graece. such the

1485)
Those

at

eight
piece frontis-

hundred
are

diagrams.
most

given
I

in
could

ancient these
to

which

discover.

The the but the

MS.

containing
catalogue
was

(among
the eleventh

others)
or

is

assigned

in

printed
the tenth

twelfth

century,

librarian

of

opinion figure
familiar
to

that in the

it

might
centre

belong
shows

to

century.
of

The the

the

Greek

original
which

Square
this

of

Logical
p.

sition, OppoThe the

has and

survived

day

(see

31).

triangular

lunular

figures
and the this the
on

represent

respectively
Datisi.

syllogistic moods
To
the

Darapti,
list
of

(I believe)
most recent

imperfect

writings
am

on

Symbolical
add
at

Logic, given
last S.
moment

in

preface,

enabled memoir the

to

the C. is vol.

important
of

new

of

Professor of

Peirce

the the

Algebra
American

Logic, Journal

first part matics, Mathe-

which

printed
iii.

in

of

(isth
of

September,

1880).
of

Professor that of

Peirce

adopts
as

the basis

relation of his

inclusion, instead

equation,

the

system.

BRANCH

HILL,
HAMPSTEAD

HEATH,

N.W., October,
1880.

yd

REFERENCE

LIST

OF

INITIAL OR SOURCE

LETTERS

SHOWING

THE AND

AUTHORSHIP PROBLEMS.

OF

QUESTIONS

PROFESSOR Manchester.

ROBERT

ADAMSON,

Owens

College,

PROFESSOR

ALEXANDER

BAIN,
Moral

University
Science

of

Aberdeen.

Cambridge College

University.
Examination

Tripos,

or

Papers.

Dublin

University. University.
HOPPUS,
PROFESSOR ERASER.

Edinburgh
REV.

JOHN
in

formerly

Professor

of

Logic,

"c.

University
Civil
Service

College,

London.

India

Examinations.

London
M.D.

University,
and
D.Sc.

Second

B.A.,

Second

B.Sc.,

M.

A.,

Examinations.

PROFESSOR

MOFFAT,

Queen's

University

in

Ireland.

Oxford

University.
PARK,

PROFESSOR

Queen's

College,

Belfast,

and

Queen's
R
=

University.
GROOM

PROFESSOR

ROBERTSON,

University

College,

London.

WHATELY'S

Elements

of Logic.

CONTENTS.

CHAPTER

pAGE

I.
"

THE

DOCTRINE

OF

TERMS

II.
"

QUESTIONS
KINDS

AND

EXERCISES

RELATING

TO

TERMS.

III.
"

OF

PROPOSITIONS

18

IV.
"

EXERCISES

IN

THE

DISCRIMINATION

OF

SITIONS PROPO-

25

V.
"

CONVERSION

OF

PROPOSITIONS,

AND

IMMEDIATE

INFERENCE

31

VI.
"

EXERCISES

ON

PROPOSITIONS

AND

IMMEDIATE

INFERENCE

56 64

VII.
"

DEFINITION

AND

DIVISION

VIII.
"

SYLLOGISM

71

IX."

QUESTIONS
TECHNICAL
"

AND

EXERCISES

ON

THE

SYLLOGISM
.

94

X.

EXERCISES

IN

THE

SYLLOGISM
. .

103

XL
"

CUNYNGHAME'S

SYLLOGISTIC

CARDS
.

107

XII.
"

FORMAL

AND

MATERIAL

TRUTH

AND

FALSITY
.

XIII.
"

EXERCISES

REGARDING

FORMAL

AND

MATERIAL

TRUTH

AND

FALSITY

122

XIV.
"

PROPOSITIONS

AND

SYLLOGISMS

IN

EXTENSION
.

126

XV.
"

QUESTIONS

ON

INTENSION

135

xxviii

CONTENTS.

CHAPTER

PACK

XVI.
"

HYPOTHETICAL,

DILEMMATIC,

AND

OTHER

KINDS

OF

ARGUMENTS

137

XVII.
"

EXERCISES

IN

HYPOTHETICAL

ARGUMENTS
. .

145

XVIII.
"

THE

QUANTIFICATION

OF

THE

PREDICATE
.
.

149

XIX.
"

EXERCISES

ON

THE

QUANTIFICATION

OF

THE

PREDICATE

159

XX.
"

EXAMPLES

OF

ARGUMENTS

AND

FALLACIES
. .

164

XXI.
"

ELEMENTS

OF

EQUATIONAL

LOGIC
. .

.179

XXII.
"

ON

THE

RELATIONS

OF

PROPOSITIONS

INVOLVING

THREE

OR

MORE

TERMS

223

XXIII.
"

EXERCISES

IN

EQUATIONAL

LOGIC
.

227
.
.

XXIV.
"

THE

MEASURE

OF

LOGICAL

FORCE
. .

249

XXV.
"

INDUCTIVE

OR

INVERSE

LOGICAL

PROBLEMS
.

252

XXVI.
"

ELEMENTS

OF

NUMERICAL

LOGIC
....

259

XXVII.
"

PROBLEMS

IN

NUMERICAL

LOGIC
....

276

XXVIII.
"

THE

LOGICAL

INDEX

281

XXIX.
"

MISCELLANEOUS

QUESTIONS

AND

PROBLEMS
. .

290

DOCTRINE

OF

TERMS.

[CHAP.
it should of
a

is it abstract
name

or

concrete

If

concrete,

be

the
Now

of is

thing, not
has

of

the

attributes
of

thing.
;
or

colour

an certainly

attribute the

gold
of

or

vermilion

never

theless,colour
blue.
Thus

attribute
say

being yellow
is
an

red

or

I should

that is

yellowness

attribute with with


a

of

colour, and
or

if so, colour

concrete

compared compared
becomes

ness yellowgold
or

blueness, while
If this view is

it is abstract

cobalt.

abstractness right,

question thought
Thus uncle
as

of

degree.
3.

Again, a
cannot
an

relative
to

term

is

one

which

cannot

be

except

in relation be

something else,the
but
as

correlative.
of
an

nephew
or

thought
end
be

the
be

nephew thought
But
as

aunt;

instrument
some

cannot
or

but the

the

instrument

to

operation. meaning
What

question
to
as

arises, Can

anything
else ? What

thought except
is the is

in
a

relation
but the sun,

something
that of the
on

of
a

table but the

which
person
to

dinner
?

is put ?

chair
to

seat

some
sun

Every

planet
Even

is related meteoric

and

the

planets.
space
All
are

stones to

moving
the
sun

through empty attracting them. philosophy.


4.

related

by gravity
in

is

relative,both
of

nature

and

As

to

the

distinctions and

and general, singular,

proper

terms,
seem

connotative
to
me

non-connotative in

terms,
confusion.

"c., they
I have

to

be

involved

complete
in

shown

in

the

Elementary
are

Lessons

Logic (pp. 41-44) that


There
would be

Proper
an

Names

connotative. certainly of

impossible breach
the

continuityin supposing that, after


of
'

narrowing

extension

thing
man,

'

successively down Englishman,


time, the
educated
so

to

animal, vertebrate, mammalian,


at

and Cambridge, mathematician, great logician,

forth,
maining re-

thus

increasingthe

intension

all the

single

step of adding Augustus de

Morgan,

Professor

in

I.]

DEFINITIONS

AND

EXAMPLES. all
utmost

University College, London,


instead however
terms

could
to

remove

the

tation, conno-

of

increasing it
many other

the

point.
the

But of

this and may be

questions in

doctrine
that

decided, it is quite clear


ill-suited for
alone

in any

case

this
in my first positions Pro-

part of

logic is
This
somewhat

furnishinggood
is sufficient
to

exercises
excuse

reasoning. passing

ground going
offers
a

rapidly and
on

over perfunctorily once

the

part of logic,and
which

at

to

the

subjectof
of the

wide

field for definitions


to

useful

exercises.
several
a on

Accordingly,
kinds of

after
a

giving brief
few
answers

terms,

questions,and
problems,
of I

fair
to

supply
the
more

of unanswered

questions and

pass

and satisfactory

prolific parts

logic.

DEFINITIONS

AND

EXAMPLES.

5. A
same

general
of

term
one

is

one

which

can

be
or

affirmed, in the

sense,

any
"

of many

two (i.e.

more) things.

Examples

lake, steam-engine. Building,front-door,

6.

singularterm
sense,

is

one

which

can

only

be

in affirmed,

the

same

of

one

singlething.
Victoria, Cleopatra's Needle,
the

Examples

"

Queen
Park.

Yellowstone

7. A
two
or

collective
more

term

is

one

which

can

be which

affirmed
cannot

of be

things taken

together,but

affirmed

of those

or thingsregarded separately distributively.

Examples

Regiment, century, pair do:en, book (a collection of sheets


"

of of

boots, baker's

paper).
j; 2

DOCTRINE A

OF

TERMS.

[CHAP.
for
a

8.

concrete

term

is

term

which

stands
man,

thing.

Examples

"

Stone, red

thing,brute,

table, book,

father, reason.

9.

An

abstract of
a

term

is

term

which

stands

for

an

attribute

thing.
"

Examples

Stoniness, redness,

brutality,humanity,

rationality. tabularity, paternity,


A

10.

connotative

term

is

one

which

denotes

subject

and

implies an Examples
Sir
member
"

attribute. Member of Parliament


or

denotes
any other

Gladstone,
individual

Stafford
of

Northcote,

parliament,
bird and

and

implies
a

that

they
or

can

sit in
or

parliament;
or

denotes

hawk,

eagle,
all

finch,

canary,

implies

that

they

have

the attributes

of birds.

11.

non-connotative

term
can

is

one

which

an signifies

attribute

only, or (ifsuch
"

be) a subject only.


denotes whiteness Smith

Examples

Whiteness without
a

only,

an

attribute
to

subject. John
some

(according
a

J. S. Mill, and
person

other

denotes logicians)

subject or
12.

only, without
names are

implying attributes.
always
which connotative.

Concrete also
are

general
all
name

Such

adjectives, without
of
a

exception.
is the that of

Every

adjective is implies that


of blood
or

the
the

thing

to

it is added, and
name

thing

possesses

qualities. Red implies


name

of other the

red

thing, and
term, the

it is red.

Redness redness.

is

abstract

the

quality

I.]
13. A of

DEFINITIONS

AND

EXAMPLES.

5
to
a a

positive concrete
its

term

is

applied

thing

in

respect
abstract

possession
denotes
"

of

certain

attributes

positive

terra

certain

attributes.
paper, rock
;

Examples

Useful,

active,

usefulness,

rockiness. activity,
A

14.

negative
of

term

is

applied

to ;

thing

in respect the
term

of

the

absence

certain

attributes

if abstract

denotes

the absence
"

of such

attributes.

Examples

Useless, inactive, not-paper;

uselessness,

inactivity.
15. An
or

absolute without
be.

term

is the
to

name

of

thing regarded
else, if such

per
there

se,
can

relation

anything

Examples
1

"

Air, book, space,


term
some

water.

6.

relative with

is the other

name

of

thing regarded

in

connection

thing.
cause,

Examples
17.
as

"

Father, ruler, subject, equal,


term

effect.

categorematic
a

is

one

which

can

stand

alone

the

subject of
"

proposition.
noun

Examples
phrase
1

Any
or

substantive

any

adjective, any

any

proposition used
is any

substantively.
word which
can

8.

A
as

syncategorematic term
the

only
some

stand

subject of

proposition in

company

with

other

words.

Examples

"

used Any preposition, conjunction, adjective

adjectively.
19. every

Differences
one

of

opinion

may

arise

concerninng
and

almost
not

of

the

definitions

given above,

it would

DOCTRINE

OF

TERMS.

[CHAP.
to

be

suitable

to

the purpose

of this book

discuss

the

matter

further. In
to

every

case,

too,

we

ought
there
term

before
no

treatingany
about
but

terms

ascertain

clearlythat
An confused

is

ambiguity
one

their
two
or one

meanings.
more

ambiguous
before The
upon
not

is

not
we

term,

terms
sense

and together,
we

should
to

singleout
the however
Nos.

definite

endeavour of

assign
has

logical
been

characteristics. dwelt sufficiently and

ambiguity
in be
of

terms

the

Elementary Lessons,
here.

iv,

vi.,and
the

it need further

pursued
the

For

study
i. and

subject
;

of

terms

the reader

is referred book and

to

the

Elementary Lessons

Mill's

System ofLogic,
i. ;
;

i., chapters
ii.; Levi

ii.; Shedden's

Logic, chapters i. Martineau,


Hamilton's

Hedge's Logic, part ii., chapter


vol.

Prospective Review,
Lectures
on

xxix., pp.

133,

"c.
to

Logic, vol. Hi,,lectures


to

viii.

xii. ;

Woolley's

Introduction

Logic, part i., chapter i.

QUESTIONS

AND

ANSWERS.

20.

Describe
terms
"

the

logicalcharacters equalisation.

of

the

ing follow-

Equal, equation,equality,equalness,

and inequality,

Equal things ;
it

noun-adjective; concrete, as denoting equal connotative, as connoting the attribute of equality;


a

is

relative general, positive,


cannot
as an

and
form

syncategorematic, because
the

adjective

subject

of

position. pro-

Equation, noun-substantive,originally abstract,as


either equality, or
the action
of

ing mean-

making

equal.

It is

I.]
now

DEFINITIONS

AND

EXAMPLES.
to

generally used

by mathematicians
to

denote

pair of

affirmed quantities

be

equal.
and

It is thus

concrete, general,

positive, perhaps absolute,


Inequality is
and
a

categorematic.

noun-substantive,abstract, singular, tive, negathe of action

categorematic.
means

Equalisation
or

of

making equal, an
a

bute attriis thus

circumstance

things, not

thing.

It

abstract, singular, positive, categorematic.

21.

What

are

the

logical characters

of

the

terms,

drop
A

of

oil,oily,oiliness ?
being
a

drop of
concrete,
a mass

oil

concrete,

finite

thing,its name
the

will

be
from

relative (as having dropped general, positive,


of

collective oil),
as

as

regards

particlesof

oil, connotative
and

implying

the

qualitiesof oiliness,"c.,
and

categorematic.
Oil

is concrete, like

connotative positive, collective,


of and oil, of

gorematic, cate-

drop
one

only

differs in

not

admitting,
a case

as

regards

any

kind

oil,of the plural. It is

of what vol. that


a

I have

proposed (Principles of Science, p.


to

28 ; ist I

ed.,
find

i., p. 34)

call

substantial and

term, but
the
older of

which

Burgersdyk, Heereboord, homogeneum,


with
also See
a

logicianscalled
the
same name

totum nature

the

parts

being
210.)

and
p.

the whole.

(Heereboord, Synopsis Logicae,


i. p. is concrete,

83.

Mind,

vol.

Oily is

and noun-adjective,
as

general, positive,
attributes of

connotative,

denoting

oil and

implying the

oiliness, doubtfullyrelative, syncategorematic,1680.


Oiliness, noun-substantive, abstract, singular, positive,

categorematic.
Where

distinctions

are

omitted, it

may

be

understood

that

they

are

regarded

as

inapplicable.

DOCTRINE

OF

TERMS.

[CHAP.

i.

22.

Describe

the

logical

character

of

the

terms
"

Related,

relative,

relation,

relativeness,

tionship, rela-

relativity.
I

have the

already prevalent
terms.

dwelt,
abuse of

in the is
of

the word

Elementary
'

Lessons

(p.
other than

25),
like

on

relation,'

and

abstract
reform

Nothing popular
use

more

nearly
;

impossible
but I

to

the

language properly
one

will

point
name

out

once

again

that

relation

is
of

the
to

abstract

of

the

connection attribute

or

bearing
of those be
'

thing
The
to

another,
in
'

this

being
are

an

things.
related,'
aunts,

things
be

question

properly
fathers,
not
"

said

to

or

relatives.'
all

Thus,
relatives

brothers,
relations.

sisters,

and

cousins,
abstract

are

Relationship
of

is

an

term

signifying
to

the

attribute
when

being
this
was

related

it

was

invented
concrete

replace
term.

relation The

wrongly
a

used
mother

as

relationship
the relation relatives. used the

between which

and

her

daughter
such related

is

simply
persons

exists Relativeness

between is
abstract

two

or

an

uncommon

term

sometimes where further


the

to

replace
not
one

the of

sense

of

relation,
is
of

case

is

family
due

relation.
to

Relativity
and

abstract

term,

probably
had
better

Coleridge,
the

which

metaphysicians

have

monopoly.

io

DOCTRINE

OF

TERMS.

[CHAP.

Europe

Advocate

Injustice
Brace
Dumbness

Being
Whale

of

partridges

Lawyer
Time Manchester

Planetary System
Classification

2.

In

the

case

of

the

following terms
which
are

distinguishwith
abstract and those

special care
which
are

between
concrete
"

those

Nature

Ether Ethereal
Ethereal
ness

Natural Naturalness Naturalism Author

Ethericity
Scarce

Authority Authorship
Animal

Scarcity
Scarceness Truth Trueness

Animality
Animalcule

Verity

3.
terms

Investigate
as

the

ambiguity
concrete
or

of

any

of

the

following
"

regards their Weight


Time

abstract Science Schism

character

Intention
Vibration

Space
Relation

4.

Supply

the

abstract
terms
"

terms

corresponding

to

the

following concrete
Wood Stone Conduct

Conduction

Atmosphere
Alcohol

ii.]

QUESTIONS
Witness

AND

EXERCISES.

11

Axiom
Gas

Equal
Table

Fire Socrates

Boy
In

5.

the
to

case

of such

of

the the

following

terms

as

you

consider
terms
"

be

abstract,

name

corresponding

concrete

Analysis
Psychology
Extension Production Socialism

Nation

Vacuity
Realm

Folly
Evidence

6. number and

Do
?

abstract

terms

admit

of

being put
terms

in

the

plural
time
"

Distinguish between
in
can

the

which
at

are same

abstract

concrete

the

following list,and opinion


be used

the in the

indicate

which

in your

plural:

colour, warmth,
7. the of

redness,

weight, value, quinine, equation, heat,


space.

hotness, solitude,whiteness, paper,

[c.]
of

Investigate the
form
;

logical characters
the
form
;
a

and

ambiguities
"

term

in all human form of prayer

followingexpressions:
;
a

a a

religion
school

forms
;
a

the

form

of
;

thought
a

form

mere

form printer's

form

of government

; form

good

form

; essential

form.

8.

What

error

is there

in the

? following descriptions

Peerless

"

syncategorematic, general, abstract, positive,

relative.
Bacon
"

equivocal,concrete, general,substantial, positive,


lute. categorematic, abstract,general, negative,abso-

relative. Black
"

12

DOCTRINE

OF

TERMS.

[CHAP.

9.

Analyse
of

the
each

followingsentences
term

as

regards the logical distinguishing


or

character

found
as are

in

them,
concrete
"

especially between
collective
or

such

abstract,

distributive, singularor general


science
et

Logic
Entre
"

is the rhomme
is

of

the formal

laws

of

thought.
it be and
a

le monde

il faut I'humanite.
;
so

Art

universal
it

in

its influence
from
a

may

in

its

if practice,

proceed
In

sincere
be

heart
merest

quick

observation. the
most

this

case

it may

the

sketch, or

elaborate

imitative

finish."

10.

Burton,
term
an

in

his

Etruscan Is

Bologna,
it

p.

234,

uses manner

the

abstract
to
one

Etruscanicity.
abstract
term

possible in
to

like

make ? If

corresponding
for the

every

concrete

so,

supply
Newton.

abstracts

followingconcretes

"

Sir Isaac

Royal Engineers.
Postal

Dictionary.
What faults logical do

Telegraph.
in the

11.

you

detect

following

? expressions The

standard
three

authorship of
is

modern
of

times.

The The

great nationalities
not

Western man's

Europe.
nearest

legalheir

a necessarily

relation.

12.

Coleridge, in
thus defines
an

celebrated
Idea and
"

note

to

his Aids

to

tion, Reflecnamely,
is
an

An

Idea

is the

of indifference
:

the

real objectively
if

the
as

real subjectively

so,

that

it be

conceived possesses

in

the

Subject,
; but

the

idea
an

Object, and
it is then the
as a

objective truth
is

if in

Object,
as cising exer-

Subject, and
powers
an

thought necessarily
Thus
a an

of

of

Subject.

Idea, conceived
Law. and
a

subsisting in

Object,

becomes

law

(in a mind) contemplated subjectively

is

an

Idea''

I.]

QUESTIONS
the

AND

EXERCISES.

13

Analyse

meanings

of

the

terms

Idea, Object, Subject,


passage, with

Real, Truth, Law,


to especially

"c., in the
concreteness

above
or

respect

their the

abstractness.

[L.]
to

13.

Name

negative terms
"

which

correspond

the

followingpositiveterms
Illumination White Certain Constant

Variable
Famous

Notorious Valid

Dying
14. Name

Plenty positive terms


which

the

correspond
"

to

the

followingnegative or apparently negative terms Immensity


Inestimable Disestablishment Falsehood

Unravelled
Infamous

Unpleasant
Want

Presuppositionless
Shameless

Unloosed Indifferent Headless


In
as

Empty
Intact

Ignominious
the

15.
as

examining
and
of

following list
those

of

terms,
are

distinguish,
the

far

between possible,

which which

reallynegative
simulate

in

form

origin, and negatives


"

those

only

character

Annulled Disannulled Antidote

Undespairing
Invalid Headless

Infrequent

Independence
Individual
Indolent

Eclipse Undisproved
The

Infinite

Disagreeable

14

DOCTRINE

OF

TERMS.

[CHAP.

Impassioned
Immense

Despairing
Infant

Purposeless
6.

Deafness

Can
are
'

you
true

find any double may


are

examples perhaps
each
a

of

terms

in the

dictionary

which and of
two

negatives?
be

'Paired,' 'Impaired,'
affirmed and

Unimpaired,' things which


the

respectively
not

equal,unequal,
of
true

unequal.
and

Analyse
show

meaning
it is
or

of is
not

the

following terms, negative


"

whether

double

Indefeasible Uninvalided

Indefatigable Uninjured Undecipherable


Undeformed

Undecomposable
Undefaceable Indestructible

Indistinguishable
denotation
to

17.

How
term

are

the

and

connotation of the

of

crete con-

related
term

the

denotation

corresponding
connotation

abstract
1

8.

Explain the
reference
to

difference the
terms

of

denotation

and

with

Law,

Legislator,Legality, [L.]
the

Crime.
19.
terms
"

Compare

the

connotation

of

following sets

of

Abbey Abbey

Caesar

I Westminster
i' Mineral
"

\ Roman
/

Road of communication

Oxide

of iron

"J Means

I Ore
20.

I Railway
the

Distinguishin opinion on
the

following list
at

such time

terms

as

are

nori-connotative,naming
whose

the

same

the

logician

subject you

adopt

"

II.]

QUESTIONS
A^irtue Virtuous The
mother

AND

EXERCISES.

15

Socrates

Barmouth of the Gracchi


The Lord

Gladstone
Form What the ?
to
name

Chamberlain

21.

list of twelve

purely non-connotative
connotation
;

names.

22.

is, if any,
First ;

the

of

these
j Santa

terms

Charles

Richelieu

John

Smith

Maria

Maggiore
23. names, What

Try
and

half-a-dozen

perfectly
involved in

non-relative non-relative. each


of

then

inquirewhether implied
or

they reallyare

is the

relation
?
"

the

followingterms

Metropolis
Realm

County

Alphabet
Sun

Capital city
24.

Show, by examples, that the

division

of

Names the

into

general and
into abstract 25. What

singular does
and kinds
concrete.

not

coincide

with

division

[L:]
can are

of words what

stand

as

the

subject of

proposition,and
26. of

kinds

excluded?

[o.]
quantity : omnis,
in the

Distinguish

between Latin

the

distributive,collective, or
of

singularuse
omnes, 27. What

these

adjectives

cunctus,
is

cuncti,ullus, quidam, aliquis. peculiarabout


?
"

the

use

of certain

terms

followingextracts

(1) Frenchmen, (2) (3) (4)


His
Howard. In quo

I'll be

Salisbury to beyond
that

you. of
a

family pride was quisque


foe

Talbot

or

artificio

excelleret,is in

suo

genere

Roscius
'

diceretur.
meets

When

foe.'

16

DOCTRINE

OF

TERMS.

[CHAP.
junctions con-

28.

How ?

does

Logic

deal

with

verbs, adverbs, and

29.

How

many ?

logical terms
Which is mind is
matter

are are

there

in
?

the

following

witty epigram
What What

and
? ? No

what

they

matter.

Never

mind.

30.

How

many
sentences

logical terms
?

are

there

in

each
many

of

the

following
are

Ascertain
such
term.

exactly

how

words

employed (1)
The

in each

Royal

Albert

Hall
on

Choral
the

Society'sConcert
Gore of

is

held

in the Albert
the

Hall

Kensington

Estate

purchased by
Exhibition
of is

Royal

Commissioners

the

Great

1851.
a

(2)

"A

name

word

taken

at
our

pleasure
mind
a

to

serve

for

mark
some

which

may
we

raise had
may

in

thought sign

like

to

thought
to

before, and
be
before
to

which
a

being
of

nounced pro-

others,

them

what

thought
31.

the

speaker

had

in his mind."

Words,
"

says

Hobbes,
men

are a

insignificant (thatis
name

without whose

meaning),

when

make

of

two

names,
:

are significations

contradictory and
few

inconsistent

as

this

name,

an

incorporealbody."
a

The

following are
names,

instances student

of

such

apparently
to

selfto

inconsistent
the

and

the

is

requested

add

list
"

(1) Corporation

sole.

(2) Trigeminus. (3) Manslaughter


(4)
An of
a woman.

invalid breach

contract.

(5) A

of

necessary

law

of

thought.

CHAPTER

III.

KINDS

OF

PROPOSITIONS.

i.

IN

this

chapter
to

propositions
the forms ancient of

will

be

described

and in

classed which thus

according
four

Aristotelian

doctrine, recognised,

principal
stated

propositions

were

tabularly

"

Affirmative.

Negative.

S*

Universal

Particular

Singular
indefinite
must

propositions

are

to

be

classed

as

universal,
of

and

propositions,
be The

in

which
at

no

indication

quantity
or

occurs,

interpreted
student
text

discretion
to

as

universal familiar

particular.
what
I

is -books of their

supposed
say upon

be

with

the first

ordinary

the of form

subject. propositions,
and with

give

series

Examples logical

brief
A
next

comments

upon

peculiarities.
in the

copious
chapter

selection
for the

of student

exercises
to treat

is in

then like

supplied
manner.

CHAP,

in.]

KINDS

OF

PROPOSITIONS.

19

EXAMPLES.
'

2.

Books

are

not

absolutelydead
is indefinite
or

things/

O.

This Hamilton
p.

proposition
would
as

pre-indesignate,as Logic, Vol.


Milton I.
to

call it
we can

(Lectureson hardly
were

(III.),
have it to

244) ; but,
that
'

suppose

thought
mean

all books books


are

living things, I
is
to

take

some

not, "c.,' that

say,

particular

negative.
'

3.

The

weather
means

is cold.' the
be

A.
state

The

weather and

present

of
as

the
a

atmosphere,
which
'

may

best

described

surrounding singular term,

makes
Not

the assertion all the

universal. of the officers and


were

4.

gallant efforts
at

escort
save

of

the

British E.

Embassy

Cabul

able

to

them.'
At
'

first

sight this
'

seems

to

be
'

particular negative,like
a

Not

all that

is gold glitters
'

; but

little consideration

shows

that made

gallantefforts
in common,
as a

is

collective

whole, the efforts


successful
'

being
of the
a

and
The
were

therefore

either then
to

or

unsuccessful

whple.

meaning
not

is,
save

The

whole

"c., gallantefforts,
universal bad

able

the men.'

It is

negative. general is
better than looks
but
at two

5.

'

One

good
a

ones.'

A.

This
even means a

saying of Napoleon
definite

first like the


' '

or particular

singular proposition ;
not

one

bad bad

general general

'

any

one,

but

'

any

one

acting
6.
'

alone.

No

non-metallic E.

substance

is

now

employed

to

make

money.'

20

KINDS The

OF

PROPOSITIONS.

[CHAP.
the

subject is
as
'

negativeterm,
non-metallic
make

and

proposition might
are

be

stated

All
to

substances

not

any

of

those
'

employed

money.'
vexation.'
so, this is A
;

7.

is Multiplication

If all

is multiplication of vexation. is
not

there

are

certainly

other
8.

causes

'

Wealth

the

highest good.'
is
one

E.
are

wealth Affirmatively, the

of

the

things which

not

highest good.
'

9.

Murder
most

will out.'

A. this is
an

Like

proverbs,
its material

unqualified
be doubted.

universal

proposition;
10.

truth may is
a

'A

little knowledge looks like that


a

dangerous thing.'

A. is

This

particular affirmative, but


'any
small collection
of

really

A,

as

meaning

knowledge

is,"c.'
'

11.

All

these

claims

upon
out

my

time

overpower Ed.
p.

me.'

A. that

Dr.

Thomson

points

(Outline, 5th

131)

all is here
'

clearlycollective.
whole is greater than any is of its

12.

The

parts.'

A.

Though meaning
' '

this apparently singular, any whole


run

a general axiom, really

is greater, "c.' wild in Great Britain


at

13.

No

wolves

the

present

day.'
14.

E. 'Who
never

seeks

and

will

not

take, when

once

'tis offered,

shall

find it more.'
to
one

E.

This
'

seems

be who

compound
is

proposition,but
has
not

the

ject subwhen

is, Any
once

seeking,but

taken

it

was

offered.'

HI.]
15. 'The
known

EXAMPLES.

21

planets are

now

more

than

hundred

in number.'

A. collective Of

Clearly a
therefore
not

singular affirmative
course

proposition,and

universal.
the

the

planets separatelycould

have

predicate here
come

affirmed.

5.

'

Figs

from
but
we

Turkey.'
cannot to

I.
assume

Indesignate ;
statement

without
'All

express from

that

it is intended

say,

figs come

Turkey.'
17. 18.

'Xanthippe
'

was

the wife
free who

of

Socrates.' enslaved

A.

No E.

one

is

is

by

his

tites/ appe-

19.

'Certain A.

Greek

philosophers

were

the

founders

of

logic.'

Apparently
group the of
men,

I ; but
each

if of

'

certain

'

means was

certain in

definite his

whom

essential and

time,

proposition

becomes

collective

singular, hence

universal.

20.

'

Comets

are

subject
; but

to

the law
a

of

gravitation.' A.
of such

Indefinite it may
21.

affirmative

in

matter

universality

be

interpretedas
ends
; but been

A. in
as

'

Democracy
indefinite have

despotism.'
to referring

I.
matter

Again

in which
be

no

rigorous laws particularly.


'

detected

it should

interpreted

22.

Men

at

every I.

period

since

the time

of Aristotle

have

studied

logic.'

Obviously particularas regards

'

men.'

22

KINDS
'

OF

PROPOSITIONS.

[CHAP.
O. O.

23.

Few

men

know
men

how do
not

little they know.'

That
*

is,

'

Most

know, "c.'
A. is is

Hence

24.

Natura

omnia

dedit omnibus.'
nahtra

because Singular affirmative,

singular term. thoroughly


biguous am-

The

assertion
as

is

one

of

Hobbes',
and
or

and

regards
of

omnia

omnibus,
distributive
Nature

which

might

be
No

capable

either

collective

meaning.
did
must not

doubt, however, the

meaning

is that
;

assign
taken

anything

to

any

particular person

if so, both

be

collectively.
'

25.

There
a

are

many

cotton-spinnersunemployed.'
assertion
'

I.

Really
at

kind be

of numerical

; but
a

if to
'

be

classed

all,it
26. 'A

must

I,

'

many

being only

part of
the

all.' of

few A.

Macedonians

vanquished

vast

army

Darius.'

Collective
acted
not

because affirmative, singular


It

the

few

of

course

together.

is

question whether

the

predicate

is

also

singular.
True Faith and Reason
are

'

27.

the

soul's

two

eyes.'

A.

Collective

singular.
ought always
to

28.

'A

perfect man
A.

be

busy conquering

himself.'
'

All

'

perfectmen
A

ought, "c.
man

29.

truly educated

knows
A.

something

of

thing every-

and There
sentence
man

everything of something.'
seem

to

be

two not

and predicates, the case, because

hence the

compound

; but

this is both.

trulyeducated

must

know

in.]
'

EXAMPLES.

23

30.

Some

comets

revolve
as

in

hyperbolic orbits.'

I.

Particular

affirmative

it stands.

'

31.
'

The

dividends
'

are

paid half-yearly.'A.
all
so

The

dividends

includes

known.

'

32.

Ov
must

TO

fj.eya
mean

"v

tori,

TO

ev

/Aeya.'

and

A.

This but that

that

not
are

all great

things are
There and

good (O),
are

all

good
"

things
great

great

(A).
;

three
;

classes

of

things

and

good

great

not-good

not-great and
33.

not-good.
alone which
can

'It

is force A.

produce

change

of

motion.'
It
=

what
a

can

produce,
of

"c.

The is
-

meaning
kind

is,Whatever
of

produces
there

change
assertion

motion

some

force ; but
"c.

is

no

that force

whatever

produces,

'

34. As
'

We

have

no

king but
; but
'

Csesar.'

it

stands, A
is
our

the
'

meaning conveyed implies that


who

Caesar

king

Nobody

is

not

Csesar

is

our

king.'
35. be
not

'It

is

true

that
at

what

is settled

by custom,

though

it

good, yet

least

it is fit.'

Complex 36.
make
A
'

; three

propositionsin
make
man,

all.

God

did

not

and

leave

it

to

Aristotle

to

him

rational.' and

simple

singularnegative proposition;
follows
for conjunctively,
to assert

the

'

not

'

applies to
could man.'
not

all that
have

of

course

Locke
not

intended

that

'

God

did

make

E.

24

KINDS

OF

PROPOSITIONS.

[CHAP.

in.

'

37.

Dublin

is

the

only

city

in

Europe,

save

Rome,

which

has

two

cathedrals.'

Compound
Dublin

sentence

implying
two

three

propositions,
A.

namely

"

has

cathedrals.

Rome

has

two

cathedrals.

A.

All

European
Rome,
have

cities,
not two

not

being
cathedrals.

Dublin

and

not

being

E.

38.
fear,
and

'

The

affections

are

love,

hatred,

joy,

sorrow,

hope,

anger.' disjunctive
or

Really
or

proposition.
This

Affection

is

either

love,

hatred,
is
an

"c.

implies
"c.

that

love

is

an

affection,

hatred

affection,

26

PROPOSITIONS.

[CHAP.

(16)

XaXcTTa

TO.

KCiAu.
rotten

(17) (18) (19) (20) (21) (22) (23) (24)


(25)

There's
To

something
or

in the state

of Denmark.

be
are

not

to

be, that is the question.


do all I have said
unto

Ye

my

if ye disciples, posse
no

you.

Possunt
There
can

qui

videntur. effect without


a cause.

be

Rien Pauci
All

n'est beau
laeta
arva

que

le vrai.

tenemus.

cannot

receive
I
a

this

saying.
that I fear to
can

Fain

would
not

climb, but

fall. like that it

(26) There's
takes

joy

the

world

give

away. know
me

(27) (28)
(29) (30) (31) (32)

Not
Two

to

argues make from

thyselfunknown.
a

blacks
men are

won't
free

white.

Few
He

vanity.
runs

that another

fights and day.

away

may

live

to

fight

We

are

what is
none

we

are.

There Two
Better

good

but

one.

(33)
(34) (35) (36) (37)

lines straight
late than

cannot

inclose

space.

never.

Cruel Omnes
Le

laws

increase
bona

crime. dicere.
a

omnia

genie

n'est

qu'une plus grande aptitude


in solitude is either

la

patience.

(38) Whosoever
beast
or a

is

delighted

wild

god.
summa

(39)

Summum
Non
omnes

jus

injuria.
inulti.
succurrere

(40) (41) (43)

moriemur

Haud

ignara mali

miseris

disco.

breeds (42) Familiarity Some

contempt.
read
to

cannot politicians

the

signsof

the

times.

(44) Only

the

ignorant affect

despise knowledge.

iv.]

EXERCISES.

27

(45)

Recte

ponitur;

vere

scire

esse

per
to

causas

scire.
across

(46) Only Captain


Channel.

Webb

is

able

swim

the

(47)
(48) (50)

Some E pur

books si
muove.

are

to

be

read

only

in parts.

(49) Civilisation
Some Sunt
men

and
are

are Christianity

co-exlensive. falsehoods. telling

not

incapable

of

(51) (52) (53)


(54)

nonnulli
lusus.
true

acuendis

puerorum

ingeniis

non

inutiles
All Me

is

not

that

seems

so.

miserable.

The

Claimant, Arthur
same a

Orton, and
person.

Castro

are

in all

probabilitythe

(55)

The
to

three
two

angles of
of grammar

triangleare
overload

necessarilyequal
the memory.

right angles.
exitium artis
are

(56) Many (57) (58)


(59)
Nullius Summae

rules

patiturnatura
occultare results
artem.

videri.

est

Wonderful
recent

the

of

science

and

industry in
it alteration

years.

(60)
(61)

Love

is

not

love

which

alters

when

finds.
A

healthy nature
is
no

may

or

may
not

not

be

great ; but

there

great
dederis

nature

that is
semper

healthy.
opes.

(62) Quas
(63) Quod (64)
Ilacra

solas

habebis homines.

volunt, id credunt

crap"
would

ov

SiKaiw^T/crerat.
mundi. mother's
servants.
son.

(65) Antiquitas seculi,juventus (66)


(67)
That
Men

hang

us,

every
are

in great
ever

place

thrice

(68) Justice is (69) (70)


A Men

equal.
bear
a

friend
are

should
not

friend's
were.

infirmities.

what

they

28

PROPOSITIONS.

[CHAP.
in

(71) The (72)


(74) (76) (77) (78) (79)
Nemo

troops

took

one

hour

passing sapit.

the

saluting

point.
mortalium labuntur omnibus anni. horis

(73) Fugaces
AUTOS
cyw

elfju.
sunt

(75) Communia
Dictum
The The To Romans fear of

amicorum
est.

inter

se

omnia.

sapientisat
the

conquered
leave

the

Carthaginians.
is die.

Lord, that is wisdom.


we

live in hearts

below

not

to

(80)
(81)
2.

'Tis
Dum

only

noble

to

be

good.

spiro spero.
over

In

looking
have the

the those

following
which have

list of
a

propositions
and

distinguishbetween
those which
All
a

distributive

collective
have

subject.
been discovered

(1)

asteroids

during

the

present century.

(2) All
(3) (4)
The

Albinos facts dress


is

are

pink-eyed people. aboriginal life


out
seem

of

to

indicate

that

developed
omnia

of decorations.

Non Dirt

omnes

decent.
are

(5)
(6)

and

overcrowding apostolisunt

among

the

causes principal

of

disease. dtiodecim.

Omnes

(7) Many

artisans side and

are

unemployed.
of
a

(8)

The

diagonal
animal.

square

are

surable. incommen-

(9) Omnis
(10)
3.

homo
est

est

Nihil

ab

omni

parte beatum.
how many distinct and assertions the
are

Ascertain in
each of

exactly
of
the
these

made

sentences,

assign

logical

characters

propositions.

iv.]

EXERCISES.

29

(1)

'Tis

not

rny

that profit it.

doth

lead

mine

honour

mine

honour,

(2) True,

'tis a

'tis true. pity ; pity 'tis,

(3) Hearts,
love

tongues,

figures,scribes,bards, poets,

cannot

think, speak, cast,


to
a

write, sing, number, kingdom


quod
for
ante

ho

! his

Antony.
horse sapere,
:

(4) (5)

horse,
est

! my
non

horse.

Istuc

pedes
sunt,
defect

modo

est

videre

sed

etiam
neither

ilia, quoe
in
excess

futura
nor

prospicere.
of

(6) Virtue
but

consists in
a

action,

certain
our

mean

degree.
and
state
are

(7) (8)

The

gloriesof
substantial

blood

shadows,

not

things.
perfume
the

'

To To To

gild refined gold, to paint the lily,


throw smooth the
a on

the add
with eye

violet,
another

ice, or
or

hue

Unto
To

rainbow,
the

taper
of

light
to

seek

beauteous

heaven

garnish

Is wasteful

and

ridiculous
the
man

excess."
heaven

(9)

All
Are

places that
to
a

eye

of

visits,
havens. the

wise
of

ports

and
is

happy

(10)

The

age

chivalry
fit.

gone,
ever,

and

glory

of

Europe

extinguished for
is
or

(n)
(12)

Poeta Not

rascitur, non
all

speech
truth i.-;

enunciative,
falsehood.

but

only
War,

that in which

there

(13) Devouring Famine, Plague,


Each Death's

and

able
servile

to

undo

Mankind,
are.

emissaries in men's
the of

(14) Many

are

perfect

humours,

that

are

not

greatly capable of
is the
more

real
one

part of business, which


that

constitution
than

hath

studied

men

books.

30

PROPOSITIONS.

[CHAP.
c'est
is

iv.

(15) (16)

Vivre, Justice by

ce

n'est

pas

respirer,
but into
of

agir.
expediency
reason

is

expediency,
maxims,

it

speaking
has
trated concen-

general
the

which mankind. stare,

experience
and

(17)

Men,
it

wives,
were

children,

cry

out,

and

run

as

doomsday.
far which
between

4. in the

Distinguish following

so

as

you
are

can

the

propositions
and

list

to

you
pp.

explicative
68
"

ampli-

ative. Outline

(See
of
Homer A the

Elementary Necessary
wrote

Lessons,
Laws

69. " 81.)

Thomson's

of
and solid
two

Thought,

(1) (2)

the

Iliad is
a

Odyssey. figure having


are

parallelepiped
which every
on

six

faces,

of

opposite
the
to

parallel.
of
a

(3)

The

square

hypothenuse
the
sum

right-angled
on

triangle
sides

is

equal

of

the

squares

the

containing
is
a

the

right angle.
bird.

(4) (5)

The Axioms

swallow
are

migratory

self-evident

truths.

5.

Classify
as

the
are

following generally negative,

signs
used
or

of
to

logical
indicate

quantity

ing accord-

they
or

universality,
affirmative
or

affirmative

particularity,

negative

"

Several,

none,

certain, whole,

few,

ullus, nullus, all,


not

nonnullus,

not

few,

many,

the

almost

all.

CHAPTER

V.

CONVERSION

OF

PROPOSITIONS,

AND

IMMEDIATE

INFERENCE.

i.

THE

student
to

is

referred

to

the

Elementary
for
the

Lessons
common

in

Logic,
rules

or

other

elementary
and

text-books,

of of

conversion easy

immediate the ancient

inference,
square of

but,

for

the is

sake

reference,

opposition

given

below.

A
..

Contraries
..
..

cr

"#"

Subcontraries

All

the

relations

of
to
a

propositions single
in

and

the will be

methods
found

of

inference

applying
and

proposition
the

fully
and

exemplified
answers.

described

following

questions

32

CONVERSION.

[CHAP.
to

2.

It

appears
some

to

be

indispensable, however,
nomenclature
two
an

endeavour relations Alexander


the
name

to

introduce

fixed

for

the

of

propositions
Bain

involving
made

terms.

Professor

has

already
and
of
terms

innovation

by

using

ofcerse,
reformers
use

Professor
the

Hirst, Professor
of

Henrici
have

and

other
to

teaching
and attached
It
seems

geometry
obverse in in

begun

the

converse

meanings logical

sistent inconscience
to state

with

those
p.

to

them

(Mind.
in
to

1876,
most

147).

needful, therefore,
nomenclature of

the
be

explicit way
with the what the

the

here
Professor
'

proposed
Robertson.

adopted
as

concurrence

Taking following

original proposition
we

all

are

",'
"

the

are

may

call

the

related

propositions

INFERRIBLE.

Converse. Obverse.

Some
No A

B
are

are

A. B. B B
are are

not not

Contrapositive.
or,

No all not

A.
not

A.

NON-INFERRIBLE. Inverse. All B A. A


are

are

Reciprocal.
It
must

All
that

not

not

B.

be
are

observed all
not true

the

converse,

obverse,

and

con-

trapositive
The
same

if the

original proposition
the
names case

is

true.

is

necessarily
latter
two

with
are

the

inverse from

and
the de

reciprocal.
excellent
la

These of pp.

adopted

work

Delbreuf,
88
"

Prolegomenes
at

Philosophiques
of

Geometric,

91,

the

suggestion
p.

Professor

Groom

Robertson.

(Mind,

1876,

425.)

34

CONVERSION.
'

[CHAP.
are
'

that materiallyfalse)
may remain

all

men

dishonest
some men

'

;
are

but

it

theless never-

true

that

dishonest.' involve
the truth

Observe, then, that


truth
of

the
the

of falsity truth
of

A
I

does

not

the
or

I,
A.

nor

does
But

of
A

involve

of falsity
As

the

truth

necessitates

that
"

of

I.

stated the

in

the

Elementary
true

Lessons

(p. 78),
universal
be
true

Of be

alterns, subtrue
:

particular is
may
or

if the
not

but

the

universal true."

may

when

the

is particular

5.

How

do

you

convert

universal

affirmative

propositions ?
They
as

must

be

converted
is
to

by
while

limitation

or

per

accidens,

it is

called,that

say,

preserving the
be

affirmative

the quantity of quality,

the

proposition must
A
less

limited from
into

universal in the

to

particular. Thus
more or

is converted

I,

as

following

troublesome Converse

instances, the
second in
each

Convertend

standing first

and

the

pair
f All

of

propositions:
contain carbon.
are

organic substances
substances for
no man

( Some ( Time

containing carbon
bides.
for
no man

organic.

I Something biding
f The
poor

is time.

have have

few
few

friends. friends
more are

I Some
f
"

who
man

poor.

A. wise

maketh make

opportunitiesthan opportunities than

he

finds. find
are

Some wise

who
men.

more

they

They

are

ill discoverers
see

who but

think
water.

there

is

no

land, when

they can

nothing

( Some

ill discoverers

think

there

is

no

land, "c.

v.]

QUESTIONS
is Diana of the

AND

ANSWERS.

35

f Great
I Some

Ephesians.
of the

great being is Diana


animals
are are

Ephesians. exception air-breathers.

Warm-blooded

without
or

J Air-breathers

(with

without

exception) warm-

blooded

animals.

6.

How

would
?
'

you

convert

'

Brutus

killed

Caesar
The

strictly logicalconverse
was

is
a

'

Some
can

one

who
be

killed killed
in

Caesar
once,

Brutus.'
Brutus

For, though

man

only
the

and

is
know

said distinctly

to

be

killer, yet
other

formal

logicwe
have been

nothing
on

of

the

matter, and

Caesar
persons.

might
An

killed

other

occasions

by

absurd

illustration is
to

purposely chosen
memory the

in the

hope

that

it may
that

assist

fix

in

the

the
matter.

all-importanttruth

in

logicwe
How

deal

not

with

7.

do

you

convert

particular

affirmative

propositions?
To

this kind
that

of
to

propositionsimple
say,

conversion

can

be

applied;
words,
thus of the

is

the

converse

will preserve
convertend.

both
In

the

quantity and
I when
of
:
"

the

quality

of

the

other
I
;

converted the

gives

another the

propositionin simple
converse

either
other

followingpairs is

( Some

dogs

are

ferocious animals
not

animals.
are

I Some
f

ferocious
men

dogs.
to

Some

have have
men.

courage

appear
to

as

good
as

as

they are.
as

"JSome,

who
are

not

courage

appear

good

they

are,

( Some

animals

are

amphibious.
animals.
D
2

1 Some

amphibious beings are

36
8. How

CONVERSION.

[CHAP.
universal

do

you

convert

negative

positions pro-

?
These also converted
E

are

simply,giving another gives E.


;
a

universal
is that both

negative proposition.
the terms of
E
are

The

reason

distributed

universal whole
man

negative asserts subject and


animal
class
one
'

complete separation between


the whole that
of the

the
'

of the
is
a

predicate.
one man

No

tailed

asserts

not

any

is found it follows

anywhere
animals in the is
a

in the
no

of

tailed

animals.
to

Hence

evidentlythat

being belonging
the
class

the

class

of tailed
we

is found

in
verse con-

of

men,

which

result tailed mode

assert

simple given

proposition, 'no examples


No No of the
same

animal

man.'
are

Further

of conversion

below.

virtue is

ultimatelyinjurious.
a

ultimatelyinjuriousthing is
wise
one man runs

virtue.

f No

into heedless into heedless look forward


ancestors. to

danger. danger
is
a

I No
f

who will

runs

wise

man.

People People
not

not to

who posterity

never

look

J
I
\.

backward
never

their look

backward
to

to

their

ancestors

who

will

look

forward

posterity.
is
not
an

( Whatever

is insentient is
an

animal.

I Whatever
How

animal

is not

insentient.

9.

do

you

convert

particular negative

? propositions
arises Difficulty rule of conversion the about tells
converse us

this
to

question,
preserve

because

the

first

the

qualityof
be its be

the

proposition ;
But
a

accordingly should
distributes class
must

negative. predicate,
excluded

negative proposition always


a

because

thing excluded

from

v.]
from

QUESTIONS
every

AND

ANSWERS. Now

37

part of the

class.

the

subject
as a

of

being
truth,

particularand predicate.
'

it indefinite,

cannot

stand say

distributed

It
are
'

is still
not

possible to
'

with

material this
'

some

men

soldiers

;
are

but
not

converted
'

gives the
soldiers
' '

absurd
are

result, all soldiers


Even

men

; or,

no

men/ the
we

if

we

insert the
say,

mark

of

quantity
are

some some

before

predicate, and
remember
all.

'all soldiers
'

not

men,'
and

must

that The
so

'

some

is

indefinite, perfectly
be
see,
more

may

include further
on,

questionwill
far
as as

cussed fullydis-

but,

can

the

particular
and fact This

negative proposition, so
indefinite constitutes in
a

long

it remains

negative

meaning,
the when
If

is

incapable of
O

conversion.

blot in the ancient

logic.
is

Nevertheless
converse

proposition
we
'

capable
excluded

of

giving
from

result

change
some men

it into the
'

tive equivalentaffirma-

proposition.
class
'

are

the class is
a

soldiers,' they
'some

are

necessarilyincluded
men are

in

the This

non-soldiers, or,

non-soldiers.'
as

proposition in I, and gives


are a

by simple conversion,
also

scribed, already denon-soldiers

converse

in
"

I,

'

some

men.'

As

further

examples take
not

( Some I Some

dicotyledons have plants with


are crystals

reticulate
leaves

leaves.
are

non-reticulate

dicotyledons.

J Some
I Some
( All

not

symmetrical. crystals,

unsymmetrical things are


men

have

not

faith.
not

1
f

Some

who

have

faith saith
of

are

men.

Not

every into

one

that

unto

me,

Lord, Lord, shall

enter

the who
unto

Kingdom
shall me,
not

Heaven.

^ Some
V, say

enter

into

the

Kingdom

of

Heaven

Lord,

Lord.

38
10.

CONVERSION.

[CHAP.

How

do

you

convert

singular propositions ?
those into
a

Singular propositions, being


term
as as

which
two

have

singular (See
be the with

subject, may predicate


is

be
a

divided

classes, according
term.

the

singular or
The

general
former

Karslake,
converted
same

1851, vol. i., p. 54.) simply,one


another of is

will

always

single thing being


name,
as

identified Victoria
'

under

in

'Queen
into

is the
of

Duchess
Lancaster

Lancaster,'

converted

the

Duchess

Queen

Victoria.'
a as

Simple conversion general term, provided


to

will also
that

apply

if the

predicate be
is

the

be negative so proposition

distribute

this
'no

term.

Thus,

'St. Albans St. Albans.'


as

not

great

city' becomes predicate


be

great
and

cityis
tributed, undiswe

But in
an

if the

general

affirmative

then singularproposition, limit the


new

must
or even

convert
one case

per accidens,and
of significate
the

subject

to

some

general

term.

Examples

of

each

follow

"

f The

better

part of valour
is the better

is discretion.

Discretion

part of valour.

( Time

is the greatest innovator.

\ The

greatest innovator

is time.

f London I The

is the greatest of all cities.

greatest of all cities is London.


is
not
a

( London

beautiful
London.

city.

I No
( Le

beautiful

cityis

styleest 1'homme
meme

meme.

L'homme

est

le

style.

f All

the allied troops who

fought courageously.
the allied troops.

I Some

fought courageously were

v.]
i

QUESTIONS
but

AND

ANSWERS.

39

Mercy

murders, pardoning
which
murders

those

that

kill.
those that

"| Something I kill.


Not all the could
run

is mercy,

pardoning

figures that
up,

Babbage's calculating machine against the general heart.


stand

would would

stand
not

Something
is all the could
run

which

againstthe general

heart

that figures (collectively) up.

Babbage's machine

u.

Show

how
All

to

convert

the works

propositions
"

(1) (2) (3)

'

mathematical

are

not

difficult.'

'

All

equilateraltriangles are triangle


has
one

equiangular.' equal
to

'

No

side

the

other
The looks
are as or

two.'
it

first

proposition, as
the universal

stands, is ambiguous, for it


'

like

negative,
to

no

mathematical
we

works

difficult.'

But, according
'

custom,

may
are

interpret it

meaning
'

that

not

all mathematical works


cannot
we are

works

difficult,'

some

mathematical
O.

not

difficult,' a proposition simply,as already


the
are

in the form

This

be

converted
must

explained (p. 36), because


and quality, works
'

preserve

negative
matical mathe-

'all
'

(or some)
We

difficult

things

not

being negative would


works.'
can,
are
'

distribute

its O

predicate
into

mathematical
some

however, make
not-difficult

I,
we

'

mathematical
convert

works

and things,'

can

this

simply into
it

some

not-difficult

things only

are

mathematical

works.'

Proposition (2), as logicallyconverted trianglesare


shown
are

stands, is in A, and
limitation into
'

can

be

by

some

equiangular easilybe

equilateral.'Geometrically
the inverse

it could

that

proposition
true

'

all
we

equiangular triangles
must

is equilateral,'

also

; but

of

course

not

4o

CONVERSION.

[CHAP.
in

allow

knowledge

of

the

matter

question

to

influence

us

in
be

logical deduction,
inferred Number from the

and

the

inverse

proposition cannot
be converted
the

original. negative,and
one

(3)is a
' '

universal

must to

simply into
is
a

Nothing having
but there the student

side

equal

other

two

triangle ;

is

something paradoxical
is recommended
to

about

this

which result,

investigate.

12.

Convert
is
an

'

Life

every

man

holds
the

dear.'

This

example
have

given
by

in

Elementary
it into
"

Lessons,

(p. 304).

Students Life is held


Some No
man

variouslyconverted
every
man.

dear

life is held holds it

dear, "c.
dear

death

(!)
be
easy
to
see

and

so

forth.

But

ought surely to
is that universal into
'

that the

grammatical object
'

is

transposed, 'life' being


'

the

object
holds

of

holds

dear.' is

The

statement

ever}'

man

life

dear,' and
to
are

a explicitly

affirmative
some

proposition,
hold life dear

be
men

converted
!

by

limitation

who

13. "What
or seems

Convert
is it that
more

the

pioposition
What
the

'It rains.'
'

rains ?

is

'

it ?

ment, Surely the environThe

exactly
'the

atmosphere.
is

proposition
fall.' The

to

mean,

atmosphere
be
'

rain letting which and


many

converse

will therefore

something
in this

is

lettingrain
other
cases

fall is the

atmosphere.'
process
not

But

the Aristotelian

of

conversion result.

by

limitation

gives a

meaningless
14.

if

absurd

Convert
never

the felt

proposition
a

'

He

jests

at

scars

who This

wound.'
on

is the

8th

example

p. 304

of the

Elementary

42

IMMEDIATE

INFERENCE.

[CHAP.

16.

What metals

is the
are

obverse
'

of

the

proposition

'All

elements

The Alexander in his

obverse

is

new

term

introduced is thus
no.

by
described

Professor

Bain, and

its

meaning
pp.
to

by him

Deductive
must
'

Logic,
be
not

109,

"In

affirming one
'

thing,we
is
fact He from

prepared
side.

deny
not

the
two

opposite :
facts,but

the

road
same

level,' it is

are inclined,'

the

its other
to

This
that of
'

process each
an

is named
of the four
'

OBVERSION."

proceeds

point out
is
Y' X

propositional
X
'

forms, A, I, E, O, admits
becomes X is
not

obverse.
X

Every

is Y'
some

'no
not-

not

Some

is Y'
'

becomes
all X

K'
Y'

'No

is Y'

becomes X

is

not-

Y'

'Some

is

not

becomes

'some

is not-F.' will be
'

ingly AccordNo metals

the obverse
are

of the

propositionabove
describe
an

not

elements.' Bain
goes
on

Professor

to
on

what

he

calls of

'

Material
matter

Obversion,' justified only


of
the

examination
'

the

proposition.
If

Thus of

from the is

warmth

is

he agreeable,'
'

after examination infers,

that subject-matter,

cold

is
I

disagreeable.'
feel sure,

knowledge

good, ignorance
up of so-called

is bad. material

however, that this mixing


with
formal

obversion

obversion Mr. Bain

is

likelyto
because

confuse

people
he

altogether. Indeed, cites,


' '

is himself

confused, for
I like
at
a

I don't
a

like

curving road,
'

straight
the
all of the

one same

as

childish in

reason,

being

no

reason

all,but
at

fact

obverse.'
two

Now,

if there
is

is any

relation
case

between material
same

these

it propositions,
; but

certainly a
do
'

obversion

in
formal

reality they
obverse
not

not

express
a

fact at all.
'

The
one
'

of
a a

I like

straight
We
; but
'

road, is

am

not

who
I

do
not to

like

road.' straight road straight


at

might perhaps infer,


there is

do

dislike curved

clearlyno

reference

roads

all.

v.]
While

QUESTIONS

AND

ANSWERS. obversion in

43

accepting
obversion, I
the
the from
utmost

the
must

new

term

the

sense

ot
to

formal
use

add

that

students

have

begun
with

it with

confusing laxity,
To

the obverse

the

converse,

"c. contrapositive,

prevent

clature nomenlogical

fallinginto complete confusion,


choose convenient
as names

it

seems

to

be

indispensable to
relations of

for

the

simpler (p.32),

propositionalforms,
to

attempted

above

and

to

adhere

them

inflexibly.

17.

What

is conversion

by contraposition ?
'All have
student

Give

the
There

contrapositive of
nothing
as

birds found
to

are

bipeds.'
difficult in

is

which

so

teaching logic
remember

to

get
of
to

the

comprehend

and

this process

attention contraposition ; particular


the

is therefore

requested
a

above

question.
get its

Having taking
a

proposition
its

in

A,

we

contrapositive by
this
as

the

negative of
take

predicate,and
the

affirmingof
and subject,

subject the negative of


are

original subject. Thus,


as a new

if 'all
affirm
'

Xs

Fs,'
that
are or

we

all not-Fs all

of them
notwe

they are

not-^Ts, getting the proposition all


is

Fs do

not-JTs,' which
not

either

or

E, according
the

as

do

to join the negative particle

predicateX.
'

Accordingly
are

the

of contrapositive
'

the

proposition
are

all birds

bipeds
It is
one

'

will be

all that

are

not

bipeds
the

not

birds.'

thing to
be
De

obtain

the

another contrapositive, from


hold
no

thing
late

to

see

that it may

inferred used
and
to

premise.
act

The

Professor

Morgan
one,

that the

of

inference
process

is

self-evident
be certainly
'

needs

analysis;
we
'

but

the

may

analysed.
and

Thus

may
are

obvert
not

the

mise pre-

All

Xs

are

Fs,' obtaining No-Jfs


then
or

Fs,' which
into
'

is

proposition in E,
are

convert

simply
not-Fs of the
are

No

not-Fs

Xs,' also

in

E,

else

'All

The

contrapositive, then,

is the

converse

obverse.

44

IMMEDIATE

INFERENCE.

[CHAP.
the

We

may

also

prove

the
not-

truth
Y
must

of be

contrapositive
X
or

indirectly ; for
but if it be
X

what it is

is

either

not-X
same

by

the
same

premise
time

also

Y,

so

that the

thing would impossible.


other

be

at

the

notmust

Kand

also

Y, which

is

It follows

that

we

affirm XXI.

of not-F below

the
; also

alternative,not-^f.

(See Chapter
; first ed.

of Science,pp. 83-4 Principles


18.

vol. i. pp.

97-8.)
of

Give

the

converse
'

of All

the

contrapositive
substances

the

proposition

vegetable

are

organic.'
As 'All

learnt

from

the

last

question,
are

the

contrapositiveis
substances.'

not-organic substances
may take
are

not

vegetable
to
'

We

this

to

be

equivalent
substances

No

stances inorganic sub-

vegetable substances,' (E), the


is
'

simple

converse

of

which

No

vegetable
of
the

are

inorganic
we

stances,' subthe

the obverse

premise.
affirmative
are

But, if

treat

contrapositiveas
'

universal

proposition,thus,
Some

All

inorganic
convert
are

substances

non-vegetable substances,'
'

we

must

getting by limitation,
which inorganic,'
cannot

non-vegetable
of

substances

is

the

subaltern

the
us

obverse, and
back
seen

by

any

process

of inference

lead is

to to

the be
a

original.

Conversion which

by

limitation

easily
a

faulty process
afterwards term, the

always
this kind

occasions

loss

of

logicalforce.
As
we

shall
a new

observe,

of

conversion

introduces

namely

the

indeterminate

adjective
to not

'some,'
terms

so

that the

inference

is

not

reallyconfined
we

the be

of
to

original premise.
the

Although
shall

may
to

able

with dispense entirely

word, owing

its

ment employit

in
from

ordinarydiscourse, we
formal

ultimatelyeliminate
to

pure

logic,and

relegateit

the

branch

of

numerical

logic.

v.]
ig.

QUESTIONS
Take
the air

AND

ANSWERS.

45

following proposition,
'

'all

water

contains

convert

it
an

by

contraposition :
affirmative
position, pro-

change

the
and

result
convert

into

To has

show

the
been

need

of

more even answers

careful

logicaltraining than
great Universities, I
I

hitherto
a

common,

in the which

give
above

few

specimens
The

of

received

to

the

question.

contrapositiveof

the

was proposition

variouslystated, as
All air does
All All
not

contain

all water.

air is

not

contained
not
a

in

water.

not-air

is

thing contained
in water.
water.

by

not-water.

Some
Some All

air is

not

contained
no

not-air not-air

contains

contains drew
to

water.

The

who logicians

these

inferences
results
as

then the

proceeded
"

by simple
Some No No

conversion is

get such
without
not
no some

following:

water water water

not

some

air. air.

contains contains student


'

air. that
'

One
is
a

too

clever of water,
'

inferred he of

All

or

every

vacuum

void

which void

converted, simply indeed, into


water

Every
in

is

'

vacuum

An there

examiner is
a

logic is
in the sufficient
of

sometimes brains
of
an

forced

to

believe
;

that the
more

void any

examinee

but is

absence often any the


case

of

training in logicalwork
results shown
agree process.

cause

the

lamentable

above.

In

it

seems a

impossible to
self-evident

with De

Morgan

that

is contraposition

46
These observe
two

IMMEDIATE

INFERENCE.

[CHAP.
due
to

absurd

answers

are

mainly
'All the

the

failure

to

that in the
'

proposition
air,'form
the then
'All is
'

water

contains

air,'the

words

contains both

grammatical
and

predicate,
the

comprehending
copula.

logical predicate
the
water

logical
water

Logically

proposition
is what does

is

'All

is
The
not

containing air,'or
then contrapositive water.'
'

contains
not

air.'

All

that

contain the

air is

Uniting
is what

the

negative particle to
contain air.'

predicate
'

water,' and

converting by limitation,we
does
not

obtain

Some

not-water

20.

Describe each other


"

the
the

if logical relations,

any,

between
and

of

following propositions

each

(1)

All

organic
are

substances

contain

carbon. which
do

(2) There
not

no

inorganic
carbon.

substances

contain

(3)

Some

inorganic

substances

do

not

contain

carbon.

(4) Some

substances

not

containing

carbon

are

organic.
Of
of

these, (i) is
is
'

universal

the contrapositive affirmative,


not

which

All

substances Hence
'

containing carbon
converse

are

ganic inorof this

substances.'

the

by

limitation
are

is, contrapositive
not

Some

inorganic substances
of 'All

substances

containing carbon,' equivalent to (3). Proposition (2)is


the obverse is the
must

inorganic substances
of

contain
To
no

carbon,'which
obtain

contradictoryof (3).
the contrary

(4) we
substances

take contain

(i),that is,
it in the substances

organic

carbon,
substances
convert

express
are

affirmative
not

form,

'All

organic
then

containing carbon,' and

it by limitation.

v.]
21.

QUESTIONS
Take any
convert

AND

ANSWERS.

47

proposition
it

suitable

for

the
convert

pose, pur-

by contraposition,
the
and

it
an

again

simpliciter,change proposition,
the

result show

into

affirmative
may
The be
a

that

you

regain
suitable

originalproposition.
of

[C.]
will

most

kind

proposition for
as

the purpose

universal

such affirmative,

(1) All
The

birds

are

bipeds.
be

contrapositive may

stated

in the form

of

E.

(2)
Which

No

not-bipeds are

birds. the obverse of

is converted

into E, simpliciter
are

(i).

(3)
When

No

birds
into

not-bipeds.
the

thrown the All

affirmative

form

by

second

obversion,

last becomes

(4)
As

birds

are

not-not

bipeds.
is

double

this negation destroys itself, obverse of

equal

to

(i).

Notice

that the

the

obverse

is the

original.
of

22.

Give

the

converse
'

of the
are no

contradictory
coins which

the
not

proposition,
made
The

There

are

of

metal.'
is stated
'

premise
it
means

in

complex
are are

form
made of

with of

double

negation ;
which The

No

coins
'All

not

metal,'

is the obverse

of

coins in

made
square

metal'

(A).

contradictory,as
is
a

shown in
can

the

of

opposition
coins
are

(p. 31)
not

proposition
metal,' which

O, namely,
be

'Some

made that

of

converted

only by
whence

tion, nega-

is,by joiningthe negative particle to the predicate,


Some

thus

'

coins

are

not-made-of-metal,'

by
are

simple
coins
'

conversion the
answer

'Some

things

not-made-of-metal

required.

48
23.

IMMEDIATE

INFERENCE.

[CHAP.

(i) (2) (3) (4)


(5) (6) (7)

All

crystals are
solids
are

solid.
not

Some Some No Some Some All the

crystals.
not

not-crystals are crystals are


solids
are

solids.

not-solids.

crystals.
are

not-solids solids
are

not

crystals.

crystals.
any,

Assign

logical relation,if propositions and


is
a

between

each

of these

the

first of them.

Proposition (i)
obverse is

universal

affirmative limitation
In

(A)
is

; its

simple
sub-

(4);

its

converse

by
is

(5);
to
'

the

contrary
we

of this take

converse

(2).

order

obtain All

(6)
not-

must
are

the

contrapositiveof (i), namely,


subaltern
we

solids

not

the crystals,'

of

which

is

(6) ;

and

converting (6) by negation


inverse, but is
that
not

get

(3^. Again, (7)


We may

is the say

inferrible from
from

(i). (6)
with

further

(4) can
logical
not

be force

inferred
to

(i),and

is
can

exactly equivalent
be
cannot

in
are

it ;
to

(5) and

inferred,but
be

equivalent
is
not

the

original ; (2)

inferred

from

(i),but
24.
we

inconsistent

its truth.

What

information
from

about

the
'

term

not-A
s are

can

derive

the

premise

All

Bs

'

This
not

though apparently a question,


of
a

very
;

simple
the

one,

does in
a

admit

very

simple

answer

it is said
on

important
one

theoretical that
as

point of

view.

It may affirms

be

hand,

the

proposition only
us

of all As

that

they
from
not

are

"5, this tells


class A. Thus
any

nothing
what

about

things
may the

excluded
or

the be

is not-^4 from

be

B,

it may This

B, without
true.

interference Not-A other

premise.
may infer the

is

quite
'

About
on

we universally

nothing.
all

But,

the

hand, if we

convert

proposition

So

IMMEDIATE

INFERENCE.

[CHAP.

This
more

question differs
involved form.
'

from The

the

last

only

in

being put
more

in

premise
are

when

simply
contra-

stated becomes

All

not

solids

not

the crystals,' limited

positiveof gives
'

'

All

are crystals are

solids,'and

conversion

Some

solids

crystals.'
non

27.

Nihil

potest placere, quod

decet.

vert Contraposition con-

this

proposition, (i) simply, (2) by


; and

show

by

what the

logical processes contrapositive to

we

can

pass

back

from

the
This

original.
xi.

[c.]
65) equals, Nihil
is unbecoming
it
can

c. premise (from Quinctilian, non can

quod
be

decet, potest placere; nothing which

please. Being simply


to

universal

negative,E,
can

converted

into

'Nothing

which

please

is

unbecoming.'
In

order

we apply contraposition,

must

put the premise

into the

form

of

A,

thus

'

All

unbecoming
which

things are
is 'All
not

the contrapositive of unpleasing things,'

unpleasingthingsare
a are

not

which unbecoming things,'


term

having

double

negative in
We

each
can

equals
the

'

All

pleasing things

becoming.'

regain

originalpremise by

to applying contraposition

this last result.

28.

Convert,
from

and

give

some
'

immediate

inferences that

the

following:
for
this
a

Nothing

is harmless

is mistaken
The
and
'

virtue.'

predicate of
that

proposition is clearly 'harmless,'


for
a

is mistaken

virtue

'

is

relative
then
'

clause

describingthe subject.
mistaken into
for
a

The

proposition is
'

Nothing simply
is mistaken

virtue

is

harmless/ (E), Nothing

converted harmless

another
for
a

proposition in E,
virtue.' obversion
to

Applying

the

originalproposition we

get

v.]
'

QUESTIONS
that is mistaken
for
a

AND

ANSWERS.

51
'

All

virtue

is

not-harmless,' or

is

harmful.'
we

By immediate
conduct.'
see

inference

by complex conception,
for

infer 'All

foolish conduct

mistaken

virtue

is harmful
ception, con-

foolish

(Concerning

inference

by complex

Thomson's

Outline," 88, and Elementary

Lessons

p.

87.)
29.
we

Because infer
man

every

Prime

Minister

is

man,

can

that
?
of

every

good

Prime

Minister

is

good
The

process
as

immediate

inference

by
allows of

added
us an same

minants, deter-

described

by

Dr.

Thomson,
to

to

join an

adjectiveor determining mark


judgment, narrowing both
Of
course,

both but

terms to

affirmative
extent.

terms,
be the

the

however, it must
and
same a

same

determining ambiguous
that
very he

mark
not

in each

case,

if

an

adjective be
Minister

it is

the logically

adjective in
Prime

its several
means

meanings.
is
an

Now

good applied to
who
one

able,
from
man

active, upright minister, but


men means are

probably
ranks of

different
A

good
who

in

other in

life.

good

is

good

the

ordinary

business is
erroneous.

and

domestic

relations

of life.

Thus

the inference

(See Elementary Lessons, p. 86.)


that of

It will afterwards
no

be shown

when

the
can

propositionis fullyexpressed
occur.

such

failure is

inference

Strictly speaking the premise


=

Prime and it follows

Minister

Prime

Minister, Man

that inevitably Minister


=

Good, Prime
30.

Good,
two

Prime

Minister,Man.
circles Can
of
to

Euler

employed
a

overlapping
the

represent
raise any ?

particular proposition.
to

you
a

objection

accuracy

such

diagram

52

IMMEDIATE

INFERENCE.

[CHAP.
in
a

Such

circles

have
In

been
my

employed
Elementary
metals

great

number

of

logical works. represented by

Lessons
are

(p. 75)
not

the

proposition particular
the

'some

brittle,"is

: followingfigure

FIG.

i.

It

does

not
a

seem

to

have

been

noticed sufficiently the exclusion

that

though

such

diagram correctlyshows
metals from
any

of

part of the

class

part, that
at

is all parts,
same

ot

the class brittle


another

substance, it indicates
class metals

the

time

that

part

of the
Thus

is included
to

among the
two

brittle
positions prothe

substances.

the

diagram corresponds
of

I and other. with


not

O, instead
it has E been
;
so

showing

either apart from O

Now,

fully explained that


that when
that
no
'

is consistent
metals
are

the truth of it brittle,'

we

say
are

some

may
'

be
some

metals
are

which brittle, The

is

contradictoryto I,
should
best
to not

metals

brittle.' it would

diagram
be

prejudice this question,and


the part of the circle the circle in of brittle

therefore
metals

remove

bounding

which
to

falls within
a

substances, or

else

have

broken

line,as

Fig. 2.

FIG.

QUESTIONS
In

AND

ANSWERS.

53
'

the

same

way

the

proposition I, for instance,


be

Some

crystals are
included either in

opaque,' would
within
a or

represented by

broken

circle shown

complete

circle,in

the

manner

Fig. 3

Fig. 4.

FIG.

3.

FIG.

31.

What
sentence
1

is

the

logical
will need

force

of

the

following
of
Ethics
:

from

Sidgwick's
not

Methods be
a

materialist
determinist

naturally
be
a

determinist
'

materialist
force
are
'

Taking
second be
a

'

naturally
becomes

'

to

give
'

universal

to

the

first

proposition, it
The
not

All

materialists
us

determinists.'
need

proposition informs
materialist,' that
are

that

determinist
the

is to

say, This the

at

least, 'some
the

determinists

not

materialists.'
converse

proposition is
is the The

sub-contrary of
of
'

the

of
are

first,and

tradictory con-

all determinists
us

materialists.'
from

second

proposition,then, prevents
and that among

supposing
terms.

materialists
We learn

determinists
there
are

to

be

two

co-extensive materialists
some

persons
;

called
hence

who

are

all found
are

determinists
among among

called

determinists

found
not

materialists

; other
as

determinists, however,
to

are

and materialists,

those The

who

are

not

minists, deter-

they
would the

cannot

be

materialists.
as

first

proposition
second
as

be

described technically the inverse

A,
the

and

the

contradictoryof

of

first.

54
'

IMMEDIATE

INFERENCE.

[CHAP.
are

32.

All
we

equilateraltriangles
thence infer have that

equiangular.
and vice

'

May

triangles having

unequal angles
versa

unequal sides,

? that all
may

The

asserts proposition

sided triangles have equalthat

equal angles; triangleswhich


sides.
But
as

hence
have the

we

by contraposition infer
cannot
are

not

equal angles
we

have
not

equal

propositionstands,
sides have

in justified

(see p. 32), and reading it reciprocally


which
true
as

that triangles inferring

have
a

not

equal
of

not

equal angles. This


it is the the
contra'

is

matter

geometrical science,but proposition, namely,


the equilateral,'

positive of
be

another

inverse of

all

equiangular trianglesare
must

truth

which

separatelyproved.
Can form "s
are

33.

we

ever are

convert

proposition
one

of

the
'all

'all As ^s'?
cannot
a

Us'

into

of

the

form

Certainly we
As
are

infer

that

all Us

are

As

because of the
so

all
con-

j?s.
B

As

general rule the


a

predicate
the

vertend the

will be could

wider
not

term

than

subject A,
Professor

that

inverse

be

inferred.

Henrici,
for

{Elementary Geometry, Congruent


describes
convert

Figures, p.

14),
we

stance, in-

space

as

but three-way-spread,

cannot

simply,and

say that every


not

three-way-spread is
that the
are

space.

It nevertheless

happens
be

uncommonly
to
mean

original
all

proposition is reallyintended
which defined every
are
can as a

'all As Thus
can

Us,'
be into

then

simply

converted.

if space
convert

three-way-spread ofpoints,we
space. of

three-way-spread of points is
the form U and (chapter xviii.),

Such

definitions

of

proposition afterwards
considerable

described
care

by

the

symbol

is

requisite

v.]
in

QUESTIONS

AND

ANSWERS.

55

discriminating
S. Mill
has

between
to

the the

propositions simple
as a

and
of

U.
a

J.

pointed

conversion
common

universal
of It
error

affirmative

proposition
book

very

form

(System of Logic,
be
too

v.,

chapter
the
p.

vi.,section reciprocal
32,
cannot

2).
and be

cannot

often
as

repeated
described
the form

that
on

inverse

propositions
from
an

inferred

original of

A.

34.

In

what
?

cases

does

predication
that
in
as

involve
processes

real
of of

existence conversion

Show

some

assumptions
in
nature

to

the
made

existence
; and

classes

have

to

be

trate illus-

by examining
and if
so

whether

any
are

such

tions, assumpin
some

what,
all
.S

involved

the

inference not-6"

that
P.

if

is

P,

therefore

is not

The

above

question
The
to

must

have

been of

asked

under

some

misapprehension. nothing
whatever the

inferences with
of real

formal
;

logic

have

do

existence and
to

that No

is, occurrence doubt,


tradiction, con-

under
if all S is

conditions

time
order

space.

P, it follows
some

that, in
must

avoid
to

logical
not

not-S
'

be

admitted
are

be

P.

For

instance,
more

if

All

heathen
years

gods
old/

described that
'

in Some in
no

writings things writings


assertion
ever

than
are

1,000
not

it follows
are

which
more

heathen
years
nor

gods
old.' could

not

described involves
an

than real

1,000

This such

of

existence,

inference

be

drawn,
existence
a

unless, indeed, the


in time and

original proposition
This

itself asserted in

space.

subject is pursued

subsequent

chapter.

CHAPTER

VI.

EXERCISES

ON

PROPOSITIONS

AND

IMMEDIATE

INFERENCE.

i.

Examine

the contain

following
consistent
be
true

pairs

of

propositions,
such be
may
true

and that
; true.

decide if the vice Give between

which
first

pairs
of if the the

propositions,
second the
may first

pair
second
name

the true,
the

and

versd,
the the

be of of

be
if

technical
two

logical relation, pair.


are

any,

propositions
( Some
/ \

each

metals
metals
are

useful.

All

useful. useless.

f No

metals useful useless useful metals


metals

are

(2)

"

( Some

things

are

not

metals.

( Some
/ \

things
are

are

metals.

I All

things
are

metals.

(4)

Some No

useful. useless.

are

( All

metals useless

are

useful.

I Some
Draw all
'

things

are

not

metals.

2.

the

immediate is
of

inferences

you

can

from

the

proposition
3.

Knowledge
converse

power.'
the

Give
'All

the

contrapositive
contain
of

of

the

position pro-

organic
the

substances

carbon.' intuta quiz

4.
'

Give
are

all all

logical

opposites

indecora,

Unsafe

things unbecoming.'

58

IMMEDIATE

INFERENCE.

[CHAP.
with the
utmost

(3) (4)
10.

All

trespassers

will

be

prosecuted

rigour of
Nemo
me

the law. lacessit.

impune
obverse,

Give
of
All

the the

converse,

inverse,and
"

reciprocal

of

each

followingpropositions:
are

(1) (2) (3)


(4) (5)
ir.

mammalia

vertebrate

animals.

Sir Rowland That


a

Hill is
a

is dead. in
an

which
statesman.

merit

author

is often

fault in

Whatever In

is necessary victoria.

exists.

veritate

Give

the contrary,

subaltern, contradictory,

converse,

and obverse, inverse, contrapositive,

reciprocalpropositions
"

corresponding

to

each of the

of the followingpropositions:

(1) All (2) All (4) The (5) (6) (7)


None He
a

B.A.'s three
men

University of

London

have

passed

examinations.
are

sometimes

thoughtless.
which
wears a crown.

(3) Uneasy

lies the head

whole

is greater than solid bodies has been


are

any

of its parts.

but who
rope.

crystals. by
a

bitten

serpent

is afraid

of

He

who before
be

tries

to

say

that

which
say

has

never

been

said
never

him, will probably


after him.

that which

will

repeated
as

12.

Give
for the

many

equivalent logical expressions as


"

you

can

propositions
treasury
blame.
was

(i)

If

the
to

not

full,the

tax-gathererswere

vi.]

EXERCISES.

59
not

(2) Through (3)

any

three be

points
that

in

straightline

circle may
It

described.

is false life.

to

say

only

the

virtuous

prosper

in

[R.]
are

13.

What

relations logical
"

there

between

the

following

? propositions

(1) All elementary (2) There


(3)
Some No
are no

substances

are

undecomposable.
which
are

compounds

not

posable. decom-

compounds

are

not

decomposable.
are

(4)

undecomposable

substances

compounds.

w
14.

From
'

the
can we

proposition
infer that

'Perfect
'

happiness

is

possible imis

Imperfect
the

happiness

possible
15.
Is

'

it the
'

same

thing to
are
'

affirm

of the falsity
to

position pro-

Some

birds

and predatory,'

affirm
'

the truth
?

of
1

the 6.

proposition Explain
follow the

Some

birds

are

not

predatory
case

statement

that

in the

of

subcontrary
falseness

truth propositions,
cannot

may truth.

follow

from

but falseness,

from

17.

Give

in succession

(i) the obverse, (2) the


contrary,
the

converse,

(3) (6)
are

the
the

subaltern, (4) the

(5)

the

contradictory,
'All

contrapositiveof
acts.' the
same

proposition

wise

acts

honest
1

8.

Concerning
"

propositionanswer
related

the

following

questions : (1)
How How

is its

converse converse

to

its subaltern?
to

(2)
(3)

is its subaltern

related

the

converse

of

its

How

is its subaltern

related

to

its

contradictory?

[BAGOT.]

60

IMMEDIATE

INFERENCE.

[CHAP.
contrary
of the
tradictory con'

19.

What
of

is the the

converse

of
'

the

proposition
to

Some

crystalsare
of
'

cubes

How
20.

is it related What
feet

the

original proposition?
of the
converse

is the

converse

No

men

are

ten

high
the

'

21.

Name of
:
"

logical process

by

which
to

we

pass

from

each
one

the

following propositions
All No No All All

the

succeeding

(r) (2)
(3) (4)

metals
metals

are are

elements. non-elements.
are are

non-elements

metals.
not

non-elements metals
are

metals.

(5)
(6)

elements.
are are

Some

elements metals
but
a

metals.

(7) Some
22.

elements. is
a a

(i)

'None

logical author
as

trulyscientific
examine
can

author.' the be

Taking
from

this

proposition

premise,
which

and following propositions, inferred the

decide

of them

premise.
author is
no

(2) A

truly scientific logical. truly


are

author

who

is

not

(3) Some
who

scientific

authors

are

not

any

authors

not

logical.
author is
not
a

(4) A

not

trulyscientific
who
are

logicalauthor.
authors
cannot

(5) Those
be

not

truly scientific trulyscientific.


is
are an

logical.
are

authors (6) All logical

(7)
(9)

No

trulyscientific
not

author

author. illogical

(8) All
No
No

authors illogical is
a

trulyscientific.
author. is
not
a

author illogical
one

trulyscientific
author

(10) (n)

is

trulyscientific

who

logical

author, Some

logicalauthors

are

not

trulyscientific

authors.

vi.] Give,
relation other.
'

EXERCISES.

61

as

far

as

possible,the
each of the

technical above

name

of

the

logical
each

between

propositions and

23.

Some

small

sects

are

said

to

have

no

discreditable extrude
out

members,
who

because
to

they

do

not

receive

such, and

all

begin

verge

upon

the

character.'

Point

how

this statement
24.

illustrates
we

logicalconversion.
that them because of because ?
every
a

Can

logicallyinfer
cold
contracts

heat

expands

bodies, therefore
25.
Does

it follow
therefore
a

that

city city
of
a

contains

cathedral,
creation of
the 26. All

the

creation the

involves cathedral

the volves in-

cathedral, or
of
a

creation

creation

city?
are

English
Does

Dukes

members

of

the

House

of

Lords.

it follow that the

by
of

immediate of
an

inference

by complex
Duke
?

conception
creation
of

creation

English
of

is the

member

the House

Lords the

27.

Give
"

every

possible converse

of

following

positions pro-

(1) (2)
(3)
28.

Two All

lines straight trade-winds students the

cannot
on

enclose
heat.

space.

depend
do
not

Some Give

fail in

anything.
converse

[M.]
and
"

logical opposites,

contra-

of positive, If
a

Euclid's

twelfth (so-called)
meet
on

axiom

line straight

two
same

straight lines,so
side of it taken

as

to

make

the interior
than
two

angles

the

together less
which

rightangles,those
shall
at

straightlines being continually


upon
two

produced
the

length meet
are

that

side

on

are

angles which
29. How

less than

right angles.
to

is the above

propositionrelated
two

this other

"

If

straightline

fall upon

parallelstraightlines, it

62 makes

IMMEDIATE

INFERENCE.

[CHAP.
same

the
to two

two

interior

angles

upon

the

side together

equal
30.

rightangles ?
'Some

[R.]
of

From

members

Parliament
No. 3

are

all

the
we

ministers' infer
members that

(Elementary Lessons,
'some

p. 325,

can [4]),

place-seeking prejudiced
are
'

and

incapable

of

Parliament ministers

all the

prejudiced place-seeking
that
true
are

and

incapable
Is it

? argue because
two

31.

perfectlylogical to
both

sub-

contrary

propositionsmay
both it
be

be

at

the

same

time}
to

thereforetheir contradictories, which


other, may
32. Is

contrary

each

false ?
argue

perfectlylogical to
both sub-contraries is both

thus?

"

If

contrary

propositions are
which Now the
as are

false,their respectivecontradictories,
to

each

other,

are

both

true.

this result may is


the

possible, it
be false.

is therefore

possible

that

contraries What

33.
two

logical relation, if
xi.
a

any, 'A

between
false

the

assertions

in
to

Proverbs, chap.
the Lord
:

i,

balance
is

is abomination

but

just weight chap.

his

delight
34. and

'

Examine

the

verses

of

Proverbs,
the
two

x.

to

xv.

assign the
make What
'

relation

between the
verses.

opposed

assertions

which 35. madness

nearly all
is the
to
'

nature

of the is
a

step from
'

'anger is
?

short

madness
at

long passion
base

[R.]
triangle are proposition by
any

36.

'

The

angles
can

the be

of

an

isosceles this

equal.'
appeal
37.

What

inferred
and

from

obversion,
to

conversion,
the
can

without contraposition,

geometrical proof?
assertion
we
'

From what

The

improbable

is

not

sible,' impos(i)
the

learn, if anything, about

possible, (2) the

probable, (3) the

not-improbable, (4) the

impossible, (5) the not-impossible?

vi.]

EXERCISES.

63

38.
the

How

would

logician
of four

express

the

relations

between

following

statements

interlocutors

(1) (2)
(3) (4)

None

but

traitors all traitors.

would

do

so

base

deed.

And

not

Some

would.

No

not

even

traitors.
Science

[College

Moral

Examination,

Cambridge.]

39.

What

difficulties

or

absurdities
?

do

you

meet

in

verting con-

the

following

propositions
dictionaries.

(1) (2)

Some
No

books

are

triangle
other
two.

has

one

side

equal

to

the

sum

of

the

(3)

Every
A

one

is

the

best

judge
scientific

of

his

own

interests. and

(4)

few
of

men

are

both

discoverers

men

business.

(5)
(6)

Whatever

is, is
men are

right.
wise in their
own

Some

conceit.

(7)

"

The

eye

sees

not

itself, by
some

But

by

reflection,

other

things."

CHAPTER

VII.

DEFINITION

AND

DIVISION.

i.

ALMOST

all of notions

text-books the
correctness

of

Deductive
of

Logic definitions,
and of
me

give
and

rules
for

for

judging
up

dividing

into
to

subaltern
treat

genera these home


of In

species. logic
very

On the

attempting,
manner

however,
this
are

parts
to

in

of

work,

it has the

come

strongly
and

that involve

they
the
to

beyond
matter

sphere

Formal

Logic,
is

of

thought.
;
one

form

there
same

nothing

peculiar
may

definition
to

in

fact

the and

very
a

proposition
to

be is
3

definition
to
us

person
to

theorem the
but

another.
9
as

It
9
at

open
x

for
8 of
as

instance
i

define
2,
as

number

3;

or,
one

; or,

7 +

"c.;
a

having

selected the other the the

will

any

these

equations
The

definition,
in

equations theory
fact

follow

theorems. lines is
of
some

perplexity partly
some

which
from

of

parallel
there
one

involved

arises

that

is

choice and

definitions,
the other.
may

mathematicians is

choosing
too, that
to

way
same

It

quite apparent,
different ledge knowHerschel
serve as a

the

proposition people.
of William For

afford

different

instance,
Herschel'

'John
would

was

the

only
of

son

definition William

John

Herschel and
To

to

any

one

who
to

knew
one

only
who be the
a

Herschel,
knew

of
one

William who knew be

Herschel

only
theorem.

John.
Similar between

both made

it

might

remarks

might
and

concerning

distinction

ampliative

explicative propositions.

66

DEFINITION

AND

DIVISION.

[CHAP.

5. Criticise the

followingdefinitions
is
a

"

(1) A (2) A (3) La (4) (5) (6)


A

square
are

four-sided the

figure of
is

which

the

sides

all

equal and
of has

angles all rightangles.


system

member

the solar continued

anything over

which

the vie

sun

influence.
la mort.

est

le contraire

de

lemma

is

proposition which
the is
one

is

only

used

as

subservient An archdeacon functions. Life is the

to

proof
who

of another exercises

proposition.
archidiaconal

definite both

combination

of and

heterogeneous successive, in
and

changes,

simultaneous with
external

correspondence
sequences.

coexistences

(7)

gentleman
subsistence. bodies

is

man

having
whereof

no

visible

means

of

[ORTON.]
are

(8) Equal
the

those

every

one

can

fill

place of
the
a

every

other.

[HOBBES.]

6.

Examine is

definitions

"

(1) Tin (2)


Vice

metal

than lighter

gold.
of

is the is
a a

opposite of
substance
caseous

virtue.
rags.

(3) Paper

made

(4) Cheese
(5)
Rust

is

preparation of
of

milk. iron. which

is the red

desquamation
function

old

(6) A (7) A

transcendental
not
an

is any

function

is

algebraicfunction.
a

right-angledtriangleis right angle, and


are

triangle containing
the

one

of

which

containing plant

sides

or

are

not

equal.
part of
an

(8) An (9) A

organ

is any
to
a

animal

or

priated appro-

distinct function.

man

is

self-knowing animal.

vii.]

EXERCISES. An A

67

(10)

animal

is

a a

sentient

organised being. figurehaving


be the its

(n)

triangle is
is

three-sided
to two

angles
of

togetherequal

rightangles.
may

(12) A

man

one

who

Prince

Transylvania.
7. of In what

[HOBBES.]
the
or definitions, following some

respects
?

are

them, defective

(1) Logic (2) Logic

is

guide
art

to

correct

reasoning.
in
correct

is the

of

expressing thoughts

language.

(3) Logic
(4) Logic

is

mental

science.
of

is the science

the

regulativelaws

of human

thought.
8.

Does

the

eleventh
a

chapter

of

the

Hebrews,
of

or

any ?

contain part of it,


9.

correct

definition logical

Faith

Give
may

examples
be

of indefinable

words,

and

explain why

words
10.

indefinable. the Proximate Genera


for

Give
"

the

following

species
Man

Plant Hound and and


an

Monarchy
Science. the

Triangle
11.

Define
name a

by

genus

differentia accident

followingterms;
case
:
"

and

proprium
Island
Bank

in each

Parallelogram
Bill of
Tree.

exchange

Dictionary
1 2

What
.

accident
13.

of

difference, species, g'enus, property, and Examination 1 [D.]


are

the

attribute, Distinguishspecific property,


term

and

accident,

using the

Circle

as

an

example.
F 2

[B.]

68

DEFINITION

AND

DIVISION.

[CHAP.

14.

Is

it

possible

to

define

the

terms

gold, coal, legal


?
out

-nuisance, civilization, Cleopatra'sneedle


15. of the
Define

the

term

boat, and
the

then

point

how
:
"

many

following things

definition

includes

Bark,

fire-engine, ferry-boat, floating pontoon,


1

wherry,
one

canoe.

6.

the following objects under Classify


"

or

other

of

the

: specie heads, cash, bills,

Cheque,

promissory

note,

shilling,money,

token-coin, note. bank-

bank-note, I.O.U., paper-money,

sovereign, Scotch

(See Money
of

and

the

Mechanism

of Exchange:
Section
on

national Inter-

Series,p. 248. Scientific

the

Definition

Money.)
17.

Distinguish Logical

from

Physical

Division

and

Definition.
1

[o.]
anything
admit of
more

8. ?

Can

than

one

tion defini-

[o.]
between Distinguish precisely
a

19.

the definition

and

the

descriptionof
20.

class. the difficulties which arise

Explain
of

concerning
criticise

the the

definition

parallel straight lines, and


definitions
in every
:
"

following suggested

(1) Lines
(2) (3)
Lines Lines

which

are

part equidistant.

of similar direction. which

being
never

in the
meet.

same

plane

and

indefinitely

prolonged
21.

Examine

the
Man

definitions following
is is
a

"

(1) (2)

bundle

of habits.
sense.

Law

common

VIL]

EXERCISES.

69
which

(3)

Reverence

is

the

feeling
of

accompanies
worth in others.
on

the

or recognitionof superiority

(4) Hunger

is the

product
food.

man's

reflection

the

necessityof
22.

[p.]
logicaldivisions,and

Which
are

of
?
"

the

following are

which

not

(1)
(2)

Man The

into

civilised and into

uncivilised.

world Australia.

Asia, America,

Europe,

Africa,

(3) Grammar
(4)
War

into syntax

and

prosody. logicaland
and

into civil and

aggressive.
which
are

(5) Syllogisms into (6) Sequences


of

those

illogical.

phenomena

into casual visible. and

causal.

(7) Energy
(8)

into

potential and

Geometrical

figuresinto plane
is either natural

tri-dimensional.

(9) Allegiance
and
I.

and

perpetual,or

Jocal
Book

temporary.

(Blackstone,Commentaries,

Chap. 10.)
the
term

23.

Divide

Inference,

so

as

to

include

the

various

species usually discussed


The

by logicians.
classes
one or

[E.]
which the
were

24.

followingwere qualifiedto
vote

the in

of persons other of

in

1868 of

United

States man, male

America male

"

Male

citizen,male
freeman,

inhabitant,every
male person,
man.

white

white citizen,
male

white

adult, free white


a

free white citizen,-

Form

scheme
of

of

logical division
classes.

which

shall

have

place for
25.

each

the above

Divide

logically University, colour, chair, science,


"

man. species, art, church, undergraduate, virtue, statesreligion,

[o.J

70

DEFINITION

AND

DIVISION

[CH.

vn.

26.

Form

scheme

of

division

of

sciences

to

include

the

species"
rational,

Deductive,
abstract,

experimental, inductive,

concrete,

descriptive,

explanatory,

empirical.

27.

Apply

the

rules

of what

logical
is
wrong,

division and

to

the

following
what is

instances,
deficient
:

correcting
"

supplying

(1)

Discursive

thought
and

may

be

divided

into

the

Term,

Judgment

Syllogism. Singular,
and

(2)
(3)

Notions

are

Concrete,
are

Universal. and Universal.

Propositions

Affirmative,

Negative,

28.

To in

what

extent

are

the

rules

of

division,
the

usually

given adopted

logical
in the

treatises,
Natural

repudiated

by

classifications

Sciences?

[L.]
?

29.

When

is

division
?

inadequate
And when
not

When

indistinct

When

cross

division
?

arranged

according

to

proximate
Give

parts

[MORELL.]
scheme shall find of

30.

an

accurate

logical places:
"

division

in

which
Part

the

following
;

things
; Vox

Name; Noun,

of

Speech

Term

logica
;

Verb

Adjective

Syncategorematic

term

Word.

CHAPTER

VIII.

SYLLOGISM.

i.

MEDIATE of

Inference,

or

Syllogism,
offers
a

forms

the scope

principal
for useful of

part

Deductive
I

Logic,
in
the forms and and
most

and
first

wide brief

exercises.

give,
and

place,
I then
;

epitome
them

the

syllogistic
by
full of

rules

;
answer

exemplify
I

dantly abun-

question
largest
and
the
more

lastly,

supply
of been

chapters syllogistic published.


the tinction disand

the

varied which
has

collection
ever

questions
Some of

problems

perplexing
and
are

questions,

involving

of

formal

material
treated

falsity of
apart in

syllogisms
the

their

premises,

succeeding

chapter

(xii.).

RULES

OF

THE

SYLLOGISM.

(1) Every
These and the
terms

syllogism
are

has

three

and

only

three

terms.

called

the

major

term,

the

minor

term,

middle

term.

(2) Every

syllogism

contains

three

and

only

three

tions. proposi-

These

propositions
the minor

are

called
and be the

respectively
conclusion.
once

the

major

premise, (3)
(4)
was

premise,
term
must must

The No

middle
term

distributed
in the

at

least. which

be in

distributed

conclusion

not

distributed

one

of the premises. nothing


can

(5)

From

negative

premises

be

inferred.

72

SYLLOGISM.
"

[CHAP.
must
one

(6) If
;

one

premise
be

be
to

negative,the conclusion
prove
a

be negative

and

vice versa,
must

conclusion ?iegative

of

the

premises
From

negative.
rules

the

above

may

be

deduced be

two

subordinate
to state at

rules, which
once.

it will nevertheless

convenient

(7)
drawn.

From

two

particular premises
be

no

conclusion

can

be

(8) If

one

premise

particular, the

conclusion

must

be

particular.
FIGURES
S
=

OF
M
=

THE

SYLLOGISM.
term.
Third

minor

term.
First Figure.

middle
Figure.
-

major
Fourth

term. Figure.
-

Second

Figure.
-

pSe.
Minor Premise.

M S
-

--

P M

M M

M M S
-

P S P M

S S-P

S
P

Conclusion.

Q-P

MOODS
The the

OF
a

THE

SYLLOGISM. table
of

following is
of

compact
numerals

the valid
the

moods

'of

syllogism, the
group

showing
a

figuresin syllogism :"


AGO
2.

which

each

makes propositions AAI


AEE
2.4.

valid
All

AAA
I.

3.4.

1.3.

EAE
i. 2.

EAO
3. 4i.

EIO
2.

IAI
3. 43-4-

OAO
3-

MNEMONIC

VERSES.

Barbara,

Celarent, Darii, Ferioque,prioris ;


secundae
;

Cesare, Camestres, Festino, Baroko, Tertia, Darapti, Disamis, Bokardo, Ferison, habet
;

Datisi, Felapton, Quarta insuper addit,

Bramantip, Camenes, Dimaris,

Fesapo,

Fresison.

74

SYLLOGISM.

[CHAP.
;
out

3.

Examine
into
a

the

following argument
and

throw the

it

form, syllogistic
:
"

bring

figure

and

mood

It cannot

be

true

that

all
is

repression is
and

chievous, mis-

if government
is sometimes The universal the conclusion affirmative beneficial. is stated
'

repressive

yet

[B.]
of
a

in the form

denial
'

of

the

all

is repression
'

mischievous

; hence

contradictoryof this,or
is the
which the real

some

repression is
The in middle the
'

not term

chievous' misis

conclusion.
not
we

'government,'
In

does

appear

conclusion. mischievous 'beneficial.'


'

looking for
the
must
as

major term,
but

do

not

find
term

in We

premises,
assume,

only

its
we

opposite
are

then, that
to

intended

to

take

ficial' 'benethere

equivalent
a

'not-mischievous/
four
"

otherwise

would

be

of fallacy

terms.

To

be

brief, then, the

takes syllogism Some


All

this form

government

is is

not

mischievous.

government
some

repressive(orrepression).
is
not

Therefore,
valid

'

repression
in the third

mischievous.' mood

It is

syllogism

and figure,

OAO,

or

JBokardo.

4.
In

In

what

figures is
have All No
No

the

mood

AEE

valid

the first figurewe

M S

is P. is M.

S is P.

The which Process

negative
of the

conclusion in
Term.

distributes
the

the

major

term

P,

is undistributed

major premise

; hence

Illicit

Major

viii.]
In

QUESTIONS
second

AND

ANSWERS.

75

the

figurewe
All No No

have P S

is M. is M.
P.

Sis

The

major
and

term

is

now

properly distributed
term

in the

major
once,

premise,
in the

the

middle

being
the

also

distributed

minor

negative premise,
show of that the

syllogism
third in

is valid

in

Camestres. The reader


may

in

the and

figurewe
the
fourth

have

again
a

Illicit Process

Major,

figure

valid

syllogism Camenes.
What
rules in

5.

of

the

syllogism

are

broken

by
and

arguments
DIE?
The
answer

the

pseudo-moods,

OAE,

cannot

be

better
p. in

given
In

than

in the O

words A
E

of the the

Solly (Syliabus of Logic,

86).
the

the

mood

predicate
in the
term must

is

distributed

major
both

premise, and subject


that

subject in the minor


conclusion.
be

premise, and
Hence

and

cate predisome

it follows the conclusion

either
was

distributed

in

which
term

not

distributed
be

in the in
once

premises, or
either which
that
or

else the
We

middle cannot,

cannot

distributed
at

premise.
form the

therefore,
but

determine be
of

will take, fallacy be either


an

may

quite
major

certain
or

there

must
an

illicitprocess

minor,
the

else
O

undistributed both

middle.

Again,
are

in

mood in the

E,

subject and
no

predicate
is distributed that

distributed in the
be

conclusion, whereas

term

minor
an

premise,
middle
and

and

it therefore

follows It

there

must

illicit process
term

of the
cannot

minor.
be

is also in the

evident the minor

that

the

distributed
in

premise,
the

that
term

if
must

it is distributed be

major premise

major

undistributed, and

76

SYLLOGISM.

[CHAP.

consequently
middle 6.

there
or

must

be

fallacy either

of

buted undistri-

illicitmajor.

None Germans

but

whites
were

are

civilised ; the
:

ancient
were

whites

therefore

they

civilised.
This
appears
at

[w.]
first

sight to

be

in Barbara,

the

terms

standing apparently in major premise


it does that
none

the order
assert

of the first
that
are

figure.
are

But

the

not

all whites
so, and

civilised ;

only
to

asserts

but

whites
the
are

this is

lent equiva-

the

of contrapositive
All

proposition
whites.

civilised

Joining to

this the minor The ancient

premise
Germans is in the
that
were

whites,
with figure,
term two

we

see

that

the

argument
so

second middle

affirmative in Middle.
two

premises,
both There

the

is

tributed undis-

cases,

producing Fallacy
a

of Undistributed
between
an

is also

difference

of

tense

the
;

premises
this

which

might perhaps
not

invalidate

argument

but

point need
None Gauls
but
were

be

further

noticed

here.

7.

civilised Whites

people
:

are

Whites

the
were

therefore

they

civilised.
At

[w.]
seems

first

sight this

to

be

in

the

second

and figure,

invalid
as

; but

convertingthe major premise by contraposition, example,


are we

in the last
'

get

valid

syllogismin
were

Barbara

"

thus, All whites


8.

civilised ; the Gauls of

whites, "c.'
to
error

All
and

books

literature all of
man's

are

subject

they
of

are

invention
are

; hence
error.

all

things

man's

invention

subject to

[H.]

VIIL]
This
may

QUESTIONS
seem

AND

ANSWERS.

77

at

the

first

reading
is of the A A

to

be

correct

ing, reason-

especiallyas
there is
is in the

the

conclusion

materially true;
Minor
of the man's Term.

but The

fallacyof Illicit Process

argument
and
are

pseudo-mood
should be 'some

third

figure,

the conclusion

thingsof

invention

subject
9.

to

error.' is content
covetous
man

He rich he

who
;
a

with

what

he

has

is

truly
what

is not
man,

content

with
is

has

no

covetous

therefore,

truly

rich.
The

middle
term

term

is
as

'

content

with
of

what the

he

has/ and

since and

this

appears the

subject
E

major
is in the

premise
first of

predicate of
the

minor, the argument


A E.
the

figurein
the

pseudo-mood
Term,
the

There
.

is Illicit Process
E
not.

Major

because

conclusion does

distributes

its

predicate and
The from

major premise A
may We be
true

conclusion the

in

matter

but

does

not

follow

premises. they
but
have.'

could
'

only make
All

the argument
are

good
content

by taking as
with
what

major premise,
This

the

trulyrich
valid if

would

give a
with

syllogism in only
have.'
to

Camestres,

the

original premise,
are

converted,

yields 'Some
10.

trulyrich

content

what is you

they

Protection

from
;

punishment
be of

plainly due
maintain it appears

the
this

innocent person you is


are

therefore, as
not
to

that

ought

punished,
his

that
The
above

convinced
"

innocence,

[w.]

equivalentto
are

The This

innocent
person

not to

to

be

punished
;

is not

be

punished

Therefore, this
Put

person

is innocent. obvious of Negative fallacy

in

this form

there

is

an

78

SYLLOGISM.

[CHAP.
express the

Premises;
affirmative The This
In

but form

we as

can

also
:
"

premises

in

an

follows

innocent person

ought ought
to

to

be

exempt

from from

punishment punishment.

be

exempt

this

case

it is apparent

that the
'

middle

term

'

ought
both

to

be

exempt

from

punishment
3 of

is the

undistributed

in

the

premises,against Rule
11.
'

syllogism.
heareth
my words ye
are :

He

that

is

of

God

ye
not

therefore of
In

hear

them

not,

because

God.'
order
:
"

[w.]

the usual
He Ye
.".

that is of God
are

heareth
;

my

words

not not

of hear

God
my AEE

Ye

do

words. in the first figure, and


of

The the
my

propositionsare
'

involve
'

Fallacy of
words
but

Illicit Process
as

the

Major

Term.
the

Hear

is distributed is

the

predicate of
as

negative
of

conclusion,
affirmative

undistributed The

the

predicate
would

the

major premise.
if
we were

argument
to

become

valid, however,
and universally, He
12.

allowed the
=

quantifythis predicate
to

assume

meaning
he who

be
my

that is of

God

heareth

words.

Any
error are

books

conveying plain
that

important
; but
as

truths
such do

out with-

deserve it is

attention few

books deserve

few,

books

attention.

Carefully distinguish the


this argument.
A be

truth

and

fallacy in
13 ; it may

good example suggestedby Whately's


put
:
"

No.

thus

VIIL]

QUESTIONS

AND

ANSWERS.

79

Any
Few
.".

books
books

conveying, "c.,
do do convey, "c.

deserve
;

attention

Few

books

deserve

attention.

This

is in
to
mean

the

first that

and, figure,
'A

if

we

the interpret deserve

clusion con-

few
in the

books

do

attention,'
that the
But

that
rest

is
do

to

say,

affirmatively only, without


mood

implying
Darii.

not, it is valid
p.

usually
'

(Elementary Lessons, indeed, in


books
are

67),

we

interpret few
do
the

negatively ;
such
not

the

example
without Most

itself this is the

plain meaning,
makes minor

few,' implying that all but this few


truths
error.

convey mise pre-

important
into

This

O,

'

books

do

not, "c.,' and


a

the
case

argument of Illicit

consistingof
Process

in

the first figure is

of the Major Term.

13.

That

man

is who

independent places
his

of

the

caprices of
in A

Fortune,
moral

chief

happiness
:

and

intellectual
is

excellence
of the

true

philosopher
Fortune
:

independent
a

caprices of
is
one

therefore his chief

true

philosopher
in moral

who

places

happiness

and

intellectual

excellence.

[w.]
Middle, the middle Fortune,' being
dicated pre-

A
term

case
'

of

the

of fallacy of the

Undistributed

independent

caprices
of
true
'

of

both affirmatively

one

who

places
The

his

chief

happiness, "c.,' and


mood

of

'

philosopher.'
conclusion

pseudo-

is, therefore,A A A
the better

in the second the

figure.
may
'

The

fallacy

is

none

because
If the

be

considered
that
man

true

in

matter.

premise had

begun

Only

is
a

independent, "c.,' we
valid
Barbara. syllogism,

might

have

put the argument

into

So

SYLLOGISM.

[CHAP.
climate and
as

14.
to

It is

an

intensely cold quicksilver ;


this it is does

that
the

is sufficient

freeze

climate

of

Siberia
At the

intensely
like
a case

cold.
of

first glance this looks


we soon see

Undistributed

Middle
'

; but

that

the
to

major premise is really

Any

climate cold

cold sufficiently climate.' The

freeze

quicksilver is
is thus valid

an

intensely
Barbara.

argument

in

15.

No

one

who
is

lives with

another any
on

on

terms

of in

confidence

justified,on
Brutus
lived

pretence,
terms

killing him:
with
on

of
not

dence confi-

Caesar:

therefore he

he

was

fied, justikilling [w.]

the

pretence

pleaded,

in

him. This
any

is valid in Barbara, the middle

major

term

on being 'justified

pretence, "c.,' the


the minor

term,

'one

who

"c.,' lives,

and

term,

'

Brutus.' is

The less

conclusion, however,
it
not

obviously weakened,
We

or

is

general than
Brutus
was

might

have

been.

might
on

conclude

that
It is the

in killing Caesar justified that he


was

any

pretence.
on

only

inferred
he

not

in killing him justified


course

pretence

pleaded, which

is of

included

in

'

any

pretence.'
16.

Inquire
called

into
:

the

validity
Coal

of

the
is of earth

following properly
a

afgument

Whatever
name

substance
consists in of
a

by

the

aceous carbonas

substance this

found

the

; now,

specimen
and
was

consists

carbonaceous

stance, subit

found Coal.

in the

earth, therefore

is

properly called

[L.]

82

SYLLOGISM.

[CHAP.
conclusion
as a reason

logicianscalled
and the minor

the analytical,

being put first,


or

premise
overthrown God is
not

adduced
to

proof.
the

The

major premise, assumed


'

be

obvious, is

to
are

effect that

All who
whom

are

in the wilderness well

among

those

with

pleased.'
be of

More

fully stated,
suffer

indeed, the assumption might


any
not

that those

all who

from is is to

signal calamity
well

are

some

with

whom

God

pleased.
a

To

be

overthrown This

in the

wilderness
makes
a

suffer from which

signal calamity.
can

view

sorites

the reader

put in order.
term

19.

If

the

major
of

of

syllogism
? this

be do

the
we

predicate
know
In

the the

major
minor

premise, what

about

premise
of

[L.]
sort, great

questions answering syllogistic


must

attention briefest afford


may be If the

be

given

to

throwing
Such

the

reasoning
The
above

into the

and

clearest

form.
in
:
"

questions,thus

treated, question

capital exercises
thus

reasoning.

answered

the major major premise is affirmative its predicate, and


case

term,

is undistributed
; in this

must

likewise

be

undistributed

in

the conclusion the minor be

the

conclusion, and consequently


affirmative.
If

premise,
order

must

be

the
must

major
be

premise

negative,
to

then

the

minor

premise

in affirmative,
case

avoid

negative premises ;

thus

in any

the minor

premise is
that fourth
in O

affirmative.

20.

Prove first the


or

cannot

be
and

premise
the

in

the be

figure ;
the second

that

it cannot

major

figure,or

minor

in the
If

third.
the
that

[M.]
O,
the

either

of

premises be
its

conclusion
term

must

be

negative, so

predicate the major

will be

viii.]
distributed. the other

QUESTIONS
But
as

AND

ANSWERS.

83
its
be

distributes
must

only
course

predicate,and
affirmative
and

premise, which
restricted.

of

conditions universal, only distributes its subject,the syllogistic


are

much

Thus

as

premises

in

the

first

give an figure
subject of O
Illicit Process

Undistributed and
of

Middle, the middle


A.
The

term

being
O

predicateof
the

premises A
the second

give
the

Major

Term,
its

the
In

predicate of A,
then be

major term, being undistributed.


cannot

figure O
the

be

the

major, because
minor, because
the

subject would
In

major
cannot

term,
be

and

undistributed. the

the

third
term

figure it
then Illicit Process the fourth

the

major

would

be

predicate of A,
of the
A

major premise, and again arise.


A

thus

Major would
will

in Finally,

figure O
Middle.

give

Illicit Major, and

O, Undistributed

21.

If it be

known

concerning
that

syllogism
middle
can

in the is infer

Aristotelian distributed
as

system,
in both

the

term
we

premises, what
?

to

the

conclusion

The

syllogismcannot
term, both

be in the second

because figure,

the

middle would
first

being
have
to

the be

predicate in
would

both

premises, these
5.
In

negative, against Rule


have
to

the

figure the minor


to

premise
its first

be

negative,in
term;
but of
a

order

distribute

predicate,the

middle

negative minor
Major
Term.

in the
In of the

gives Illicit Process figure


however, figure,
be

the

third

the middle

term

being subject
these be

both

premises will
fourth

twice

distributed

if

both

which universal,
In the

happens
of

in the moods
term

Darapti
is predicate

and

Felapton.
of the

figurethe middle
the
a

major
a

and

subject

minor

we

must,

therefore, have

negative major
in the mood

and

universal
We

affirmative

minor, which

happens

Fesapo.

find,then,
G 2

84
that
a

SYLLOGISM.

[CHAP.
term
can

doubly distributed
or

middle

prove

only
the

conclusion, I particular
and fourth

O,

and

these

only

in

third

figures.
Take
an

22.

apparent

syllogism subject premises,


the of the and

to

the

fallacy
whether

of
you

negative
can
or

inquire by
into

correct

reasoning premises

converting
the

one

both

affirmative

form.

[India Civil Service, July 1879.]


Take

premises in

the

first figure"

No No

YisZ;
X

is K
have

Obvert

the

major premise (seep. 42), and


All Kis

we

"

not-Z;
Y.

NoJTis The A
E E

premises would
in the first

give no minor,
No All X Y

conclusion,the pseudo-mood
Illicit Process
"

figure involving
we

of the

Major

Obverting

the

have
"

is Z is
not-

Y.

There middle

are

now

four
all. The

terms,
reader

and
may

therefore

no

common

term

at

easilywork
62

out

other

examples.
Vol.
I. p.

(See Principlesof Science, p. 75-)


Prove that the
minor

; first

edition,

23.

third

figure
and

must
a

have

an

affirmative conclusion.
In

premise,

particular

the

third

figure the major


if the

term

is

predicate of
be

the

major premise. Now,

minor

premise

negative,

yin.]
the

QUESTIONS
will be

AND

ANSWERS.

85
distribute
must

conclusion the

negative (Rule 6), and


term, avoid
of but the

its
be

predicate
affirmative

major
to

major premise
Term. It

in order

negative premises. Thus,


the the

there

will arise Illicit Process


reductio ad

Major
minor

follows, by
cannot

absurdutn, that
must

premise premise

be

negative and Again


the

be

affirmative.
of

predicate

the

minor

is the
term

minor will be

term, and, the

this premise being affirmative,

undistributed, giving a particularconclusion.

24.
a

Show

that

if the

conclusion

of

syllogism
term

be

universal but
once

proposition, the
distributed
in the

middle

can

be

premises.
by
the

Questions

of

this

sort

can

be
of

most

answered briefly
terms

counting premises.
subject.

the

available if the

number

distributed
be
a

in

Thus,

conclusion
one

universal
term

tive affirmafor

proposition, we
But,
at
as

need

distributed
must

its

the
most

premises
two

both terms,

be

affirmative,

they contain subjects.


middle universal
terms term

distributed is

namely
in which

their the

Hence
can

there be

only

one

place
the other

distributed.
be

On

hand, if the
and
;

conclusion
be be

negative, both
in the
cannot

major

minor
as one

require to
three

distributed

premises
and

but,

premise only can


than
terms

negative,we
the
the

possibly have
of the

more

distributed,the

subject
and

predicate
one.

of

the Two

negative premise, and being required only


was one

subject major

affirmative terms, middle

for

minor the

there

remains which

distributed

place
the

for

term,

to

be

proved.
this result
21

Observe

that

is

contrapositive of

that

proved

under

Question

(p. 83).

86

SYLLOGISM.

[CHAP.
of the

25.

Given the rule

the that

six
two

rules

syllogism, premise
be

and

particular premises
if
one

prove ticular par-

nothing-, show
the

that

conclusion

must

be

particular.
following ingenious third, R,
denial
of

This

may

be
De

demonstrated

by

the

reasoning of
"

Morgan

{Formal Logic, p. 14).


and

If two

P propositions, P

Q, together prove
of R
true

it is

plain that
For P

and

the
cannot

denial be

prove

the

Q.
Now,
prove

and

together
Q

without

R.

if
R

let possible,

and (a particular) Then the


P

(a universal).

(a universal) and the denial (particular)


of

of

(particular) prove
can

denial

Q.

But

two

culars parti-

prove

nothing."
that
as

26.

Show

the
a

proposition premise.
the

is

seldom

admissible
When O is the Rule

minor

minor

premise
will
As
we

conclusion distribute have


must two

must

be

negative by
the

6, and
term.

therefore
must

its predicate,

major
Rule

not

negative
the

premises by
and
term

5, the

major premise
its

be

affirmative,

will
must

not

distribute be the
term

predicate.
of the the
be be

Hence

major
Now,

subject
becomes

major

premise.

since
of the

the middle

undistributed

predicate
the minor
Thus

major premise, it must


in

the

predicate of

premise,
we

order that O

that
can

it may
be

once

distributed.

conclude

the

minor
Baroko.

premise only

in

the

second

giving the figure,


27. Show that
a

mood

universal
as a

negative proposition major premise,

(E)
Since may
serve

is has

highly
both

efficient
its
terms

[p.]

E
as

either distributed,

of them

the

major

term,

which, the

conclusion

being

viii.]

QUESTIONS
be

AND

ANSWERS.

87
term

negative,must
serve

distributed.
the be
to

The
term at

other
once.

will

then
mise pre-

to

distribute
therefore

middle chosen avoid

The

minor

may

will, provided that it be


There

affirmative
no

in

order

negative premises.
all of the four

is

restriction of
E
as

and figure,

accordinglywe
nineteen

find valid moods

with
no

major premise in eight of


the

figures;
moods

in

fact,

less than E.

recognised

begin

with

28. In

Name what

the

weakened
can

moods
there Do any

of the be of
no

syllogism.
weakened
nineteen
a

figure
and

mood,
moods

why
than
is

the

commonly
the

recognised premises
one

give

weaker ?

conclusion

would

warrant

By

weakened

mood
a

meant

which

gives a particular might


the
have been

conclusion drawn. thus


the
'

when The

universal

conclusion
from

information This
of

obtained
can, of

premises
only
when

is

weakened.'

course,

happen

conclusion

the

stronger

mood

is universal.

Hence,

in the third there mood


can

which figure,
no

gives only particularconclusions,


mood.
In the

be

weakened
a

other
will

figureseach
a sponding corresame

which

has weakened

universal mood

conclusion with

have of

conclusion
a

the

quality.
E A

Thus

Barbara E A

gives
O
;

mood A
E

I ;

Celarent,
fourth
E

Cesare,

Camestres,
of
are a

; in the

figure only
Thus the

Camenes

admits
moods

weakened

form, A

O.

weakened
of the

five in number.

Bramantip
to

fourth

figure is the singlemood

alluded

in the latter part of

the

question. impossible
the
to

Considering by
limitation it is

that

it is

employ

conversion
of the

without
too

weakening
of

logical force

premise,

bad

the Aristotelian

logiciansto slight

38

SYLLOGISM.

[CHAP.

the weakened done.

moods

of the

syllogism as

they

have

usually

29.

Can
relation
"

we

under

any X

circumstances and Z

infer the

between

from

mises pre-

Some Some

Fs Fs

are

Xs

are

Zs

[IndiaCivil Service,July 1879.]


Not

if

'

some

Fs

'

bear The

the

sense

attributed

to

the

pression ex-

in
some over one

Logic.

indefinite
it
must
never

adjective of
be

quantity
Y
'

is

so

that indefinite,
the
same were

interpretedtwice
some

with

meaning.
intended other

But

if the

in

the
same

premise
Y'
as a

by
and

the arguer
term

to

be

the

some

in the

premise, the
one,

would

practically
a

become

distributed in the

the

premises might give


Dr. Thomson
"

valid

conclusion

mood

Darapti.
" 77, p.
in

has word
of

remarked

(Laws of Thought,
appears
or

132), that
the
common

the
senses

(some)
'Some

to

be
'Some

employed

two

other,'and

in certain,'

language." purely indefinite


word,
F.'

Observe, however,
sense
'

that it is in the

former used
'

that F'

logicians have
must not

always

the
some

so

that

some

be

identified

with

30.

Is the That

following argument
has of
no

valid

syllogism
the
no

which

parts parts

cannot ;

perish by
soul has

dissolution

its

the
cannot

parts
the
This III.

therefore, the
of
its

soul

perish by

dissolution
is

parts.
the It Port

example

quoted

from 6.

Royal Logic,Part
remarked order
to

Chap,

ix., Example
advance

is there

that

several

persons

such

in syllogisms

show

90

SYLLOGISM.

[CHAP.
as

that

is

open
"

to

misinterpretation,

in

the

example

Potassium Potassium

is

metal
on on

;
water water. ;

floats

Therefore,
Examine

Some

metal

floats

this criticism

carefully.
Dec.

[Moral
What
first ed. I

Science the

Tripos, Cambridge,

1876.]
;

said

in

Principles of Science, (pp. 59-60


was

Vol.

I. pp. metal
=

71-2)

that

my

inference, namely,
water,"
result
'

"Potassium
a more

potassium floatingon
than The
the
'

is of Some here

exact

character
on

Aristotelian
'

metal
an

floats

water.' for
'

some

after

all is
we

only

indefinite
in mind
or
'

name

'potassium,'and
'

unless
'

constantly
and
'

bear
be

that the all,'

some

means

in is apt

logic
to

one,

it may
'

more

reasoner

confuse
matter

some

with

the

plural

several.' Groom

This

view

of

the

was

criticised

by

Professor

Robertson

(Mind, 1876,
of the

p.

219).
if any,

32. in

What the

is the

nature

argument,
'

apparent
because
be

enthymeme,
the
an

The
'

field
?

is

neglected
This
may,

soil is poor
in

of course,

argument
soil is
poor

Barbara, thus

"

Every field of
This
.*.

poor

neglected ;

is

field of

soil ;

This

field is may
cause an

neglected.
also
mean owner a

But

the
reason

statement
or

that the

soil

being poor
it ; in this
The

is the
case,

why

the
but

neglects
causal
out

it is

not

argument

relation.

student, therefore,must
the
bear

always
of

look

for

in ambiguities
may

conjunction 'for,''because,' "c.,


one

which The

certainly
between

of

two

if

not

more

senses.

relation

viil.]

QUESTIONS
and conclusion
cause

AND

ANSWERS.

gi
to

premises

has

nothing
effect.

whatever

do

with

the relation

between

and

33.

Explain
bad

"
"

It

is

scarcely
that
any but

ever

possible

cidedly de-

to
a

affirm

argument
detracts

involves

syllogism
the value

this

nothing

from

of

the

rules." syllogistic

[R.]
Scarcely any syllogism in
or one

in

ordinary writing or
; it is

discourse
that

states

full form

always presumed

the hearer is

reader
the
as

is

enough
make
as

of

logicianto supply
may

what

wanting.
such

Now
a

missing premise
to
so a

generallybe supplied in
breach the
new

way
to

good syllogismformally speaking, that


any of

is
It

say,

to matter

avoid

the

rules. syllogistic is

is another
The
us

whether
the the

premise
would

materially
that

true.

value
to

of

rules is,then, syllogistic

they

enable
to

assign
define

premises

which

be

requisite oblige
else

support

the
to

conclusion the

put forward.
nature

They

thus

the arguer
to

of his

assumptions, or

yield up

his conclusion.

34.
to

How

shall first

we

reduce

the

following syllogism

the

figure?
liable
to
err

All None None

men

are

;
err

who who

are

liable

to

should
are

refuse
men.

advice

should

refuse

advice

This Camenes the

argument
In

is in the absurd
name

fourth
m

in figure,
us

the mood

this and

the letter
final
s

directs
us

to

transpose
the
the

premises,

the

directs

to
we

convert

conclusion

simply; making

these

changes,

obtain

92

SYLLOGISM.

[CHAP.
natural

same

argument
"

in

the

more

form

of

Celarent,
advice

thus

None All
men men

who
are

are

liable liable to

to

err

should

refuse

err; advice.

No How
to

should

refuse

35.

shall first

we

reduce

the

following syllogism

the

figure ?
vertebrates animals animals O in
are are

All birds Some


Some

are

;
not not

winged winged
are

vertebrates

birds.

The

premises

A the

the

second is
a

figure,and
valid

the

conclusion Baroko.
The

being O,
letter
k

argument

syllogismin
Reductio
act

directs

us

to

employ

the

as impossibile, explained

in

the

Elementary Lessons, p.

149.
" "

Or

we

may

convert

the

getting major by contraposition,


are

All not-vertebrates
Some

not
are

birds ; not-vertebrates
not

winged winged

animals animals

Therefore, Some Taking


term, the

are

birds.
as

negative

term

not-vertebrates

the

middle

this is valid

in Barbara.

36. Can

we

reduce

the

mood

Camestres

per

? impossibile

Taking

the

symbolic example
All Xs
are are are

"

Fs Ks

No

Zs Zs
of

Therefore,
and

No

A's,
that
X
'

assuming
minor

for

sake
'

argument
Zs
are

the

conclusion true, which

is

false,the contradictory
put
valid
as

some

will be

premise
Darii

with
"

the

originalas major, gives the

in syllogism

vin.l

QUESTIONS

AND

ANSWERS.

93

All

Xs

are

Fs

Some

Zs

are

Xs

Therefore,

Some

Zs

are

Fs.

But

this

conclusion
Zs

is

the

contradictory
so

of

the

original

minor
the

premise
conclusion Darii

'

no

are

Ys,'

that

we

cannot

contradict

of

Camestres
of

without

producing original

syllogism
Thus mood
all

in

to

contradict the conclusion

one

our

premises. by
that
a

we

prove
the

of
It

Camestres be found

indirectly
on

of

first

figure.
the

will

trial

the

moods

of

imperfect
one or

figures
other of

may

be

similarly
of the

proved
first
or

indirectly
so-called

by perfect

the

moods

figure.

CHAPTER

IX.

QUESTIONS

AND

EXERCISES

ON

THE

SYLLOGISM.

i.

ASSIGN
out

the in

moods succession

of

the

following

valid

syllogisms,

pointing

"

(a)
(b) (c) (d)

The
The The The

conclusion middle
term

major
minor

term, term,

and and and


;

the the

major
minor
of

premise premise
the three

containing
containing propositions

it it

; ; ;

(e) (/)
(g)

The Their The


The The

quantities symbols
order
in

qualities

which the

they

should

be

technically placed

(h] (i)

figure
mood,

of and

syllogism
its mnemonic

;
name.

(1)

No

birds feathered feathered

are

viviparous
animals animals
are

All
No

birds

are

viviparous.
for
are

(2)

Robinson and
all

is

plain

spoken;
men

he

is

Yorkshire

man,

Yorkshire
not

plain
;

spoken.

(3)

Birds
Bats

are

viviparous
animals
not

animals
;

are

viviparous
are

Bats,

therefore,

birds. natural
laws

(4)

Whatever

investigates investigates
is
a

laws
;

is

science

Logic Logic

natural

science.

CH.

ix.]

QUESTIONS

AND

EXERCISES.

95
;

(5) Quicksilver is liquidat ordinary temperatures


Quicksilver is
Some
a

metal

; at

metal, therefore, is liquid

ordinary temperatures.

(6) True
Whales

fishes

respirewater
not

containing air

; ;

do

respirewater
not

containing air
true

are Whales, therefore,

fishes.

2.

Arrange
of

the

following

valid

syllogisms premise
and

in

the

usual

strict order
Name the

major premise, minor


mood
to

conclusion.
In
mining exa-

figure and

which the

they belong.
directions of

syllogisms, always
question.

follow

the

first

'(i) Iridium
metals

must
are

be

lustrous

; for

it is

metal, and

all

lustrous.
are

(2) Some

pleasures pleasures
are

not

praiseworthy;

hence

some

not not

virtuous, for
virtuous. that virtue do
not
are

whatever

is

not

praiseworthy is

(3) Epicureans
but all

do
true

not

hold

is the chief hold


true

good,
so

philosophers
are

that

it is

accordingly,epicureans

philosophers.
because
are

(4) Some

towns
are

in

Lancashire

unhealthy,
such
towns

they

badly drained,

and

all

unhealthy.
3. Draw

conclusions

from
mood

the

followingpairs of premises, employed


with
"

specifyingthe figure and


.

Every

virtue is
a

is

accompanied

discretion

I There
/ \

zeal without
a

discretion.

f Sodium
( Sodium

is is

metal
a

not

very

dense

substance.
;

(3)

All No

lions

are

carnivorous animals

animals
are

carnivorous

devoid

of claws.

96
/

SYLLOGISM.

[CHAP.
union
;

Combustion

is chemical is

(4)

Combustion -|

always accompanied by

evolution

(
f All

of heat.

boys
are

in the
no

third form

learn

algebra ;
form under
twelve

(5)

There -j

boys

in the

third

years

of

age.

( Nihil
/fi"

erat

quod

non

tetigit :
ornavit.

I Nihil

non quod tetigit

4.
are

Examine

the

following arguments
the

and

point out
mood
as

which before
;

valid
case

naming syllogisms,
of

figureand

in the of the

such

as

are

pseudo-syllogisms,name
is broken

the rule

syllogismwhich
name

thereby,

and

give

the

technical

of

the

fallacy
"

(1) All
No Some

feathered
are reptiles

animals

are

vertebrates animals
;

feathered
not
are

are reptiles

vertebrates.

(2) Some
Some Some

vertebrates

bipeds

bipeds
birds vices
are

are

birds ;
vertebrates.

are

(3) All

reprehensible;
not not

Emulation Emulation

is is
are

reprehensible
a

vice.

(4) All

vices

reprehensible;
is
not not
a

Emulation Emulation

vice ;

is

reprehensible. [L.]
art
are are

(5) Some
All

works works
of

of
man

useful works of
man

of

art
are

Therefore

some

works
;

useful.

[L.]

(6) Iron
All Iron

is

metal
are

metals

soluble

is soluble.

98

SYLLOGISM.

[CHAP.
fore there-

(13) The (14) The

nation it may wise its


man

has

right to good government; against bad


an

rebel has

governors.

of pleasures; for virtue infinity the midst


of

has

delightsin
it.

the severities that

attend

6.

Point
a

out

which

of the

followingpairs of premises will


name

give

conclusion, and syllogistic


cases.

the

obstacle

which

exists in other

(1) No (2)
(3) (4)
No
All Some

is B is B is
not not

;
"

some some

B
not
some no

is not

C.

A
B

C is B.
not

is B.

is B

;
some

C is B.
not

(5) All (6) All (7) All


(8)

not

is C;

A
not

is B.
B. ^4 is not C. B. B.

A
not

is B
B

; all not

C is

is

not

C ; all
;
no no

not not not

All A All

is

not

B A

is is

(9)
7. To
"

C is not

.Z? ;

what All B All B but

moods is A is ^
; ;

do

the C

followingbelong?
is A but
; therefore

"

(1) (2)

only

only

C is "."

"

nothing

C is A

; therefore

nothing

C is Br
De
to

See and

Dante's

Monarchia,
the

as

translated

by

F. C. Rev. of

Church,
R.
ing, reason-

appended
sometimes

Essay
Many

on

Dante, by the

W.

Church,

1878, p. 195.

curious be

specimens
drawn

pedantic,might

from

the

De

Monarchia.
8.

Supply premises
"

to

prove

or

disprove

the

following

conclusions

(i) The

loss of the
not

Captain

proves

that

are turret-ships

sea-worthy.

ix.]

QUESTIONS

AND

EXERCISES.

99

(2) The (3) The


war

system cottage-hospital
Prussians
to
are

should

be

adopted.
of rights

in refusingthe justified if

Garibaldi

they find

him

fightingagainst

them.

[E.]

(4) Private property should be respected in war. ought to be admitted to the franchise, [o.] (5) No woman (6) The law of libel requiresto be amended. [o.] (7) Capitalpunishment ought to be abolished. [o.] meetings, (8) Royal parks ought not to be used for political [o.] not of merit. are a safe test (9) Written examinations of merit. [E.] are a safe test (10) Written examinations "(n)
The

Annuity-tax should
national

be

done

away

with. be
a

[E.]
secular

(12) Any

system

of education

should

system.
how different moods

[E.]
may

9.

In

many

the argument
"

plied imcan

in the maintain that

followingquestion be
all

stated ?

No

one

who persecutionis justifiable ineffective."


correctness
'

admits

that

persecutionis
How would

sometimes the
formal
'

of
'

the ?

reasoning be

affected

by reading deny

for

'

maintain what

[c.]
and
can figure,

10.

What

and conclusions, each

of

mood

be

drawn

from but

pair of

the
are

? following propositions

(1) None (2) Some


(3)
All

gentlemen
of

members
are

of the club.
not

members

the

club
to

officers.

officers members

are

invited
of

dine.
are

(4) All
11.

the club

invited

to

dine.

[c.]

Express the followingreasoningsin

each

of the four

syllogistic figures.

(i) Some
the

medicines

should

not

be

sold without

buyer's name,

for

they are

registering [E.] poisons.


H 2

100

SYLLOGISM.

[CHAP.
; hence
some

(2) No

unwise
men

man are

can

be

trusted

tive speculathey
are

unworthy

of

trust, for

unwise.

[E.]

12.

Can

the

following argument
if so, what of ridicule
to

be

stated
term

in the form
?
"

of

and syllogism,
"

is the middle is
a

The

power

since dangerous faculty, and unjustly,


to

it

tempts
some

its possessor
for the

find

fault

distress

of gratification

others."

13.

If the
as
'

proposition
the
a

'warmth
of
or
a an

is essential

to

growth'
you ?
treat

occurred
'

premise

would syllogism,

warmth

as

distributed

undistributed

term

[E.]

14.

Show
as

that

the

following singlepropositions may


that
is
as

be

regarded

enthymemes,
"

equivalentto imperfectly

expressed syllogisms:

(1) (2)

Have If

thou

nothing
were

to

do

with

that

just man.
ride.
the power

[w.]
of of

wishes

horses, beggars
are as

would
to to

(3) Large

colonies
as

detrimental

State,
human

overgrown

limbs

the

vigour

the

body.
read been is
an as as

(4) If (5) All (6)

had

much

as

my

neighbours,

I would

have
law of

ignorant
of and liberty

[HOBBES.]
consequently

abridgment
asked
all

happiness. being
of it who what
was

Thales

the
"

most

universally
; for

enjoyed
have

things,answered nothing
my

Hope

they
touch

have

else. if thou
canst

(7)

will

give

thee

daughter

heaven.

ix.]

QUESTIONS
If all the
to
we

AND

EXERCISES. of

101

(8)

absurd

theories

lawyers and they are


no

divines

were

vitiate the should

objects in
no

which and

conversant, in the

have

law

religionleft

world.

[BURKE.]
between the

15.

Distinguish
senses

causal, simply logical, or

other,

of

the

copulative

conjunctions

in

the

following
"

1i) (2)

It will

rain, for certainly


are

the

sky

looks the

black.

The

people good. plant is


ancient
must

happy

because

government

is

(3)

This

not

rose;

for it is

monopetalous. soothsayers, and


will lead
a

(4) The

Romans have of
:

trusted been
the

their

therefore
state

frequently deceived.

(5) A

favourable

exchanges
cause

to

portatio im-

of

gold

this will which

corresponding
an

issue

of

bank-notes

will

occasion check

vance ad-

in
and

prices;

which

again will

exportation
to turn

encourage

importation, tending

the

exchanges against us.


1

[GILBART, 1851,
syllogism in
to

p.

284.]
are

6.

Form

an

example

of

which middle

there
and

two

prosyllogisms,one
to

attached

the

the

other

the

minor that
and
a

term.

[H.]
sorites with
of
n
n

17.

Prove

valid

premises
(n" i)
s

must

have

terms,

is

capable

giving

"

conclusions.

8.
v.

Can
Scene

the

following Shakspearean
stated

passage
a

(Hamkt,

Act

be i.)

in the

form
was

of

sorites ?

"Alexander returneth
loam
;

died,
dust of
;

Alexander

buried, Alexander
; of

into
and

the

dust

is earth

earth
was

we

make

why
not

that
a

loam, whereto
barrel ?
"

he

converted,

might they

stop

beer

102

SYLLOGISM.

[CHAP.
the

ix.

19.

Throw

the
:

reasoning

of

following passage

into

form syllogistic
"

Carbon,

which of and

is

one

of

the be

main

sources

of
water

the its

nourishment

plants,cannot
cannot

dissolved
be

in

in

simple form,

therefore
absorb

absorbed dissolved

in that

form

by plants, since
All the

the

cells

only
must

substances.
have

carbon
them

found
a

in

plants

consequently
and
and

entered carbonic

in

form

soluble of

in water,
carbon

this

we

find

in

acid, which

consists of the

oxygen." [A.]
as

20.

Complete
sound

such
but

following arguments
"

may

be

considered

incomplete syllogisms :
of
as

(1)

The

people
famine, and
country,
you

the you

country
are one

are

suffering
the

from the

of

people
famine.

of

must

be

sufferingfrom
of

(2) Light
does

cannot not

consist
possess

material

particles,for

it

momentum.

(3)

Aristotle

must

have for
many

been

man

of

extraordinary
have duced pro-

industry;
so

lie could
works.
was

not

otherwise

(4)

Marcus

Aurelius
;

both

good
that

man

and

an

Emperor
be

hence and is
some

it follows vice
versa.

Emperors

may

good

men,

(5) Nothing
valuable

which
;

unattainable

without is
not

labour

is with

knowledge

attainable

labour, and

is therefore
are

valuable. ar.d
no no

(6)

All

gasteropods
animals
are are

mollusks,

vertebrate

mollusks

; therefore

gasteropods

vertebrate. is
not

(7)

Suicide

always

to

be

condemned

; for

it is but
has been

voluntary death, gladly embraced

and

voluntary

death

by

many

great heroes.

CHAPTER

X.

TECHNICAL

EXERCISES

IN

THE

SYLLOGISM.

1.

Prove,

from
term

the

general
in

rules

of

Syllogism,
the

that

when

the
must

major
be Prove

is

predicate

its

premise,

minor

premise

affirmative.

2.

that,
the

when

the
cannot

minor
be
a

term

is

predicate
affirmative.

in

its

premise,

conclusion

universal

M
3. Prove

that
term

there

must

always
in the

be

in

the

premises

one

distributed
4.

more

than

conclusion.
of
a

Prove

that

the

major
can

premise
never

syllogism,

whose

conclusion
5.
Prove

is

negative,
when

be

particular
is will

affirmative. tive, nega-

that conclusion

the

minor

premise

universal
be

the
6. Prove

(unless weakened)
the
we

universal. the

that, if in
and

first

figure
a

we

transpose

major

premise
7. the 8.
correct

conclusion,
third

obtain

pseudo-mood.
be

In

the

figure,
what

if the will

conclusion the in

substituted

for

major
Prove

premise,
that
has
no

figure
the

be

[BAGOT.]
figure
its
can

syllogism
a

fourth
among

be

which universal

particular
for
be

negative
its conclusion.

premises,

or

affirmative

[L.]
the mood is
not
a

9.

If

the

major
the

term

universal determine conclusion

in the

premises
and

and

particular
it
one.

in

conclusion,
that the

figure,

being

understood

weakened

[c]

104

SYLLOGISM.

[CHAP.
transform the mood
A E

10.

Why
the
.

is it

impossible figureinto
have

to

from
1 1

second

the first?
a

What it ? What
must

figuremust figure
it ?
must

negativeconclusion

Why

must
12.

must

have

particular conclusion?
the fourth

Why
13.

Why Why
O
If ?

the

major premise
minor

of

figurenot figure

be

O?
14.
must

the

premise
of be of
?

of

the

fourth

not

be

15.

the

minor

premise

the

first

figure

were

not

what affirmative,
1

fallacywould
committed
of

committed? first

6.

If

the

major premise
be kind

the

figurewere

I,

what

fallacywould
17.

What of

proposition does
Why
of
be

not

occur

in
?

the

premises
1

the first figure ?

does the

it

not

occur

8.

If

the

major

premise

second
?

figure
of

were

what particular,
19.

fallacywould

committed the

What

is remarkable

about

conclusions

the

third
20.

figure?
What

kind

of

proposition cannot
can syllogism

be

proved by
subaltern

the

fourth
21.

? figure In

what be

mood

of the for

position pro-

substituted

its subalternans the affecting

(universalof
conclusion
must

same 22.

as premise quality)

without
a

If

one

premise
that

of

syllogism be O,
affirmative

what

the

conclusion
23. form 24.

be ?
a

Prove

universal of the first O excluded

proposition can
first and
second

the conclusion

figurealone.
from the

Why Why

is O

? figures

25.

is it that the moods

and

I O

are

true

in all the four figures ?

106

SYLLOGISM.

[CHAP.

x.

38.

Determine
than

how in

many
the

universal conclusion.

terms

may

be

in

the

premises
39.

more

Determine

how
than

many

particular
conclusion.

terms

may

be

in

the

premises
40.

more

in in

the

Determine

what

cases

there

may and of

be

in

syllogism [P.] Darapti,

an

equal
41.

number

of

universal
reduce

terms

particular.
Festino,

How

do

you first
the

Camestres,
?

Fresison

to

the

figure

42.

Exemplify
process

reduction

of

Baroko

and

Bokardo

by

the

per
that

impossibih. Cesare,
be

43.

Show

Disamis,
per

Camenes,

and

other

moods

can

likewise

proved

impossibile.

(See

Kars-

lake,
44.

1851,
Reduce

Vol.

I.

p.

81.)
to

Celarent

the

fourth it?

figure.

To

how

many

other

figures
Reduce

can

you

reduce

45.

Felapton
what

to

the

second

figure.
can

46.

To

other Barbara?

moods,

respectively,

you

reduce

Darii,

Ferio,

CHAPTER

XL

CUNYNGHAME'S

SYLLOGISTIC

CARDS.

1.

IN

this

age

of

mechanical
one

progress

it

may

be

matter

of

surprise
two

that

no

has ago
and

produced
and

syllogistic

machine.
true

About

centuries

Pascal

Leibnitz incited

invented

calculating
accounts

machines,
of these

Swift,

perhaps
the

by

the
of

machines, possession
the of

described
a

professors
About

Laputa

as

in

the
ago of

thinking
Smee,

machine.

thirty

years

late kind for

Alfred
of

F.R.S.,

proposed

the

construction with
More
a

mechanical the
a

dictionary,
ideas defined which
upon

together
in

contrivance

comparing
constructed

it.

recently meaning
they
rules

have any
not

machine worked than four

analyses
its

the

of do of form.

propositions
involve
more

keys,
terms.

provided
Yet

distinct
been

the

the So

syllogism
much the

have
worse

never

put
rules

into of the

mechanical

for

the

syllogism.
2.

Some

approximation
been

to

syllogistic
Mr.

machine

has,

however,
B.A. He

recently
devised

made certain

by

Henry

Cunynghame,
if

has

cards,
a

which,

placed

one

upon that of

the is
a

other,

infallibly give
and when

syllogistic
indicate the

mood

when
absence

possible,

it
contents

is

not,
of

the

conclusion. into in such

The
a

cards,

too,

can

be solid

condensed

hollow
a

cylinder
as

turning
all

upon

cylinder,

way

to

give

possible

syllogistic

io8

SYLLOGISTIC

CARDS.

[CHAP.
This

moods

in

one

turn to

of

the
a

handle.

device, though
is

hardly perhaps
the
nearest

be

called
to

machine, syllogistic
such
a

probably
is possible.

approximation
am now

machine

which

I
to

enabled, by Mr.

Cunynghame's

kindness,
for the

describe

these

ingenious

and

devices interesting

first time.
3.

The
a

cards syllogistic of

consist cards.

of

set

of

eight larger
is

and

set

eight

smaller

Each

larger card
to

3^

inches

high by 2\
one

inches

broad, and

bears, near

its upper

edge,
the

of the

eight possible propositionsconnecting J/,


P,
the

middle four

term, with

major

term.

There

are,

of

course, M In

propositions of
four

the forms
more

A,

E,
P

I, O, in which
is the
a

is the

subject,and

in which

subject.
certain
can

the lower

part of each

largercard
between

is S

written,in
and the

position, each possibly be


on

proposition
from
a

/*, which

drawn

syllogism having
for its
'

proposition
Thus,
6" it ;
'

the upper the


as

edge only

of the card Some

major premise.
is P,'
we

under is
but

major premise
the
universal

find

Some

P,'

conclusion affirmative
of

that

can

be

drawn
'All P

from

the of ^

proposition,
'

is M,'

admits
'

any
'

one

three

possible conclusions, namely,


or

No
4.

is P;

Some
of

S is the

P,'

Some is also

S is

not

P.'

Each

card

lesser

set
on

z\

inches

wide,
one

but the

only
with
or, one,
as

3 inches

high, and

bears

its upper

edge

of

eight possible minor


the middle Jlf,
we or

premises connecting S,
term.

the minor

term,

In

the lower

part of each

smaller,
cut

may in
some

call

minor it,
two

card

(with one

is exception),

cases

rectangularopenings, so adjusted placed


minor the is upon
a

that

if any
so

minor the

card

be and

larger or
inscribed

major
upon if any, in

card,
them

that

major

premises

be

one visible,

below

other, the conclusion,


seen

belonging
the minor

to

those

premises

through

the

opening

card.

If two

conclusions,a

universal

proposition,

xi.]
and its subaltern
we

SYLLOGISTIC

CARDS.

109

minor,
the
see

are

both
'

both possible,
P is

will be
the

seen.

Thus, if
'

take
we

major
below in
and
'

Some the the

MJ
'

and Some

minor

All

M mood

is S,'

conclusion
fourth

.5 is If
we

P,'

the

being
'

Dimaris S is

figure.
upon the

take
'

the
M

minor is

All
see

M,'

place it
S is

major

No

P,'

we

below

No

P,'
'

the

correct

conclusion
is
not

in the

mood

Celarent, together with


weakened
P
or

Some

P,' the
If M the

corresponding
major
not

subaltern

conclusion. minor
'

be
no

'

Some

is

not
can

M,'
appear

or

the
at

Some

is

S,'

conclusion
cards
are

all.

5. The
page. of
on

shown

in which

complete they
No the
are

detail

on

the

next

The

principle on

constructed conclusion
at

is that is written

conclusions. excluding illogical the

major
and it be
can

card
no

but

such

as

premise

the

top

will

warrant, card
if

conclusion unwarranted
out

is left uncovered

by
the

the

minor The

by
cards

the

minor

premise.
to

student

readilycut
and
cut

according
the
It
next

directions but
the

given
must

above

the

figures on
to

page,

they
struction con-

be

exactly

scale. card
be

will

assist

if each into minor


6.
seven

major
divided

divided
half
an

by

pencil high,

mark each

horizontal

spaces,

each

inch

card
The much

being

into

six similar is
more

spaces. and The

syllogistic cylinder
less easy the
to

compact
illustrate.

handy, ciple prinwas

but

describe
but

and

is
needed best

exactly
to

same,

considerable

ingenuity
in

combine
The

the order

cards
of

together into cylinders


minor
as

the
ferior in-

way.
or

the is

premises
"

on

the S
is

moving
is

cylinder
Si
6*
are

follows:
M

'All
'

J/,'

'

All

S,'

"

Some

M,"
is
in
etc. not

Some

is S,'
M P

No

S is M,' is
not
'

'No The Mis

Mis

S,' 'Some

A/,'

'Some
'

S.'
All

major P,'
'

premises
Some P

like

order,

All

is M,'

is

M,'

CUNYNGHAME'S

SYLLOGISTIC
MAJOR
CARDS.

CARDS.

MINOR

CARDS.

CHAPTER

XII.

FORMAL

AND

MATERIAL

TRUTH

AND

FALSITY.

i.

THE

rules

of

syllogistic
assumed be
true

inference
to

teach

us

how,
other

from

certain

premises
which
of

be

true,
those

to

draw

propositions
But

will

under the

assumptions.
to

if, instead
them
to
as

supposing

premises

be

true,

we

regard
arise
Such
as

materially
conclusions
have of
not

false, various
which
been may

puzzling properly
treated
to

questions
be drawn. in results furnish and
to

the

questions
manual

adequately
as

any

popular
great
admirable

logic;

and and

they
at

lead
same

of

practical

importance,
in the
to

the

time

exercises
I

discrimination draw

of

good

bad

reasoning,
subject
in

propose

special
with

attention
answers.

this

the

following

questions

2.

Is
true

it

possible
?

to

draw

false

conclusion

from

premises
of

It

is

possible,
if
sense. we

course,

to

draw

any

conclusion of
a

from

any of

premises,
common

disregard
But

the
we

principles speak
be in in

logic

and work
to
mean

when
we

logical

of

drawing drawing
of
of
a

conclusion,
conclusion
and

must

understood with
Now

logically,
Rules of

accordance

the the

Laws
nature

Thought
the

the relation

the

Syllogism. premises
and

logical
if the

between
are

conclusion is

is

this, that

premises

true

the

conclusion

true;

ii2

TRUTH

AND

FALSITY.

[CHAP.

truth

is, as
truth.

it were,
; not

carried whole

from truth

the

premises
of

into

the

conclusion

the The
a

but necessarily,

nothing
be

except
answered

question

above

must,

course,

with Is

direct

negative.
to

3.

it

possible
?

prove

true

conclusion

with

false
To
prove

premises
a

true

conclusion,
must
mean

or

to to

prove

that

tain cer-

conclusion
the

is true, of the
p. of

establish
"

its truth To

in

opinion
the

persons 90,
some

concerned.
to

prove,"
which tion relastated the

says

Wesley, 1832,
truth

"is

adduce

premises
Now,

establish of

conclusion." in
are a

premises
is that admitted

and

conclusion

as syllogism,

just above,
must

if the
to

premises
likewise
as

true

the

conclusion if certain

be

be

true.

But,
cannot

persons

regard
such

the

premises
as

false,they

possibly
the

regard
the
"

premises

or establishing proving

truth of
p.

conclusion.

Solly,indeed, points out


a

(1839, note,
false
can

9)

the

of possibility
form

true

conclusion
But

from

premises in

every
that
on

of

reasoning."
is

this remark
or

only
to

mean

the other
4.

conclusion

materiallytrue,

known

be

true,

grounds.
If

the

premises
the

of

syllogism
false ?
if the

are

false,does

this make
No. The and the

reasoning
is
correct

reasoning
other

form

of

the
mood

mises preof

conclusion

agree

with

that of any

valid
of

or syllogism

development
the material
most

of the Laws of any of

Thought,
sitions propo-

wholly regardlessof
per
a se.

truth

the

The

ridiculous
"

proposition may
to

make

: good syllogism

for instance

has angles equal Every griffin is Every triangle


a

two

right angles ;

griffin ;
"c.

Therefore, Every triangle has,

ii4

TRUTH

AND

FALSITY. moral

[CHAP.
science. The
a

more

complicatedtruths
is
a

of

and physical
XI.

(See

Principles of Science, Chaps. (a},then, adopted


true

and

XII.)

sentence

only

on

the

suppositionthat
blindly accept
in
a

man,

having
its false

false
to

premise,
say,

will

all

results, that is
manner.

will

reason

purely

deductive

The
learnt

sentence

(b)is
can

erroneous, from

because,
false
But

as

we

have
a

fully
true

(p. 112), we
in mind had

premises
there
was

draw

conclusion the

form. good logical

doubtless and

in

writer's

confusion he
to

between

formal

material

falseness,and
it is

said that establish have

by premises materiallyfalse
the material
correct.

impossible

truth

of

any

conclusion, he

would

been

6.

An

apparent
examined

syllogism
is found

of
to

the break
term

second the

figure
rules of tributed. undisis
marked re-

being
the

syllogism,
On that
one

the

middle

being
it is
can

further
of

examination

the
true.

premises
What

evidently
we

false,and
from
or

the

other

infer truth

such

circumstances

concerning
?

the

falsityof

the

conclusion

[c]
is
can

As

in the the
rule
once

second

figure the
the

middle

term

predicate in
break
be

both third

premises,
of the
at

apparent

syllogism
the be
be

the

syllogism,requiringthat
both therefore A

it shall
are

buted distritive. affirmaor

least,only when

premises A,
A

The In

premises must
if the

I,

A,

II.

the first case,

premises
^s
are are

All

Ks, Ys,

All Zs

xii.]
and
we assume as

QUESTIONS
the first
; thus
one

AND

ANSWERS.

115

to

be false,we

obtain

its

dictory contra-

true

Some All The Zs

Xs
are

are

not

Fs

Fs.

conclusion

must,

by

Rule

6, be

and negative,

there

will be

Illicit Process
to

of the

Major.

Assuming

the second

premise

be

false,we
All

get
Xs
are

Fs
are

;
not

Some
Whence
we

Zs

Fs.

may

infer correctly Some


Zs
are

in the mood
not

Baroko,

-X"s.
assume

In

the

case

of

I, we
false,we
All No Xs Zs

obviously cannot
get
are are are

to

be

false ; but

if I be

Fs
Fs Xs.

; ;

Therefore,
In the

No

Zs
we

premises
Cesare. its
If

I
of

A,

cannot

assume^A
Term
; but

to

be false without be
be

Illicit Process
have

the

Major
1 1

if I
can a

false,we
taken
as

Lastly,in
the

only
of
not

the

major
us

false, and
Festino.

contradictory then
conclusion
does

gives
the

syllogism in
or

apparent

pseudowhat
we

syllogism in question
thus

correspond
false logically

with
as

obtain, the
the
new

conclusion

is

compared

with

premises assumed.

7.

If

(i) it

is false with

that

whenever

is found
untrue

Y that

is found

it, and
found

(2) not
without

less
the

is sometimes of

ment accompanithat may

Z,

are

you

justifiedin denying
there
also
you
I
2

(3) whenever

is found

n6

TRUTH

AND

FALSITY.

[CHAP.
however this may

be

sure can

of
you

finding
in

?
same

And

be,

the
Fin of

circumstances

judge
[R.]

anything
This
can

about

terms

of Z?

excellent

example

reasoning by
to

contradictories rules of

be

easily solved

by adhering
X

the

simple
is found
'

opposition, and
be
as

gradually undoing
the

the

perplexities.
Y

The

supposition that, whenever


may

is found

with
are

it,
'

stated this is

as

universal

affirmative
'

all Xs Xs
are

Ys Ks

;
'

but

false,its contradictory
That

some

not

is the

true

condition.

(2) X

is sometimes
mean

found that
'

out withXs real

the
are

accompaniment
Zs
'

of Z, would
to

some

not

but, being asserted

be
are

untrue,
Zs.'
sure

the

condition whenever

is its
Z

contradictory
all Zs
'

'all

Xs may

Thirdly,
of

is

found, there also you


'

be
you

finding
these

Y,
must

means

that
assert

are

Ys Zs
"

'

;
are

but, if
not

deny this, you Putting

that

some

Ys.'

propositionstogether,thus

(1) Some (2) AH


Hence,
we

Xs Xs
are

are

not

Ys ;

Zs
are

; not

(3) Some
they
mood
make

Zs

Ys ;

find

that the

valid

syllogism

in

the

third
a

figure,and

Bokardo. be and
I

The
converted

conclusion, being

particular negative,cannot

directly ;
'

we

can

only
are

obtain

by
Thus

obversion
we

conversion
answer

some

not-

Ks

Xs.' the

must,

presume,

the

last part

of

problem negatively.
What that
"

8.

is the
a

precise meaning
'
"

of

the

assertion

proposition

say

All

grasses

are

edible'

is false ? of the of propositionsis generally falsity with

The

doctrine
to

supposed

be defined

precisionin

the ancient

formula

xii.]
of

QUESTIONS
of

AND

ANSWERS. If

117

the square

opposition.
of
are

universal

affirmative

position pro-

is false, its true,


so

the particular contradictory, negative,is


case

that, in the
'

the
not

example
edible.'
of
to

given above,
I,
and vice

we

infer, that
the
But
to
case

some

grasses
we

from Similarly,
versa.

of falsity

E
seem

infer
to

the

truth

it does

not

have

occurred

logiciansin general
be

inquirehow
of

far similar

relations other
more

could

detected

in the
of
'

disjunctive and
for all ?

complicated
assertion
If this

kinds that
be

propositions. Take,
endogens
what
not
are

instance, the
that

All

parallel-leaved plants.'
one or more

false,
are

is

true

Apparently

endogens
or more

parallel-leavedplants, as, plants


are

else that But

one

parallelhappen
are

leaved that
three
not
no

not

endogens.
the

it may

also

endogen
which the

is

parallel-leaved plant at
of simple falsity

all.

There

and alternatives, show of

the

does original is
not true.

the

possiblecontradictories
whether
of

But of

question
usual

arises

there

is

confusion doctrine
of

ideas

in the

treatment
a

this ancient
a

opposition, and
not

whether

contradictoryof
involves
of

proposition is
the

any is

propositionwhich
not

the it.

of falsity I

original,
that any

but

the

sole

condition is made that


not

apprehend
sufficient

assertion
It

is false which
to assert

without hidden
we

grounds.
moon

is false

the

side
can

of

the

is
tradictory, con-

covered

with but

mountains,
because
we

because

prove

the
must

know

that the
If
'

assertor
a

have

made
of

the

assertion
were

without
to

evidence.
assert

person

ignorant
are

mathematics

that be
or

all involutes
a

scendental tran-

curves,'he
because, whether
Professor
one can

would
are so

making
not, he

false
cannot

assertion,
know
that

they
Newman
a

it.
no

F.

W.

has

correctlyremarked
terms

reallybelieve
understand is

proposition the

of pp.

which 35,

he

does This

not

on (Lectures

Logic, 1838,
does

36).
know

unquestionably

true

for, if he

not

ii8

TRUTH

AND

FALSITY.

[CHAP.

what them
a

things he
to

is

speaking about,
his mind.
act

he

cannot

possiblybring
who
swears

comparison in
a

witness
as a

that

prisoner did
not

certain whether
of
to

when,

matter

of
or

fact, he
swears

does

know

the the

prisoner did question


the

it

not,

independently falsely,
evidence that Dr.
a can

whether

rebutting
of A

be

brought
wished

prove
to

perjury.
an

It is reported

man,

who

be

thought
in

acquaintance
of

Johnson,
sermon

remarked

to

him

coming
'

out

church,

'

good

to-day,
very
can

Dr.

Johnson.'
This

That

may

be, sir,'
I'm
not

replied
sure

the

much know

over-estimated
it.'
a

doctor, 'but

that you
It

hits the

point precisely.
that
a

will be

shown

in

subsequent
has

chapter
an

position pro-

of moderate
number less
more

complexity
the

almost
are

unlimited
more one or or

of
in

contradictory propositions,which
with

conflict
of

original.
the

The

truth of the
not

any

these
the

contradictories
of falsity
or more

establishes does original


of

of falsity

the the

but original,
truth there of any

establish

one

its

contradictories, because
that

always

remains

the
of

alternative
the
terms.

nothing
even

is known

concerning
that view
no

the relations
at

It may the

happen
In

relation

all

exists
an

between assertion

terms.

this
a

of the

matter,

then,
and

of

the

of falsity
not

proposition means
between

its simple deletion.

The
but

is contrariety

knowledge
also

knowledge,

between

knowledge
the Fs' in

and
It

ignorance. ought
of
to

be

remembered,
the

in

dealing with
Xs Now
are

trine doc-

that falsity,
one or more

falsityof
Xs
are

'all

only

implies that
one are or a

not

Ks.
no

practice
; there agree

few

exceptions are

often

of

importance
in
a sense

in many

cases

exceptions which singular


sense

with, and
all

in
a

a falsify, general proposition.

Thus

points of
two

revolving sphere
at

describe

circles, excepting
of

the

points

the

poles.

Other

examples

singular

xn.]

QUESTIONS
will be found Henrici
a

AND

ANSWERS.

119

exceptions
XXIX.

in the

Principlesof Science, Chapter (Elementary Geometry,


be

Professor
P-

points out
must

1879,
true

37)

tnat

proposition
be
true

considered
of

to

be

in

general, if it only
in
a

in

an

infinite number
of

cases,

and

false

finite number the


truth

exceptions.
as

This

subject
and

of

and
be

of propositions falsity

premises
Logic,

conclusion
I. p.

may

pursued
II.

in

Karslake's

Vol.

83;

Whately,

Book II.

Chap.
i.
"

iii. " iv. ;


Part

2;

Aristotle, Prior

Analytics, Book
Part

Chaps,
Watts'

Port II.

Royal Chap.
Most

Logic,

II.
8.

Chap.

vii.

Logic,

ii. "" 7 and of the

scholastic

such logicians,
treat

as

Thomas

Aquinas,

Nicephorus Blemmidas,

this

subject elaborately.

"

9.

Trust

"

(said Lord
"

Mansfield
own

to
sense

Sir

A.

Campbell)
your
to

to

your

good
beware
of

in forming

opinions
the

; but

of

attempting judgments. right ;


"

state

grounds
will

your be

The

judgment
will

probably

the

argument

infalliblybe
and

wrong."
show its

Explain

this

phenomenon,

logical [p.]

significance.

If
reasons

you
are

give

reasons

for

decision, implying
the
reasons

that which

those
you quently subse-

and sufficient,

are

upon for

did

make
to

the

decision, it is possible
whether
from such
reasons

critics

inquire

logicallysupport
If

the

conclusion
will be

derived detected

them.

they

do there

not,
may

the

judge
no means

in

paralogism
But, if

which
no reasons

be

of

explaining away.

be

given,

120

TRUTH

AND

FALSITY.

[CHAP.
to

it will

seldom

be

possible for
a

critics

make

any

such in

detection. detail
case

It is

impossible, as
well
as

general rule, to publish


upon

the is

law

as

the

evidence
were

which

law

decided, and,

even

if it detect

published,

it would
man

generally be
does the
not

impossible

to

bad
on

logic in
which
a

who

assign the precise points


in which he argues

he

relies,and
mass

way

about

complex

of

details.

Although
and
any
an

it may discreet

be, from
for
a man

his
to

own

point

of

view,
reasons

venient con-

avoid
can

giving
avoid

for

important public decision,


open

if he
means

it, yet

it is

question
increase There

how

far such

of and

escaping

criticism his

is

likelyto

the
are

carefulness many
cases,

impartialityof

judgments.
verdicts

including nearly

all the

given by juries on
undesirable the of and result
to

points of fact, where


any
statement

it would of
reasons.

be

highly
Where

require
upon

depends
the

oral

testimony,
of

the
of
to

haviour bebability pro-

witnesses, degrees
the
real
to

estimation is
of

degrees

of

it guilt,

quite impossible
the and conclusion

define
to.

and

publish
must

premises
sense

come

We
same

trust

common

judicial tact.

The

remarks

may

apply

to

various
votes

arbitrations,magisterial
of members of of

decisions,

administrative
But
as

acts, where
to to

liberative deare

bodies.

the

grounds
of
a

decision

precise

and
seems

brief,so
absurd he

be

capable
that

complete judge
will

ment, state-

it less well

suppose
to

judge
If he

because

needs

disclose

his argument.
can

displays bad
clearlynot possibly have
who
was

logic,where
be
a

bad

logic
Lord

be

judged,
advice
to
a

he
may
man

is

fit to

judge.

Mansfield's when

been
to

prudent
act

and novel

good

given

forced

in

circumstances, and
his decisions would

in

distant

colony (Jamaica),where

have

CHAPTER

XIII.

EXERCISES

REGARDING

FORMAL

AND

MATERIAL

TRUTH

AND

FALSITY.

1.

Compare
both
as

the

following
their of
a

syllogisms,
formal their

or

gisms, pseudo-sylloand conclusion conclusion


as

regards
truth it is
case.

correctness, and

regards
then

the

material how each


All No

premises
true

explain
in

that

materially

is

obtained

(1)

existing things
abstract

are

real

things

; ;

ideas ideas

are

existing things
real

.-.

No

abstract real

are

things.

(2)

No All

things

exist
are

abstract abstract

ideas ideas

real
real

things things.

.".

No

are

(3)

All No

real

things

are

existing things
are

; ;

abstract

ideas ideas

existing things
real

.-.

No

abstract

are

things.
which
we

2.

If

there

be

two

syllogisms,
are

of

know

that
how

their
may

major
we

premises
the be both

subcontrary figure
true

propositions,
of both
?

determine

and in

mood

May

their

conclusions

matter?

3.

Prove
of
one

by

means

of and thus the

the

syllogistic rules,
the

that
a

given
valid in
no

the

truth

premise knowledge

conclusion
our

of

gism, syllocase

the sufficient

in
of

possession
other

is

to

prove

truth

the

premise.

[c.]

CH.

XIIL]
4.

EXERCISES.

123

It is known
a

concerning

supposed syllogism middle,


can we or

that
one

it
of

involves
the any

fallacyof undistributed
is false in under
matter

and
can we

that
not

premises
conclusion

draw

these

circumstances

5. Construct
of
one

two

such syllogisms,

that the

the

major premise
of the be both

shall

be

the

subcontrary
that these the

of

conclusion
of
to

other, and
true

such

also
Are

conclusions sufficient

shall

in matter.

data

determine

the

figure?
6.
correct matter

If

one

premise
form, does

be

false

in
that

matter,
the

and

the

syllogism
is false in

in
?

it follow

conclusion

7. Examine

the

doctrine

'

that, if the
may be

conclusion
true
or

of

syllogism be
but

true, the

premises
be

either
or

false ;
of

that, if the
be

conclusion false.'

false, one

both

the

premises must
8. from
"

Interpret the
Mr.

logical force
Essay
foolish
on

of

the

following
Roman

passage
:
"

Freeman's have
been

the

Holy
that and

Empire

It may

to

believe

the German
that the

King
Roman

was

necessarilyRoman
was

Emperor,
of

Emperor
9.
one

Lord necessarily

the world."

Taking
of the

syllogism of
to

the

third

and figure, whether


to
or

assuming
not, with
be

premises
of
can

be

false,show
thus

the

knowledge figureand
10.

its falsehood frame


to
a new

supposed belong.

in

our

possession,we
the

syllogism :
it will

if so,

point

out

mood

which

What
as

do

you

mean

by
a

(i) Formal,

(2)

Material
a

truth,

applying (a)

to

single proposition, (ft)to

syllogism?

124

TRUTH

AND

FALSITY.

[CHAP.

ir.

Give

careful

answer

to

the
in

miscellaneous
322.

example,

No.

88, in Elementary
Is the
or

Lessons

Logic, p.

12.

following
?
can
"

extract

sense

or

nonsense,

logically
the

correct

incorrect

We

may

doubt the

whether any

ancient

method mood
a

of reduction ;

prove

of validity
we can

syllogistic

for, as

from

false

premises
ad

obtain illogically doubts


cast

true

conclusion, the
its
as validity a

reductio

has impossibile

upon

method

of

proof."
of the assertion
not

13.

What
to

is the say

precisemeaning
Castro
cannot

that
to

it be

is false Orton
?

that

be

proved

14.
cannot

If P
be

asserts

that

oxygen,

hydrogen,
the
?

and

nitrogen

liquefied,and

denies
to
mean

assertion, what

preciselymust Analyse
each

Q be understood
all of that

15.

is

implied

in

the

assertion
"

of

the

of falsity

the

propositions: following
a

1i ) Roger
(2)

Bacon

was

giant.
Newton
was

Descartes

died

before is
not

born.

(3) Bare
(4) All

assertion kinds

the necessarily
one

naked

truth.
not

of grass

except

or

two

species are

poisonous.

6.

Let

X,

Y, Z, P, Q, JR, be six propositions: given


Of

(a)

X, Y, Z,

one

and and
is true

only only
;

one one

is true

; ;

(b} Of P, Q, J?, one (c) If X (d) If (e) If


Y

is

true

is true, P is true,

is

true

; ;

is true, R

is true

XIIL]

EXERCISES.

125

Prove

syllogistically, (/) (g)


If X

that

is is

false,
false,

is is is

false
;

If

Q
R

false

(A)

If

is

false,

false.

[c.]

17.

How

do

you

meet

the

following
false

difficulties

(1)
conclusion

True

premises
;

may in

by
a

reasoning
of

give
there other.

correct

because,
one error

train

reasoning
the

may

be

two

errors,

and

may

neutralise

(2)
per
se

Since is

truth

applies
of because

only
it

to

propositions,
that
a

and

term

incapable false,

truth,

follows

term

must/^r
either
true

se

always
false.

be

everything

must

be

or

CHAPTER

XIV.

PROPOSITIONS

AND

SYLLOGISMS

IN

INTENSION.

i.

To

any

one

desirous

of is

acquiring
so

thorough
as a

command

of of

logical
the

science,
or

nothing

important meaning
indeed is

careful

study
positions, pro-

intensive and

comprehensive
This
some

of
not
an

terms,
easy

syllogisms. by
the the fact that

task,
have
have of

as

is

shown
upon

great
Sir

logicians
W. fatal

who

written fallen

subject,
errors,
or

especially
at

Hamilton,

into

grave
Most

the

best

ambiguities again,
it in
a

expression. ignore quite


the

of

the

common

text-books,
or

either

subject

altogether,
to

else

treat

manner

disproportioned questions points


read
to

its

difficulty

and

importance.
some

The
more

following
obscure
have

and the
of

answers

touch but the

of

the

of

matter, the the

student
Lessons

is assumed
in

to

the have books:

fifth studied

Elementary

Logic,
of

or

else

subject Logic,

in

one

or

more

the
v.

following
vii. work
;

Port

Royal

Part

I., Chapters
pp. 45
"

to

Spencer
was

Baynes'
first
to

Translation,
draw Part
to

1861,
to

55
in

(this

the Watts'

attention
I.

the

subject

modern

times);
Levi

Logic,
articles

Chapter
;

vi., ""

and

10

Hedge,
Laws

34

38
"

Thomson's
52;

Outline

of
""

the

Necessary
t"

of

Thought,

Spalding,

1857,

30

33

Walker's
II.
most ;

Commentary
Treatise
treatment
on

on

Murrcrfs
Logic,
of the

Compendium,
IV.

Chapter
The

Bowen's

Chapter subject
is

elaborate

found

in

CH.

xiv.]

QUESTIONS
the

AND

ANSWERS.

127

Peirce, Proceedings of
Sciences, 1867,
read
what Vol.

American pp.

Academy
"

of
If

Arts

and

VII.,
on

416

432.
must

the

student

Hamilton's is said about

Lectures

Logic, he
in this

observe carefully

Hamilton

chapter.

2.

State the

the

proposition
form.
it

'

Men

are

mortals

'

in

intensive

This
asserts

proposition, as
that all mortals.

stands, is clearly extensive, and


men

individual
When

will

be
to

found
turn

among
a

the

things called
into the

asked

such all

tion proposikinds
of

intensive

form,

students 'All

make the

blunders, saying, for instance,that

noted qualitiescon-

by
'

the
the

term

man

are men

connoted
are

by

the

term

mortal,'
mortal.'
to

or

All

properties of
the

properties
men,

of

This

is

certainlynot
are

case,

because

in addition
are

being mortal, Again,


a

rational,are
would
say,

vertebrate,
attributes
means

erect,
man

"c.

student of

'The This

of

connote

the attributes
connote

mortality.'

nothing, the
say,

verb
'

being wrongly
possess

used.

To man,

again,
the
as

that

All

which
of

the
to

propertiesof
the

possess

properties
was

mortal,' is
which

leave

proposition just properties


of

it

before,

'All
men.

possess

the

man,'

being simply
mode

In

passing from
must terms.
are

the be
men a

extensive

to

the

intensive
of
so

of

thought, there
of of
we

complete inversion
a

the

relation

the

As
a

are

part of mortals,

the
If

qualities
we

mortals may
care use

part of

the kind
to

qualitiesof
of

men.

like
case,

different

copula
error.

but, in
The
same

that

much

is necessary modes
of

avoid

following are
truth, the
second the

different
first of

expressing correctlythe
an

each

pair being
intensive

extensive of

and

the

corresponding

form

assertion.

i28

INTENSION.

[CHAP.
mortals
;

All
All

men

are

included
mortals

among
are

"

of qualities

included

among

the

qualities

(.
(

of

men.

Mortals

include
men

men

I
(

Properties of
Man

include
mortal

the
;

of properties

mortals.

is

species of
mortal

\ The
( Men

genus
are

is in the
;

speciesman.

part of mortals part of

\ Mortality is
3.

humanity. particular and


form
? much

Can

we

exhibit
in the

negative

sitions propo-

intensive

This

question has
the
most

not, I remarks

think,been
to

investigated by
in the works
to

logicians,and
Hamilton universal and

be

found

of the

other

logicians apply only Taking


the

affirmative
'

proposition.

particular
that
'

affirmative,Some
or more

are crystals

opaque,'it asserts
opaque
'

One

crystalsare
the
more

among
'

things.' It follows, no
is among
the

doubt, that
of
one

quality
the

opaqueness

the

qualities
I may be

or

namely, crystals,
extensive
as

particular crystals
Thus

referred treated

to

in

proposition.
A is treated.
'

much intensively the

Taking
we

negative proposition,
infer
that
'

No

iron bars

are

parent,' trans-

cannot

No

propertiesof transparent
This inference would
as are

objects are
be

propertiesof
false ; for, there

iron may

bars.' be many

quite

such properties,

gravity,inertia, extension, "c., indestructibility, possessed


we can

which bars.

alike

by transparent
'

objects
'

and

iron

All

infer is that
are

Not

all the or,

propertiesof transparent
of the

things

in

iron

bars,'
not

Some

properties of separation

transparent things are


in extension involves

in iron

bars.'

Entire

in intension, or separation only partial

130

INTENSION.

[CHAP.
Illicit Process
P of

illogical.It Major
other Term
means

involves
;

fallacy of
may
But

the

in

short, S
M.

comprehend
Bowen

through
that
was

besides
error

Professor choice it of
as

thinks

Hamilton's such
as no

lay in
would

the

language, which
Hamilton
too

one

understand

tended really into

it.

The

matter,
way,

however,
and
I

is

important
to

be

passed places
In

over

in this which

proceed
treated

notice
of

other

in

Hamilton

has

intensive

syllogisms.
his sixteenth
Lecture
"

on

Logic (Vol. III.

p.

295),we

find the
"

following:
Extensive
B

An

Syllogism.
B
.

An

Intensive

Syllogism.
A

is A

C\sB ^is C\"A


; ;

Cis C
All But
man

is A

is mortal is
a man

Caius
But

is

man

Caius

all

man

is mortal Caius

Therefore, Caius
Between the

is mortal.

Therefore,

is mortal."

syllogismsas
the order
It

thus stated there is no


of

difference
a

whatever,
difference

except
in the

transposition
of is

premises,

mere

writingwhich
true

is immaterial goes

to
on

the
to

point

in

question.
follows
:
"

that

Hamilton

explain as
"

In

these been

examples,
said, that

you

are

aware,

from in the

what
two

has

viously pre-

the
a

copula
under

different

is precisely of quantities
of

counter

meaning
;

; in the

quantity

contained extension, signifying

in

the

quantityof

contains comprehension, signifying

in it."
concrete

Afterwards

the

example
Caius

of

syllogismbefore
form

given,is
"The
man

thus

stated fully
term

in the intensive contains in

(p. 296).
Middle
term

Major
;

it the

xiv.]
But

QUESTIONS
the Middle
;
term term
man

AND

ANSWERS.

131
term

contains

in it the Minor

mortal

Therefore, the Major


term

Caius

contains

in it the Minor

mortal."
the

To

say of

least,this is

very

clumsy

and

misleading
matter

mode

explanation ;
under
are

for after all,the


that

point of the

is left untouched,
are

namely,
in the

it is individual
sense,

things which
or qualities, sense.

contained

extensive in

and

attributes,which
not

contained that Caius


taken

the

other man,

Is

it

absurd that

to
man

say

contains

without
A way

plaining ex-

is here
of

? intensively the
same one

thing
that

does the

not

contain
man,

any

its

in qualities

class
or

regarded,contains extensively
Nor

of its members
matter

significates, namely, Caius.


by

is the

much

mended

referring back
Hamilton
"

to

the

previous explanation relations, saying,


in the
one

(p. 274),
"Thus the

where

illustrates the God


is

proposition,
is contained

merciful,viewed
under

God quantity, signifies notion


as

is contained under
"

the that is, merciful,

God

the

notion

merciful; viewed

in

the

other,
God

means,

God

comprehends merciful,that is,


notion

the notion is in
the
matter

comprehends
unless
sense we

in it the

merciful''This
and

again
a

all wrong,

notion interpret

containing
with the

different totally first


statement.

in the
if

second

as

compared
understood

Even

Hamilton
to

himself, he ought correctly


and
not to

have

stated
to

it

vocally, unequimatter

have

left the

reader the
same

put the
words

right by
different is
even

careful
senses. worse

of interpretation His

in

two

subsequent expositionof the sorites

for he different
one

gives

the

identically same
asserts,
and

premises
other in

twice

over

in

order, and

without the

that explanation,

is in it were

comprehension
stated in then
a
'

other

extension.
'

Suppose
struck

police
Robinson
K

court

that struck

Brown

Robinson,' and

132

INTENSION.

[CHAP.
learn

Brown.'

Should
'

we

not
'

be

surprisedto
used in
a

subsequently
in

that the verb


one case

struck
a

was

psychological sense

and
'

physical one
struck
'

in the

other, the real meaning


as a

being
Yet

that

Brown
then
not

Robinson struck
than
over,

very
on

disagreeable
the
to

fellow,'and
this would

Robinson
be
worse

Brown

head
state

'

?
a

for
an

Hamilton

syllogism or
of

sorites twice then


assume

with

unimportant change
takes
one

order, and
to

that

the reader other

ment state-

be

in extension

and

the

in intension. with
the

am

obliged,therefore,to
that Hamilton did
not

coincide

opinion
of

of

De

Morgan
The

really missed
understand is put in the
De what

the

point
was

the

question,in short,
about.

he

writing
the
:
"

whole

matter

clearest

lightby
the

few following
"

lines from

Morgan's Syllabus,pp. 62-3 recentlyintroduced


have have

The

who logicians
and

have

tion distinc-

of extension

comprehension,
and quantities, the is
some same.

altogethermissed imagined according


to

this
the W.

opposition
'

of

the

that Sir

quantities remain
Hamilton,
All X

Thus,
V is
X1
a

propositionof
is
a

prehension, com-

but extension. Aristotle


few In

'Some this
Laws

is all

proposition
he

of

the of

logicians have Thought


:
'

abandoned which drew


to

both the

and

the

from
genus

clear words the

of his dictum

the

is said view

be

part

of

species;
all

but

in

another
All

point
animal class

of

(aXAws)the
notion the
In the Here

species is part
in notion
all the all
:

of the
man

genus.'

is in man,

is in

animal,

in class.
man

first,

notion class

animal
man

part of the notion part


of the class

; in

second,
is the

the

animal.

oppositionof
The
same on

the

quantities."
is
more

view
the

stated fully

in De

Morgan's
Phil.

Third Trans.

Memoir

Syllogism (pp. 17-19;


On
tne

Camb.
I conclude

1858,

pp.

188-9).

whole,
so

that Hamilton's and

treatment

of the

subject is

doubtful

confusing

xiv.]
that it had

QUESTIONS
better
not

AND

ANSWERS.

133
course

be

studied

in

an

elementary

oi

reading.
De

Morgan,
about

in the

the

paper

just referred
the doctrine
as a

to,
of

gives

some

remarks

historyof
of

intension

and has

extension, and
'

speaks

Hamilton

logicianwho
the

recently contended
of the

for the revival,or


of extension

rather
and
as

full introduction,

distinction

comprehension.'

He

names correctly

the Port insist


very
on

Royal Logic
the

being

the the

first
use one

modern made other


of

work

to
'

distinction, though
But he
names

it is

not

extensive.'

only

work,
Ed.

the

Institutions

Philosophic^ describing logic was Logic,


and
in Watts' upon

of

J. Bouvier
distinction. extensive. which the
III. I

(3rd
De Not

Mans,

1830),
of the

as

this
not

Morgan's reading
to

modern

mention is
3 ;

familiar

in
I.

doctrine Section
find

frequently dwelt
Chap.
VI.

(see Part
10,

Chap.
in

Section

elsewhere),
1816

the
manual

matter

excellently explained
the American

the

brief

of In

logician, Levi formerly


in the is which the
in

Hedge
used

(pp.
in and

42-44).
Dublin
in the and

Murray's Manual, Glasgow,


the

much

subject is fully explained, Commentary


fact I
one

clearest
on

possiblemanner,
II. This

of of

John
best

Walker

Chapter
that
as

the

pieces
remarked

of

logical exposition
he had

know,

and

Walker

treated

because point fully,


to

he
of

regarded it
the
not

absolutelynecessary
pages, the
a

the

understanding
students the

subsequent
familiar with extension
that

which

were

often between
some

puzzling to
regret I
in this

distinction
term.

comprehension
must
matter

and

of

With

hold,
are

then,

the
and

pretensions of
unfounded.

Hamilton

mistaken
The

whole has of

subject
been

of

extension

and
much

comprehension
care

or

intension

investigatedwith
the

and

fundity pro-

thought by

American

logician,Professor

134

INTENSION.

[CH.
referred
to

Xiv.

C. This

S.

Peirce,
memoir
a

in the should

memoir be

already
studied

(see
who

p.

127).
to

by
of

those

wish

acquire
relations.

thorough

understanding

logical principles

and

6.

It

is

asserted of
a

by

some

logicians
must

that

the

predicate
in

proposition
the
your

be
is upon

interpreted regarded
this
in

intension

while Give
the

subject opinion
of the

extension.

point,
upon

and
recent

explain logical
this

bearing

question

controversies.

[c.]
the effect that
of
a

I should sition

answer

question
to

to

proposi-

being, conformably
of the
nature
can

the
an

opinion equation,
to

Condillac,
absurd
to

necessarily
suppose
or

of be

it

is
own

that

things
A

equated
in

their

qualities
the

circumstances.
of
a

proposition
class of

extension
with As

expresses

identity
or

thing

or

things

the
De

same

thing
says

class

under

another III. p.

designation. 529),
"

Tracy

(Ideologic,Vol.
ide'es
A the

Dans

tout

jugement, egales
an en

les deux

comparees in
of may

sont

necessairement
expresses member and

extension."
between

proposition
attributes

intension the
be
one

identity
those
of

the

other.

The
or

subject
the

pursued

in

my

Essay

on

Pure

Logic, passim
in
Dr. of
;

Logic of Quality apart


S. Mill's

from
I.

Quantity"

1864,
;

in

J.

Logic,
of

Book

Chapter Bailey
on

V.

and

Martineau's

review in his

Samuel

the

Theory

Reasoning,
Vol. II.

Essays

Philosophical

and

Theological, 1869,

CHAPTER

XV.

QUESTIONS

ON

INTENSION.

1.

'Christian,'
Arrange

'animal,'
these with the terms,
most

'Episcopalian,' (i)
in the order and the

'organised,'
of
sion, exten-

'

man.'

beginning
order of

extensive with

(2)
most

in

the

comprehension,

beginning

prehensive. com-

[L.]
Arrange
composer

2.

the

following
for the

in

the

same

manner:

"

General,
of

animal,
his
Third
own

pianoforte,
of Gaul.

Roman,

historian
De

campaigns,
Memoir,
pp.

conqueror
20,

(See

Morgan,

21.)
of extension
of

3.

Arrange
terms

in

order in

and

intension
II.

such
as are

of

the
names

given
of

Question
genera,

Chapter
and

the

subaltern,

and

species,

can

be

arranged

in

series.

4.
or

Analyse
of

the

following
and

terms

in

the
:

counter

quantities
government,

wholes

extension

intension

Man,

law,

triangle,
Show
of

vegetable^

[L.]
analysis
whole. of intensive

5.

that

the

an

equals

the

synthesis
6. Invent

an

extensive

[c.]
and it both in the

syllogism
in

in

Barbara,
forms.

state

extensive

and

the

intensive

[L-]

136

INTENSION.

[CH.

xv.

7. the

What conclusion

is

the

place

of

the

Major

and and

Minor

Terms intensive

in

of

(a)

an

extensive,
?

(b)

an

(comprehensive)

syllogism

8.

Can

the

distinction

of

extension

and

intension

be

made

to

apply

to

the

inductive

syllogism

[c.]

9.

Select

from

pp.

91

to

98

examples
and
state

of

the them

moods in the

Celarent,
intensive

Cesare,
form.

and

Camenes,

10.

What in

is

the and the

difference intent is

of
Is

meaning
the and
extent

of

genus

and notion

species always

extent

of
?

less

as

intent

greater,

vice

versa

11.

Interpret
:

the

following

propositions

in

extension

and

intension

"

libel

is

malicious himself

and

injurious
to

statement.

He

who

believes claims

be

always

right

in

his

opinion

infallibility.
to

It

is

impossible
that
can

be needs

and

not

to

be.

He

swim

not

despair

to

fly.

138
3.

HYPOTHETICAL

ARGUMENTS,
indeed
is to
:

ETC.

[CHAP.

Logic
he

is

worthy
be

of

being cultivated,
as

if Aristotle is not

regarded

infallible ; but

Logic, therefore,

is not

worthy [w.]

of

being
Clearly
'

cultivated.

false

hypothetical syllogism.
to

The
'

antecedent

is, if Aristotle is
in the minor

be

regarded
In the

as

infallible

denied ; this is
the
:
"

premise.
be

form categorical
as

pseudo-

argument
Those

might
who

stated

somewhat

follows

regard regard
we

Aristotle of

as

infallible
j

must

consider

logicworthy
We do
not

being
as

cultivated

Aristotle
not

infallible ;

Therefore,
There

do

consider of the

logic,"c. Major
Term.

is Illicit Process

4.

We
for

are

bound

to

set

apart
if the
us :

one

day

in

seven

religious duties,
is

fourth but
seven
we

ment commandare

obligatory
one

on

bound

to

set

apart
; and

day

in

for that

religious
fourth

duties

hence
is is
'

it appears

the

commandment
The
'

obligatory
fourth
'

on

us.

[w.]
is obligatory
; it is the

antecedent
;

if the

commandment

the consequent which


is

is

we

are

bound, "c.'

consequent
the

affirmed, so

that the argument


It the
are

involves
be

Fallacy of Affirming the


follows
:

Consequent.
on

may

put
mandment com-

as categorically

Those

whom
;
we

fourth

is

are obligatory are

bound, "c.
those The
on

bound,
fourth

"c.
mandment com-

therefore,we
is Undistributed the second

among

whom

the

obligatory. Middle,
the

is evidently that fallacy

of
in

pseudo-mood

being

figure.

XVI.]
5.

QUESTIONS

AND

ANSWERS.

139

(i)
had
events not

If

the

prophecies
written

of

the

Old

Testament
of

been
of

without of

knowledge exactly
and

the

the

time

Christ, (2) they could


;

correspond
had
not
:

with

them

(3) and

if

they
would
the

been

forged by Christians, (4) they preserved


are

be

acknowledged
and ledged acknow-

by

Jews

(5) they
the the
were

preserved
and

by exactly
therefore

Jews, (6)
events

they correspond
time written
were

with

of

the

of Christ without

they
of

(7) neither
events,

knowledge by
The

those

(8) nor

forged

Christians. argument
will be found
to

[\V.]
consist of
two

above

valid

destructive
Thus of

hypotheticalsyllogisms woven

together in
and

statement.

(i)
the is its

and

(2)

are

the

antecedent is its
The

consequent

first

syllogism; (6)
conclusion. its antecedent

negative
second

minor,

and

(7)

negative

syllogism has (3) and

(4) for
minor

and

consequent,
for

(5)

for

its

negative

premise,

and

(8)

its

clusion. con-

6.

In

how
of

many the

ways

can

you

state

the
'A

stance subwolf
'

categorical proposition
will the devour

let into
Isaac

the in

sheep-fold
his

the

sheep
of

Watts,
has well
to

Essay
out

on

Improvement
Thus,
the

the

Mind,
may be

pointed
the
same

the real

varietyof expression which


assertion.
as

given
the let
: a

lents equiva-

for
"

above wolf wolf


:

proposition, he
into
the

gives

following:
will
be voured de-

If

you

fold, the
the
not

sheep

The

will devour fold be

sheep, if
left shut

the

sheep-fold

be

left open

If the

carefully, the

140

HYPOTHETICAL

ARGUMENTS,

ETC.

[CHAP.
devoured
:

wolf

will devour

the

sheep sheep
A

The into

sheep
the

will be
open

by
is
out

the
no

wolf, if it find the


of

way

fold

There

defence
of

the
:

from

the wolf, unless


be

it be
among
are

kept
the

the

fold

slaughter will

made There

sheep, if
modes of

the wolf

can

get into the fold."


result

various in the

hypotheticallystatingthe

contained

original. categorical

7. In

strictlylogical point
to

of view,
to

ought
say in

it to

be
'

offensive

Captain
does

Jones
run

of

him

If

Captain Jones
to

away
'

battle, he

will live

fightanother
deeply, not
what

day

This

question touches
of

only
is much

the

soldierly tation repuimportant, Captain


his this forward be

Captain Jones, but,


of

more

the

import precise
as a

propositions.
in

It puts but

forward

Jones

running

away

battle,

it puts
would

only
to

as

the hypothesis,

result of It is

which
a

living
what

fight another
such life ;
mere a

day. things

quite

different be
taken form
a

matter,
to

meaning
common

proposition might
are a

imply

in

often

said in the
name

of

inuendo.

The

coupling though bring the


a

of

man's
were

with

disreputable
of

action, even
raises unless the
to

the action
"

expresslydenied
the assertion the mind
to

him,
at

question, Why
terms

was

made
of the

all ?

together in
were

hearer
'

If in company

gentleman
reason

suddenly
that is
a

remark

There
run

is

not

the least in his


last for

to
'

believe
;

Captain Jones point


to

did

away any every

action

here
an

blank that

denial

ot

ground
one

believing
construe to

assertion
a

effect; yet
denial
as

would of
a

such the

mal-a-propos

evidence
a

wish

raise

question,and
a

possibly start
is

rumour,

which Thus

would
we

presently take
see

tive disagreeableaffirma-

form.

that

the

logic of conversation

xvi.]

QUESTIONS
different in apparent
; not

AND

ANSWERS. from the in

141

widely
science

nature

strict
the

logic

of

that

it is

really different
we

end,

when

thoroughly analysed.
inaccurate,
seldom be
or sure even

But

constantly

deal
so

with that

illogical,
we can

untrustworthy
that
an

persons,

assertion
we

will

be

construed
to

and

repeated
is
were

in the

form

which
in the

originally gave
of

it.

There

too

much

truth
to
us

saying

Talleyrand,

that words

given

to

disguiseour

thoughts.

8.

If Brown foolish
Brown you

says

to

Jones,
no

'

Because
to

Robinson

is

have

need

be

does foolish,'

assert

categorically

that

Robinson

is

foolish
There
Brown the
can

?
no

be be of

doubt understood

that,in
to

the
make

logic of
an

common

life,
upon
not

would

imputation
remark
was

wisdom

Robinson,
the

if especially
course

the

explained by
in strict
'

previous
very

of

the

conversation.
the

But

logic it
'

seems

doubtful

whether

conjunction
'

because

should

be
'

from interpreted differently The fact of


on

if/

as

in is is

the
no no

last
reason,
reason

question.
"c.'
for you
'

Robinson the
A

being
of

foolish

Foolishness

part

Robinson
must

being
to

foolish.'
the

logical copula
existence it

not

be

understood
of

assert
or

physical

and
a

occurrence

its

subject

predicate ;

only

asserts

relation

between

them.

9.

If P
but

is

Q, and

is R, be

it follows

that that truth

P
no

is R such
of

suppose
as

it to

discovered
how is

thing

Q
P

exists,
is

the

the
?

conclusion,
I do
not
see

R, affected
is in deductive is
to

by

this

discovery

how

there The

logic any
the
effect

question
that

about

existence.

inference

if the

142

HYPOTHETICAL

ARGUMENTS,
and The

ETC.

[CHAP.

P propositions P

is

is R

are

true, then
of

the conclusion
may

is R

is
one

true.
or

non-existence

possibly
which
case

render the

both

premises materiallyfalse,in
but

reasoning vanishes,
for
; and

is

not
are

defective. logically
creatures

If

I argue,

instance,that satyrs
creatures

half

man are

and
very

half

goat

half
are

man

and hideous
or

half
; the

goat

hideous, therefore equallygood


course,

satyrs

very

reasoning
cannot,

is of

whether the
to

satyrs exist
conclusion is

not.

We

say that

materiallytrue,
truth
man can

if there
But

be if I and in
term

no

objects
argue

which
are

the material
creatures

apply.
and
half

that

satyrs

half

goat,
exist

such

creatures

exist in this
case

Thessaly, therefore, satyrs


non-existence truth of
cannot

Thessaly
would

; in

the

of the second the

middle

affect

the material

the

premise,
material

and, if this be
truth
I
assumes

held

false,we
De

affirm

of

the

conclusion.
to

ought

a'dd

that

Morgan
must

in

more

then

one

place
even

that

the middle
; thus

term

have

existence, or
"In

existence objective

he

says

(Syllabus, p. 67):
term

all
"c.

syllogisms
This is
one

the

existence

of the middle

is

datum"

of the

few

points

in which

it is

possible to
III.

suspect
The pp.
see

him

of unsoundness.
may refer
to
on

student and

Hamilton's

Lectures, Vol.
of 3.

454-5, also

p. 459,

'Sophisms

Unreal

Middle;'

Whately's Analytical Outline,"

10.

Lias

lies

above coal

red

sandstone

red

stone sand-

lies above coal.


This is books of

; therefore

lias lies above

[w.]
one

many

examples
which

to

be

found the

in

the

logic
form.

of

arguments
said that

simulate be solved

syllogistic

It is often

they can

syllogistically ;

xvi.]
but

QUESTIONS
cannot

AND

ANSWERS.

143

this certainly of the

be

done The Lias

by

the

ordinary rules
that
\ve

and

processes
even

syllogism.
'

most

could

get,

is that by substitution,
The

lies above is
It

what

lies above

coal.'
one,
one

fact is that the argument

a really

mathematical
to

involvingsimple equations.
which
has been

is

preciselysimilar
F. W.

thus treated
p.

by

Professor Mr.

Newman The
"

1869, (Miscellanies,
former
"

28),

and

J. J. Murphy.
the remarks latter,

of these

as logicians, quoted by

The

argument
Lead
:

Lead

is heavier

than

silver; Gold
than

is heavier
to

than

Gold therefore
as

is heavier
as

Silver,'1 brings
of

the
To

mind
say

conviction that its

direct

the
on

simplest
its
to

syllogisms.
reducible
to

validitydepends changes
of
:

being
least
you

is wholly unplausible : syllogism, you


have
as

for form

effect the
at
as

reduction,
hard
to

to

make

accept

the direct argument


are more

and

when

have cumbrous

got your
than

syllogisms, they
the

complicated
of Logic

and

argument
Mr.

as

it stands."

Murphy
the

(The
and

Relation

to

Language

Belfast

Natural

History
treats

1875)
thus
"

Philosophical Society, ijth February, simply as a question of quantity, argument


and silver

"Call
x, y,

the
z :

weights of gold, lead,


then
x
"

respectively

and

y
z

+
+

p
q q

y
X

p.
could
"

In

the

old
means

the foregoing conclusion logic,


of the

be

drawn

only by
That

following syllogism:

which less
:

is greater than

the greater is greater

than

the

The

weight weight

of
of

gold
lead

is greater

than

that

of

lead, and
:

the

greater than

that of

silver
than

Newman
as

has
to

inadvertentlywritten

Lead

is heavier

Gold,

which

is wrong

fact.

144

HYPOTHETICAL

ARGUMENTS,

ETC.

[CH. xvi.
that of

Therefore silver. Considered


as

the

weight

of

gold

is

greater

than

as

fact all this of stated.

course

is true, but which is here

considered stated
It
as

logic it is wrongly

That

the

canon. major premise is reallythe syllogistic a

is not

merely

general truth,like
a

the

truth

that
as

all matter
near a

tates, gravithe
a

but

logical principle,lying
the

to

first

principlesof
is
a

science

as

the axiom

that

part of

part

part of the whole."


have

We

only
the

to

assume

to
z

be
of

the
coal

height
above
the

of any

lias, y the
one

height
datum,
the head

of red and of be argue


are

sandstone, and
same

fixed
at

equations represent
that Reid doubtless
we are

argument

this section. added


was

It may that and We


we

rightin denying
because each the
to
same

when syllogistically

infer that

C
may

both

equal
; A

to

they
'

equal

to

other.
are

throw each

it into the other


are

form,
and
to

Things equal to things equal


other.'
is
The

equal

to

Care each

the same,
a

therefore

they
the

equal
The

But

this is

sive deluin

syllogism. obtaining
are

inference

reallyaccomplished
inferences inferences
to

major premise.
and
of

of

equality
Euclid

prior to
attempt
the
an

simpler
Herlinus

than

the

of

and logic,

the into it is

and has the

Dasypodius
been
more

throw

form syllogistic

ridiculed, because rightly

attempt
of the remarks Memoir

to
more

prove

simple
be found

and

self-evident

by

means

complex.
this
the

Some Second
Fourth

on on

point will
8, "c.

in De pp. 50,

Morgan's
51; his

Syllogism,1850,
p.
;

Memoir,
above Vol.

1860,

in

Mr.

Murphy's
note to

paper

quoted
129 of

; and

in Hallam's

remarkable

Section
to

III., Chapter

III., of his Introduction


p.

the

Literature

of Europe (istEd.

288

5th

Ed.

p.

in).

HO

HYPOTHETICAL

ARGUMENTS. the

[CHAP.
forms of

4.

Under

which
you

of

commonly

recognised
:

syllogism would

bring the following?


If A
If

is B,

Cis
E\$

D; F;

Cis

D,

Therefore, If A
5. Are
If
so,

is B, E

is F.

[c.]
of conversion?

capable hypothetical propositions


these
"

convert

(1)

If it has

thundered it has

it has

lightened.
not

(2) Unless
6.
correct

lightened it has

thundered.

Which
? A A

of

the

following arguments

are

logically

(1) (2)

is B, if it is C
is not A B
not

it is

not

C, therefore
as

it is

not

B,
B.

unless

it is C; is
not

it is but
as

not

C, it is

not

(3) If
(4)
7.
are

i?

B,
is D.

is B, it follows

that
A

is

not

B, if C is D

then

is

not

D,

for A

is B.

If the
as

Hypothetical
and

Modus
to

Ponem

and

Modus

Tollens

taken

corresponding
their

the

Categorical
to

First

and

Second
Barbara the

Figures,
and

typical forms
Modi
would

the

Moods
forms
to

Camestres,

what respectively,

other

of the

respectiveHypothetical
moods
of the

correspond

other

respectiveCategoricalFigures ?
is
true

[R.]
; if C

8.

If

is true, B
true.

; if B

is true, C
the

is

true

is

true, D
of

is

What

is the

effect upon

other
;

assertions C is

that (i) D supposing successively

is false

(2) that

false ;

(3) that

is false ;

(4) that

is false ?

9.

Analyse

the

following arguments

and

estimate

their

validity.

xvil.]

EXERCISES.

147

(1)

I shall

see

you

if you

do

not

go;

but

as

you

are

going (2)
The

I shall not.

Penge

convicts

were

after guilty of murder, if,


at

long continued
Staunton

neglect

their

hands,

Harriet

died. virtuous
if he alone
are

(3)

Since

the

happy,
he
must

he
be

must

be
if

virtuous
he

is

happy,

and

happy

is virtuous.
were no

(4) If

there there

dew

the weather

would

be

foul

but

is dew

; therefore

the weather

will be

fine.

[a]
(5)
If there
are

sharpersin
;

the company
are no

we

ought
the

not
pany com-

to

gamble

but

there
we

sharpers in
to

; therefore

ought
accused
I

gamble.
with

[E.]
that
Muraena
so.

(6)

"

I could

then
to

only
my the

be

of acting justice

contrary

law, if

maintained
was

purchased
But I

votes, and that


he

in doing justified
not

maintain

did

buy
to

the

votes,

therefore, I do

nothing contrary
Muraena,
c.

the

law."-

Cicero, Pro 1854,


10.

L.

iii.

(See Devey's Logic,

P-

I33-)
of
a

State

in the form of
the

disjunctive argument
verses

the matter 6
"

of

the First Book


n.

Samuel, chapter xvi. question


are

13.

Examine

whether

hypotheticaland
to

junctive disthe

arguments

reducible

the

forms

of

categorical syllogism.
12.

Dilemmatic

arguments

are

more

often fallacious than

not.

Why

is this ? the
:

[c.]
partiesto
"

13.

the logicalposition of Investigate from value

the
"

following colloquy
But

Clarissa
for my

Harlowe

Morden
say
2

// you

have

the

cousin

that

you
L

you

148

.HYPOTHETICAL

ARGUMENTS.

[CH.

xvn.

have,

you

must

needs

think
.

Lovelace
"

You

must

alknv

me,

sir,

to

interrupt
when I

you. I

If
have

have

the

value

say

have

hope,
that

sir,

say

that

value,

there

is

no

cause

for

if,
you that

as

you

pronounced
me

it

with

an

emphasis.
you

Morden

"Had

heard

out, rather

Mr.

Lovelace,
an

would
than

have

found doubt." This

my

if

was

if

of

inference

of

passage

is in

quoted
1877,

and

discussed

by
pp.

Professor

Groom

Robertson

Mind,

Vol.

II.

264

6.
"

CHAPTER

XVIII.

THE

QUANTIFICATION

OF

THE

PREDICATE.

i.

As

explained
and

in

the

preface,
this
book

have

thought
the
Dr. in

it well forms Thomson


most
cases

to

discuss

illustrate
and

in

of

exercises,

of

logical expression
and with
of

inference
These

recognised correspond
under
to

by

Sir
what

W. De

Hamilton.

Morgan

represented
also

different
some

systems
the
pressions ex-

notation.

They
and

correspond
current

of

arguments

in far

ordinary
better the
to

life.

Although
the
to

in

scientific will himself


to

point
of- the

of

view

it is

eliminate is

logical
make

wisp

'some/
with

yet
the

student into,

obliged
it

acquainted
him.
that

pitfalls

which

is

likely
It
account

lead assumed

is

the

reader of
the in

has

studied

the

brief in is the
commended re-

of of the

the

Quantification
Elementary

Predicate

given
and he
or

22nd

Lessons

Logic,

to

read of The
a more

either Hamilton's

Thomson's

Outline, Logic

else

Bo

wen's

account

(Bowen's
and task.

Logic,

Chapter
own

VIII.).
is

study
arduous the
as

of

De and

Morgan's thorny
of Dr.

Hamilton's

writings
The

following
by

are

eight
described

kinds

propositions
Thomson.

cognised re-

Hamilton,

by

Sign.
U I A

Affirmative.
All Some All

Negative.
No

Sign.
E

is all
X is
some

Y.
some

is X

Y. is
some

Y.

Some
No

not

some

Y.

is

Y. Y.

is X

Y.
no

i\ O

Some

is all

Some

is

Y.

ISO
2.

QUANTIFIED
Indicate
and
:
"

PREDICATE.

[CHAP.
the tity quansitions propo-

by
the

the

technical
of the

symbols

quality

following

(1)

All

primary
vital
of do.

forces

are

attractive. under

(2)

All

actions and

come

the

law

habit,

none

but

vital

actions

(3)

The

best
is that

part
which

of

every

man's

education

he

gives

himself. horns. the


most

(4) Only

ungulate
readers

animals
are

have

^5) (6)

Mere

very

often

idle of
Most

human

beings. vegetables
are

water-breathing

flowerless.
"

[P.]

(1)

Is

clearlya

universal

affirmative

(A).
A
;

(2) As regards its first


addition,
'all
not

part
but

is also

but

the

exclusive

'None

vital actions
do

do,'
The
'

means

that,
parts

vital

actions

not."

two

together yield a propositionin U,


are

all vital actions

all that 'best

come

under
a

the

law

of

habit.'
a

(3)

The

part,'being
so

superlative,is
'

singular

term, and
Hence

is the

predicate
an

that part

which, "c.'
U.
gulate uncon-

the

proposition is
have

identityin
to
'

(4) An

exclusive

proposition equivalent
no

all not is the

animals

horns,' which

trapositive of,
animals
are a

and

equivalent

to,

'all

horned

ungulate.'
great
I.
many
mere

(5) (6)

Means

that

readers

are,

"c., and

is in the form
Is also
a

affirmative particular

proposition.

XVIIL]
3.

QUESTIONS
Does
not

AND

ANSWERS.

151

the
is to
'

proposition Y
say, does P
not

of Thomson
'some

imply
is all

O,

that

Q'

imply
This the
seems

that

some

is not

Q
if

'

very of

because plausible, there

some room

makes left in
must

up

whole

Q,
more

is,so
the

to

say,

no

(7s

sphere
tore

for any
not

Ps,

remainder

of which

therethe fact

be the

Q.

This P' may

argument,

however,
may

overlooks

chat

'some there

in

question
be
no

possiblybe
excluded

the whole
from

of

P,

so

that

remainder

Q.

4.

Is
true

the ?

proposition

'

Some

men

are

animals

'

[E.]
true
or

The the
we

proposition is
we

untrue

materially according
word be
states
some.

to

sense

put
mean

upon
'

this troublesome
or more

If position pro-

take

it

to

one

it may
course

all,'the
less than

is
known We necessary
to must

true

in
one.

fact, but

of

is

every

carefully distinguish between

the and

strict
that

and

logical interpretationof
De

'

some/

which

applies in colloquy.
"

Morgan
the

says

(Formal Logic,p. 4),


of
a

In

common

conversation
the

affirmation

part is

meant

to

imply

denial
are

of

the

remainder.

Thus,
to

by

'

some

of

the
'

apples
are

ripe,'it
ripe.'
this
"

is

always
is the
are no

intended

signifythat
in

some

not

There
use

in providing difficulty word

formal
the
one.

logic for
Thus
'

of which

by stating explicitly
into O.

two

propositions
some

colloquiallymerged
are

of

the

apples

ripe

'

is

really I

5.

What

results
'some

would
As
are

follow j5s ""s


'

if
as
'

we

were

to

terpret in-

implying

that

'

some

other

As

are

not

X52

QUANTIFIED
The

PREDICATE.

[CHAP
is in the
I is form
true

proposition
the

'

some

As

are

Us

'

I, and
it A
be is the

according to
true

table

of

opposition (p. 31)


which
"s.'
at

; but

A
some

is the

contradictoryof O,
As
never are

would

form

of

'

other A could the

not true
own

Under

such

cumstance cir-

be

all,because

its truth is

would

involve

truth

of its

which contradictory,

absurd.

Briefly
"

If A

is true, I is
of

true

; and

if I

implies O,

then

implies the
Several

truth

its

own

contradictoryO.
to

have logicians W.

come

grief over
in
none,

this troublesome that all


same
'
'

word, notably Sir


is

Hamilton, who
of all

holding
throws the

some

formally
into in

exclusive confusion.

and

logical
great

systems
mistake

Woolley

commits

saying (p. 77), "In affirmativeand


some

every

particularproposition,
each A

therefore, the
other
:

negative mutually imply


then
some

if

only

is B,

is

not

B, and

vice versa"

6.

Explain
'

the Some
""

precise
Xs of ?
are

meaning
not

of
some

the
Fs

sition propo'

(the
its its

proposition importance.
This is
one

Thomson).
Give
your

What

is

contradictory

opinion

of

of

the

eight
of

forms

of

proposition

which

Hamilton, in
of
'

pursuance

the

thoroughgoing quantification
into his is
to

the

predicate, introduced
Y'
means
'

system.
say,

Now,
'

if
'

some

any

some

Y,' that
be any differ from O
of the

if the

some

is undetermined this which


But

and

may
not

where
'

in the
some

sphere
is
not

of

Y, F,'

does proposition

any

is the if
'

proposition
Y' is
a

old

Aristotelian
class
a mere

Logic.
F, less
empty

some

determinate the

part of the

than

the

whole, then

propositionbecomes

154

QUANTIFIED
chief interest of this
the
a

PREDICATE.

[CHAP.
w

The

proposition
value of

arises

from

its

important bearing Logic, and


upon
means

upon

Hamilton's Hamilton

System of
insisted

his

position as

logician.

the the

of thoroughgoingquantification

the

which predicate,

recognitionand
which
the Thus

employment
of the

of all the

eight propositions
to

introduction
was

quantifiedpredicate
key-stone
But be

renders

conceivable.
arch

the

put

into and this

the De

of the
seem

Aristotelic
to
me

logic.
to

if,as

Thomson

Morgan

have
and

conclusivelyshown,
absurd,
the

proposition,w,
and
De all the

is

valueless

key-stone
does
not

crumbles overtake
are

arch

collapses.
because of

The

same

ruin

Morgan's system,
the
same as

his

eight propositions
;
nor

not

those

Hamilton views
of

does

it and

affect

in any

appreciable degree
who

the

Thomson

George Bentham,
De

did

not

insist upon

the

thoroughgoing
inherent
the

quantification of the predicate.

Morgan 4),
of

has

admirably expressed
He says

the
on

guity ambi-

of this word.

(FifthMemoir
some

Syllogism,
very

1863,
ask

p.

"'He of

has
'

got
he has and

apples' is
got
some

clear:
a

the

meaning
Some
he has

not

apples,' in
will be
one

company

educated will
no

men,

the he

apples
may

those

of
;

discord.
some

think

that
at

have that
he

apple
not

that

apple
or

all ;

some

has

got

some

particular apples
The

species of apples."
be

subject

of

particular propositionsmay
p. of 1^2,

pursued
Shedden's Thomson's

in

Spalding'sLogic, 1857,
Logic; Hughling's Logic
Outline,
fifth

and

elsewhere;

Names,
77
;

1869, p.
Hamilton's

31

edition, section
279
;

Lectures,
De

vol.

iv.,pp.
Mr.

254,

Devey,
is his

1854,

pp.

90"94;

Morgan,

1863, Fifth Memoir.


A.

J.

Ellis in

exact particularly

in

his

treatment

of

this

question

articles

in

the

Educational

Times,

1878.

XVIIL]
7.

QUESTIONS

AND

ANSWERS.

155
'

Solly
B is

says

(p. 73)
some

"

If the is
not
some

premises
B' the

are

some

A,

reason

may
some

logicallydeduce
But

that

C
in

is
one

not

A, four
in

this

conclusion forms."

is not Is

of

the valid
in

legitimate
the mood
The

the

argument
and
if
so,

quantified syllogism,
?
follow

what

propositionsare
Some Some
Some B

as

"

is is

some

riot

(any B}
some

O
w

C is

not

A
in

The

middle
as

term terms

is distributed
of
w

once

the

minor
can

premise,
no

and,

both One
of

are

there particular, and negative, is


as so

be

illicit

process.

premise
the

is

is the conclusion.
the

No

rule

syllogism
It appears

broken, and
I O
w

argument
sixth mood

is
of

therefore the first

valid.

in

the

figure of Thomson's

table.

8.

Which

of

the
make

following conjunctions
valid you

of the

positions procase

syllogisms
as

In

of those
reasons

which
for
so

regard

invalid, give

your

treating them.

The

pseudo-mood
of the

in the first
the

figure gives illicit


E does

process

major term,

because

conclusion
A

butes distrinot. 77,

its The

predicate, and
I

the
77

major
a

premise

pseudo-mood

draws

negative conclusion,

156
from
two

QUANTIFIED
affirmative Table
of It is
an

PREDICATE.

[CHAP.
in

premises, but
Modes,
obvious

is

by oversight given
i,

Thomson's

figure

mood

xii.,second
I E
77.

negative form. of
In p.
error

misprint for
103,

line (Out-

the

Laws
as

of Thought, section

5th ed.,
Lessons

p. in

188.)
Logic,
the
was

the
1

table

reprinted in
same

the

Elementary
page
as

88
was

the (accidentally

in

Thomson),
It

corrected
out

in

the Mr.

fifth and
A.

later editions.

pointed
E Y

to

me

by

J. Ellis.

is valid in the second

first

figure.
breaks
no

In the

figure U
of

O
w,

"o

rule, but
are

the
some

clusion con-

instead

being
in the

(some

Xs

not

Zs)
are

might
any

have

been

stronger
U
4,

form
and

O U O

(some
w w

Xs

not

Zs).
U

The

moods

appear
not.

in
The

Thomson's mood
In
t\

table, column
O third term,
is valid.

though
is

does

the

figure A
since
the

to

subject
O

to

illicitprocess its

of

the

major
which

conclusion

distributes A
same

cate, predimajor

is the Y E

undistributed O is
not

predicate of
to

in the

premise.
because

subject
and

the

objection,
this last case,
E
;

distributes

its

predicate ; but, might


have

in

the conclusion Y E E
appears

is weakened, in O

been mood

hence third

Thomson's does
not

table, tenth
appear.

of

and figure,

9.

In

what

mood

is

the
est

following argument aequiangulum


trilaterum
;
;
omne

Aliquod

trilaterum
est

triangulum

(omne)
est

ergo,

ali-

quod triangulum
The first

aequiangulum
figureis
I
;
as an

premise,
a

'

some

trilateral

equiangular
second,
'

is plainly figure,'

proposition
U
; the

in

the

all

trianglesare
universal

all

trilateral

is figures,'

plainly a doubly
'

in proposition

conclusion,

some

triangular

XVIII.]

QUESTIONS

AND

ANSWERS.

157

figure is equiangular,'is
is distributed
there in the

in I.

The

middle

term,
not

trilaterum,

minor

premise, though
nor

in the the

major

is

no

illicit process, the

other

breach

of

syllogistic
the mood mood

rules, so
I U I of

that the

argument

is
It

valid
appears

syllogism in
as

first column

figure.

the

twelfth

in

the

first

of Thomson's
New

table
pp.

of moods.

See,

however,
this

Baynes'

Analytic, 1850,

126-7, whence

example
Does

is taken.

10.

the
mood

following argument
of
man

fall into

any

valid

the

syllogism lawyer
any

?
;

Some

is all

Any therefore, Some


all (i.e. the
rest

lawyer
man

is not

stone

(i.e.lawyer)

is not

any

stone

are

stone).
by
IV. De

This
case

example
Hamilton's Vol.

is taken mood
p.
term

Morgan
as

(1863,

p.

10)

as

of

b,

stated
term

in his Lectures

on

Logic,

IV.,
a

287,

thus, "A

coparti-totally

inclusive, and

totally co-exclusive,
of
each

of
was

third, are
called

co-exclusive parti-totally
De the
not

other."

It

by
to
are

Morgan

the

'Gorgon
it

Syllogism,' alluding,I
produces
the upon all mankind E Y
o",

presume, who

petrifyingeffect
lawyers.
not

It is

plainlyin
in of The later Thomson's E Y

mood

and

though
mood is that
from

it does
a

appear form

table, may
the of the

be

considered

weakened
first

O,

seventh matter,

negative
however,
to

of the

figure.
in his

point
of the

Hamilton,
the
some.

writings,proposed
mark of IV.
some

depart

Aristotelian
As

sense

particular quantity
p.

stated wished most,


"

in
to

his Lectures, Vol. introduce is that


"

281, the view


should
mean

which
'

he
at

some

some

only,
of
' '

some

not

all.'

But, if
of

we

apply

this

meaning

some

to

the

conclusion

the

158

QUANTIFIED

PREDICATE.

[CH.
result
are

xvm.

Gorgon

Syllogism,
are

it
not

produces

the

ridiculous

that,
stone.

though lawyers
De

stone, all tht rest of mankind


correct,
'

Morgan

is

unquestionably
to

and

this

Gorgon
tested

Syllogism brings
and matured The and and
1861 It
'

ruin

Hamilton's

long adequately

system.
of the

particulars

discussion about found

between this in

De

Morgan Syllogism,
of

Professor

Spencer
matters,

Baynes
may be

Gorgon
the

kindred and
1862

Athencenm

and that

elsewhere.
De

is curious

Morgan

states

the

Gorgon

gism Syllo1861,
p.
10

differentlyin
p.

the

Atheimum

of
on

2nd the

November,

582,
the

and

in

his

Fifth is
not

Memoir material
to

Syllogism,
issue.

but

difference

the

final

"

ii.

The has

month

of

May
or

has

'

no

R
:

'

in

its

name

nor

June, July,
are

August

all
and

the

hottest

months

May,
all their the

June,
hottest

July,
months

August:
without

therefore,
'

are

an

R'

in

names."

This student
he

is
to

Whately's
Book

example

No.

117,
i,

and

as

he
of

refers

the

IV., Chap.
it
as an

I.,"

which

treats

induction,
It would mood versal unithat

evidently regards
been referred minor

Inductive
to

Syllogism.
the Thomsonian
as a

have
E

by Hamilton

E,

the

premise being
There
can

treated
no

doubly

proposition.
the minor is
or

be

doubt, however,
:

really disjunctive, thus June,


or

hottest The

month

is is
a

either

May,

July,

or

August.
four

major

compound
'

sentence,
no

comprising
its name,'
Lesson
'

separate
has
no

propositions,
R,' "c.

May

has

in

June
p.

(See

Elementary

Lessons,

XXV.,

215.)

CHAPTER

XIX.

EXERCISES

ON

THE

QUANTIFICATION

OF

THE

PREDICATE.

i.

Express
and

carefully, predicates, symbol tending

in the in

full

logical form,

with

quantified
;

subjects
the Thorn

following
each
case :

propositions

assign

sonian

(1)

Thoughts
wonders.

to

ambition,

they

do

plot

unlikely

(2)

Fools Heaven

are

more

hard
to

to

conquer
soon

than
or

persuade.
late,
fate.

(3)

has

all

allotted,

Some

happy
is

revolution

of

their

(4)

Justice
This Man is is them.

expediency.
the
man

(5) (6)

certainly
the

saw

yesterday.
ears

only

animal

with

that

cannot

move

(7)

Wisdom

is the

habitual

employment
in

of

patient

and

comprehensive
and
remote
means

understanding
to

combining
the

various
of

promote

happiness

mankind.

(8)

It

is

among

plants

that

we

must

place

all

the

Dia-

tomaceae.

(9) (10)

When

the

age
at

is in

the

wit

is

out.

Every
Some
Some

man

forty

is either
are

fool fools

or

physician. physicians.

(n)
(12)
(13)

men

at

forty forty moi,

neither both Louis

nor

men

at

are

fools the

and

physicians.
said.

L'Etat

c'est

as

Fourteenth

160

QUANTIFIED

PREDICATE.

[CHAP.
made of

(14)

There

are we

no

coins
a

excepting

those of

metal,

if
or

overlook

few

composed

porcelain,glass, (3p6x""v.
a

leather. said
which

(15)
(16)

Antisthenes
All animals
not

8e7v Krao-Qai
have have
a

vow

rj

larguage
have

have
a

voice, but

all which

voice
among

language.
has
a

(17.)The

elephant

alone

mammals

boscis. pro-

8)

Prudence proper of time

is that
to

virtue

by which

we

discern

what

is

be

done

under

the various

circumstances

and

place.

(19) (20) (21)

Whatever There A
are

is,is right.
arguments
an

and

arguments.
and
a

dispute is
is
a

oral controversy,

controversy

written beth

dispute.
of

(22) (23)

There

workys
Roman
was

actyf lyf othere

gostly othere
of
us a

bodily.
The

only

who
the

gave Roman

us

summary

Aristotle summary

only

who

gave

of

Euclid.
that the last moment the
to
same.

(24) Zenobia
and of

declared her

of

her

reign

life should
some

be

(25)

As

it asketh

knowledge
so

demand
some

question
sense

not

impertinent,
a

it

requireth

to

make

wish

not

absurd.
of dark
men

(26)

Mankind To say another

consists that
way Mr. of

and
was

fair excited

men.

(27)

Raffles

was

only

saying that
educated
are men

it

was

evening.
not
men.

(28) Though

all well

are

discoverers.

all discoverers

well for

educated
gay

(29)

No and

man

is esteemed
women.

garments

but

by

fools

62

QUANTIFIED
"We

PREDICATE.

[CHAP.
'all
a

4.

have

been
'

assured Y is
some

that

is

some

Y'

is

contradicted
cannot

by
made

all

X,'

proposition which being


p.

be

good

except

by

some

declared

not

all."

(De Morgan,
this

Third

Memoir,

1858,

24.)

gate Investi-

point.
'
'

5. Take and
stone

stone

and

'

solid

'

as

subject and
in

predicate,
w,

convince is
not

yourself
some

that

the

proposition
be

'some

solid,' cannot
the

contradicted
rj,

by

any the

propositions of
same

forms

U, A, I, Y, E, O,

having

subject
6.

and

predicate.
the various

Write
are

out

judgments, including U
to

and
puns

Y,
are

which

logicallyopposed
State kind in the of and
case

the

judgment,
judgment
which
of

'

No thus

admissible.' what is the

of each

formed
to

opposition
also the

in

it stands

the

original judgment,
each
7.

kind

opposition between

pair
"The

of

the

new

judgments.
'

[c.]
are some

judgment,
because
a

No

birds the

animals,' is only,
and

never

actuallymade
the power
8. If Draw of

it has

semblance this
statement.

not

denial."
from

Examine the

[P.]

inferences
was

following:
then

Sir Thomas unless

imbecile,
was

Oliver

was

right:
not

and

Sir Thomas

imbecile, Oliver

was

wrong.

[P.]
9. Examine

the the

following arguments
nature

; in those

which

are

false
those

point out
which
are

and

name

of

the
usual

fallacy;

arrange

valid

syllogisms in. the


the

form, and

give

the

symbolic descriptionof

mood.

(i) All

the householders
are

in the

kingdom, except
all the who who
male pay pay

women,

and legally electors,

householders

are

precisely those
that
all
men

men

poor-rates
poor-rates

;
are

it

follows electors.

EXERCISES.

163
moon comes

(2)

All

the earth

times and

when

the
sun,
are

between
cases

the solar time


no

the
nth

the

sole

of such
a

eclipse;the eclipseof (3)


All
men are are

of nth

February
of

is not

therefore, the
the

February

will exhibit

sun.

[THOMSON.]
all mortals
are
men

mortals, and
sure are

all those
are

who those

to
sure

die ;
to

therefore,all

all

who

die. Arthur person

(4) The
he

Claimant is Castro

is

unquestionably
who is the
same

Orton
as

for

Arthur

Orton.

10.

Which

of

the E Y

following
O,
Y A

moods

are

and legitimate, I Y

in what
A E E

:" figures

A, Y

Y,

I,

Y,

[M.]
the
Figure I.

n.

Examine

of validity

the
II.

followingmoods
Figure
III.

Figure

UAU YOO

AAA

YEE OYO

AYY

[c.]
and mine deterI U

12.

Exemplify
in how
many

any

of

the

following moods,
is valid
:

figures each

U,

I,

YU

Y, TjUrj,

coUo".

CHAPTER

XX.

EXAMPLES

OF

ARGUMENTS

AND

FALLACIES.

THIS

chapter
and

contains Fallacies

large

collection
from many

of

examples
They

of

Arguments
form

collected

sources.

additional
are

illustrations
in the

and

exercises
The

to

supplement
student
is
to

what

given
in
or

the
case

previous
of

chapters.
example
In the

determine
a

each

whether
former
case

it contains he
is
to

valid

fallacious

argument.
into
of
a

throw technical

the

example

regular
form,
the

form,
whether

and

assign
mood of

the the

description syllogism,
"c. In
some

that
of

categorical syllogism,
or

or

hypothetical
t\vo
or more

or

disjunctive syllogisms,
will be
plicated com-

examples
forms
must

two

or

more

different

of

reasoning,
course

together.
exhibited When the

They

of

be

analysed

and

separately.
the existence
must

of

fallacy
to

or

paralogism
reduce this

is
to

suspected,
a

student
or

endeavour
of

distinct
the

paralogism pseudo-mood
however,
Aristotelian
fallacies

breach
or

the

syllogistic rules, exhibiting


of

pseudo-form
may
as

reasoning.
the

In

many

cases,

the

fallacy
text-books

be

of

kinds
or

described
Material.

in

the

Semi-logical
in
for the

These Lessons reference It has full

have
XX.

been

explained XXI.),
but of

Elementary
of below.

(Nos.
a

and list of found

convenience is

simple
been

the

kinds

Fallacies
to

given
in

not

practicable
of the

undertake
of

this
The

book student

exemplification

subject

Fallacies.

is

CH.

xx.]

ARGUMENTS

AND

FALLACIES. Lessons

165 named,
"

therefore
any De
as

referred

to

the

Elementary
the

or

to

of

the

followingwritingson
Formal

subject :

Morgan's
accurate

Logic, Chapter XIIL,

as

amusing
III.,

it is

and and

instructive; Whately's Logic, Book


most

perhaps
text-book

the

best

interesting part
edition

of this celebrated
on

; Edward

Poste's

of Aristotle

Fallacies.

Paralogisms.
1.

Four

Terms.

Breach Middle. of

of

Rule
Breach
or

I. of Rule Term. III.

2.

Undistributed Illicit Process


Rule IV.

3.

Major

Minor

Breach

of

4.

Negative Negative
vice

Premises. Conclusion
Breach VII.

Breach from of and

of

Rule

V.

5.

affirmative
Rule VIII. VI.
can

premises,

and

versa.

Breaches
one or

of

Rules

be

resolved

into

other

of the above.

Semi-logicalFallacies.
1.

Material
i.

Fallacies.

Equivocation. Amphibology. Composition.


Division.

Accident. Converse
Irrelevant

2.

2.

Fallacy of
Conclusion.

Accident.

3.
4.

3.
4.

Petitio
Non False

Principii.
Cause.

5. Accent.
6.

5.
of

Sequitur.
Questions.

Figure

Speech.

6. 7.

Many

1.

France,

having

warm

climate, is
in

wine-producing

country.
2,

[E.]
describes
be

Livy
to

prodigies

his

history;

therefore

he

is

never

believed.

[E-]

166

ARGUMENTS

AND

FALLACIES.

[CHAP.

3.

All

the

metals

conduct do
so,

heat and

and

electricity;for
are

iron, lead, and

copper

they
in

(all)metals.

[*]
4.

charitable he

man

has what
of

no

merit

relieving distress,
himself.

because
5. you have
or

merely
is the
fraud

does result
or

is
all

pleasing to
this

[E.]
day
read

What hear led of


an

teaching?
some never one

Every
who
to

forgery, by
he

might

innocent

if life,

had

learned

write.
6.

[E.]
use

The

of
causes

ardent

should spirits and

be

prohibited by law,
which it is
one

seeing that
the chief
7.
some

it

misery
law
to
are

crime,

of

ends
men

of

prevent.
fit to be

[E.]
ministers therefore
of

Pious

only
men

religion ;
of

ignorant
be

are

pious;

ministers

religionmay
8. No

ignorant men.
should ba allowed all
for the

[L.]
sake
is may
an

punishment
may
not
come

of the

good
and of
we

that
are

of it ; for

punishment good
the

evil,
come

in doing justified

evil that

it.
9. We know
we

[E.]
that God exists
because

Bible

tells
must

us

so

; and

know

that whatever Divine

the

Bible

affirms

be

true
10.

because The
or

it is of end
the

origin.
is either the the of
to

[E.]
protection
It of

of

punishment
therefore
of

society

reformation

individual.

Capital
does
to not

punishment ought
in fact prevent while society,
1 1
.

be

abolished.
so

crimes
on

violence, and
alternative

fails

protect

the other
is

it is absurd.
we

[E.]
look for

The

glass
is

falling ;

therefore

may

rain.
12.

[E.]
This
a

dangerous
avow

doctrine,
for

for

we

find

it

held up-

by
13.

men

who

their disbelief demand

in Revelation.
is

If there

is

education, compulsion

unnecessary.

[E.]

xx.]
14

EXAMPLES.

167
are

Actions
a

that

benefit
to

mankind

virtuous;

therefore

it is

virtuous

action
is
a

till the

ground.
institution
;

15. wrong 16.

Slavery
to

natural

therefore

it

is

abolish fool

it. is
fit for

No

high place;

John

is

no

fool;

therefore
17. He

John

is fit for
a

high place.
for
no

[E.]
Mahometan
holds

is not

Mahometan,

these
1

opinions.
Mind active.
He these If
must

[E.]
; matter

8.

is active

is

not

mind;
for

therefore

matter

is

not

[E.]
be
a

19. hold
20.

Mahometan,

all

Mahometans

opinions.
we are

[E.]
believe
of them

to

philosophers, knowledge
tell that
us we

is

impossible,for nothing
of mind.
21.

one

set

that
can

we

can

know

of

matter,

and

another

know

nothing

[o.]
age the is wiser than of
our

Old

youth;

therefore

we

must

be

guided by
22.

decisions assassins
to

ancestors.
not to

[o.]
be

Political
act

ought

punished,

for

they
23.

according
education

their consciences.

[o.]
is unnecessary
j

If

is

popular, compulsion
will
not

if

unpopular, compulsion
24.

be

tolerated.

[o.]
when

Nations
for

are

justified in
people
has
a

revolting right
to

badly
ment. govern-

governed,

every

good

[E.]
25. These
to two

figures are
other.

equal

to

the

same

figure,and

therefore 26.

each

Opium

produces sleep,

for

it possesses

soporific

virtue.
27. Wealth
to

[E.]
is in obstacles

proportion
:

to

value, value
is in

to

efforts,
to

efforts

ergo,

wealth

proportion

obstacles.

68 28.

ARGUMENTS

AND

FALLACIES.

[CHAP.
a

When lost.
A
manor

Croesus

has

the

Halys crossed,
this

mighty

army

will be
29. baron 30. teach 3
1
.

cannot
now

begin
founded.
non

at

day,

because

court-

cannot

be

Poeta

nascitur,
of

fit ;
verses

how !

absurd

it is then

to

the Aio

making
te

Latin

^Eacida, Romanes
rule has

vincere

posse.
a

32.

Every
has

exceptions ;
therefore

this is
are

rule, and
some

fore therethat

exceptions; exceptions.
that in

there

rules

have
33.

no

[E.]
mind
; the

All

perceives is

existence the

of

objects
ot

consists

being

perceived ;
on

therefore mind.

existence

depends objects necessarily


34. for Some

[E.]
;

objects

of

great
of

beauty merely please the eye


great
to

instance, many
answer no

flowers

beauty,

and

accordingly

they
35.
as a

purpose is
a

but

the sight. gratify


of

[H.]
and,
those

miracle and

violation

the

laws

of

nature;

firm

unalterable
a

experience

has

established the very


nature

laws, the
the

proof against
as

miracle, from
argument

of
can

fact, is

entire

as

any

from

experience
close
force

possibly be.
36. The

[E.]
imagination
The the
and

affections the
of
more

have

union
to

together
latter.
are

vivacity of
prospect
us

former
any

gives

the
we

Hence

pleasure with
any

which

acquainted
we

affects
own

than
of

other
nature

pleasure
we are

which

may

superior, but
consists and
of

whose

wholly ignorant.
37. for Common salt
a

[E.]
metal

and

metalloid,
one

it consists
the

of

sodium
a

chlorine, of which

is

metal, and

other

metalloid.

[E.]
good.
peas
; A

38.
man

"The

truth

is,that luxury produces much


for
a

gives half-a-gumea gardening


does

dish
"
"

of

green

how

much

this occasion?

(Dr. Johnson.)

[o.

70

ARGUMENTS

AND

FALLACIES.

[CHAP.
it

48. If
would
be

the
about

earth

were

of
as

equal density throughout,


as

2\

times

dense
the

water must

but be

it is about
of

5| times as density.
49. external space
;

dense

; therefore

earth

unequal

Whatever
to

is conditioned world world

must

depend

on

some

cause

itself: this this

is conditioned

by
some

time

and
cause

therefore
to

depends
that

upon

external

itself.

"

[E.]
is

50.

It

sometimes
none

happens
but
a

an

electrical
are

current

excited, where
into
an

magnetic

forces

called directly
is excited
to
or

play
it. The

; for

such

current,

in certain
a

cases,

in

electric non-conductor

by moving

magnet

away

from 51.

[R.]
Quaker
upon
asserts

that

if

men

were

true

Christians,
would be that
no a

and
need

acted of

their

there religious principles, he

armies. is

Hence

draws

the

conclusion

militaryforce
52. is natural 53.

useless, and

being useless, pernicious.


least

Detention

implies at
be have

possession;
without
what

for

detention

possession.
can

Nothing
must

conceived

extension has

what be

is extended

parts

and

parts

may

destroyed.
54.

[o.]
an

Had

armistice have been


not

been
been

beneficial
upon

to

France those

and
:
an

Germany,
but such

it would has
not

agreed
; it is

by

powers

the
have

case

plain therefore
to

that either

armistice the

would

been

advantageous
Adam

of

belligerents.
55.

[o.]
of of
nature
as soon as was

By the

law

created it
was

he

was

governor

mankind,
governor of laws

for
his

by rightof

nature

due

to

Adam
When

to
men

be

posterity.
useless
;

[o.]
men are

56.

are

pure,

are

when

corrupt, laws
57.
There

are are

broken.
many

arguments

which

we

recognise

as

xx.]
valid which
the
man

EXAMPLES.

171

it is

impossible to
be the law law

express
as
a

in

form syllogistic

therefore

syllogism is
can
a

valueless
to

test

of truth,
law

[o.]
a

58.

No

himself;
an

for

implies obeys

superior who
but the 59. them
own same

gives
person

and be

inferior who
ruler

it ;

cannot

both

and
women

subject, [o.]
to

It the

is

injusticeto
for

the the

intellect

of
many

refuse
as our

suffrage ;
or

reigns
have

of

queens, for

Elizabeth

Anne,

been

famous

literary

productions.
60.
must to

[o.]
every
man

To be

allow

unbounded

freedom

of

speech

advantageous
interests
of

to

the State, for it is highlyconducive

the

the

community

that

each

individual his ments, senti-

should

enjoy
Your of

an

unlimited

libertyof expressing
will
for

[o.]
6
course 1. sorrow

is

and fruitless,

not

change
very
reason

the
I

destiny. Very true,


individuals the
a

and

that

am

sorry. 62. Because


some

[o.]
have in

their

very

hood child-

advanced
of
reason,

beyond only
makes needs

youthful giddiness
proper

and

debility
to

it

system

of

education

make

other Haste
man

young

people wise
Waste,
and

beyond
Waste

their years.
makes

[o.]
fore there-

63.
a

Want;

never

loses
at

by delay.
war price is desirable, an

[o.]
is
any
an

64. If peace
as war

any

evil

; and

is

confessedly

evil,peace

at

price

sirable, is de-

[o.]
65.
'

The is of

two

propositions, 'Aristotle
are

is

and, living,'

Aristotle both

dead,'
them

both
or

intelligible propositions; they


both be is

are

true

of them either
true

false, because
or

all

intelligible propositionsmust
66. No form of

false.

[E.]
tions, revolu-

democracy
never

subject
the
mass

to

violent
of the

because

it

excludes

people

from

political power.

[E.]

172

ARGUMENTS

AND

FALLACIES.

[CHAP.
to

67. The
Law

student

of

History
finds

is

compelled Society has


I shall

admit
never

the

of

Progress,for
It is fated of that
cases

he

that

stood

still. 68. either


I shall
or

[E.]
that
not
a

recover,

in is

which

the

employment
much

of

physician
in

useless, and

therefore assertion have

inexpedient.
that
men

[E.]
occupied
is Lord

69.
affairs

The
cannot

public
Sir the

time
as

for

literary occupations Derby,


Mr.

disproved
Bacon,
and

by
G.

such

instances

Julius Csesar, Alfred,


of French.
a

C.

Lewis,

the of

Earl the

Gladstone,

late

Emperor
Whatever universe has
no

70.

had
no

beginning

in time time

has
:

limits in space the

the

has

beginning
is

in

therefore

verse uni-

limits in space. mollusc


an a

71.

The

aggregate

of

the

second

order; in its

for there

is

no

sign

of

of multiplicity

like

parts

embryo.
72.

The
of

farmers

will not
; for

pay
no

in

rent

more

than

the

net
a

produce

their farms

trading class

will continue

losingbusiness.
73.

[L.]
of

The

knowledge
of

things is study study


the

more

improving Physics
must

than

the

knowledge
be
more

words.
than

The

of

therefore

improving
The moral

of

Languages.

[E.]
governed
its
so

74.
as

world
; for

is far

from

being

so

well

the

material
are as

the

former, although it has


not

laws,
stantly con-

which

invariable,does
the latter.
has
a

observe

these

laws

[p.]
gold
may
must

75.

England
;
a

coinage

and

is that

very

wealthy
countries

country

therefore, it gold coinage


parents
should interfere
are

be
be

inferred

other

having
their

wealthy. judges
to

76. Most
children
to

the best
be
sent

of the age
as

at

which
desirable un-

school, and
are

it is

with

those

who

the

best

judges

of

xx.]
their
be

EXAMPLES.

173 that
to

children's
to

it interest,
send

follows

parents
school.

should

not

compelled
77.

their children which


comets
return
"

Among
are some

the of

bodies
the

do
but

not

move

in

elliptic
do
some

orbits
move

all

bodies

which

in

ellipticorbits
return

periodically ; hence,
be
comets.

bodies

which

cannot periodically
are

78. Some
for all male

rate-payers rate-payers
are

clearlynot
electors,and
for

fit for their


some

duties who

are

electors of

accept

bribes

clearly unfit

their

duty

electing

representatives.
79.
ease

Whatever and

is done it is
We

skilfully appears
once

to to

be

done

with

art, when

matured
therefore who
more

habit, vanishes powerfully


the
can cited ex-

from

observation.
to

are

emulation of

by those

have

attained
we

highest
with the

degree
least 80.

excellence, and

whom

therefore

reason

hope

to to

excel.

[L.]
that when
must
we

It is absurd

maintain
a

cannot true ;

avoid
for
some

thinking or
persons in which 81. How

conceiving
be
not

thing,it

be

cannot

in

darkness believe.

without

thinking

of

ghosts,

they do
can

[R.]
that

any

one

maintain
remorse

pain

is

always

an

evil,who
sometimes
82.
can

admits be
a

that
real

involves

pain, and

yet may

good?
in which the the
of

[c.]
transmission animals
or

The
be

time

is past

of
even

news

measured

by

speed
not

of

steam

; for the

telegraph is

approached by

either.

[DE MORGAN.]
"

83. We

enjoy

greater
on

degree

of
can

than political liberty

any
excuse

civilised
for
a

people
seditious

earth, and

therefore

have

no

disposition.
understand
the other

84. Those competent


to

only
treat

who

languages
their
own

are

correctly of

of principles

174

ARGUMENTS such

AND

FALLACIES.

[CHAP.
of

since the

competency

requires a. philosophicalview
in

nature

of
matter

language
must

general.
merely phenomenal,
I must

[L.]
be
so

85.
too.

If

be

[E.]
A

86.

miracle
nature.
are

is

incredible, because

it contradicts

the

laws

of

[E.]
no

87. There
agree, 88.

practicalprinciples wherein
none a

all

men

and

therefore

which
metal

are

innate.
all alkalies

[E.]
contain
a

Potash

contains is
an

; for

metal, and

potash
not too

alkali.
; for

[E.]
hope dies, all
the that dies.
to
a

89. Quench
90.
are
a

hope
bad
:

when
have

That

is

you I

impudence
you
are

say

you

while materialist,

know

dancing
colour. and bable pro-

master.

91. 92.

Blood

is

colour;
in

for it is

red,

and

red

is

Every incident
; therefore

this

story is very

natural

the story itself is natural si

and

probable.
brevis
:

93.

Dolor,

longus,

levis ;

si

gravis,
the
:

ergo,

omnino
94.

fortiter sustinendus. ''Whether


we we

[L.]
live
the
unto

live, we
die
are

Lord;
we

and live

whether

die, we die, we
on

unto

Lord

whether

therefore, or
95. hommes hommes

the

Lord's."
on

\_Rom. xiv. 8.]


n'est pas heureux ainsi
; les

Quand
faux faux

n'a

point d'amis,
n'ont
ne

et et

trompeurs trompeurs
the

point d'amis:
pas

les

sont

heureux. Church the


was

96. Philippiwas
in

city where
:

the first Christian also the

Europe

was

founded of
;

it

was

place where
and army Cassius
was

republican
defeated

army

Rome

under the

Brutus

finallydefeated
at
was

hence

republican
the

finally
in

the

city where
is
to

first Christian

Church

Europe
97.
stable

founded.
a

Switzerland
power is
not

republic,and,
be

you

will grant,
any

more

found

; nor,

again, is

political

xx.

EXAMPLES.

175

societymore

settled than

that
can

of the in
no

United

States. of

Surely,

then, republican France

be

danger
as briefly

revolution.

[*.]
98. Expand
argument
into
a

syllogism,as dialogue
I. Act

you

can,

the

contained
VI. Part III.

in the

of
i,

Shakespeare's
between

King words,

Henry
"my
99.
autem constare

Scene

the

title's good," "c.,

and, "succeed
beatissimos

and
esse nee

reign."
constat ;

[o.]
beatus

Quoniam
sine
; nee
esse

debs
nemo

virtute ratio

potest

virtus

sine

ratione

usquam

inesse
confitendum

nisi in
est.

hominis

figura;

hominis
too.

specie deos
order thereof of

[H.]
upon confusion, whereout

Without
the
want

there

is

no

living in public society,


of
; and

because

is the

mother

division destruction.
101.

necessityfolloweth
Ecclesiastical

of

division,
s.

[HOOKER,
is

Polity, v. 8,
and that is

i.]

Whatever

contradictoryto
incredible
order the

universal
;

invariable
sequence

experience is antecedently
of and facts

and
nature

as

which

is called with
of that

of

established,
miracles
or

in

accordance

universal
order
are

experience,

alleged violations Justice is


to

antecedently improbable.

[E.]
102.

the
man

profitof others;
to

therefore

it is

profitable un-

the

just
both

be

just.
seller

[o.]
profit: in
the
the
;

103.

In both

trade these
one

buyer and

home in the

trade

profits remain profitgoes


will
be
to

in

country

foreigntrade
the the
same

the
more

foreign trader

; therefore

population
than in the

profitably employed
enriches.
of the of money
:

in

home
104.

foreign trade.
in
money Hence of the of

[o.]
the trade money
out

Whatever
of any

brings
in the

value

branch

trade, or
balance
more

country
in
;

consists altogether, and than any

it of

brings

trade

which

carries it is
a

the

country

it draws

into

losing trade

and

therefore, money

176
should bounties 105. what is
be

ARGUMENTS attracted

AND

FALLACIES.

[CHAP.

into

the

country,

and

kept

there

by

and

prohibitions.
may be
may

Distinction
not
uncommon

reasonably expected,
be

because
and

reasonably expected,
with for

distinction
106.
"

is

not

uncommon. am

[E.]
I moved envy
no cause

Neither
or

if you

are

equal to,
you
to

less than

myself, I
to

have

for it ; and you, and


not

if

be

greater, I
evil of
men

ought
have
my

endeavour

to

equal

speak
107.

you."
been

[L.]
derided, and
I
am

Great proves

derided:

which

that

system

ought
MORGAN,
are

to

be

adopted.
p.

[DE
108.
most

Paradoxes,

387.]
for when

Preventive the

measures

always invidious,
them

successful
109. Treason

necessity for
prospers
:

is the least apparent. the


reason

never none

What's

Why,
sine

when
no.

it prospers,

dare

call it Treason. sine

Neque
neque

quies gentium

armis, neque
Lib.
are

arma

stipendiis ;
Men

stipendia sine

tributis habere

queunt.

[TACITUS,Hist.
in.

iv. cap.

74.]
all
not

are

not
are

brutes;

brutes

irrational:
men are

irrational
free from
112.

beings
best

irresponsible ; therefore,
all taxes

responsibility.
The
of
are

[H.]
taxes
:

on now

consumption
all
;

and and

taxes

on

the the
are

transfer former

of property
are

the

latter

many

of

levied
and
on

by
on

stamps

stamp

duties

therefore duties
;

good

taxes,
taxes

taxes

justiceare good

all stamp

therefore
Protest

justice are
Law

taxes.

(See
edition,

BENTHAM'S
1816. 113.

against
remarked

Taxes,

second

pp.

53,

54-)
that
"a
man

Dr.
was

Johnson
not nature

who What

sold

penknife
name

an necessarily

ironmonger."

is the this

and
was

of ?

the

logical fallacy against which

remark

directed

178

ARGUMENTS

AND

FALLACIES.

[CH.
in
our

xx.

night,

and

which

we

continue
on

to

do

even

dreams."
"

(MACAULAY,
123.

Essay
Claimant
manner

Bacon.}
has

[E.]
many of

The and

undoubtedly
were

peculiarities
the

of

gait

which
not

characteristic

missing

baronet.
to

Are

these

therefore
of

proofs

of

identity equipollent by
is
the

the

evidence

imposture
the

afforded
man

absence
to

of

tattoo-marks
?

which

genuine

proved

have
124.

possessed
Even if

it

could
men

be

shown

that

animals

perform
the aid
of

certain
reason,

actions it would
them

which

could

only perform
follow

by

by
by

no

means

necessarily

that

animals

perform

its aid. neither Mind


nor

[c.]
Matter,
:

125.

If

there's

Mill's If Believe
126. If
we

existence,
you
as

too,

we

shatter

still believe
well

in

Mill,
and
Matter.
we

in

Mind

[E.]
may infer

accept
was

Aristotle's
not
one

testimony,
of the
Ionian

that that
between he from
not

Anaximander

philosophers
a mean

accepted
Water

as

the and

One Air
;

material

principle
the
to

term

for, in
nature

Physics,
have

we

read

that

held the

the

substances

in

been
of

produced
and in

primordial
process

principle
condensation stated
was

by

process

secretion
;

by
De

of it is

and other

rarefaction mode forward of

while

the than

Ctxlo the

that
not term

production
any

last-named
such
a mean

put
for

by

who What
most

adopted

their does

principle.
this inference

syllogisticform
naturally
assume

(figure and
?

mood)

[R.]

CHAPTER

XXI.

ELEMENTS

OF

EQUATIONAL

LOGIC.

i.

THE

symbols
:
"

employed

in

this

system

are

the

following
A,
groups

B,
of

C,

or

other

capital
the

letters,
common

signify qualities,
part,
or

or

qualities, forming
of

intensive
classes of

meaning,
objects.
b,

terms,

or

names

of

objects

and

a.

c,

or

other
;

small
thus
a

italic

letters,
the
This

are

the
of

corresponding
one
or more

negative
of
was

terms

signifies
A.

absence notation

the

qualities signified by by
De

for
p. of

negatives
38.
the that The
terms

proposed
=

Morgan,
of

Formal
of A

Logic,

mark

is

the

sign
it

Identity
;

Meaning
=

between

which

stands

thus
are

indicates with the

the

qualities

signified
B.

by

identical

qualities

signified by
The the
Thus

sign

-i-

signifies
of the and

unexclusive
both

alternation,

including
or

ordinary
A
-i-

meanings
B
means

the

conjunctions
of
A
to
or

and of

and.

qualities
if

those coincide.

B,

or

those

of

both

B,

they

happen
forms
a

Juxtaposition
is thus the AB
sum

of

two

letters

term

whose the
A
two

meaning
letters
B.
N 2
:

of
means

the
a

qualities
of the

signified qualities

by
of

union

and

i8o

EQUATIONAL
2.

LOGIC.

[CHAP.

The
:
"

Laws

of

Combination

of

these

symbols

are

as

follow The the


the
sum sum sum

Law

of

Commutation.
A B

AB

BA

that

is

to
same

say,
as

of of may
Law

qualitiesof qualitiesof
be

and and

B A.

is

evidentlythe
The way

of

arriving at
have
we

the

but the result is identical. different,

The
same

of

Simplicity.
over we

AA

A
same

if
as

we

the

twice qualities
once.

get

the

if

named

them The and

Law

of

Unity.
A
are

"]"

the

qualitiesof
A.
AB
-I-

the The The

of qualities
Law of

simply

the

of qualities
-i-

Distribution.
A

A(B
of B
"

C)

AC. the

of qualities

with

those

those

of C

are

and
same
as

those
Law

of AB

those

of B

AC.
-i-

or

The is

of

Indifferent Order.

"!"

B, which

evident. sufficiently

3.

The

Laws may

of

Thought
thus be

are

the

foundation
:
" "

of

all

reasoning,and
The The
Law Law

symbolicallystated
A
....

of of

Identity
Middle

A.
AB
~

Duality or
. .

of
.

Excluded The The Law

/
.

of Contradiction

Aa
"

o.

successive
or

applicationof
more

the Law

of
to

Duality to
the

two,

three, four,five
of
all

terms, givesrise

the

development Logical
below.

possible logical combinations,


the first few columns
terms

called
are

Alphabet,
The

of which
are

given

combinations
94

for six ed. (first

given

in the

Principles

of Science,p.

Vol.

I. p.

109).

XXI.]
THE

ELEMENTS..

181

LOGICAL

ALPHABET.

4.

The

one

sole and
OF TERM
BE

all sufficient rule

of inference

is the

following RULE
FOR
PREMISE
ANY

SUBSTITUTION.
SUBSTITUTE
WHAT IN

IS
WITH

STATED THAT

IN

ANY

TO

IDENTICAL

MEANING

TERM.

The

term

may any

consist
group

of of

any

singleletter,any
connected alternation.
law

juxtaposed by
the

letters, or

alternatives

sign
5. have
my
as

-i,

the

sign of

unexclusive

It is assumed

as

necessary
was

that every
Law
see

term

must

its

negative.

This

called

the
;

of Infinity in
p.

firstlogical essay

(Pure Logic, p. 65
A.
1

also

45) ;

but

pointed out

by

Mr.

it is assumed J. Ellis,
6.

by

De

Morgan,
to

in his

Syllabus,article
CRITERION
two
or more

Thence

arises what stated


as

I propose

call the

OF

CONSISTENCY,

follows

"

Any

propositionsare disappearance of

contradictorywhen, and.
are

only when,
occasion
the

after all possible


total

substitutions any

made,

they

term,

positive or
in p.

from the Logical Alphabet. negative, The principle of this criterion was
the and
the

explained
tended

65

of

Essay
the

on

Pure

Logic referred
Mr. A.

to, but

subsequent inquiry,
to

writings of

J. Ellis, have

show

supreme

importance

of the criterion.

82

EQUATION
processes

AL

LOGIC.

[CHAP.
of

The

of the
are

this

equational system
chapters
of

Logic

are

fullytreated
Science,and
which follow.

in

first
now

seven

the

of Principles
the

they

amply

illustrated

by

problems

6.

How

do

you

express

in the

new

logic the
indicated

four

Aristotelian the
The

forms

of

proposition
O ?

by

vowels
is

A, E, I, and

answer

"

A.

Every
No A

is B.

A A

AB

(i). (2).
(3).

E.

is B.
A A

A* CAB CA* of the

I. Some O. The Some

is B. is
not

CA B. CA

(4).
class A

first expresses of of the


class

the

coincidence
A

with

part
mode second
the

B,

namely
the A's

B, which
form

is the

equational
The among the

assertingthat
expresses In the
a

part of the B's.


A's
are

that similarly the third form

the
some

found

not-B's. and

is that
B.

expressed by
some

symbol C,
are

proposition asserts
part
of

A's

(C A)
of

identical arise

with

the

class
to

Some

difficulties the

may

about
some,

this form,
as

owing

the

ambiguity negative

Aristotelian The the fourth third.

elsewhere

discussed

(pp. 151-158).
form
of

propositionis evidentlythe

7.

How

shall of

we

express

equationally
in

the

tion asser-

Hobbes

(De

Corpore Politico,I.
the
state

i.

13),

that
is
1

'Irresistible
'

might

of

nature

right
'

Might

is the

principal part
in this

of the

subject, but
the

it is

qualified or

restricted

proposition by

adjective

XXL]
'

QUESTIONS

AND

ANSWERS.
'

183
state

and irresistible/

by

the

adverbial

in

the

of

nature.'

Thus

putting
A B
=

irresistible ;

C
D

in the

state

of

nature

might
is

==

right.
and
have
state must

The But

subject
Hobbes

clearly ABC;
of
course,

is affirmed that
all

of

it.

cannot,

meant

right
true.

is irresistible

might

only in the

of

nature

is this

As, indeed, irresistible

might
be

overcome

everything
the
to
sure

opposed
of the

to

it,there

can

nothing
It

else

rightin
seem

sphere
have of

its

action, so
A
even

that
=

the

proposition would
is
not

form

C
of

C D.
Hobbes.

easy

to

be

the

meaning

8.

Represent
that
is born full of

the of

meaning
a

of

the
is of

sentence

'

Man and

woman,

few

days,

trouble.'
is

The

relative

clause,

'

that
we a

born

of

woman,'
that

is

and evidently explicative,


are

cannot

suppose
Hence

there

any
A

men

not

'

born

of

woman.' C

taking days
; ;

man

of

few

B the

born
seems

of

woman

full of

trouble

meaning
"

to

be

expressed

in

the

two

positions pro-

AB;

A=ACD.
But

if

we

may
"

treat

the
'

sentence
man

as

an

imperfectlyexpressed
of
a woman

syllogism
is, "c.'"
and
B
=

namely,
then
B

because

is born

he
=

the and

premises obviously
the conclusion in the second, is A

become

AB,
for B

C D,

by substitution
=

in the

first of

its value

ABCD.

184
9.

EQUATIONAL
Show how
A
to
=

LOGIC.

[CHAP.
contra-

obtain

equationallythe

positive of
This is

AB.
the

explained in
I. pp.

Principles of Science,
but

p.

83

Vol. edition, (first

97-102), Thought
-I-

may

be

thus

briefly

repeated
"

By the Second
for A

Law b
=

of
Kb

ab.

Substitute

its

equal
=

AB.
"!"

b or, b

AB" ab.

ab

-I-

ab,

"

Concerning
43"47,
"c.

the

contrapositive

see

above,

pp.

32,

10.

Show
A
=

how B.

to

obtain

the

complete

contra-

positiveof
As before b
=

Ab

-I-

ab

B"

"!"

ab

-I-

ab

ab

similarly
a
=

a" the

-I-

ab

aA

-I-

ab

"

-I-

ab

ab.

Having
it is

now

two

propositions
a
=

ab
"

b, ab, and get

plainthat

we

may

eliminate
a
=

b.

11.

What

descriptions
'

of
'

the

terms

'glittering
from the

thing

and

'

not

gold
?
not

can

you

draw

following

assertions
is

(1) Brass (2)


Let
A
=

gold

Brass

glitters.
;

brass

golden

C The

glittering thing. A";

premises

are

(i)

(2)

AC.

186

EQUATIONAL
Celarent.
No
men are are are

LOGIC.

[CHAP.

perfect;
men

(i) (2)

B A A

B"r. AB.

All
No

kings kings
the
B

perfect; (3)
case,

AB"r. in

Solved,
of the

as

in of

last

by

direct

substitution

(2)

value

given

in

(i).

Darii. All mathematicians have well- trained

\
C
=

CD. ABC.
ArJCD.

intellects ; Some Some


women

J
are

mathematicians have

;
I "
)

AB
AT5

women

well-trained

AD

intellects.

Here

stands and
the
we

for

the

indefinite
treat

adjective some,
as an

and

for and the

women,

then

AB

undivided

term, in

obtain

result

by

direct

substitution, exactly as

previous moods.

Ferio.
No
...

foraminifera
inhabitants
;

are

fresh

water

)
~

V C

Ca.

J
of

Some

components
;

chalk

are

fora-

)
,

,,

minifera Some

components

of

chalk

are

not

)
"

AD

-D
:=

fresh-water

inhabitants.
a

J
term

Except
form

that

negative
In

takes there
the

the is
no

place
moods may

of

the

positive term
between

D, in the last mood,


them.
so

difference
of

in the said

fact, all

four

first figurepresent
to

that great similarity inference.

they

be

be

of

one

form

of

xxi.]

QUESTIONS

AND

ANSWERS.

187

Cesare.

The
All All

absolute
known

is

not

phenomenal phenomenal
not

; ;

(i)

C
A A

C".
AB. ABr.

things are things are by


the
any

(2) (3)

known

the absolute.

We from
we

cannot

direct substitution
as

obtain

the conclusion
b in

the
may

premises,
take B

appears

in

(2) and

(i).

But

contrapositiveof (i) as
=

described in the

before

(p. 184), namely, premise


is then

B"r, and

substitution

second

practicable.

Camestres. of

All

laws human human

nature customs customs

are are are

invariable;
invariable
laws of
;

(i)

C
A A

CB.
Kb. Ac.

No
No

(2) (3)
the

nature.

As

in the
b

last
=

mood,
be, and

we

must

take

of contrapositive

(i),namely
13.

substitute
treatment

thereby
of

in

(2).
Camestres,

The
and

equational
Camenes

the

moods in
pp.
; but

Cesare,

is described

also
I.

the

Principlesof
or

Science, pp.
the

84-6

edition, Vol. (first


pp.

99-101),
the

in

Substitution

of Similars,
way
as

47-49

following
M.A.

is the
of

briefest

of

getting the
Mr.

Aristotelian
W. H.

conclusion,

Camestres,
Let

suggested by
be

Brewer,

the

premises

(1) (2)
Multiply together,and
AB
=

A B

AC.
Be.

we

get
=

ABG:

o.

Thus

there

is

no

such

thingas

AB.

J88

EQUATIONAL
14.

LOGIC.

[CHAP.
be

The

remaining
In

moods
case

need
D
=

only
:

symbolically

represented.

every

some

15.

Exhibit Sir W. the

the

force logical
"

of the

motto

adopted

by

Hamilton World

(1)

In

there

is

nothing

great

but

Man.

(2)
Let A B
=

In

Man

there

is

nothing
C
D

great

but

Mind.

in the world
man

possessingmind
great.

The

conditions

may
A
=

be

represented as
ABD
-i-

"

(1)

Kbd.
B*/.

(2)
As

BCD

-i-

it may

be

understood,

though

unexpressed,

that

xxi.]

QUESTIONS
men are

AND

ANSWERS. and

189

(3) all
mind
are

in the
we

world,
further

that

(4) all possessing

men,

have
-

(3)
The

AB.

(4)
thus reduced
abcD abed.

C
to

CB.

combinations
ABCD Kbcd

are

Observe
but
as a as

that, if mind

were

regarded
man,
we

not

as

an

attribute
treat

physical part
one

of

could

not

the

assertion

of

relation. simple logical

6.
'

What
All

is the

meaning
which
'

of
come

the
to

assertion

that
are

the with

wheels silver

Croyland

shod

If

we

take
A B
=

wheel

;
to

coming
shod

Croyland silver,

C the

with

in the form assertion,as it stands, is evidently


AB But
=

ABC

(i).
no

it
to

was

always understood, joined


shod in the mind with
A

doubt,
the

that this

adage
'No

was

be
are

with

tacit

premise

wheels

silver,' expressed by
=

"c

(2).
be of

There these
and

seems,

at

first

to sight,

contradiction
wheels shod The

between with silver

premises ;

for

(i) speaks
there that Of
are

(2) denies
to

that

such
are

things.
no

explanation
wheels

is obvious,

namely,

there

such

things as

coming

Croyland.

the four combinations

A, containing

igo Alk

EQUATIONAL
is

LOGIC.

[CHAP.
A"C

negatived by descriptionof

that the

and (i),and ABC A is given thus,


A
=

by (2), so

Kbc,

or,

by

substitution

of

for Ac,
A
=

Kb,

that the

is, no

wheels which
an

come

to

Croyland.
was

This

is of
to

course

inference

the ancient in

adage
former

intended

suggest,
fens of

Croyland being Cambridgeshire,


could This that all make

abbey lying among days


no

the

where

wheeled

vehicle

its way. illustrates the

question

important logicalprinciple
be

propositionsought, strictly speaking, to


We

interpreted
in the

hypothetically.
destructive
If any

have and
we

only
see

to

put these

premises
a

hypothetical form,

that
"

they
thus

make

reasonable

hypotheticalsyllogism
wheels
no come

to

Croyland
are

they
with

are

shod

with

silver ; but
no

wheels
come

shod

silver ;

therefore,

wheels

do

to

Croyland.
are

17.

Ruminant

animals

those
have

which horns

have
; the
a

cloven

feet, and
animal

they usually
which
it

extinct
cloven and
are

left this
was as
a

footprint had
ruminant beasts have
animal of prey

foot; therefore
had horns.

Again,
animals

no

ruminant of prey.

it cannot

been

beast
The above
No.
=

problem 78).

is

given
our

in

the

Elementary
thus
"

Lessons

(p. 321,
A B

Taking
;

symbols
D
=

ruminant

extinct beast

animal
;

having having clearly

cloven horns
;

feet ;

of prey

C
we

have

(i).

xxi.]
The may be
statement

QUESTIONS
that
as

AND

ANSWERS.

191
T

ruminant

animals

usually

have

horns

formalised

PA

PAC

(2),
of

that PA

is

to

say, horns.

certain
Next D
we

particularportion
have
DB

the

class

A,

have

(3).
get

in (3) by (i) we Substituting


D=DA

(5);
animal
was a

showing
we

that

the

extinct

ruminant. is
erroneous

But,

as

cannot

substitute

between
If

(3) and
there it had

(2),it
is

to

assert

that it had
number but
no

horns.

by usually we
a

mean

in

the

far bability, pro-

greater

of cases,

then

considerable

certaintythat
as an or

horns.

Again,
prey
are

we

have

additional

premise, that

beasts

of

not

ruminant,

flE

(4),

which, taken

with
D
=

DA

(5),
lishing estabcannot

our

previous conclusion,gives a syllogism in Cesare,


that D is
not

E,

or

that This

the
we

extinct

animal

have

been

beast

of prey.

might sufficiently prove


of

the symbolicallyby multiplying and

respective members

(4)

(5) together,giving
DE
=

0EDA

o.

This this D
out

shows
can

that also

inconsistencyarises
E.

from

supposing
be

that

be

The

same

result
De.

might

worked

by combinations, givingD
1

The

first edition

of the

Lessons

reads

always

instead

of

usually.

192

EQUATIOXAL
18.

LOGIC.

[CHAP.

Take

the

proposition 'All precisely what


what
it leaves

crystals are
it doubtful.

solids,'
it

and

ascertain and

affirms, what

denies,

[A. J. ELLIS.]
Taking
in the form
A
=

and crystal,

solid, the proposition

is

AB.
are

The

conceivable

combinations

four

in

number,

namely,
AB,
Of
to

A", 0B, and

ab.

these, only A"


say,

is inconsistent
'All

with

the

premise, that
denies

is

the

premise
because
; hence

crystals are
'

solids'

the

existence strike
as

of such

things as
then

unsolid there

crystals.' We
be
no

cannot

out

AB,

would

such

thing
of

left crystals

the

premise
that any

affirms

the existence

solid that stand

in crystals,

the

sense

other

propositiondenying
be

crystalsare

or solid,

that solids may


our

would crystals,

in contradiction
we

to

premise.
out
or a

Again,
then be
no

may

not

strike

b, because

there
to
a

would avoid

such
of

things as b,
our

not-solids. there
must

Hence be
we are

contradiction
as our

premise,
of

such
to
we

thing
to

'a

non-crystalwhich adopted
Criterion
one case

is not-solid.'

If

hold
must

Consistency (p. 181),


of ab
so exists,

say

that

at

least

that to avoid
case

self-

contradiction, some,
must

that
to

is
l

at

least
to

one

of

not-crystal,
This
firms con-

be the

allowed

exist

and
we same

be

not-solid.

conclusion

which
the be

obtained previously

by
But

the the

Aristotelian combination
the in truth doubt
1

logic from
aE may

premise (p. 48).


or

removed

left without leave


are

affecting
it

of

the

premises, which
'

therefore

entirely
exist'

whether
the

which not-crysta'ls
of the verb

solids
pp.

Concerning

logicalsense

exist,see

141-2.

194
sense,

EOUATIONAL

LOGIC.

[CHAP.

those

people
relation. it does
;

who

talked have

nonsense,

assuming
But
we

there
cannot

to

be

such

persons,

must

been

absent. who

invert
were

this
away,
sense

Because
not

those

talked who
were

nonsense

follow
all have

that those been

present

talked

they

may

silent.

20.

De who the

Morgan
wishes
to

says
test

(Syllabus,p. 14), Any


"

one

himself

and

his friends of the

upon

question
and

whether would
be
on

analysis
useful the

forms

of

enunciation himself
Is either

or

not, may

try
:
"

them

following question

of the

following propositions true,


?

and

if

either, which

(1)

All
are

Englishmen
to

who
among

do

not

take

snuff

be
use

found

Europeans
do

who

do

not

tobacco. who
among
not
use

(2) All Englishmen


are

tobacco who

to not

be

found

Europeans

do

take

snuff.
answer

Required

immediate

and

tion." demonstra-

Assigningsymbols
A B
=

as

follows
; ;

"

Englishmen
Europeans
obvious

C
D

taking snuff;

using tobacco
are propositions

it is pretty

that

the

above

thus

symbolised

"

(1) (2)
We of
are

Ac M

to

compare which
may

these
be A
=

with assumed

the
to

well-known be

relations

the terms,

(3)

AB;

xxi.]
that
'

QUESTIONS

AND

ANSWERS.

195

is, Every Englishman (4)

is C
use

an

European/
CD
;

and

that
out

is,

'

All who

take

snuff
we

tobacco.'

Now,

in is

working

the combinations, conditions

find that the class A^

composed

under

(3) and
Ac
=

(4)as

follows
-I-

AB^D

ABrt/.

The

truth of

is,then, that Englishmen English Europeans


of In
not

who

do

not

take

snuff

consist

taking snuff, but


neither in
may

using
nor

tobacco, and

English Europeans
short

taking snuff
be

using
who

tobacco.
some use

(i) is
not

erroneous

ignoring the
Europeans
Ad

fact that

Englishmen
tobacco
to

using snuff
the

do

for

smoking.
of description

According
ABcd,
which

assumption (2)
with
true

is

coincides
Thus
are

the
that

descriptiondrawn
all

from do do

(3) and (4).


not not
use

it is
to

Englishmen
Europeans

who

tobacco

be

found the

among

who

take snuff; the the


no same are

negation of
of the follows

larger term, using tobacco,


one,
our

includes
But Ad

negation
means

narrower

using

snuff.
about

it

by

that because from will


as

inference

is the

from

(2)as

therefore (3) and (4), be


seen

these

conditions

as identical,

in under

the

following
several

of the class A descriptions suppositionsand conditions

furnished

the

"

(1) (2)

A A A

ABC ABCD ABCD

"["

ABa/ AEc

-I-i-

(3)and
21.

(4)

AEc.

What from

can

we

infer about

the
?
"

term

Europeans

the

following premises
are

(1) All Continentals (2) All English are (3) No English are

Europeans
;

Europeans
Continentals.

196

EQUATIONAL
C
to

LOGIC.

[CHAP.

Taking A, B,

represent

Continentals, English, and


become

the premises Europeans respectively,

(1)
(2)

A= B
=

AC. BC.
aB.
are

(3) B
The

combinations

left

uncontradicted
we are

the

four-

A"C,

aBC,

abC,

abc, whence
who

learn
not

that

Europeans,
of
are

C, consist of Continentals
who
are

English,

English
neither

not

Continentals,and
nor

of any

others,who

Continentals

English (abC).

22.

Criticise

Thomson's of number several of

"

Immediate

inference
From
. . .

by
a

the

sum

predicates. judgments
in in
sum

sufficient
same

A, having
U may

the

subject, a

judgment
is the

be

inferred, whose
other

predicate

of

all the

predicates."
been Vol.
P
=

[p.]
answered
I. p.

This

question has
61

in

the

Principles of
in A and
are

Science, p.
of the

ed. (first
P
=

73). Judgments
P
=

form
up

PQ,

PR,

PS,

"c.,

by

summing
second
But
a.0

the
of
not

by predicates
P
=

successive
may

substitution
P
=

in the
....

side

PQ,
a

we

get

PQRS
U

this does described

give

proposition of
is of the
form

the form
P
=

which,

by Thomson,

X.

23.

Represent
Thomson's

the Laws

following

argument
107
or nor
:"

from

of Thought, "
is either

All S

C
nor

or

;
;

is neither is not

therefore, S

P.

xxi.]
The

QUESTIONS

AND

ANSWERS.

197

premises

are

: respectively

"

P S We

PC Scde.

-i-

PD

-I-

PE.

get the conclusion


two

in the

briefest way
stand
=

by multiplying
:
"

the

premises together as they


PS=
P

; thus
o

(C

"!"

"!"

E)

Scde
to

-i-

"!" o.

Each

alternative is
no

is found

be

contradictory,so
to

that

there The
Dr.

such

thing as PS,
is

that is

say,

no

is S.

argument
as

not,

however,

correctly described
U
same

by
the

Thomson forms

in the

mood syllogistic

E,

nor

are

other

of
are

argument

given

in

the

section

gistic. syllo-

They

in disjunctive

character.

24.

If Abraham either

were

it justified,

must
now

have he
was nor

been
not

by

faith

or

by

works

justifiedby
works
was

faith

(according to James),
therefore

by

(according to Paul) :
not

Abraham

justified.
in deciding difficulty
on

[w.]
the

There of
the be

is

some

best

method
of
to

symbolising this
conditions
the best A B
=

argument,

owing
but

to

the

vagueness

when

analysed ;
"

the

followingseems

: representation

Let

Abraham

C
D

justified by justified by

works

justified ;
are :
"

faith.

Then

the

premises

AB A A

AB Ac.

(C

-i-

D).

Kd, found
to

These of

premises will
A

be

erase

all the the

tions combina-

excepting Abed,

which

gives

conclusion.

198
The
not

EQUATIONAL
combinations
be examined.
of
a are

LOGIC.

[CHAP.
and modes need of

altogetherunaffected
student
may

The

try other get A


=

the premises, but representing method.

should

hb

by

every

25.

It

must

be
been

admitted, indeed, that


accustomed in
to

(i)a

man

who
cannot

has be

enjoy
of
a

liberty
slave be
:

happy
of the the

the

condition

(3)

many

negroes,

however,
of

may

happy (2) they

in

condition
never

slaves, because
to

have

been

accustomed

enjoy

liberty.

[w.]

Let

A B

man

accustomed in condition negroes.

to

enjoy liberty ;
of slave ;

happy
certain

The

premises

may

be

stated

in the forms
A hb.

(i) (2)

C The

aC. CB.

supposed possible

conclusion

is C

(3)
in that the

The
hbc

combinations which C that

are seen

as

margin,

from

it will be
=

aBC

aEC

"!"

abC

abC
abc

is, are

either

B,

happy,

or

b,

not

happy.

The Process

fallacyis
of the

that

of

Negative

Premises

or

of

Illicit

Major.

xxi.]
26.

QUESTIONS
If
that

AND

ANSWERS.

199

which

is devoid is

of devoid

heat
of of

and

devoid it

of

visible that

motion

energy, motion

follows
but

what

is devoid
cannot

visible

possesses

energy

be

devoid

of

heat.
Let A B

possessingheat

possessing visible possessing energy,

motion

C the universe taken is


:
"

being 'things' unexpressed,


as

and

'devoid Then

of the

being

the

negative of
ah

'possessing.'

condition

abu

we By contraposition

obtain,using Mr.

MacColl's

notation

for the

negative of

ab

(SeePreface) :

(A4

-1-

"zB

-I.-

AB)

Hence

bC
"

MC,

two

alternatives self-contradictory
be

disappearing.
is

It

can

also the

readilyshown

that

this inference

equivalent

to

condition.. original

27.

Prove

the B
=

logical equivalence
AC

of

the

position pro-

"!"ac

and

Ac

"[ aC.
of

This the

might

be

shown

by recedingto the combinations


but each
=n"

Logical Alphabet,

it is

more

neatly proved
hence
the of

by

equating the negatives of


If of M B
=

member

of the first equation.

N, then
must

also

(p. 184);
with the

negative
AC
"!" ac.

be

identical of
B

negative
of the

Now

the

negative

is b ;. that

compound

and

200

EQUATIONAL
member is the
In

LOGIC. of

[CHAP.
the

complex
two

compound
MacColl's

negatives of

the

alternatives.

Mr.

notation

(AC

-I-

ac)'
=
=

(AC)'(ac)'
(aC
the
"!"

Ac

-i-

ac)(Ac

-I-

aC

-I-

AC).
to

On

multiplying out, expression for


found
to

nine

products
aC
-i-

are

found

be

all

self-contradictory excepting
the
b. Vice

Ac, which

is therefore of
Ac
-I- aC

versa so

the that

negative
the

will be

be

AC

-I- ac,

propositionsare

clearly equivalent.

28.

If

no

is BC, and

what AC ?

can

infer

about

the

relation

of B

The

condition

is
A
=

Ab

-I- Ac.

Substitute in either side of


ABC and
we
=

ABC,

get
ABC
=

AB"C

-I-

AECc

o,

or

cannot

be

AC.

29.

It the

is known

of A B

certain
is not

things
;

that

(i) where
B

quality
where

is,B is,C
a

(2) where
are.

is,and
from of

only
these

and

Derive of the C

conditions
in which A

description
is not

class is.

things
The

present, but

premises are

: clearly

"

(1) (2)

A
. .

Ab,
CD.

B
. .

202

EQUATIONAL
than half

LOGIC.
to

[CHAP.
solve

more problem, requires

page

it, using,

moreover,

fractions.] sundry impossible logical

31.

Given
and prove

(i) that
that that all A

everything
is

is either

or

C,
A
:

(2)

all C

B, unless

it is not

is B.

[c.]
the assertion that that

The every every

first condition not-B not-C is

is

expressed by
have bC.

C, which
Thus
b

carries with
we

it the

equivalent

is B.

(i)
to

The
are

second

assertion

is less easy if
'

because interpret,
not-A.

we

not

told what
appears,
"

happens
however,

that is C, is it,' be that C

The
must

meaning
be

to

if it is A

B, that is

AC These

ABC.

(2)
"

conditions

give the
ABC. AEt.

combinations
"BC.
aRc.

abC. from the B.


first

two

of

which

we

learn

that

is

always

32.
'

If

we

throw
A is B is B
'
"

'every
or a

is

B
'

'

into
A

the

form
is it

every
not

B,' we

have

every

which But
As
'

contradiction that thus there


we

in terms.
can

evidently implies
are

be

no

which
A

not

Bs, and

return

to

every

isB.'

xxi.]
The
De

QUESTIONS
above is
a

AND

ANSWERS.

203

transcript with
example

altered

symbols
p.

from But

Morgan's
both

sixth

(Formal Logic,
from
De
an error

123).
not

the

contradiction

arises

simply by
b.

in

plying multithe rule this

alternatives of

Morgan
not

follows

for rule

the
can

resolution

dilemmas,
when
the

observing
are

that

apply only
have
"

alternatives

different.

Equationally we

AB AB" AB"

"!" -I"!"

AB. AB" AB
=

hence
. .
.

hb Kb
.

He

gets

AB.

It is

rarelywe

find De

Morgan

tripping.

33.

Every
C
; D and is

is

one

only
and

of

the

two

or

both it

B is

C, except
-T

when
no

is A

E,
is

then

neither

therefore

D.

This

problem
124,

was

proposed by
solution has

De

Morgan given

in the Formal

Logic, p.

and
101

been i. p. has
"

in the The

Principles premises,
may be

of Science, p.
as

ed., vol. (first


Robertson

117).

Professor in two

Groom

pointed out,

stated

propositions,namely
A D
=

AB^ D^BC

-i -i-

Some
to

objection has, however, solution,and


over we

been

taken
a

by

Mr.

Monro

my

the

student

will find
It
seems

good
rather which

exercise ful doubtare

in

going
how

the

solution
treat

carefully.
the

should the

combinations

and

not

B';,but
De

difficulties lie conditions.

wholly

in

the

tion interpreta-

of

Morgan's

204

EQUATIONAL
34. From but Hence B A

LOGIC.

[CHAP.
C follows each with
D
;

follows D
and
are

B, and
inconsistent

from

and
A

with

other.
each

are

inconsistent

other.

This
De

problem

which

is

formally
123,

the

same

as

one

of
the

Morgan's {Formal

Logic,p.

Example

3),

has

conditions

(1) (2) (3)


The consistent combinations
K"cd.

AB;

C
B

CD;
B*

are

abCD.

abed. We that
A

see

that A
never

and found C

never

occur

together,and
of

in
B

fact and

is

excepting
and
D.

in the presence

the absence
I

of both
an

committed

error

in

treating this problem


52,
B

in

the

Substitution
as

of Similars
the
D

(pp.
both

53), by regarding aEcd


may
occur

negatived by
C and

premises.
must

in the absence

of A, but

be

absent.

35.

What

are

the

combinations
are

of

the

qualities
to

A,

and

C which

possible according (i)


"Where
A

the

following
B
or

conditions? C
at
are once

is present,
at
once

and

either
;

both

present
C
is

absent

(2) where
the

present, A
under

is

present."

Describe

class

not-B

these

conditions.

xxi.]
The

QUESTIONS
conditions
are

AND

ANSWERS.

expressed equationallyas
A
=

(1)

ABC AC.

-I-

(2)
The ABC
Abe

consistent
AEc and

combinations AbC and


are

are

shown

in the

the

margin;
condition,

removed

by

first

and
two

aEC

abC
ones

by

the

second. contain

Selecting the
b, we
have the

remaining

which
"

abc

required description
b
"

Abe

-I- abc

be.

Where

is absent, C

also will be

absent

36.

'

The

logical
in

value

of

two

affirmative
is

mises prezero.'

the the

second

figure

absolute

Examine

truth

of this statement

[P.]
be

The

two

premises

assumed

to

be

universal

may

symbolised as (1)
(2)
The first A
=

AB;
CB. A^C and

negatives
and AbC. is A
A

the

combinations
so

Abc,

the in
we

second

AbC
to

abC,
There
we

that

the

premises overlap
If

regard

remain

five combinations.

inquire what

get the value


-

ABC

-I- AEc

AB,
C

which
we

is no

more

than

(i).

For

the that

descriptionof
no

similarly
less

get (2). Thus


A

it is C.

plain

relation
B
we

is established
even

between

and
; for

Concerning

have

information
B

ABC

-I- AEc

-I-

aEC

"!" aEc

AB

-I- dE

E.

2o6

EQUATIONAL
Of the

LOGIC.

[CHAP.
draw
more

negative terms, however,


thus #BC abc. B":

we

significant

descriptions ;

"!" ac.

b
c
"

-I- abc.

It

cannot

be

truly

said

that

the

logical value

of

the

premises

is absolute

zero.

37. D

Given

that

(i)everything
both A and C C
nor or

that

is B A A

but
nor

not

is either

neither is both

C
B

;
:

and
prove

(2) that
that
no

neither
A

and

is B.
from Moral

[Adapted

Science

Tripos,

Cambridge, 1879.]
conditions

The

are

(1)

B"/

B"/(AC
=

-I-I-

at).
D) (a
-I-

(2)
Confining
AB.
rest
no
we see our

I- D

(C
to

b\

attention
K"cd

the

combinations

containing

that

is contradicted C
or

by

(i),and
there

the
are

which

contain
or no

either is B.

D, by (2). Hence

ABs,
The

equation (2) may


AB

be

more

stated briefly

as

AB*/.

The

only combination
aBCd.

containinga

removed

by (i) and

(2) is

38. Illustrate

the

use

of

symbolic
"

methods

by

expressing the propositions

xxi.]

QUESTIONS

AND

ANSWERS.

207

(1) No
and

is B

except
some

what that.
never

is both

and

D,

only
C
A

of D B is is

(2) Either
where

or

absent

except
both
are

or

present, but

always

absent

then.

[C.]

The

first

propositionappears containing only


the
We in
some

to

deny

the presence be

of

any

combination and
we

AB
cases

except
then.

there
To say

also present this


some

D, and
must

express

introduce

another above

letter term,
true

E,
E

so

that where is

E
not
are

is present
B
at

holds

; where

is absent, A

all.

find then

that the

followingcombinations

negatived :
ABC^E.
AB^DE. ABa/E.

ABCD^. ABCde.

All

this may

be

expressed
AB
=

in the

one

equation

ABCDE.

The
mean

proposition (2) is

not

easy

to

but interpret,

seems

to

39. is

Every
M,
X

is either

P, Q,
every

or

R;
is M

but

every

every

is

M,

; therefore

every

is M.

De

Morgan,

who

gives

the

above

(Formal Logic,p.

123,

2o3

EQUATIONAL

LOGIC. form

[CHAP.
of the dilemma.

Example
It is thus

5),describes
solved

it

as

common

: equationally

(1)
(2) (3)

X P

X(P.|.
PM
;

Q.|. R);

Q
R

QM;
RM.

(4)

Substituting by (2) (3) (4) in (i),


X X
=

X X

(PM (P
-I-

-i-

QM
-I-

-|.

RM)

R)
"

M.

in Re-substituting

the last

by (i)

40.
no

Every
C

is either
;

B, C,

or

D A

no

is

is A

therefore,every

is D.

[De Morgan,
The

Formal

Logic,p. 122.]

premises are
(i)
A

clearly
"

AB 0B.

-i-

AC

-I-

AD.

(a)
(3)
In

rtC.

(i) substitute
then strike

the values
out two

of

and

given in (2) and


"

(3),and

terms self-contradictory

ArtB

-i-

AaC

"!"

AD

AD.

41.

If A
but E

be

B, E
A

is F ; and is

if C C
is

be
D
;

D, E

is F

either
is F.

B,

or

therefore,
123.

(De Morgan,

Formal

Logic, p.

2io

EQUATIONAL
43.

LOGIC.

[CHAP.
B is E
; every

Every
C
is E

is B

or

or

; every

; and

every

is D. ibid. p.
123,

[De Morgan,
Thus

Example

4.]

symbolised

"

"

(i)
(2) By
B
=

AB

-i=

AC CE.

-i-

AD.

BE.

(3)

(4)

ED. then

obvious

substitutions, by (2) and


get
"

and (3) in (i),

by (4) in

the result, we
A
=

ABDE of

-i-

ACDE

-i-

AD.

But

the

first two

these
are

alternatives

are

superfluous ;
in the wider

they
term

both
AD.

involve Hence

D
"

and

therefore

contained

AD.

'

44.

If

the

relations
A

and

combine

into

C, it
that

is clear

that

without
and

C
B

following means
without C

there
means

is not

B,

that
is not

following

that

there

A.'
on

[De Morgan,
The

Third

Memoir

the

Syllogism, p. 48.]
appears
to
mean

relations that AB

and

combining

into C
or
"

simply

is

accompanied by C,
AB
=

ABC. have

To

find A

without

C
Ac
=

we following,

necessarily

ABc

"!"

Abe. ABC. Kbc.

Insertingfor

AB he

in this last its value


=

AECc C
=

-I- Kbc

Similarlyfor
EC
=

B AEc

without
"!"

following
AECc
-I-

aEc

aEc

aEc,

XXL]
45.

QUESTIONS

AND

ANSWERS.

211

Suppose (2)
exhaustive

class A

to

be

divided
on

(i) on

one

principleinto
into

and

B, and

another divisions

ciple prin-

and

D,

the

being
all A is

; suppose

further
;
can

that you

(3)

C,

and

(4) all
is

is D all D

conclude

that

all C

A,

and

is B

[E.]
to

The S

meaning

of this A

problem
D

appears

be

that and

the class

will, as regards

and

B, consist
; if so,

of

SA"

SaB,

and the

as similarly

regards C and
conditions be

there

will under

first

two
"

only

four

possible combinations,

namely

SAM).
But

the

further
so

condition

(3) negatives SAM3,

and

(4)

negativesSaBC^,
C,
we

that, on

find

it is

(withinthe
that
we

for the description of inquiring class S),KbCd; D is similarly may


are

#B"rD.

Both

questions

then

be
not to

answered look

in

the

affirmative, provided

beyond

the

sphere

of the class S.

46. What

are

the
or

classes
not

of

objects regarded
the
exist

as

possessing
A,
with the
A

possessing
may

qualities
and

B,

C,

D,

which

consistently

the

fundamental that that


no

Laws

of

Thought,
possesses does

conditions and

class

both
not

B, and
B

everything
C
but
not

which
D ?

possess

possesses
that

[L.]
both
A

The will be

first condition

no

class

possesses

and

expressed in sufficiently
and
B from

the

premise
The

Kb, which
condition
p 2

prevents A

meeting.

second

is

212

EQUATIONAL
=

LOGIC

[CHAP.
sixteen tions combina-

obviously b
in it will be
\bCd "BCD
T"p "
J

bCd.

On

going
of the

over

the

the

fifth column that

the
first first

Logical Alphabet (p. 181),


four, containing AB,
are

obvious

negatived by
(0B) remain
"

the

premise.
:

The

third
and

four

untouched

of

the

second

fourth AbCd is

fours and

containing b, all
abCd.
the The
answer

are

negatived except
list of

adjoining
to

combinations

abCd

therefore

the

question.

47.

How

can

we

represent analyticallythe precise

meaning
affirmative
say
not

of

the

opposition
and As

between its and

universal

proposition
All
are

contradictory,
some

between
Bs

Bs,

As

are

?
affirmative is in before be
Ab
more

The its

universal

is

symbolised
the
Now
mean
'

as

AB, Ab,
are

and
as
'

logicalpower
shown

to

negative margin.
to

combination
some
'

the

As A
at

Bs

AB
was

explained
or

one

least,it
be
the

At"

|
may
one

all As.'

But,
the

even

if there of

at,

found, it establishes subject


to

existence elsewhere of

combination,

remarks

made number
as a

(p. 142).
enters not

In
at

this

qualitativetreatment
so

logic
as

all,
force

that

one

counts

for

much

million.

The
to

of

the

particularnegative proposition
which had been removed

is,then,

restore

the

combination

by

the universal

affirmative.

48. If
add the

to
a

the

premises

of

an

affirmative first
now

sorites

we

proposition affirming the predicate, the


to
an

subject
become
series

of

last

conditions
numerous

equivalent
identities,
Thomson's
or

equally
U.

of in

doubly

universal

propositions

form

XXL]

QUESTIONS
we

AND

ANSWERS.

213

if Symbolically,
B add become
=

have
so

the
on,

series
up
to

of X
=

premises
XY,
and

A
we

AB,
then

EC,
the

CD,

and Y
==

condition the
same

AY,

the
as

premises

immediately

in

force logical

To
not

give
be very

perfect demonstration
easy
; but

of

this

theorem

might
of

the student
several of
to

may

convince the
as

himself combinations

its truth

by observingin
with
a

trials that
a

consistent
never one

the

premises
of
the

sorites

shown of
a

above,

contain
in the

negative letter
order the

the

righthand
Thus

positive
binations com-

usual

alphabet.

the

consistent

with

first two

premises
abc;

are

ABC,
those for the first three

aEC,
are

abC

and

ABCD,
Hence the the last

tfBCD,

a"CD,

abcD

and

abed.

predicate appears
the first

in every

combination
in the the first

except

last, and
In all

subject only
first

combination.

affirming
and the

the

subject
the

of

last

predicate, then,
contains
must

combinations

except

first,which neither,
the
....

both

terms,
There

last,which
in
XY every
....

contains
case

disappear.
which
. . . .

remain
....

only
abcde stated

two

combinations
xy

ABCDE

and

proceed

from

the

identities

in

the

question.
Suppose
no a

pillarof
is of
at

circular

section

to

be any

so

shaped
the
to

that

lower the the

section section

less diameter bottom

than

upper

section,
section

but
at

the

is

not

greater than

top ;

we

have

here

physical analogue

the

heap

of

propositions described

above.

214

EQUATIONAL
49. Is

LOGIC.

[CHAP.
correct

Professor

Alexander from

Bain his

in

the

following

extract

Deductive

Logic

(P- 159)?
(1) (2) (3)
It may
"

Socrates

was

the

master

of Plato.

Socrates The
master

fought at
of

Delium.

Plato

fought
whether than
was

at

Delium.

be fairly

doubted
more

the

in transitions,
For

this
the

instance, are

anything
'

equivalentforms.
the
out
master

proposition, (4)

Socrates

of
two

Plato, and

fought at Delium/
obviously nothing
No
or one can

compounded
more

of

the

premises, is
abbreviation. of the

than there
a

grammatical
any

say

that

is here

change
of

meaning, original
need

anything beyond
Professor

verbal

modification

form." Bain
of
more

in

writing the
For

above

was

clearlyin
his

of

means

accurate

analysisthan
we

logicalstudies

had

afforded
A

him.

if

put
B
=

Socrates;
C
=

master

of

Plato;

one

who

fought at Delium,

the

premises

are

certainly
(j)
A A
= =

B;
AC. is B

(2)
The conclusion
two

(3) as

it stands
AB^r and

BC, which
the It is

negatives premises
possible,
is better
"BC. As

only

combinations addition
draw
two

aEc, whereas Afa, aBC.


=

in negative

the three A"C, conclusion

indeed,
than
to

to

the

(5)B

AC, which
A"C
be

(3) by

combinations, namely,

and
made of A
=

the

supposed proposition (4),it cannot


a

into any

non-disjunctive proposition without


for, whether
we

change
BC,
or

meaning
ABC,

make

it into

(6) A

it

XXI.]

QUESTIONS
from
cannot

AND

ANSWERS.

215

differs in force
in

the

premises (i) and


into any

(2),which
fact the It

sitions propo-

fact

be condensed

singlenon-disjunctive
is that
two

proposition of equivalent meaning. supposed

The
of

the

proposition(4) consists
in
'

merely
sentence.

(i) and
not

(2)

re-stated

one

compound
;

is

'the
case

proposition
would Bain
the have

at

all

it is

'

the

propositions.'

The

been

considerably altered, indeed,


'Socrates
was a

had

Mr.

interpreted(i) as
form been A
=

master

of Plato,' of would
not
so

AB.

The

type

of

the that
we

premises
he does

then pret inter-

have

altered essentially from

; but

it is obvious
not

(3),in

which
page of

have
of

'the

master/
Bain's
of
a

'a

master.'

Altogether

this

Professor

work
most

affords
acute

remarkable

evidence maintain kind


of

the

incompetence
of

logician to
some

accuracy calculus

logicalvision developed
selves, them-

without in the
I

the aid of

like that

latter part of

this work. which


out

append
the

logicaldiagrams
the

almost

explain
bracket

combinations

pointed

by each
number

being

those
to

negatived by
bracket.

propositionwhose

is attached

the

ABC "ABc
-

(2) ;
""

-i "Abe

"AbC-

(3)

(5)

Abc

(i)"aBc
abC abc

"
"

aBc abC abc

50.

How

far

does

the

conclusion of

of

an

totelian Arisall

syllogism
information
The four

fall short
in the
A

giving
?

the

contained
say

premises
AB,
B
=

premises

of Barbara,

BC,

tive nega-

combinations, AB"r, A^C,

Abc, "B^

The

conclusion

2i6

EQUATION
=

AL

LOGIC.

[CHAP.
AEc

AC

negatives only
of
the

two

of

these, namely,

and
half of

Abe. the

Measured

in this way,

then, it contains
;

only
course

information

premises
harm.

but

of

if the

conclusion

gives just that


the
rest

information
no

which

is is

desired, the
as

overlooking of
a

is

Enough

good

as

feast

"

or

rather

better.

51.

Take such
as

the

premises
As

of
are

syllogism
Bs,
and

in

Barbara,
Bs
are

(i) all
determine

(2) all
they

Cs, and
what doubt.

precisely what
and

they affirm,
leave
in

they

deny,

what

To
ABc -AbC -Abc aBC aBc abC

answer

this

question,we
of in and the
Abc

must

form

the
"

eight
strike

combinations
as

A,

B, C
;

and
we

their

negatives,
out

margin
as

(2) then
"

A^C condition

being
AEc

in

conflict
aEc
as

with

(i),and
conflict
are

and the

in being similarly

with

condition remain
these
we were

(2), that

all Bs

Cs.
abc.

There But

four do
not to

combinations,ABC,
stand
remove on

aBC,

abC, and

the

same

logicalfooting, because
would
abc be
no

if
as no

ABC,
we were

there
to

such

thing
be

left ; and

if
c

remove

there
or

would

such

thing as

left.

Now

it is the Criterion that every there and

condition
term
some

of

logicalconsistency(p. 181) negative shall


which abc. and both
are

separate
must

and

its

remain.

Hence

exist

things by

described
as

by ABC,

other
two

things described

But

regards the
case

remaining
we

combinations, aBC
remove or either,

abC,
of add
or

the

is different ; for without

may

these
to

wholly removing
the
new

any

term.

We

might
are

the
=

premises
ABC,

condition

that all BCs


;
or we

As,

BC

which

would

negative "BC

218

EQUATIONAL
53.

LOGIC.

[CHAP.
assertion
about is of

Show
two not terms

that
A

you and

can

make

no

(and
else

these
in

only), which
assertion

either
=

contained

the,

identity (A
The

B),
=

or

contradictory
two out

thereto.

proposition
thus
"

removes

of

the

four

combinations

Consistent Combinations.

Inconsistent

Combinations.

AB. ab.

A". aB.

Now,
and
from

if any

new

assertion be
If
an

negatives
either
A AB

either

or

both

of

hb

"B, it
A A
=

must

assertion
removes

contained
or

in and

inferrible
must
tradict con-

B.
=

it

ab, it
a

B, because

either

and

or

and

b will then It
one

from disappear entirely

the

Logical Alphabet.
could
remove

might

be

said

perhaps
and
one

that

new

assertion

tent consis-

inconsistent
cannot

combination,
done
such

for

instance, ab and
a

A";

but

this

be

except
as

by

contradictory
Al", AB
some

assertion. 0B.
or

Any
and

other

pairs

AB
removes

and

and
letter

ab

"B,

being removed,
contradiction.

and entirely

involves

54.

Is it all

(i) logically (2) physically possible that things


that
are

material and

subject
the
be
same

to

the time
to

law all

of
not

gravity,
material law?

at

things should

subject

the

same

[L.]
is
to

It is

that possible, logically

say,

in accordance and
of all

with

the
not

Laws

of

Thought,
should is
not

that all

things
to

material law

things
In

material
case

be

subject

the
law

gravity.

this

what

subject to

the

of

gravitywould

be

xxi.]
found among

QUESTIONS

AND

ANSWERS. it is not

219

not-things.

But all

logicallypossible
this is

that all be
to

(materialthings)and
to

not-(material things)should equivalent


to

subject

the

law

of

because gravity, of any class


not
one

denying

the existence
This class

subject
condition
be

the law
be
not-

of

gravity.
is

would

by
the

material, and
which

by the other
But of

condition

it would law

material,
described
must
mended recom-

impossible.
as

by

already
student
the

(p. 181)
be

the
to

Law

Infinity, every negative. question


The with

logical term
is
of

assumed
to

have
out

its

work

this

aid

letter

symbols.
As
to the

second of
course

part of the question,what


not

is

not

logically
we are

possible is
restricted
all

possible. physically
it is
all

Hence

to

the

inquiry whether
and
of law

physically possiblethat
should
be be

things
to

material the

things
This

not-material
can

subject
on

gravity.
far,that

only
property
forms
a

answered
of

logicalgrounds
is essential
to

thus

if the

tion gravita-

material it is
As

things and
not
a

part of the
not-material

definition

of

them, then

possible that
of is

things should
of inertia is

gravitate.
perhaps
the

matter

of fact the
test

possession
but

ultimate inertia

materiality ;
an

gravity is proportional to
test.

and

equally good

55.

It is observed
occur

that the

the

phenomena
will

A, B, C

only
What

in

combinations

AB":, adC, and


express

abc.

propositions
between these

the
?

laws

of relation

phenomena

Of remain.
a

the

eight
A.s
we

combinations
see

of
occurs

A, B, C, only these
with and

three
and B is

that

only with B,
that A
=

with

and

only

with

b, k

is

obvious firstly

220

EQUATIONAL
law. ABC
But
as

LOGIC.

[CHAP.
leaves
a

the chief

this law

of relation
must

the second

bination com-

uncontradicted,we
this,which
may

have
AB laws
=

law

to

remove
=

be

either that the

AB"r, or
A
=

else and
both

B B

B*r.
Br

Observe,

however,
are

overlap
B
can

and be

pleonastic, because
Hence the

they

deny
of

that laws

#BC. is

simplest

statement

the

of

relation

A
AB

B,

56. Given

three
"

terms,

"

for
you

instance, water,

blue,

a.r\." fluid,

how

would of

proceed
?

to

ascertain

the

utmost
can

number exist
among

purely logical relations


them

which

[L.]

The

relations
be

of

any

three

terms

or

things

or

classes

of

things must
Laws

governed

in the first These


or

place by

the

universal nations combiat

of

Thought (p. 180).


of

laws

restrict the
to

three
each

things,present thing
cases.

absent,
or

eight
absent

the

utmost

for

may But

be
any

present

giving
of

2x2x2=8

special logical relation


has the of the the
effect

which further
water

may

exist

between

the

things
;

these restricting is
a

combinations
the

relation

that

fluid, prevents
the
or

existence

combination the series of the

water,
any
one

not-fluid.
or more a

Conversely the removal


of

from

eight
Thus,

combinations

expresses

existence
of

of

relation

relations

negativing the
the
;

existence
the
two not-

these

combinations. water, the

removal
water,

of

combinations

not-blue, fluid
law

not-blue,
Thus

fluid,expresses

that

all

water

is blue.

the

XXI.]

QUESTIONS
of any

AND

ANSWERS.

221

logicalmeaning
of

condition

is with

represented by the
those of the

state

the

combinations that the will


be of
utmost

agreeing

conditions. distinct

It

follows

possible
three

number

logical

relations

ascertained the

by taking
terms

eight possible
one
or

combinations
more

and

striking out

of

the

combinations
of

in every
cannot

possible variety of
exceed either
2x2x2=

ways. of

The
the

number

these

ways may

256;
present

for each
or

eight

combinations

be

absent,
ways.
one or

giving
But
more

2X2X2X2X2X

256
cases

this calculation
of the three

will include
terms

many

where

and

their

negatives disappear
in the
from

gether, alto-

representing contradiction
different similar similar
in
to

conditions.

Many
relations
=

selections, too,
character
A
to
=

proceed
form ab

logical
law A A
=

and
to

; thus ;
so

the the
law

AB
-I-

is be

Ab, and
=

BC

is similar is
-143 the
;

AB

-I- ab

; and

forth.

The

tion investiga134

fullydescribed
ist

in the

Principles of Science (pp.


as

ed. vol. i. pp.

154-164)
and

also

in the Memoirs

of

Manchester
v.

Literary
pp.
are

Third Philosophical Society, It is found


for
"

Series, vol.

119-130.

that

the

256

possible selections

thus

accounted

Proceeding
"

from
,,

consistent inconsistent

logicalconditions
"

192

63
i

no

condition

at

all

256

The careful

consistent

logical conditions
fall into
no more

are

found, however,
than fifteen in the

on

analysis to
or

distinct

forms,
table
"

types of relation,which

are

stated

following

222

EOUATIONAL

LOGIC.

[CH.

xxi.

CHAPTER

XXII.

ON

THE

RELATIONS

OF

PROPOSITIONS

INVOLVING

THREE

OR

MORE

TERMS.

i.

THE

doctrine the well-known


of

of

the

opposition
square,

of
an

propositions,
and

bited exhiesting inter-

in

is

important
it is
now

fragment
that

ancient

logic only

;
two

but

apparent

propositions predicate, relationship


combinations,
in sixteen remain themselves
do

involving
not

terms,
open Two up
terms

one

subject
question
of

and of

one

sufficiently

the

the
four

of

propositions.
and

admit and

only

these nine
of

can

be

present
involve

absent

only
There resolve

ways,

which
of
two

contradiction. relation which

only

seven

cases

logical

into

only
Science,

distinct

types
;
a

of

sition. propovol.

(Principles
p.

of
the

pp.

134-7 of

ist

ed.
term

i.,
the

154-7.)
of

With

introduction

third

sphere
are

inquiry
as we

becomes have

immensely

extended. different into


of

There
of

now,

seen

(p. 221)
resolving
The

193

cases

selection distinct
of

of

combinations
of

themselves modes under

fifteen relation

types

relation.
to
'

possible

one

proposition
'

another,
any Such group

including
of

the

sion expres-

one

proposition
complex.
in
number
:

propositions,
of

become
seem

considerably
be
seven

modes
one

relation is

to

thus

proposition

as

regards

another
"

224

EQUATIONAL

LOGIC.

[CHAP.

(1) Equivalent. (2) Inferrible,or


contained
in

the

other, but

not

valent. equi-

inferrible (3) Partially

and

otherwise and
not

consistent. inferrible.

(4) Consistent

but

indifferent

inferrible, (5) Partially partially contradictory.


indifferent, (6) Partially partially contradictory.

(7) Contradictory.
2.

Let

us

take

as

an

example
=

the

proposition
vapour,
its related

Steam and

aqueous

give

pretty complete

analysis of
steam;
aqueous vapour. the form
;

positions. pro-

Let

A B
=

C The

proposition being evidently of

(i)
the

BC,
will be
as

combinations
ABC

contradicted

in be

the

margin.

The

will equivalent proposition Not


steam
=

"

ABc

not

aqueous
not

or

not

vapour.

(0

Abe

An

inferrible
any

but
one

equivalent negatives
of the will

assertion
one,

will be
a6C abc
or

which
not

two,

three, but

four

combinations therefore such


be

negatived by (i).
4

There

4-^-3

r!X2lX2X3

*-*-3-*-?.
or

14

inferinfer

rible and
steam

distinct propositions. logically


; steam

We is

may
not

"

is aqueous
steam

is vapour aqueous
; and
so

what is not forth.

vapour
;
non-

is not aqueous

; what

is not
steam

steam

vapour

is not

226
A any

EQUATIONAL

LOGIC.

[CH. xxn. by taking


one

propositionof
combination such AC
=

the
which

sixth
may

class be

is discovered

spared
and

with

which

cannot,
we

as

abC

and

ABC,

looking

in

the

index,
non-

find

1"C, or
as a as

steam-vapour

is identical

with

aqueous

vapour,

partiallyconsistent, regards (i).


steam

dictory partiallycontraor

proposition
true

It

may

may
not

not

be

that

what

is

not

and

not

aqueous that

is

vapour,
steam

but
not

it is

contradictory to (i) to
of

say

vaporous

is

aqueous. An

example
7 is found
=

a one

simply contradictory proposition


which
=

of
as

Class
AB b ;
c

in

removes

ABC

only, such

ABf
=

again
deletes

aB,

or

not-steam

is aqueous

deletes

AB^
a

c.

2.

As

second
=

example,
or

let

us

take
;

the

propositions
"

(i)

Hand

right-hand
=

left-hand
=

(2) Right
=

is

not

left.

Putting
are

hand

; B

right;
AB Bt.
are

left ; the conditions

evidently

(1) (2)
The consistent and
A

A B

-|- AC.

combinations the student

shown

in

the

margin,
following

may
one

verify the

T^P

which list,

gives

specimen
of of related the

of

each

of

(2)
(i)"
Me

the

seven

c'asses
number

assertions,
Index

the

reference also

Logical

being
abc

added.
B

(1) Equivalent.

B^;
aB
"

bc=abc.
a~Bc. No.
=

No. No.

153.
9.

"c. (2) Inferrible, "c. inferrible, (3) Partially


a
=

abc.
; db
=

15.
No. AB. No. No.

(4) Consistent,

"c.

AB

ABC C
A
=

abc.
=

67.
59. 179.

inferrible,"c. (5) Partially indifferent, "c. (6) Partially

AC

; A

AB^;
35.

ab

abc.

(7) Contradictory,

be.

No.

CHAPTER

XXIII.

EXERCISES

IN

EQUATIONAL

LOGIC.

NOW

give
to

small the

collection student
and

of
to

examples acquire
a

and

problems
mand com-

designed
of

enable

complete
views
for of

the for

equational
the
most

combinational devised in

logic.
book,
and
a

They
but
few
a

are

part
utilised
as

specially

this

few

have been

been

examination from the


a

papers, papers of

have

adopted
These

indicated form
as

other

examiners.
to

questions

perhaps
quoted

partial
in the and
be

answer

Professor when
the

Sylvester's
we

remark,
that of three

preface,
problems

especially

observe

the

questions
can

involving
almost
ad

relations

terms to

multiplied
volving in-

infinitum,
or

without
more

resorting
terms.

like student

questions
will

four, five,

The

readily

gather
in
pure

that

the

number,
is

variety, and
infinite, and

complexity
is such
as

of
we

problems gain
no

logic
of

simply
old

glimpse
i.

in

the

Aristotelian
the of

text-books.

Represent
With the metallic
make

equationally exception
substance coins.

following porcelain
has

assertions there is

"

(1)

no

non-

which

been

employed

to

(2)

With

the

exception
the has
last
not

of

zinc

and

the

metals there

discovered is
no

during
which

hundred
been

years,
to

metal

employed
which with

make

coins.

(3)

"The And

worth that is

of

that

is that this

it thee

contains,
remains."

this, and

[SHAKESPEARE.]
Q
2

228

EOUATIONAL

LOGIC.

[CHAP.
how
also
see

(4)

It

is
a

dangerous
is

to

let

man

know him let him

far he

is but

brute, without

showing
to

his his

grandeur. grandeur,
it is the
one

It

dangerous again
his baseness. him

without
to

It is

[even more] dangerous


both
to

leave

ignorant
to

in

ways

; but

high advantage
and the other.

represent

him

both

(Pascal,Pens'ees.}

2.

Represent

in

the
:
"

form

of

equational logic any

of

the

following arguments (1)


Milton
was a

great poet, and


a

fearless be

opponent
honoured
be
;

of
a
:

injustice ;
therefore

great

poet
of

should

fearless opponent Milton


are

should injustice
be

honoured

should

honoured. habits
"

(2)

The

virtues

either

or passions, faculties,

they
on

are

not

passions, for passions


determination faculties
;
nor are

do

not

depend they

previous
we

they faculties,
; therefore

for
are

possess

by

nature

habits.
can

(3)

There

be

no

person the
a

really fit to exercise

absolute fit a

power, person

because

qualifications requisiteto
consist
now

for such combined


cannot

position would
with

in native
such
a

talent
talent

early training ;
in

be De

possessed
Morgan,
of

early

childhood.

(Suggested by
(4)
One
of

Syllabus,p. 67.)
was

the

masters
was a

chemistry
;

Berzelius

Berzelius One of the

Swede
of

masters

chemistry
a

was

Swede.
a

[D ]
star;
;

(5)

This

heavenly body
stars

is either
;

planet
do
a

or

fixed

all fixed this

twinkle

planets
it is

not

twinkle
star.

body twinkles, therefore

fixed

xxiii.]

EXERCISES.

229

(6)

Show

me

any

number

of that for

men,

and
will

I with

will
one

say

with

confidence, either
raise aliens their
among
to
are

they

accord there
are

voices them.
De

liberty,or (The
stump
of

that

orator's

mode,
that
all

according Englishmen
3. Infer
or

Morgan,
of

saying

lovers

liberty.)
can,

[B.]
of
position contra-

all that

you

possibly
the

by
'

way

otherwise, from

assertion

all A

that is neither

nor

is D.'

[R.] equationallyMiscellaneous
Lessons in

4.

Express

Example

No.

39

in Elementary 5. What bodies which


6.

Logic, p.

317.

propositionconcerning
doubtful
nebulas

nebulas
of
a

and
class ?

vaporous
of

leaves
are

the
nor

existence
vaporous

things

neither

bodies
B

Represent

the fact that A

differs from

in

two

lent equiva-

equational propositions.
7.
are

Prove

equationallythat
metal-elements is
-I- A
or

the

proposition,All
a mere

elements

either
8. What AB Prove A
=

elements, is
between ?

truism.

the and
A

difference
=

the

propositions

-|. A
are

9.

that

if all not-Bs vice


the
versa.

not-As, and

all Bs

are

As,

then
10. no

B,

and

Show
are

that

negative premises

No of
a

As

are

Bs B

and

Cs

Bs, imply the logicalexistence


A
nor

class

which

is neither
11.

C.

Prove

the gem gem gem

equivalence

of
or

the
rare.

following assertions:

"

(1) Every (2) Every (3) Every

is either rich which which which is is


not not

rich is
rare

rare.

is rich. rich
nor rare

(4) Everything
gem.

is neither

is

not

230

EQUATIONAL
12.

LOGIC.

[CHAP.
not

Show

that if metals
are

which
for
are

are

either
as

valuable

or

not

destructible destructible
be

unfitted

use

money, for
use

it follows
as

that
must

metals

which

fitted

money

valuable. the
B?

13. from

Does
A
'
=

proposition A

-|-BC

differ in

force

14.

All

animals

having
a

red

blood

corpuscles are
with from
a

identical

with
What of

those is the

having

brain

in connection may
not

spinal cord.' proposition


a

description
a

you

draw

this

things having
?

brain

in

connection

with

spinal

cord

15.

Luminous

body

is either

self-luminous

or

luminous
and

by
but

reflection

; melted

gold

is both

self-luminous
is
not

nous lumi-

by

reflection.

Unmelted reflection.

gold

self-luminous these
terms

is luminous

by

Represent
the

premises
nous (i) lumi-

and symbolically,

draw

descriptions of
luminous,

body, (2)

self-luminous
not

body, (3) body

luminous

by

reflection, (4)body

(5) body

not

self-luminous,

(6) not-melted
1

gold, (7) not


are

unmelted

gold.
which
are

6.

'There

no

organic beings preciselywhat


it leaves
of

devoid

of

carbon.' what
17.

Determine

this

proposition affirms,

it

denies, and
the

what

doubtful. the

Prove

equivalence
are

following statements
no

"

No

right-angled triangles right-angled ; triangles.


18. All

equilateral ;

equilateral

trianglesare
are

no

right-angledequilateral figures

scalene

have triangles What


above
are

their

three
or

angles equal
'

to

two

right angles.
added
to

the least will make

simplest
three

assertions
All

which

the

it

equivalent to

trianglesare
to two

all

figures which
'

have

their

angles equal

rightangles

XXIIL]
19. What above and All

EXERCISES.

231

equal-sided
least it

squares

have

four

right angles.
added
are

is the makes
have If

extensive

proposition which
to
'

to

the

equivalent
'

All

squares

equal-sided

four

right angles
were

?
assert to

20.

an

orator

to

that
the

Afghanistan
assertions

is

very

poor
a one

country,
were

but
to

it is essential consolidate
a

securityof India, but


into the

reporter

these

two

assertion

that

very

poor

country,
to

Afghanistan,
the

is the
how

Afghanistan
far would of

which

is essential

the

securityof India,

the the

reporter
orator?

have

misrepresented

logical

meaning
21.

Express the following argument


of
sense

equationally: Every
"

organ for eye,


22.

has
case

nervous

communication
all

with
organs

the

brain
the

such
ear,

is the
nose,

with
and

the

five

of

sense,

tongue,
to

skin. from the


the

If

requested
carbon
'

draw

assertion
term
'

'All

coal

contains
answer

description of

metal,' what

would What

you
values

give?
will you
the obtain

23. and that class

for the
that
a

terms

man, man,

brute,
and

under gorilla,
all of
men are

conditions
and all

is a gorilla

included
?

excluded gorillas

from

the

non-brutes

24.

Assuming
vessels
of

that the

armed

steam-vessels and the

consist

of

the

armed
not

Mediterranean

steam-vessels
can

of

the

Mediterranean,
:
"

inquire
-

whether

we

thence

infer the

following results
are no

(1)
(2)

There
the All All

armed

vessels

except

steam-vessels

in

Mediterranean. steam-vessels
not
are

unarmed

in the the

Mediterranean.
are

(3)

steam-vessels

of

Mediterranean

armed.

232

EQUATIONAL

LOGIC.

[CHAP.
consist number of unarmed Calculus
of of of all

(4)

The

vessels

of

the

Mediterranean
any

unarmed

steam-vessels,
any

armed vessels

steam-vessels, and
without
steam.

number
"The

(Boole,
Dublin
"

Logic,"

Cambridge
vol. iii.pp. 25. which metals?
26. How
are

and 190

Mathematical

Journal, 1848*

i.)
describe class
of the

would

you

otherwise

class of

things

excluded

from

the

white, malleable,

Show
are

that
not

the

description
or

of

the

class
B

of and

things
C), is
both

which
as

A, (either
not-A

if

not

then
or

both

follows and How

"

either not-B.

and

not-C,

if it be

then

not-A 27. B
=

do
=

any

two

of

the

three

equations
the

B,

C, C
Frame

A, differ in logicalforce
a

from

third ?
one

28.

sorites with

one

premise negative and

and particular,
29. A

represent
the

it

equationally.
of
.
.

Contrast
=

logical force
-I-

each and
=

of A
=

the

tions proposi.

AB

"!"AC
group

AD

-|.

ABCD
.,

with
A
=

that of the

of

propositionsA
which

AB,
can

AC,

AD,
which. Show

"c.

point

out, moreover,

be

inferred

from
30.

that, under

the

condition
the

of

our

Criterion
are

of
no

Logical Consistency (p. 181)


such

assertion

that there the

things
that

as

fresh-water
are

involves foraminifera, which which


are are

tion asser-

there

foraminifera

not not

fresh-water

beings, and
but
leaves

fresh-water doubtful

beings
the

foraminifera,
which
are

occurrence
nor

of

things

neither
31.

fresh-water From the

beings

foraminifera.
are

premises, ^All gasteropods


are

mollusca,

and the

no

mollusca

vertebrates,' obtain
and

descriptions of

classes

gasteropods

invertebrates.

234

EQUATIONAL

LOGIC.

[CHAP.
that
sea-water
sea-

36. What
is drinkable
water

is the and
scarce

difference
scarce, ?

between

saying
that

not

and

saying

drinkable

is
If

not

37.

from
'

the

premises

'All consist

rectangles are
of

grams,' parallelofigures

and
whose
are

Parallelograms
sides
are

all four-sided

opposite

we parallel,'

infer that all

rectangles opposite
of

parallelograms,being
how parallel, the
far does

four-sided

figures with
fall short

sides

this inference

being

equivalent to
38.
To say and

premises?
Adam Smith is
as

that
a

is the much

father
as

of say
no

Political that he
one

Economy
a

Scotchman of Political

to

is

Scotch
can

father be
a

Economy,
Give

and

that

but

he
of

father

of the science.

the

symbolic proof

this

equivalence.
To

39. else vice

lay down
is

the
A

condition
and B
or

that
else

what both
may

is either
A and C

or

B, is what
versa,
to

is both
state

and down

what disjunctively

be

laid
A

in
B

two

non-disjunctivepropositionsasserting that
and also B the
must

without

is C

be

A.

40.

Reduce

two

assertions

hbc

and

"

ac

to

single one.
41.

Give
B
-I-

good
and

many also

inferences

from

the

proposition
and
those

AC,

carefully distinguishing equivalents,


which
are

between

those
are

inferences

equivalent

which

not.

42.
to

Develop symbolically the


the

term

Plant

(A)

with

ence referunder
;

undermentioned that

terms

(B, C, D, E, F),
are

the

conditions

acotyledonous (b]plants (D) plants


are are

flowerless

(c) monocotyledonous

(E) parallel-leaved

dicotyledonous (F) plants

not

parallel-leaved ; and

xxiii.]
every

EXERCISES.

235
or

plant

is either

acotyledonous, monocotyledonous,
one

dicotyledonous, but
43.

only

of

these

alternatives. the

Completely
"

classifytriangles under

following

conditions

are (1) Equilateraltriangles

isosceles. isosceles.
not

(2)

Scalene

triangles are

not

(3) Obtuse-angled triangles are


have (4) Acute-angled triangles have (5) Obtuse-angled triangles
are (6) Equilateraltriangles

right-angled
acute

three
not

angles.
acute

three

angles.

not

right-angled.
added
to

What the

other of

conditions

must

be ?

comply

with

results

geometrical

science

44.

Among
curvature.

plane figures the


Show
must

circle

is the
same

only
as

curve

of

equal
that
a

that

this
be
a

is the
a curve

to

assert
ture, curva-

plane figure
in which
case a curve

either

of

equal
a

it is also
of

circle,or

else, not

circle

and

then

not

equal

curvature.

45.

Which

of

the

following propositionsare

equivalent to

the first in the

list ? is
not
a

carbon (1) Crystallised

conductor.

(2) (4)

Carbon

which

conducts

is

not

crystallised.
is
or

substance (3) Conducting crystallised

not not

carbon.

Conductors
substances.

are

either

not

carbon

crystallised crystallised. crystallised. composed

(5) Carbon (6)

is either which

not

a are

conductor
not

or

not
are or

Conductors

carbon

(7) Crystalsare
of

either non-conductors

not

carbon.
are

conductors (8) Crystallised

carbon. alternatives combined

46.
with

Prove

that

any
set

set

of exclusive

part of that

produces only

that

part.

236
47. Show

EQUATIONAL
that the conclusions

LOGIC.

[CHAP. Celarent,
case

of each

Cesare,
half

Camestres, and
information

Camenes

give
in the

in

only

the

contained

premises.
statement

48. Verifyby various

trials the
a

that

no

inference

by substitution
combinations
49. Show of

within
not

group

of

propositions can
of

negative

negatived by
Cesare
as

the group

premises.
to

that

and

Camestres and

belong

the

same

type
50.

assertion

Barbara

Celarent.

Assign
to

the

premises
proper

of the

following
of

moods
:
"

of

the

Syllogism Bramantip,
51.
'

their

types

assertion

Darapti,

Camenes.

Prove

that salt
=

any

proposition which chloride,' can


the assertion

is
be
'

contradictoryto
inferred,so
common

common as

sodium

far
"=

it is is

from contradictory,
not

salt

what 52. the

sodium it
or

chloride.' does it
not

Does

follow will

that

any

proposition of
be

mth

type
to

(see pp.
one

221-2)
nth
Laws

always

equally

tradictory con-

of

the

type ?

53. and solved

Refer

to

Boole's

of Thought,
the

pp.

146

"

149,

taking
to

the

premises
in
our

of

complex
as

problem
:

there

be

expressed (1) (2)


ac
=

system

follows

afE
=

("d
AD*

-V

fiD);
-I-

AD* A

(BC
=

be);
;

(3)
work
of the
out

(B

-I-

Eff)

Cd
and

-I- cD

the consistent

combinations,
AQ",
"c. De

infer

descriptions
ACE,
and
e

classes

B, AC,

D,

e,

AB,

AEe,

ab, AE,
that D

BD,

DE,

De, C, CD,

Verify by showing
and
so

multipliedtogether give
54. and If Brown
all
asserts

forth.

that all metals will be

are

reputed elements, reputed


elements

that

reputed

elements

ultimately decomposed,
are

whereas

Robinson

holds

that all metals

xxiii.]
which
amount

EXERCISES.

237

will
of

be

ultimately decomposed,
between of them

what
?

is

the

exact

logicaldifference
the

55.

Compare
and be square

logical force
out

all the
are

following

positions, pro-

point
inferred is
not not
an

which
other

pairs
ones.

equivalent, and

which

may A

from

(1) (2) (3) (5)

equal-sided rectangle.
not

What

is is

equal-sided is
square

square.

What

is

not

equal-sided.
squares.

(4) Equal-sided rectangles are


No A

rectangle which
square but
a can

is

not

equal-sided is unequal-sided
does

square.
nor

(6) (7)

be

neither

anything

rectangle.
not

An

unequal-sided square
letters lustre
to

exist.
:
"

56. Taking

represent
=

qualities thus
; C
=

having
water

metallic
D
=

; B

malleable
;

heavier

than

white

coloured

fusible

with

difficulty ;
each

form conducting electricity; metals


"

descriptions of
copper,

of

the

gold, silver,platinum,
and
of

iron, lead, tin,


then
; BC ;

zinc, antimony, sodium,


the extension the

potassium,
:
"

and
AB

exhibit BCD
;

followingclasses
and
so

HCF;
57.

Ab;

be; B^j

forth.
of

Express symbolicallythe following classes


stone.

things
"

(1) Hard, wet, black, round, heavy,

(2) Thing
not

which

is

hard, wet,
either

either
or

black
not
or

or

red, but

round, and
which

heavy

heavy.
wet,
or or

(3) Thing
and

is either is either
a

not

hard,
and

not

not

stone, but then


to

black

then

round,

heavy

stone.

58. Referring
ed.

the

Principlesof
all the

Science

(pp. 75-6
of A
as

ist

vol. i. p. the

90), develop
A
=

alternatives

limited

by

description
AB

{C

-i-

(E

-i-

F)]

238
and infer
De

EQUATIONAL
of descriptions

LOGIC.

[CHAP.

the

following terms, Ace, Acf, ARrD.

(See
the

Morgan,
12

Formal

Logic, p. 116;
Camb.
Phil,

Third

Memoir

on

Syllogism,p.
59.
A

in the

Trans., vol. x.)


"

Represent straightline
an

this argument
can

symbolically:
circle in
;
two

cut
a

points, and
these
a are

larly simiall the

and ellipse,
of

hyperbola
sections in
two

but

possible kinds
can

conic section

; therefore

straightline

cut

any It

conic

points.
the deaf congenitally

60.
are uses

being

understood
an an

(i)that only
deaf deaf
educated express

mute;

(2)that
not

uneducated

person

is mute, is
not

but

signs; (3) that


does
use

person

mute,
bolically sym-

and

signs ;
the
; educated

these
of persons "c.

conditions who
are

and
mute

describe

classes

deaf;

; deaf-mutes

persons, of

61.
sum

Show

ho\v
one

by

the

process

substitution the

alone

to

up into
is mortal Prove

disjunctiveproposition
; Thomas

assertion is mortal. the

that

John
62

is mortal

; William

that

the

premises
can

of

syllogisms in
in
the

moods
of
a

Darapti
also that

and

Felapton

be

expressed
with

form

and assign its type. singlenon-disjunctiveproposition, this is


not

Show

the

case

the moods

of

the other

three

figures.
that the

63. Prove

following propositions or
:
"

groups

of

involve propositions

self-contradiction
=

(1)
v (2)

B AB

-i-

b.

( B

Kb.
AB
;

(3)

BC

; C

aC. of

64. Analyse the force of


'Some A

Hamilton's

form

proposition,
and 'some'

is

not

some

A,' putting for


terms

'some'

the respectively

letter

and

Q.

XXIIL] 65. What


nor

EXERCISES.

239 'Some
are

does
us

the

assertion

things are
not-A? of

neither

B 66.

'

tell How

about far do

things which
the

conclusions

the
and of

syllogisms
Fesapo,

in
as

Darapti, Felapton, Bramantip, Camenes,


deduced
on

p.

fall short 188, respectively

containing
the

all

the

information

given
C
= =

in the AfrC

premises
-I-

67. Show

that

#BC
=

is

equivalentto
Name of the
or

two

AB propositions, 68. A of the have


must

AEc

and

ab

abc.

the

type.

To

say

that those

whatever either
same

is devoid
of
as

properties of
else be

have
of

B
to

or

of say

D,
that

devoid
of

those

C, is the
B

what those

is devoid
of

propertiesof
A. What Prove

and

D,

but

possesses

C,

must

this.
or

69.

statement
'

statements
a

must

be
not

added

to

the

proposition,
or

What

is

not

square
to

is either the
of
a

equal-sided
in
the
an

not

rectangle/ in
to

order
the

make

assertions

whole

equivalent
What
and the

definition

square

that it is

equal-sided rectangle?
70.

is the difference between

the assertion

that A
be?
=

ABC
71.

pair

of

assertions
one can

b
"

ab, and

Prove AB and
=

that from

of

the

A propositions,

ABC,
not
vice

and
versa,

ABC,

we

infer is the

the
one

other,
which

but
can

point
three

out

which

be

so

inferred.
72.

Give
=

logical equivalents
the

to

the

proposition,
with

ACD
73.

A"CD.

Demonstrate

equivalence
=

of A

AB

-I-

AC

A/
from

AEc, and
Show
A
=

with

Ac

ABr.

74.

how AECD

by
;

substitution
also

alone
A
=

to

obtain,
and A

A
=

AB

obtain
;
ist

AC

AD.

of Science, p. 58 (Principles
75. combined

ed. vol.
set

i. p.

69.)
terms to

Verify

the

statement
same

that any set,

of alternative

with

the

reproduces

that

set

"

that is

240

EQUATIONAL
show that AA
=

LOGIC.

[CHAP.
substitute any
one

say,

A
:
"

when

for A

we

of the

terms following

ABr

-i-i-

A^C #BC A"C

;
"!"
-

ABC
A"r

abc;

-I-

76. Show
assertions
for A

by
such B

trial that if in any


as

pair of logically equivalent


B
=

Ab

and

0B,

we as

substitute CD for A,
for the

and CE

any

logical expressions, such


their

and

for of

B, and
A

negatives
we

in

like

manner new

negatives
assertions.
As

and

B,

always

obtain

equivalent

77.
the

further

example

of

equivalentassertions
"

take

pair of propositions : following


( AB
I Ac
=

ABC, Ate,

and

substitute
A
=

as

follows

"

PQ,
a
=

Qr,
Kb
=

C AbC

PR.

78. Express

ab

and

in what

the

form

of

single disjunctive proposition. belong ?


79.

To

type

does

it

Express

equationally De

Morgan's

forms

of

sition propo-

(Formal Logic, p. 62). (1) Everything


is either A neither
or

B ; As
nor

(2)
80.

Some

things are

Bs.

Verify the
A

identical
-I

equations
"

A A

"!"

aB;
-I-

AB A
-i-

(a
-i-

B)

;
-I-

BC

(A

B) (A

C).

242

EQUATIONAL
are

LOGIC.

[CHAP.
classes and of persons those

all who
who
are

not

members
peers
nor

comprise
elected
but

the

two

neither have
not

persons,
cannot

who

being
88.

peers

been
correct

elected
to

sit. what is
not

It is

say

that

because

A,
and

but C

is

B, is also C, therefore
; but
same

everything
conditions will render

that

is both be laid

is A

what

further

may

down

about

the

things which
other that
all

these

propositions
throw the

convertible

89.

Into

what

equivalent forms
Venus

might
round

we

joint statements
planets
are

is

minor

planet,and
the

minor
sun

large

bodies

revolving
the
to

in

orbits elliptic slightly


90. X
even

within found

earth's orbit ? coexist with


not

If B

is

always
it

A, except

when

is Y in

(which
the without assertion
If

commonly,
cases

though
X

and always, is),

if,

few

where

is

not

Y, C is
can

never

found any

absent

being
C ?
X

absent

also,

you

make

other
91.

about

[R.]
Z

whenever
Y

is present, X

is

not

absent, and
but

times some-

when

is absent,
of X be

is present, determines

if it

cannot

be

said that the absence


Y
or

anything
as

about Z

either
Y ?

Z,

can

anything

determined

between

and

w
92. If

it is false with

that
not

the less

attribute
false

is
the

ever

found C

existing co-

A, and

that
you

attribute

is

sometimes
about 93. B

found in
terms

absent of

from

A,

can

assert

anything

C the

[c.]
Lessons
=

Referring to
the

Elementary

in "!"

Logic, p. 196,
AC,
a,

from C
=

premises
derive the the

there

given (A

AB
terms

BD,

CD),
From of

of descriptions

the

BC,

b, d.
on

94. p.

important problem
lesson, with

of

Boole, described

197
=

same

the of

premises
the
terms

CD, "C,

BC

BD,

derive

descriptions

BC,

B,

b. d.

xxiii.]
95.
each In of
are

EXERCISES.

243

reference
the

to

this

last

named

problem,
ascertain
A
=

examine
o

following assertions, and


with
acD. acd.

which EC
=

them

consistent

the

premises (4) (5)

CD,
=

BD

(i) (a)
(3)ACD
96.
involve will
remove

ac
a

cd Ab abc

acd.

A"CD.
abed.

ABCD.

(6)
=

The

premises

AB

ABC,
What ?

AB,

and

Ac,

self-contradiction.

is the least alteration which

this contradiction

97.
and of

If AB

CD,

what

is the

descriptionof BD,

of bd

cd?

98.
and

What
are

must

we

add

to

the
we

premises, All
may

As

are

Bs tion rela-

all Bs

Cs, in order
is not A is not

that

establish

the

that what

C ?

99.

Verify the
i.

assertions
p.

of Science, p. (Principles
the six

141

first edit. vol.


are

162)
same

that

following propositions
:

all of

exactly the
A B C
=

logical meaning
a
=

BC
AC AB

"!- be -Iac

"C
Ac aE

-I- B^. -\- aC. -|- Ab.

b
c

-I- ab

too.

Write
r
=

out

five similar
-I- pq.

logical equivalentsof

the proposition

PQ
that
to

101.

Prove
=

ab
A
=

abC ABC

is

equivalent to

ac

acB,

and

AB

AC

-I- Abc.

102.

How

may

the

condition

-|- B

ACD

-I- BCD

be

expressed in
103.

four

non-disjunctiveequations ?
M
=

Verify the equivalenceof

M"

and

N
R
2

Nm,

244

EQUATIONAL
for M and
:
"

LOGIC.

[CHAP.

when

we

substitute

the following successively

pairs of

values M N

A,
ABC. ACD
t"D
.1-

Kb,
cD.

(2"
-I- A"cD Vtd.

1N
"

(3)

t\

M /M

|N
each

104.

Express
a

of

the

following propositions

equa-

in tionally

series of
the

non-disjunctivepropositions:
is

(1)
heat

Either

king
solid
=

dead,
or

or

he

is

now

on

the march. either

(2) Either
or

compression
in
a

expansion

will

produce

cold
A" AB

body.
-I- fD. -I-

(3)

-I- bC
*

Cd
=

(4)
105. should

AC

(AB
20

AC) (G/
xxi. p.
c,
use

-I-

cD).
what

In
we

problem
obtain

(chap.
the

194)

description
do
not

of

classes

those

who

take

snuff, and
under the

d, those
several

who

do

not

tobacco,

respectively

conditions

(i),(2) and
200-1,

(3),with (4)?
descriptions of

106.

In

problem
A"r,ab,
and

29,

pp.

draw

the classes

cD. the

107.
sentence:

Represent symbolically
"If

logical import
that
to

of

the

It be

erroneous

to
an

suppose
error

all

certainty
that all

is

mathematical,

it is

equally

imagine

which

is mathematical

is certain."

08.

Represent
from the

equationallythe logical import


of
or

of

this

extract

Oath

Supremacy
potentate,
or

"

No

foreignprince,
any

prelate,person,
power,
or

state,

hath

jurisdiction,

superiority, pre-eminence,
this realm."
ar.e
or are

ecclesiastical authority, Observe


how -especially

within spiritual,

far the alternatives

not

mutually

exclusive.

xxni.]
109.

EXERCISES.

245

Take

the All Some


men

followingsyllogism in
'

Datisi
;

are
"

some

mortals
ia

men
iv

are

some

fools ;
mortals
;

Therefore,
and
as are

Some

fools

are

some

analyse
it
occurs

equationallythe
Show

meanings
which

of

the word
' '

'

'

some

five times.

of the
the

somes

if any

exactly equivalent.
acute

Compare
this mood
pp. 131

result

with

the

remarkably
If

analysis of
Xs

given by Shedden,
"

in his Elements
no. some

of Logic, 1864,
are

2.

Ys, and
prove

for

every
some

there
are

is

thing some-

neither
Xs
nor

nor

Z,

that

things
of

neither

Zs.

[DE MORGAN.]
Boole's equationally

in.

Solve

example

analysis of Chap.

Clarke's The

argument
may
i

(see
be ABD

Laws stated

of Thought,
:

xiv.).

premises

thus

O. O. 0.

B/
AF Ae

O. O. O.

\ KM
CDE Show that every
of be
112.

equational proposition whatsoever,


are

the
X
=

members

which

represented by
into
two

and

in the

Y,
X

may
=

decomposed
and Y
=

propositions of
will
not

forms

XY

XY,
the

which

however

always differ.

Show
result.

also that

operation when

repeated

gives no
113.

new

Take

the definition

Ice

Frozen

Water, and
:

throw

it into

equivalent propositionsof
One
Two One

the

following forms

(1) (2)
(3)

disjunctiveproposition. non-disjunctivepropositions.
and disjunctive and disjunctives
and disjunctive
one one

non-disjunctive. non-disjunctive. non-disjunctives.

.(4)Two

(5)

One

two

246

EQUATIONAL

LOGIC.

[CHAP.

disjunctives. (6) Three (7) Four non-disjunctives.


Are

these

forms ?

exhaustive,

or

can

you

frame

yet

other

equivalent forms
114.

How

many

and
to

what the

non-disjunctive propositions
-\-

will Cd

be
-i-

equivalent
dD ?

single disjunctive,A.b

bC

"

115.
of
two

Express

the

proposition
and then

AB

C-i-D

in

the

form

disjunctive

in

three

non-disjunctive

propositions.
1 1

6.

As

an

exercise

on

Chapter

XXII.,

take

the

proposition :
Stratified
one

Rocks

Sedimentary Rocks,
two

and

discover

(i)

equivalent;
and

(2)

inferrible ;

(3)

several

partially consistent,

inferrible

otherwise
not

consistent; (4)

several

and indifferent,

inferrible ;

(5) two

inferrible, partially

partially contradictory ; (6) one contradictory; (7) one


117.

indifferent, partially partially

purely contradictoryproposition.
same

Treat

in

the
:

general

manner

any

of

the

following premises

(1) (2)
(3)

Blood-vessels Either
Heat An

arteries
I
or

veins. -|must

thou is

or

both

go
contact

with
or

him. radiation.

conveyed

either

by

(4)

equation
Bacon,
mediaeval

is either
an

integrableor
was

not

integrable.
the

(5) Roger

English monk,
which have
and

greatest

of

philosophers.
a

(6) Those
with blood
1 1

animals
a

brain

in

connection have
red

spinal cord,

they alone,

corpuscles.
an

[MURPHY.]
analysis of
the

8.

Perform

exhaustive

relations

of

the

following propositions, comparing

each

proposition

XXIIL]
with
each

EXERCISES.

247

other

in all the

fifteen

possible combinations,
under which
of

and

concerning ascertaining
heads it falls
:

each

pair

the

seven

(1) A (2) Alt


(3)
119. A

BC. Me.

(4) (5)
B
=

BC-i-ofo
=

ab
=

ac.

Kb;

(6) AB
exhaustive

ABC.

Perform
of

similar

analysis of

the

lations re-

the

following propositions:
=

(1) Mercury (3)


Not-metal

metal. liquid, is
not

(2) Not-mercury

liquid.
not-mercury and is
or or

is either is
a

not-liquid.

(4) Mercury
(5) Liquid

metal

liquid.
not-metal.
or

is either

mercury

(6) Not-liquid is
(7) Not-mercury
The
of

either not-mercury is either

metal. not-metal. the in 55

or not-liquid

eight propositions in question


this

45

or

seven

chapter Analyse
we can

may

be

analysed. similarly
:

1 20.

this argument doubt


to

"

As

only

through consciousness,
doubt
of

to

doubt
sciousness." con-

of

consciousness

is

consciousness

by

121.

Illustrate
are

the

principle that
of vol. ed.

the relations

of

logical
ciples Prin-

symbols

independent edition,pp.
that
more

space-relations. (See
i. pp.
39
"

of Science, first
p.

42,

444

; vol.

ii.

469;

new

32"35,

383, 769.)
terms

122.

Show
five
or

if certain

premises involving three

leave

combinations be

unnegatived, the premises

in

question must

self-consistent.

248

EQUATIONAL

LOGIC.

[CH.

xxm.

123.

From

the

point
wisdom

of

view

of

equational

logic
doctrine i.

analyse
of the

the

metaphysical
thus

of

Coleridge's
Talk,
vol.

syllogism

expressed

(Table

p.

207)

"

All

Syllogistic
;

Logic
which
answer

is
"

i.

delusion

2.

/^elusion
;

3.

exclusion

to

the
The

Understanding,
first
'

the This

"

Experience, ought
pronounces
to

and

the the second

Reason.

says

be,'
'

adds
"

'

This

is,'

and

the

last

This

must

be

so.'

250

EQUATIONAL
weakens whereas
,

LOGIC. A

[CHAP.
=

equation always
force A
=

its force ; thus


A
=

ABC

has

the

only
B
-i-

of

BC
but

has
A
-i-

the B
=

force B

|.

Again, only
the

has

the force

",

C -|-

force
3.

|.
The form in best of
one

ostensive

instance

of

logicalpower
the

is found tension inthe


form

in

proposition which
member

embraces

greatest
in

with of

the

greatest
has
;

extension

other. ABC
. . .

This
=

kind
P
"!"

assertion
R
"!"
. . .

the
as

general
terms

"!"

and

the

increase
Thus

the while
of A
10

logical force
A
out
=

to approaches indefinitely

unity.
-I22

B of

-i-

has
A

the value
"!"

|, AB
CDE this

C
of

D
out

has
of

that
32.

16, and

B
on

that

few

other

observations

subject

are

thrown

into

the form
4.

of

the that

followingquestions :
the

Show
A

logical
...

force

of

equations

of

the

form

B,

5. Prove
=

that
-I- R

singleproposition of
there
. . .

the
all

type ABC
n

....

-I- Q

"!"

-,

being

in
to

independent
sides,has
the

letter

terms, and
i
"

no

term

common

both

logicalforce

+ 2"-i 22n
-1

which

approaches

indefi-

to nitely unity as

increases.
any

6.

Can
any

you

discover
not

equation
?

between
term

singleterm
which has
a

and

expression
other form and
?

involving that
one-half

logicalforce
7.
one

than
of

What

proposition involvingonly

and

in

member,

C, D, in the other, has

the lowest

possible

logicalforce
8.
terms

What which
?

is the
can

utmost

number

of

combinations

of

be

negatived
number

without

producing

tradiction con-

9.

What

is the

utmost

of

combinations

of four

xxiv.]

MEASURE

OF

LOGICAL

FORCE.

251

terms

which
three of

can

be

negatived
?

by

proposition

involving

only
10.

them

What

two

propositions
number of

involving

five

terms

negative
self-

the

utmost

possible
?

combinations,

without

contradiction
Show

ji.

that

successive
that
. . .

propositions
is
to

of

the of that

type
the

A
=

AB,

B
=

BC
.,

say.

in

the

form

Sorites,

leave

combinations

unnegatived,

so

the

m
.

-f
2"t
-f

logical

force

is

"
"

12.

Prove

that

the

amount

of in

surplus
a

assertion,
as

or

lapping over-

of

the

propositions,
increases

Sorites

treated the

in
law

the of

last

question, surplusage.

indefinitely.

Investigate

the

CHAPTER

XXV.

INDUCTIVE

OR

INVERSE

LOGICAL

PROBLEMS.

1.

THE

direct in

or

deductive the

process combinations with certain

of

logical analysis
which are,

consists the The


Laws

determining
Problem is

under

of Thought, consistent
"

assumed

conditions. sistent inconAs

Inverse with

given
to

combinations those conditions.

conditions,
in the is

determine

explained
inverse

Principles of Science

(chapter vii.) the


consists
agree Mr. in

problem

always tentative, and


their

ing inventthose H.

laws, and
before
us.

trying whether
An

results

with
M.

American that in

correspondent, making
in trials
we

Doolittle,points out
pay attention
or

should
to

always
their mark either

to

combinations

proportion

frequency inof of

solitariness,infrequency being
The

the

deep

correlation.
or

infrequency may
problems
and

be

that

presence
2.

of

absence. consist of series of


are

The

following inductive
of
to

combinations

three

terms

their under

negatives which
the condition

supposed
certain

remain
or

uncontradicted

of

proposition
to

group

of propositions.
them the

The

student

is

requested

discover then

such

propositions, express
to

them

equationally, and
the

assign
If in any

proper

type

in
are

table

on

p.

222.

problem
to

the the

conditions fact.

the self-contradictory

student

is

detect

CH.

xxv.]
I.

INVERSE

PROBLEMS.

253
IX.

IV.

VII.

ABC
abc

ABC
aEc

MC
Kbc aEC aEc

A"-

KbC aEC
abc

al"C
11

v-

Me
in.

viii.

x.

aEC

ABC aEc

ABC aEC

Me

abc

abC

abc

3.
to

Assuming assign

each

of the excluded

following series
or

of

combinations

consist

of those the in

contradicted
are

by

certain

tions, proposito

propositionswhich
each refer table

just sufficient
these

exclude

them

problem,
them
:

express

propositions
to

and equationally, proper

as

in the

last

question

the

type in the
I.

V.

VIII.

ABC

ABC

ABC

n.

A6t

aEC abc

abC
VI.

AB,
III.

Me
atic

T-v IX"

abc

MC
vii. iv.

Me

aEt

aEC
aEc

ABC
abc

abC
abc

abC

254 I
now

EQUATIONAL
4.

LOGIC. of

[CHAP.

give
Each

series series the

inductive combinations of
those

problems
consists such
as

involving
of

four

terms.

of

those

which certain

remain

after

exclusion

contradict
The

conditions.
are

Required
in order of

conditions.

problems

ranged

difficulty.

i.

IV.

VII.

ABCD
abed

ABCD
AEed

ABCD AECd
AB^D Abed

AEeD

aECd
aEed abed

abCV
abed
II.

ABCD AECd aBCD aECd abCV abCd


abeD
v.

abed
XI.

AB^D

ABCD ABO/
AEed

VIII.

AbCd AM3 aECd

Abed

abCd hbCd
abed afcD

abCd
abed

abed

abed

XII. in.
VI.

IX.

ABCD
ABrD

ABC;/
aECd

AECd

AbCd
aEed

abCd abeD abed abed

aECd
abed

abCd

5. I terms,

next
as

give

few

similar

five problems involving

or

six

follows

XXV.]

INVERSE

PROBLEMS.

255

II.

IV.

VI.

ABC*fc aBCde abCde


"^DE

ABCDE

ABCDE ABC^E

abcde

abCde

VII.

VIII.

ABCttef AB^E/
A"CDef

abCDef
afaDEF

IX.

AB^DEF ABrD^F aB"/EF a^CDEF

abCDef abcDef

256
6. volume As

EQUATIONAL
the reader who is in

LOGIC.

[CHAP.
of the

possession

present

will have
be well
to

plenty give
were

of unanswered
the
answers

inductive
to

problems, problems
of

it may the
and

here
set

the

like kind

which

in the
127.

of Science,2nd Principles They


are as

subsequent editions, p.
I. A
=

follows

A~Bc

-I
a
=

II. A
III. A A A V.

AC;
AC
D D
; ; ;

0B.
=

ab
=

abc.
"]"

IV.

B
a
=

CD
B* Kb
=

cdy

or

their

equivalents

-I- 6C.

ab

abCd;
0BC
=

A/5D

AB^

AB"T;

aECD.
=

VI. VII.

E V
-

; bC

""CD /; B

;
-

(a

-V

b) c

"^/v/"f.

""

"B.

VIII. IX.

(Unknown.)
^D
=

-I-/; av/E

B^F;

ACF

AC./F;

X. and

This

example
is the
to

was

set

by

me

haphazard,
which

like Nos.
any

V.

VI., that
of

say,

by merely striking out fancy


me

binations com-

logical alphabet F.R.S.,


"

dictated.

Dr.

John

Hopkinson,
solution

has

given

the

following

rather

complex

(1) d (2) b
(3) A/ (4)
E eE ^^

abd. b
=

(AF (B/

-i-

ae\

A/BrDE.
E
-I-

^ACDF).

(5)
(6)

^ABCDF.

(7) "^
Can
a

simpler answer

be

discovered

258

EQUATIONAL

LOGIC.

[CH.

xxv.

The

third

answer,
terms.

given
The from student

by

Mr.

R.

B.

Hayward,
inserted the

M.A.,
as

is

in

the

simplest
are

propositions
I formed that

the

fifth

answer

those

which

combinations
of

deductively.
answers

The deducible

may from
any

prove other

any

one

these

is

without

descending
AC is the
con-

explicitly trapositive
b
=

to

the of

combinations

thus

C
=

ac

B
=

"!"

is

equivalent

to

cd,

and

so

forth.

CHAPTER

XXVI.

ELEMENTS

OF

NUMERICAL

LOGIC.

i.

LET

logical

term,

when
so as

enclosed
to

in

brackets,
the

acquire
of

quantitative

meaning,
which Then

denote

number connoted ing possess-

individual
the

objects
term.

possess

the number sake

qualities
of

by

logical

(A)
for

objects
the

qualities
of As.

of

A,

or

say,

the

of

brevity,

number

Every
numerical of
the

logical equation equation.


numbers. A that
are

now

gives
of

rise

to

corresponding
ness same-

Sameness
Hence

qualities
B denotes

occasions the that

if A

identity (A)
=

of

qualities of
It is evident

and

B,

we

may

conclude

(B).

exactly

those

objects,
under
A

and
must

those be
can

objects
hended compredraw
an

only, which
under

comprehended
B. It follows
we can

that draw
B
=

wherever
a

we

equation
numbers.

of

qualities,
Thus,
from

similar infer
A the

equation
=

of and of

C,

we

similarly
As and

from Cs
are

(A)

(B)
to

(C), meaning
number

number
we can

equal

the

of this

B's,
does

infer
to

(A)

(C).

But,

curiously
and

enough,

not

apply
=

negative
means

propositions
that
A

inequalities.
with

For

if A differs

"-j

is

identical

B,

which

from

D,

it

does

not

follow

that

(A)
Two may

(B)

(D).
in

classes
agree

of

objects

may

differ

qualities,

and

yet

they

in

number.
s
2

260 The

NUMERICAL

LOGIC.

[CHAP.
for the
sense,

2.

sign

"!" being used

to

stand

disjunctive
that

conjunction or,
"I- is not

but

in

an

unexclusive with
+
.

it follows
not

identical
statement

in

meaning
number

It does
or

follow

from
o

the
As ;

that A
the

is either B of Bs

C, that the number


to

is
some

equal

to

added

the number
been

of

Cs

objects,or possibly all,may


Thus, if
or we

have
An
or

counted

twice
an

in this addition.

say

elector is either

elector
not

for

borough,

for

county,

for

university,
is

it does
to

follow of

that the total number

of

electors

equal
three

the

number

borough, county,
some men

and

universityelectors
in
two
or

added of the This


we

together;
classes.

for

will be

found

however, difficulty,

is avoided alternative subclass into from

with into which

great

ease

; for

need

only develop
and and of strike then

each
out

all its appears

possible
more

subclasses

any

than the
A
=

once,

convert

numbers,
A
=

connected
"!" C
we

by
get
terms

sign
BC
as

addition.

Thus,
-I-bC; but

B
one

-I- B": -I-BC

out striking

of the

BC

we being superfluous,

have

BC

-I- Be

"!" ^C.

The
any

alternatives

are

now

or exclusive, strictly we

devoid numerical

of

common

part,

so

that

may

draw

the

equation

(A)
Thus, if
A B
=

(BC)

(B.) +

0C),

elector,

C
D

county
=

elector,

borough elector,
the

university elector,
-|- C "!" D

we

may

from

proposition A

draw

the

numerical

equation

(A)

(BCD)

(EC"*)

(B*D)
+

(Ecd)

(bCd)

(bcG)

XXVT.]
3.

QUESTIONS

AND

ANSWERS.

261

The

data
:

of any

problem

in Numerical

Logic

will be

of

two

kinds

(1) The
in

conditions logical

governing
classes

the

combinations

of certain

qualitiesor

of things,expressed

propositions.
numbers of

(2)

The

individuals
those

in

certain

logical

classes

existingunder
the

conditions.

The numbers

qutzsitaof
under the

problem
in certain

will

be

to

determine

the
isting ex-

of individuals
same

other

classes logical*
so

logical conditions,
determinable

far

as

such The

numbers

are

rendered the
or or

by
of

the often
a

data. consist

usefulness

of whether not,

method
not

will, indeed,

in

showing
or

the

magnitude

class

is determined
or

in

indicatingwhat
It will appear,
we

further

hypotheses
an

data

are

required.
not

too, that where

exact

result which

is
an

determinable

may

yet assign limits


lie.

within

unknown

quantity must

4.

In
100

certain

statistical
are

investigation,among
Bs
cases cases

As

there

found
out

45
100

and

53

Cs
A

that

is to B

say,

in 45

of

where

occurs

also occurs,
it to be

and also

in 53 known

occurs.

pose SupB

that It

wherever
is

is,
to

also

necessarily

exists.

required

determine

(1)

The

number

of

cases

(allbeing As)

where

exists

without
of B
nor

B.
cases

(2)

The

number neither

(all being As)

where

exists.

262 The data

NUMERICAL

LOGIC.

[CHAP.

are

as

follow

/(A)
Numerical

100

(i) (2)

equations J (B)

45

t(C) =53
Logical equation
The with that
B
. .

(3)

BC. that the


true

equation logical
the class
are

asserts

class B mode

is identical of

BC,
Cs.

which Two the

is the distinct
of B
not

asserting
from

all Bs
:

results follow the class BC

this,

namely
with
no

ist, that number

number class
are

is identical
are

the

of
Bs

the which

; and

2nd, that there

such The

things as

Cs.
to two

logicalequation

is thus

equivalent

additional

numerical

namely, equations,

(B) (Br)
We
Law

(BC)
o

(4) (5)
of

have of

now

means

solvingthe problem

for,by

the

Duality, (C)
=

(BC) (B)

(6C)-t

By (4)
=

Thus 53
or
=

45

the

required number
obtain the number
=

of ^Cs

is 8.

To

of
+

A"s,
+

we

have

(A)
100

(ABC)
45 *"

(ABf)
o

(AtC)
8

+ +

(Me] (A"r).

Hence

(A")

47-

xxvi.]
5.

QUESTIONS
The
in any

AND

ANSWERS.

263
of

difference
two

between

the numbers
is

objects
to

classes between
in each

whatsoever,
the

equal
of

the

difference which other


are

numbers excluded

objects
from

class,but

the

class.

Take
classes A

(A)
and

and

(B)

to

represent

the

numbers

in

any

two

B ; then

(A)

(B)

(AB)
-

(A")
-

(AB)

(aB)

(AJ)

(aB).

6.

If the

number
Bs

of
are

As

be
not
are

x,

of Bst"e

y,

and

of the

those
number

which of As

As
not

be
B will

/,

then
+
x

which

be/

"y.

ettingdown
x
-

the have
+

several

logicalquantities represented by

y,

we

("B)
Four
terms
cross

(AB)

(A")

(AB)

("B).

out" leaving only

(AS) as required.

7.

Represent
Thomson's

the Laws of the


of the

following argument
of Thought,
army army
were
were

from

p.

168

Three-fourths Three-fourths Therefore Prussians.


some

Prussians

were

slaughtered; slaughtered
were

who

Taking

A B

members

of

the

army,

Prussians, slaughtered,

264
the

NUMERICAL

LOGIC.

[CHAP.

premises are

expressedas

(AB) (AC)
The which
number the of Prussians

f(A) f(A).
of

slaughteredwill be (ABC), true : following equation is identically


=

(ABC)
values inserting

(AB)

(AC)

(A)

(Afc);

(ABC)

f(A)
-

f(A)
+

(A)

(Afo)

J(A)

(A3,).
Prussians it
who

That

is to

say, the of

the number
and

of exceeds

is slaughtered
a

at
to

least half

army,
men

by
were

number

equal

the number
nor

in the army

neither Prussians

slaughtered.

8.

If the number
number know of Bs

of As

which
are

are

Bs

is/, and
do

the
we
arc

which

Cs

is q, what

concerning

the number

of As

which

Cs?

We

have

the
=

self-evident equations: following


ABC
+ +

AC

A"C
ABr cfec
+

ABC

ABC
B B
+

aEC

+
=

AB values

EC

A"C

aRc.

the Inserting

we given,

get

We

see

that the

data

are

the
from the

number
zero

of As up
to

which

quite insufficient for determining Cs. are They may be anything


number
we

the whole

of As also

or

Cs. number

To

make

question determinate

need

the

of Bs,

266 The

NUMERICAL

LOGIC.

[CHAP.
some

meaning is,that
to at

there the

must
sum

be of half

Cs

which

are

As, amounting
the ze/, and Cs. unknown The

least

the the

w quantities

and As the

excesses

beyond
be

Bs

which

are

number

(aRc) is wholly undetermined negative.


Thus
w

by
w'

premises, but
lower

it

cannot

is the

limit of

(ABC.)

10.

(i)
Some

For Zs

every
are

there Ys.

is

an

which

is Y

; be

(2)

not

What

inferences

can

drawn

This

general problem given by 85), and


be in
some

De

Morgan
parts

in his of his

Syllabus writings,
differ

(p.
would

29, art

other

thus

represented
from

in

my of

formulae, which
De

however, essentially,
The

those

Morgan.

premises are (XY)


=

(Z)
be

"

......

(i)
Morgan
there
meant
are

in which
are

would Xs

zero are

if De
Ys

that

there

not

more

which

than

Zs, but

just an

equal

number

where

is

some

number. positive

Developing (i)we

get

(3)

(XYZ)

(XY*)
+

(XYZ)
+
m.

(XyZ)

(*YZ)

(xyZ)
term

Striking out
sides, we
have

the

common

and of Xs

adding (Xj*) to
which
+
are

both

for the

number
+
+

not

Zs

(X*)

(Xy*)

(*YZ).

xxvi.j

QUESTIONS
after

AND
the

ANSWERS.

267
term,

Again,
reduces
to

strikingout

common

equation (3)

(XX z)
which

(XyZ)
of

(xZ)
which

m,
not

gives as

the

number
=

Zs

are

Xs

(xZ)
The

(XYz)
compare

(XyZ)

m.

student

should

these in the

results with

those

of

the

less

general problem given


first

new Principlesof Science,

edition, p. 169;
De

edition, vol.

i. p.
a

191,

and

also

with of

Morgan's

results

expressed

in

different totally

kind

notation.

ii. are

If

or

more

Xs do
we

are

Ys, and
about
Zs ?

or

more

Ys of

Zs, what
which
are

know

the

number

Xs

therefore

This definite

question represents syllogismas


and the whole
the

one

case

of

the

numerically syllogism,

treated
Z to be

by

De

Morgan
terms

(Syllabus, p. 27).
of the

Taking X, Y,
he

the three
:

adopts
u
=

followingnotation
number

of

individuals

in

the

universe

of

problem.
of of Xs. Ys.

number number

y
z

number
m

of Zs. any

Making
De "YZ letters
means

denote

positive number,
m or more

wXY
are

means,

in

Morgan's system,
means

that
or

Xs Zs.

Ys.

Similarly
Roman Thus

that

more

Ys
the
not

are

Smaller

denote that the


m

the
or

negativesof
more

largerones.
Ys,
and
so on.

wXy
draws

Xs

are

From

two

premises

wXY
-

and

nYZ,
Let

De
us

Morgan
consider

the

conclusion

(m

jy)XZ.

what

26S

NUMERICAL

LOGIC.

[CHAP. premises
may be

results

are

given by
the
+

our

notation.

The

represented by (XY)
where
m
=

equations
m

(YZ)
the n'
same as quantities

"

"', Morgan's
and the

and and

are

in De

system,

m'

and the is
n

two

unknown of XYs

but is
m

positivequantities,
or

indicating that
number of YZs

number
or more.

more,

The their

possiblecombinations negatives
the is
at
are

of the three terms

X, Y, Z, and
all is in

eight in number,
of
to

and

these

together
u.

constitute

universe,
once

which be

the

number

The
for be

problem
there
are

seen

indeterminate the numbers

reality ;
have
to

eight
and and

classes there
n,
are

of

which

determined,
u, x, y, z, m,
we

only

six known determine

quantities, namely,
them. ingly Accordnot

by
De

which

to

find

that

Morgan's
little
or

conclusion, though
no

absolutely erroneous,

has that

meaning.
y]
or more

From Xs
are

the Zs.

premises
Now
m

he

infers

(m

(XY) (XYZ)

(YZ)
-

(Y)
-

m'
m

n'
n'

(xYz)

Thus

De

Morgan
a

represents

the number
than

of the whole its side


own

class,

XZ,
It
-

by
is m'

less quantityindefinitely
true

part, XYZ.
-

quite
-

that this

if the

second has which

(XYZ)
must

(xYz)
be
at

of

equation
of in
any Xs

value, there
are

at

least

this number

Zs; but
may

as zero

(xYz)
or a

may

exceed

(XYZ)

degree, this
is

give
number

negativeresult,while
The
true
as

there

reallya large number


for the

of

XZs. XZs

and

complete expression
:

of

is found

follows
+

(XZ)
=

(XYZ)
+
=

(XyZ)
+

(XYZ)
(xYz)

(XYz)
m

(XYZ)
+
n

(xYZ)
-

(Y)
+

(XyZ)

m'

u'

(XyZ)

(xYz).

xxvi.]

QUESTIONS
these
The
seven

AND

ANSWERS.
n, and

269
y
are

Among
known.

only quantities,
m

m,

definitely quantities,
XYs

two

and

n'

are

two

indefinite
number

expressing
YZs,
numbers

the
there

uncertainty
are

in

the

of

and

while
of

two

other

unknown the

the quantities, solution of the

XyZs

and

xYzs

arising in

problem.

12. are

If

or

more

Xs
do
are
we

are

Ys, and
about ?
in the

or

more

Ys of

Zs, what
which
two

know not-Zs

the

number

not-Xs
From

the

same

premises
and

as

last

problem,

namely
"YZ

De

Morgan

draws

the
+
n

conclusion
+
u
"

(m
that is
to

"

"

z)-xz
"

say,

the number the brackets

of
or

not-Xs
more.

which This

are

not-Zs

is the is

quantity
To

in

conclusion

equivalentto
prove

that in the this result

preceding problem.
it is

requisite to
of

develop
the

all the
n, u,

combinations
xt y, z;

numbered
are

in each

the

quantitiesm, signs indicated

there

twenty-six terms
out.

in all which the

reader

may De

readilywork
Morgan,
we

Giving

them

by

and find

out striking

pairs of positiveand
used,
in the last

negative

terms,
m

only
terms

two
are

combinations
as

left, together with

and

#',which
the

problem,
is
m

to

express

fact that

De
means

Morgan's proposition;//XY
that thus
m

not
or

but really definite,

or

more,

that

is

(m

m'}

Xs

are

Ys.

We

obtain
m'

(xy)
in which find that the
De
term

(xyz) (XyZ)

"

(XyZ)
wholly

"

"

is

undetermined.
as

Thus of

we

Morgan's

method

gives us

the value

(xy)

270

NUMERICAL

LOGIC.

[CHAP.

part of itself
The

diminished ties. quantiby three unknown (xyz), number (xz) may accordinglybe of any magnitude, the lower limit
The

while
zero,
or even

assigned to problem Morgan's


made

it

by

De
fact

Morgan
a

is

negative.
one,

is in

wholly
sions conclu-

indeterminate Similar which

and
may

De be

solution

is

illusory.
and "Yz

remarks
De

concerning
Thus, from

other

Morgan
are

draws.
and "Ys

;#Xy
are

(mXs

or

more

not

Ys,

or

more

Zs) he

infers

(m
But

"

x) xZ

and

(tn +

"

z) Xz.
these
results

it will be the

found

that by analysis
:

the first of

has

followingmeaning

(xZ)
that is
to

(xYZ
limit

(XYz)
of the number

say,

the

lower

class xZ
of

is

.part of
class

xYZ, itself,
XYz

diminished

by the

another

of

unknown

magnitude.

13. As

If the
and

fractions

and
a

ft of the

Ys

be

severally unity, 8.] gives


of

Bs,

and

if
some

(3 be greater
are

than

it follows

that

As

Bs.
vol.
x.

[Cambridge Phil,
In

Trans,

part i. p.

his
above

third
as a

memoir
very

on

the

Syllogism
remarks

De

Morgan
the
"

the valid that


a
=

general

statement

of the that

conditions

mediate is
i

inference. the
=

He

logician,
demands

to
or

say,

ordinary Aristotelian
i,
or

logician,
then middle

ft

both
of
a

he

can

infer."
term.

This

represents
The
as

the condition

distributed
are

numericallydefinite
:

conditions

readilyrepresented

follow
The

premises

are

(Y)

(AY). (BY).

".(")-

xxvi.]
Hence

QUESTIONS

AND

ANSWERS.

271

(a
(a
or +

ft)(Y)
=

(AY) (ABY)
(Y)
=

(BY)
+

(A"Y)
-

(ABY)

(aBY),

ft)(Y)

(ABY)

("*Y),

(ABY)
We learn

(a

ft

i) (Y)
AYs

(aW).
are

that
+

the
"

number

of

which

Bs the

is the mined undeter-

fraction

(a

ft

i) of (0"Y),

the which

Ys, together with


cannot
a

number

be

negative.
than
a

But,

according
hence value.

to

the

conditions,
side will of the there the
are

(3 is greater
must

unity; positive
are

the

second

equation
be

have

Not this
more we

only
is
as

(a

ft

i) As
of

which

Bs, but
as

merely
there the

lowest units

limit,and
number Y once,

there

will be

many

in the
term

"z"Ys.

If
a
=

distribute
we

middle

by making

i,

have

(ABY)
The
term
are

ft (Y)
.

o.

(a"Y)
As.

of

course

vanishes

because have

the

whole

of

the Ys

Again,

if

ft

i,
a

we

(ABY)
If both
a

(Y).

and

ft become

unity,then

(ABY)
It
must

(Y).
that these
are

be

noted carefully
number of

however
As

results do

not

show

the whole
are so

which
of

Bs, but
term

only those

which
has

within about

the the

sphere

the of

Y.

Nothing
are

been

said

combinations

not-Y, which

quite unlimited

by

the

conditions

of the

problem.

14.

"

If

occurs

in

larger proportion
of

of

the
B is

cases

where

is than

the

cases

where

272

NUMERICAL

LOGIC.

[CHAP.
a

not, then

will B
cases

also

occur

in A

larger
of

tion proporcases

of the where
This 'On Mill.
ad

where

is than

the

is not." asserted in

general proposition is
and its

J. S.
is

Mill's

chapter

Chance

Elimination,' but
I

not

proved by
2,

(System of Logic, Book


;

III., chapter xvii. section


do
not

finem

fifth

edition,vol. ii. p. 54.)


of it The shortest of the

remember
not to

seeing any proof


mind self-evident. is the

given elsewhere, proof


I have

and

it is is
a

my

however, following,
been be

proof of
to

its

truth, and
The

able

find.

condition

problem

may

expressed

in the

inequality

(AB)
or

(B)

"

(A*)

(6),

in reciprocally

the

inequality
:

(B) unity from Subtracting

(AB)
each

"

(") : (AJ).
we simplifying,

side,and
"

have

(aB) : (AB) Multiplying each


obtain
side
of

(at) : (A").
inequality by (A")
:

this

(aE)

we

(A")
Restoring unity to

(AB)

"

(ab) : (aB).
simplifying (a): ("B),

each

side, and
:

(A)
or

(AB) (A)

"

reciprocally

(AB)
which
occurs

" to

("B)
be

(a),
that
B

expresses in
cases a

the

result

proved, namely,
cases

larger proportion of
where
A

the

where

is than

of the

is

not.

15. In
and

company

of

individuals, p have
Determine
some

coats

q have

waistcoats. them.

other

relations

between

274

NUMERICAL

LOGIC. tfie Manchester

[CHAP.

on

the

same

subject (Memoirs of
with

Literary
iv. p. 330,
on

and

Philosophical Society,Third

Series, vol.
a

Session
p. 331,

1869-70), written
of Boole's

knowledge,
the

as

stated

publication on
represent
means

subject.
in the

16.

Can

we

syllogism

extensive
?

form
In

by

of

numerical
remarkable in

symbols
paper Mr.

very

and interesting

read

to

the

Belfast

Philosophical Society
has

1875,

Joseph

John
the and

Murphy
matured

given
He has

kind

of

numerical
a more

notation condensed

for

syllogism. Taking imperfect


substances
one assumes

since

printed
in
"

account

of his views

Mind,
is
are

January, 1877.
one

the gases

syllogism
"

Chlorine
gases

of

the

class

of of

imperfect
in substances

part

of the

class

freely soluble
of the

water;

therefore, chlorine
in
water
"
"

is he

of

the class

freelysoluble

symbols
Chlorine,
y
=

Imperfect
in

gases,

substances the first

freelysoluble
in the +

water.

He

expresses

premise
y
=

form

p,
there

p being
are

positive numerical things


besides second

that quantity indicating in the


q,

other

chlorine takes

the

class

of

imperfect

gases.

The

premise
z
=

form

similarly indicatingthat
q

besides

imperfect

gases

there

are

things in

the

class

of substances
z
=

freely soluble
/
+
q,

in water.

Substitution which would


+

gives
seem

to

prove class

that of

besides substances

chlorine

(x) there
in

are/
water.

things in

the

freelysoluble

xxvi.]
The modern

QUESTIONS
student who wishes should
of my

AND

ANSWERS.
master

275

to

the

difficulties of with great


out

the

views logical does


not

read these

papers the

care.

Space

admit
can

arguing
follows

matter

at

full

length,and
to

therefore
views

only briefly express


as :
"

my

tions objec-

Mr.

Murphy's
true

His of
+

equations are
and
"

equations in extension, and, with his


can

use

they

only hold
this

when

his terms

are

numerical show

quantities.
with

Under
correctness

assumption
the numbers

his

equations

perfect
not
+

of certain

classes ; but
Because
x z
=

they are
x +

therefore
we

equivalentto syllogisms.
that the

cj,

learn

number

exceeds

by p

+ to

g, but

it does class of

not

therefore

follow that

chlorine
z.

belongs
short, as

the

substances
out at

represented by

In

I have

pointed logical

the

beginning

of this
ones

chapter (p.259),from
not

equations arithmetical
also
I

follow, but

vice versti.

(See
193.

Principles of Science, p.
Mr.

171 ; first edit. vol. i. p.

hold, therefore, that

Murphy's
but
at
a

forms the
same

are

not

really
I
am

representations of syllogisms ;

time
never

quite willing to
settled
and

admit

that

this is

question

yet
is very

demanding
that Hallam iii. pp.

further inserted

investigation. It
in his
note to

remarkable

History of Literature

(ed.1839, vol.
of the but which
has

287-8) a long
similar remained

containing a theory
that of
Mr.

syllogismsomewhat
hitherto all other and

Murphy,
to

unknown

alike

Mr.

Murphy

logicalwriters.

CHAPTER

XXVII.

PROBLEMS

IN

NUMERICAL

LOGIC.

1.

IF

from the
same

the
number

number of
as

of
them

members who
the of
are

of
not

Parliament

\ve

subtract

military
number of
not

men,

we

get
men

the
we

result the

if
number

from

whole them who

military
members

subtract

are

of

Parliament.
2.

Prove

this.
of
x

In

company
men,

individuals
z are

it is

discovered Find
an

that

are

Cambridge
the number

and

lawyers.
men

expression
who values.
are

for

of

Cambridge
its greatest

in

the

company

lawyers,

and

assign

and

least

possible

[BOOLE.]
'

3.

Prove number

that
of

in

any

population
and the the who
number
are

the

difference
of

between

the
to

females between of if
to

number number
are

minors females

is who

equal
are

the

difference and that


number

of

not

minors,
4.

minors
the

not

females. which
sum are

Show the

of

metals

red,
to

we

add of of

which number of
are

brittle, the
after and which

is
of

equal
the

that

the

whole

metals

addition

number
traction subnor

metals of

which the

both

red metals

brittle, and
are

after red

number

of

neither

brittle. 5. What is the value


of

the

following

expression

(A)
6. Prove
to

(AB)-(AQ?
number of

that the

the

quadrupeds
not

in

the

world
which

added

number

of

beings

quadrupeds

CH.

xxvii.]
stomachs stomachs
stomachs
x

PROBLEMS.

277

possess

is

equal

to

the whole the

number
of

of

things
not

having having
7.
are

together with
which
are

number

things

quadrupeds.
numbers number
are

If X

and

y be

the respectively
m

of

things which
are

and

Y, while
and
n

is the whole
number

which either
m

both
alone
n

X
or x

and
Y
+

Y,

the is

which

X
+

alone, what "'?


Let
" x

the

relation

between

and

8.

be

the

whole

number which
are

of

things
y

under the

sideration, con-

the

number if
m

A, and
of
x
-

number
are

which both which


9. A

are

B ; then

be
m

the +

number
u
-

things which
y

and
are

B, show
A
nor

that
B.

is the number

neither each

Taking

logicalterm
in its

to

represent

the

number

of tions equa-

things
:

included

class, verify the

following

(A (A

AB)
ABC What

(A
AD

AC)
-

AB

AC
=

ABC
AB
=
-

Kbc. AC
+

AB) (A
-

AC)
+

(A
+

AD)
ACD
-

ABD

ABCD

Kbcd.

10.

is the

product

of the

of logicalmultiplication

the

four

factors

(A
Give
11.

AB)

(A

--

AC)

(A

AD)

(A

AE)?

another Show
:

expression for
that
the

its value. is

following equation

necessarily

true

AC

"C

A"fc

A 8

aBc.

12.

What
B

happens
does
an

in Problem
exist
at

if it be that

discovered
=

that

the

class 13.

not

all,so
the

(B)

O ? between

Find
+

expression for

difference

(A)

and

(B)

(C).

2?8
14.

NUMERICAL

LOGIC. of

[CHAP.
C
B

What

is

(a) the
?

minimum that
may

percentage
coincide

that must, under the

and

the (J3)

maximum

with

following conditions
80 70 per per
cent,

of As of As

coincide coincide in

with with

50
60

per

cent,
cent,

of Bs.
of

cent,

per

Cs.

[D.]

15.

If revolutions

occur

a a

largerproportion of censorship,than
a

ments govern-

where where found


to not

the press

is under will
of

of governments the press


are

it is not, then

censorship of
of governments

be

in

largerproportion

governments

which

ject subare

frequent revolutions, than


subject.
cent,

which

thus
1

[D.]
of
A
are

6.

If p per is the which

B, and
of A

q per

cent,

of A

are

C, what
make up

least
are

percentage
both
cannot

that

those

individuals

and

C ?

[D.]
percentage
we

17. C of the
B
1

Show
same
or

that

we

tell what
up

of how

or

of

individuals C is
not

make

unless

know

much

of In

A.
case

[D.]
in which of
B
or

8.

the easy

all

B
must

is

A, and
be

all C up

is A, the

find what individuals

percentage
which the
=

of and

C C

made

by

are

both

at

once.

[D.]

19.

Prove
+

following equations :

(A")
(A
+

(AB)
-

(A)
+

+
=

(ABC)
(A
+
+

(AC).
+

B)
+

(C
-

D)
AB

B) (c

G/) (Kb

(C

D)

(a
20.

A*)

(G*

"rD) +

CD

-J- aB).
a

Prove

that for the

the
common

following equation gives


part
of any three

correct
-

expression
A, B, C.

classes

(ABC)
21.

(B)
a

(C)

(A)

(aS)
r

(aC)

(A6f).
were

In

company who knew

of consisting

individuals in number

there who

q in number

Latin, and

knew

XXVIL]
either
is the
22.

PROBLEMS.

279

Latin number
Prove q and

or

French, but
of those

not

both

; between

what
?

limits

who
lower the
+

knew

French

confined

that
q
-

the

limit
upper

is the

greatest value
least value

in

p
zr

/, and
and

limit,the
On

in

"

"

q,

q.

(See Boole,
other

Propositions

Numerically Definite, p. 15.)


23. The student

will find
M

many

numerically
De

nite defi-

problems
and it
not

in De

organ'$" Formal
27-30
;

Logic,Chapter VIII.,
in

in his
be

Syllabus,pp.
Ys but

but that

reading
means

Morgan
with
Ys.

must

carefullyremembered
are

;;/XY
more

him
His

that mXs will

that
as

or

Xs

are

solutions be

sometimes,

shown

in

the

previous chapter, Morgan:


among
or

found
24.

delusive. of found
De "To lot of

Verify the following assertion


are

say that ;;/Xs


7?

not

any

one a

to

be

any

Ys

is

spurious (that is
unless
m

self-evident

necessary) position, prox

be

greater

than both

both of

and

y, in

which

case n

it is
-

merely equivalent to

the

following,
are

(m

+ to

y) Xy, and
other.

(m

x) Yx, which

lent equiva-

each

25.
y

It is found

that there

are

in
z

certain

club

of

bers, mem-

London
data both
are

graduates, and
in requisite London
?
are more

lawyers.
to

What

further

numerical who who


are are

order and

define

the numbers of those

graduates

lawyers,and
than
must

neither
If there

26.

persons

in

town

there be
at

are

hairs
two

on

any

one

person's head, then


town

there

least
on

persons

in the
Put

with

the

same a

number

of

hairs

their

heads.
form.

this theorem

into

strict

logico-mathelogic given
170.
a

matical 27. the

[HERBERT SPENCER.]
the theorem in numerical in edition the house

Demonstrate

of Science, new Principles


28.
"

(only), p.
there is

For

every

man

in

person

who

280

NUMERICAL

LOGIC.

[CH.

xxvii.

is

aged

some

of

the

men

are

not

aged
house
are

it

follows,
men

and

easily,
not

that

some

persons form

in of

the

not

but

by

any

common

syllogism."
of

(DE
is

MORGAN,

Syllabus,

p.

29.)
Science,
what

solution

this

problem
169.
from the and

given

in

Principles
29.
"

of

new

edition,
you
on

p.

Draw

conclusions

can

following though
of the
no

There

were

some

English
from the

board

passengers

were

saved

wreck,
yet
no

and

shipwas

officers,
lost."

as

it

happened,

only

one,

Englishman

[R.]

282 It is obvious

THE

LOGICAL

INDEX.

[CHAP.
in the middle
of

that each the Index

Greek

letter
the

appearing
presence

column

of

represents
or

the

sponding corre-

combination,
of

rather

its non-exclusion. exclusion.

Absence for of

the

Greek
No.

letter
31,

represents
we

Looking,
assertion
a,

instance, to
the Vlth

learn
all

that

be, an

type, excludes
the

combinations

except

ft, y,

specified at
centre

top

of

the

table, and
the

6, specified in the
consistent
The

column
a
=

; that
are

is to

say,

combinations
and in

with
use

be

ABC,

AB":, A^C,
will be
to

abc.

principal direction,
binations. com-

of the find

Index, however,
law

the inverse

to

the

corresponding

certain

unexcluded

Taking, for instance,the


abC, their Greek
we

combinations find their headed


e,

Afa, "BC, law, then,

signs are
in

8, e,
the

77 ; to

must

look

in the last table

in the line

column

(not-a),
rj,

(not-y),and (not-/3),
middle
=

showing 8,
two

in
A
=

the
c

column. of the IVth

We

there

find

the

assertions

Kb the 3.

type (No. 230), as


in of

those

corresponding
and infallibly three
terms.
not

to

combinations With the all aid

question.
this Index
we can

rapidly solve
What

possible problems relating to


can we

for example, assertion,

make

which

shall

be

contradictoryto, and
a
"

yet shall

not

be
out

inferrible from, the the


to

premise
unexcluded

BC

-I- abc?

Working
6. Of

combinations
be

by

this

premise, we ft, y, 8, e,

find them

ABc, KbC,
8 may any be

Kbc, 0BC, and


removed that is
to

abc, or

these, ft and

simultaneously without
say without

wholly removing Looking


we

letter,
second

contradiction. Index
at No.
one

in the

table of the
A
=

Logical

81,

find the
removes

proposition ft
and 8.

AC is the
to

(of type II.)as


the and assumed.

which

This

required proposition which


one

is,as
same

it were,
way
we

quite might
34,
or

neutral
remove

In

the

y
=

6 without

so contradiction,

that

No.

Kb

be, of type XII., is another

neutral

proposition.

It

xxvili.]
may, I

THE

LOGICAL

INDEX.

283
every

believe,be
XIII.,
that

safelyinferred
of the

that in

of proposition have
at

of type least
two

premise
to

question, will
II.

propositionsneutral
it be

it,of types

and

XII.

respectively. Suppose required,as


of
a

second and that


'

instance,to
of
=

define

the

precise points
one

agreement
asserts
as

difference

two

ants, disputthree-way
his opponent
at not

of

whom

(i) 'Space

spread
the

with

points
that

elements
=

(Henrici) ;

while

holds
same

(2) 'Space
'

three-way spread,' and


is has. The three

time
to

(3) Space
the
as

has

points as elements,' but


that combinations excluded

known
are

be

only thing
below,
the

assertions

symbolised by

being

indicated

their Greek

signs :

We

see

that

the

second
to

disputant's
that
of the
or

assertions

have

logical force
which

superior
to

first
=

by ^, namely ",
In
'

corresponds
to

assertion

5,

aB

aEC. that

addition,

then,

all that the

first asserts, he space has

affirms

three-way
this
an

spread
As
a

which third

is

not

points as
power it
to

elements.' of flexibility
to

instance

of the

and be

combinational

logic,suppose
statement

required

make
can

exhaustive
drawn of

of

all the
that
'

inferences Similar

which

be

from

the

theorem

figures (A)
We
=

consist whose

all whose

corresponding angles are


sides
are

equal (B), and


A

corresponding
this way. Vlth which
The

proportional (C).'
is of the form

proceed in BC,
of

proposition

the

type, and

negatives /?,y, 8, c.
two,
or

Any proposition,then,
of these combinations

negativesone,

three

284
will be All the

THE

LOGICAL

INDEX.

[CHAP.
not to equivalent

inferrible from

the

theorem, but

it. the

possible inferences, therefore, are


table the
of Index

indicated

in in

following

Numbers,

which, taken

nection con-

with

explains itself: Logical Index, sufficiently

ft

65

B7

97

B78

113

t"5

89

These

fourteen

assertions, which
the

are

all

the

possible

or non-equivalent inferences,

equivalentsof these, were


in
any
a

detected
be

by

the

Logical

Index in

few
case

minutes
a

; it would

doubtfullypossible,and
to

most

laborious inferences exists.


dant abun-

problem, by
any

obtain

an

exhaustive if indeed alone the


any

statement

of method

other
want

method,
of

other

The

space
of

prevents my
multitudes of

givingmore
the

illustrations which the of may be

logical problems
use

solved

and infallibly be
to

speedilyby
that

of

Logical
tables it
or

Index.

It may the

safelysaid
all

in four

pages

gives

key

possible logicalquestions,
distinct
terms. logical

relations
There for the
as,

problems involving three possibilitythat


of four
terms

is

some

the
some

corresponding day
be worked

index out,
one

relations

may
manner,

when

exhibited of
1,024

in like pages is
of
no a

it will occupy

only
those

volume

rather

larger size than

of

this volume.

There

prospect whatever
terms

that the be

sponding corre-

index

for

five

will
a

ever

exhaustively
will

published,since
each faint idea relations of the

it would

fill

libraryof 65,536 volumes,


This and moderate
fact

containing 1024

large pages.

give some
of of

number possible
a

complexity
number

logical
terms.

involving only

very

xxviii.]

THE

LOGICAL

STAMP.

285

The

Logical Stamp.
books1
I

In

my

previous
five

logical
of upon the

described
of such

Logical Logical
a

Slate

with

series

combinations
I

the

Alphabet
May 1863,
of

engraved
and
labour. I

it. since
recent

first

made used

slate

in

have
The

frequently
extensive

it with

much
of

saving

introduction
to
me

india-rubber idea that the

printing
most

stamps
convenient would the in
be

lately suggested
method
to

the the

of

obtaining
on

logical
Two four

combinations

stamp

them of

paper.
and

stamps
terms
as

producing
shown
were

combinations
IV.

three
of the of

of

columns

and
me

V.
at
a

Logical
about

Alphabet
eleven

(p. 181),

made

for

cost

shillings.
have
as

They
desired One

been

very the
of

successful,

and

leave
of

nothing

to

be

regards advantage
the
work

private
the

study
over

logical problems.
the
slate
can

great

stamps
done
on

is evident,

namely,
for

that

being

paper

be

preserved

reference

without

copying.
can

The

ABCD
or

stamp
more

readily
For

be

utilised

for for

problems
it

of is

five, six,

terms.

six

terms,
and

instance,
them

requisite

to

make

four

impressions
above of

distinguish

by

writing EF,
India-rubber ordered
at

E/, eF, ef,


stamps
of

the any

respective design
can

impressions.
now

be

easily

all

the

principal

stationers.

Pure

Logic,
Lessons p. no;

1864,
in

p.

68

Substitution
p.

of
;

Similars,

1869,

p.

54

Elementary
Vol.

Logic, 1870,
Editions,
p.

199

Principles of Science,

1874,

I.,

New

96.

286

THE

LOGICAL

INDEX.

[CHAP.

xxvin.]

THE

LOGICAL

INDEX.

287

00

oil "u

II a
1*

"
II
"

X W
C
s: c:r sr c-

"J

"
u

"OO'C'O'O'O'O'O'O'O'O'O'O'O'O'O

C C
lj

E
H

to\O

hsCO

ON

fO

"**" m*O

t*^CO

OS

ro

l/^VO

txOO

O\

PO

"*" 1O\O

THE

LOGICAL

INDEX.

[CHAP.

txOO

ON

M O COCGOOOOOOCOOOOOC

en

"""

O
O

II
j

.K~

II II

["";."*

X
w

Q
c-sr STE-

Z
i" i

,-J

"
u

o
o
H-J

ffi H

CQ

II

II"

";

"s

-"j-

invo

i^oo

-^-

mvo

r-^ro

CHAPTER

XXIX.

MISCELLANEOUS

QUESTIONS

AND

PROBLEMS.

IT

seems

convenient
a

to

bring
a

these certain
may

Studies number refer


to

in

Deductive of any mixed

Logic

to

close and

by

adding
which

Questions logical
bounds student pages

Problems,
In
some

part
pass

of the the

doctrine. of
to

cases

these

questions
It of is

formal

and

deductive what
To
to

logic.
if
any,

left

to

determine assist him.

part,
certain other

the

preceding
are

will

questions,
works where

however,
the

appended
assistance
1.

references
will

proper

be
we

found.

What when
?

may
an

expect irresistible

to

happen,
force
meets

in

logical point
with
an

of

view,

infinite

resistance
2.

If B

it is
has

said those

to

be also

false
of
statement

that C and

what

has and

the vice

properties
versa,

and

D,
as

how

would relations
3.

you
of how

interpret
A, B,

this
and

affecting

the

possible

C,

?
we

In

many

ways

may

in of is

purely

logical point
that
'

of

view

contradict in
the

the
state

assertion of the
nature

Hobbes

Irresistible
p.

might Question
4. from

right

'

(See

182,

7.)

Specify
the

ways.

Analyse
the Wealth is
not

logical of Nations,

import
book
man

of i.

the

following
viii.
:

passage

chapter
a

"It

because

one

keeps
one

coach,
and

while the

his other

neighbour

walks

afoot,

that

the

is

rich,

CH.

xxix.]
; but

MISCELLANEOUS

QUESTIONS.
one

291

poor

because

the is poor,

is he

rich,he keeps
afoot."

coach, and

because 5.
we

the other Harriet


told
a

walks

In

Martineau's
a

Autobiography (vol.i.

p.

355)

are

that

certain

from lady, after receiving

Charles

Babbage
machine,

long explanation of his celebrated


terminated
"

calculating following thing more


wrong,

the

conversation is the

with

the

question :
that I want

Now,
to

Mr.

Babbage, there
If
out

only one

know.
come

you

put
"

question in

will the
If

answer

right?

you

think
reasons

this
for

question absurd, give thinking


so, and

distinct them

and with

detailed

reconcile
a

the fact that false


6.
verts verum

premises may

give

true

conclusion.

Explain
non fieri

and

illustrate the

Aristotelian
;
ex

saying :
the

Ex

potest ut falsum condudatur


show
some

falsiscontra tion investiga-

and

of its

in applications

of nature. 7. A

[R.]
argument

certain

having
the tend

been

shown
under the

to

involve
this

paralogism, inquire into


failure does conclusion.
8. In
a or

conditions
to

which

does

not

establish

contradictory
Liberal

certain
to

borough,
the whole

on

one

occasion,the
the Conservative

party objected
to

3,624 voters, and


of

party
10,000.

SjSS1

voters,

constituency being
voters

What

is the
on

least number both sides?

which
is the

can

have

been

objected to
What
to

What

greatest number?

is the made
What

most

probable number,
?

supposing the objections


with
man

be
9.

quite at haphazard
is the
of

logical, compared injunction,'This


in the

the is

terpret popular, innot


on

the

any the

account

to

be

ducked

horse-pond.' animal, the

Explain
of

difference.
"

10.

Because of
an

horse

is

an

head

horse

is the head

animal."
u
2

292

MISCELLANEOUS

QUESTIONS
this inference. Can you

[CHAP.
express

Examine the

the

of validity

? or reasoningsyllogistically, symbolically
11.

[E.]
or

the Investigate in the four


are

nature

of the

reasoning, good
:

bad,

involved

followingexamples
stronger than
therefore

(1) Elephants
than
men.

horses

; horses
are

are

stronger
than

men;

elephants
of

stronger

[E.]
was

(2) Alexander
was

the
of

son

Philip;
good
a

therefore

Philip
he that
;

the father kill


a man a man

Alexander.
as

(3)

As

good
kills but he

kill

book

; for

does kills
a

but

kill

reasonable kills Reason


; for

creature

that
you,

good

book

herself. built
are a

(4) Nay,

look

I know

'tis true

his father the

chimney
alive
What
at

in my this

father's house, and


to

bricks

day

it. testify these

[o.]

12.

methods it froze

underlie last

inferences?
the

Because

night,

therefore

pools

are

covered

with

ice.
of

During
wind
to

the retreat in the faces the


a

the Ten

Thousand

cutting northwere

blew

of the soldiers ; sacrifices

offered

Boreas, and
seemed
What

of severity

the wind

immediately ceased, [p.]

which
13.
"

proof

of the

god's causation.
in the
an

method

is

employed

following?
from
a

Brewster

accidentally took
a

impression
and

piece
and
wax,

of

in mother-of-pearl colours
to

cement

of resin upon the

bees'
of

wax, the

rinding the proceeded

repeated
other

surface in

take

impressions
thus

balsam,
found

fusible the descent iri-

metal, lead, gum-arabic, "c., and


colours
nature

always proved

the
a

same.

He

that the chemical the form of

is

wholly

matter

of

and indifference,

the

surface

is the condition

of such

colours."

xxix.]
14.

AND

PROBLEMS.

293

What
:
"

is the

between difference, logically,

the

tences sen-

Leibnitz, a

great

philosopher,has said,"c.
?

said, "c.

; and

great philosopher, Liebnitz, has


15.
What
as

[c.]
by
+
nection, con-

is successive in the
the

induction, or
that
? in
ri2
=

induction
+

proof
numbers

3+5

up

to

of

odd

[H.]
Logic, lesson
that x"
-

(See Elementary
1 x

Lessons

xxvi.

p.

220.)
by

6.

Give
a, when

an n

inductive is
a

proof

a"

is divisible

whole
such

number.
a

17.

Can

there that is
a

be

thing as
does iv.

of simple fallacy

spection in?

which fallacy

not

involve

inference

(See
1
"

Mill's

System of Logic,book
says

chapter iii.)

8.

Leibnitz

Knowledge
either
or

is either
or

obscure distinct
:

or

clear.
the

The

clear

is

again

confused

and

distinct

either
or

adequate

inadequate
if it be
at

; is further

either time

symbolical

tuitive inand

; and

the

same

both

adequate

it is perfect." intuitive, Give


an

exhaustive under
the

classification
above

of all

possiblekinds

of

knowledge
Lessons 19. in

conditions.

(See Elementary
the
terms

Logic,lesson
the

vii.)
of

Explain

logical meanings

Genus,

Difference, Property,and Species, meaning


varieties in of
D of
extent

the Accident, distinguishing

and

intent,and
ABC^,

using for
ABdD,

illustration the in which


c, d the

things ABCD,
are

AIW,

A,

B, C,

terms

and denoting qualities,

negatives
20.

C,

D.

Draw

the inferences

deducible

from

these

data: when

(1) A (2) A
and

is the

only

antecedent when

always present
p is absent.

is

present, and
is
an

always
absent

absent

antecedent
when

always present when/


/ is absent.

is present,

always

294

MISCELLANEOUS

QUESTIONS

[CHAP.
when

(3) A

is

an

antecedent

frequently present
when

is

present, and

frequentlyabsent
the

/ is absent.

21.

Point

out

exact

nature

of

the

relations

between

the and
22.

logical processes
Generalisation.
WThat

of

Abstraction, Analysis, Synthesis,


between

is the and

difference,if logical

any,

nouns

Substantive
23. Is
a

nouns

Adjective ? adjective
?

Latin
a

used

alone

in

the

neuter

an

adjectiveor
24.

substantive

Is there of what

self-contradiction is outside
? my

in the assertion consciousness


may

that knowledge
be

inside

my

consciousness 25. Can

absolute

be certainty

found

in any

conclusion

(i)

inductivelyestablished, (2) deductively established?


26. Is

[E.]
a

there
an

any

and distinction,

if

so

what,
a

between

general and
difference

abstract

notion, and
names

is there
to

corresponding
express them ?

between

the

employed

27.

It is

rule

of

syllogism that nothing


How
men
'

can

be

inferred
from

from

particular premises.
that
men some
'

then
have

can

infer

the

particularfacts
conclusion,
28. All

died, the

universal

die

? of

[E.]
demonstrative
"

Explain
the

the

limits

science, and
of

examine
be
matter

following statement
demonstration."

No

matter

fact

can

of

[E.]
and

29.

Distinguish Logical, Mathematical, Distinguish


and

Physical

Quantity.
30.

[E.]
exemplify Logical, Mathematical,
Moral
are

Metaphysical, Physical,and
31.

Necessity. causally connected


which is the
cause

[E.]
gether, to-

When
can

two

phenomena always
If

you

ascertain
so,

and

which

is the

effect ?

how?

[u]

xxrx.]
32.

AND

PROBLEMS. far

295

Investigate how
of

or

on

what

grounds

our

ledge knowto

the

following propositions approximates

certainty :
"

Nitric acid
A

does

not star

dissolve is

gold."

"

distant

fixed
a

subject to gravity."
of view the assertion

33.

Consider
the

from

logical point
of
Great

that reform
34.

increasing trade
What
been

Britain

is caused is

by

of the tariff.
A
man

kind shot

of

proof

applicable?
heart of

having

through through by
"

the

diately immesuch
a

falls dead.

Investigate the logicalvalue


men

fact dead.

as

proving
What
what

that all

shot

the

heart

will fall

[i.]
do
you

35. Under

understand
is it

working hypothesis "?


an

conditions

legitimatefor
Mill's

to investigator

employ hypothesis?
iii.chap. xiv. ;

(Huxley.

System of Logic, book

Principlesof Science,chap, xxiii.)


arguments

[L.]
nical tech-

36.

State

these

formally,and

give their

designations:

(1) (2)

"The

thinking

power

does

not

belong

to

matter;
it." since all obtain its

otherwise
"

matter

generallywould
reward
a

exhibit
; and

Happiness
do
not

is the

of

goodness

desire

good

life,all

cannot

reward."

[p.]
with

37.

Why

is

it that

exactly the
we positive,

same

amount not

of

evidence, both
assertion credence
men

negative and
are

did
we

rejectthe
refuse
were

that there
to

black

swans,

while asserted

should
there ?

any

testimony

which

that

wearing
38. What

their heads is the

underneath of

their shoulders

[P.]
and

difference house

meaning,
was
'

if any,

between

the
'

propositions, 'This
is the
on

built
?

by

Jack,'

This

house

that

Jack built

(De Morgan,

Third

Memoir

the

Syllogism,loth page.)

296
39.

MISCELLANEOUS

QUESTIONS
the
of

[CHAP.
in human affect the

Does

the
are

thesis that the result

ultimate

premises
?

knowledge
nature

mental

association if so, how

and

certaintyof Logic, and


evidence. evidence.
:
"

[E.]

40.

Define

Distinguishintuitive, demonstrative,

and

probable

[E.]
for its

41.

Explain
in the

"Certainty, therefore,has
way
"

opposite,
tainty, Uncer-

uncertaintyin

one

in impossibility
of

another. its

language

logicians, is
whether

contradictory

its contrary opposite." opposite" impossibility,


42.
statement

[P.]
truth

Investigate the
is
to to

question
the it is

the

of

be

judged by

impression which

it makes
spondence corre-

upon

those

whom

addressed, by
person

its literal

with other
43.

the belief of the

making it,or by
that
on

any

standard.
It

[L.]
been

has

pointed
occurs

out
some

by

Ohm

reasoning to
mathematics
:

the
"A

following effect

in

works

magnitude required for

the solution
and
as

of

problem

must
x

satisfya
Examine
44.

particular equation,
the
of validity logical that

the

magnitude

satisfies this

equation,it is therefore probable


Herodotus and
;
so

the

magnitude required." only


what he

this argument. recorded


not
we

It is

heard that
account

concerning Ethiopia ;
he heard
as was

it is that

unlikelythat
may ?

most

correct

accept

his

true.

Is this conclusion
a

correct

45. child the

There
of
a

is

very married

strong

probability that
will

the

eldest of that is

newly

couple
more

inherit

the than if
for
a

estate not

husband. will be
more

it is For, firstly, children


of

probable
Next,
a

there

the

marriage.
if

child

born, it is
are

probable

that it will be
a son

son,

more

boys
it will

born

than survive

girls. Thirdly,
its father. the

be

born,

probably
that

Examine

this inference,
'

[o.]
writings
\

46. Consider
are

following argument
ascribed
to

"

Many

not

genuine were

Clemens

Romanus

298
Smith's
? parties

MISCELLANEOUS

QUESTIONS
are

[CHAP.
of

proof.
From the

What

the

logical relations

the

54.
or

statement

that blood-vessels that logically Give


your
a

are

either veins

does arteries,
a

it follow
an

blood-vessel, if it

be

vein,is

not

artery ?

reasons.

55.

It is asserted in

by

some

philosophersthat
it is

all

knowledge
that
ledge know-

is inductive inductive
can

its

origin,and
can

generally allowed
; if so, you,
no

inferences be
any
case more

be

probable only
true.

than of

probably

Can is

however,
?

adduce In

instance

knowledge
edit mulier

which

certainlytrue
If

that

which explain the difficulty


amat aut
; to

evidently arises.
tertium. the

56.
one

Aut

nihil

any this
may

takes

upon

himself

simply
how

deny

truth

of

saying

of Publius be

Syrus, in

many

different

ways

the denial 57.


of
an

? interpreted

Explain

the

following apparent
is

paradox:
the

"

thinks of
that

object ; Q
to

absolutelyignorant of
is has

size the

object;

that him, therefore,the probability


a

object is
it either
case

greater than

cannon-ball

|.
no

Again, being absolutely


reason

ignorant about
greater
or

its

he size,
a

to

believe

less
Hence

than
to

pea, him

the

probabilityof
size
between

either

being \.
the

it is in

improbable infinitely
a

that
a

object
In

is

intermediate

pea

and

cannon-ball.

[JOHN
a

HOPKINSON,
must

D.SC.]
either

58.
the

defending

prisoner his
is
the
a

counsel
or

deny
that

that the deed

committed

crime,

he

must

deny

prisoner committed
that the deed

deed

; therefore

if the he
must

counsel

denies

committed commit understand the the

is

crime,

admit

that the 59.

prisonerdid
do you
?

deed.

What

by

the

logicalproof
of

of

an

assertion with
any

Compare

logical meaning
of the word known

the word
to

proof

other

meanings

you.

xxix.]
60.

AND

PROBLEMS.

299

Can
as

all kinds well


as

of the

propositions be
extensive in the
form
event

exhibited
?

in

the
in

intensive

Give
of its

reasons

support

of your

answer a

; and

being
those

in the

negative,draw
of

up

list distinguishingbetween
can,

kinds
be
so

propositionswhich

and

those

which

cannot,

exhibited.
61.

[L.]
the

Explain
inverse

meaning
of

of

the assertion

that

Induction

is the
62.
or

process

Deduction. Induction
to

Illustrate

Mathematical

in its several in the

kinds

cases,

and

discuss

its relation

induction

physical

sciences.

63. What

is the
and the ?

relation, if any,
inductive

between

the

inductive in the

syllogism

methods

employed

physical sciences
64. Estimate

upon

logical grounds
in which
Laws students

the should

possibility of
be rendered

establishing a
capable
of

school

discovering the

of Nature.

(Gore'sArt

of

Discovery?) Scientific
65. What
?

precisely
Point
out

is

meant

by
and
439
;

the

Law

of

tinuity Con-

the

grounds
p.

limits of

its

validity.

(Life of

Sir

W.

Hamilton,

Principles of Science,

chapter xxvii.)
66. and When

the

effects

of

three

distinct

causes

are

added
and

mingled together,by
can we

what
each

processes
cause

of

experiment

reasoning

assign to

its separate

effect ?

[c.]
67. Under
failure of
the
an

what

circumstances
or

are

we

to

accept
as

the

experiment
of

series of

experiments

proving
to

non-existence

the

phenomenon

intended

be

produced?

of Science,chapter xix.) (Principles

[L.]

300

MISCELLANEOUS

QUESTIONS
of

[CHAP.

68.

Illustrate and show

the

scientific value
many ways

exceptional phenomena,
may

in how with

they

be

disposed

of

or

reconciled

physical law.
between

(Principlesof Science,

chapter xxix.)
69. What
casual
even a

[L.]
difference events,
to

is the

the

causal

and

the
not

happening of
dead leaf falls the
upon

if, as

is

generallyallowed,
sufficient of its

the

ground
moment

without

causes

to

determine

precise
which

fallingand
does

the

precise spot
70. Show
the

it will fall ? that the

by example
notion
of

logicalcopula
whether in the

not

imply
71.

existence.

[E.]
functions
of
are

Investigate the
and

question

affirmative similar.
72.
a

negative propositions
is the richest Russia

reasoning

England

country in the world, and


and

has
to

gold

currency.
poor

India, in
have

proportion
or no

are population,

countries, and
are

little
of is

gold
of

currency.

How
to

far

such
a

kinds currency

facts the

logically
a

sufficient

prove ?

that

gold

cause

nation's wealth
73.
at

[i.]
distinct methods of

If

by

two

investigation you
the
currency

arrive of the

the

same

conclusion, namely,
not

that

kingdom
it is

does

exceed discovered

one

hundred that
one

millions of the

but sterling, methods would


of

afterwards

involved investigation be inclined


to

fallacious

reasoning, what
of

you

infer about

the other method

? investigation

"]
74.
A

certain

argument
into
not

having
the

been

shown under the

to

involve this

paralogism,inquire
failure does conclusion. 75.
or

conditions
to

which

does

tend

establish

contradictory
moral

Investigate
of

the

and logical,psychological, s' excuse, s'accuse"

grounds

the

saying,"Qui

xxix.] 76. Taking the

AND

PROBLEMS.

301

senses

in which

they

most

resemble

one

another, distinguishbetween knowledge,


77.

judgment, opinion, statement,

fancy, conjecture, supposition,allegation. [E.] truth, certainty,fact, opinion, probability,

Distinguish :

evidence, conviction. 78.


How
as

[E.]
the

far

can

the
of the

of inconceivability truth of any

opposite
?

be

regarded
79.

proof

judgment
the law

[E.]
Excluded

Right-angled
the
must

and

are not-right-angled

contradictory
of

predicates; therefore, according Middle,


is
as

to

proposition
be
true

'

All

trianglesare trianglesare

right-angled
not

false,it
But

that

all

right-

angled.

this also is false.

Explain
the

the above

difficulty.

[E.]
80.

Given

that

(i) whenever
or

statements

a, the y
or

b,

are

either
c

all three

true,
y

all three
or

false, then
c

statement

is

false,and
that whenever

is true,

else

is true, and
all three
x

is false ;
all three
a

(2)
and

d,

e, y
a can

are

either

true

false,then
x

the statement When


true

is
we

false,and

is true, these

or

is true,
that

is false.
x or

infer from

premises

either

is

[Hugh
in
seems

MacColl,
A
same

B.A.,

in Educational
was

Times, question
C.

6,206.
the

solution
paper
mean
"

given by
March,
other
x

J. Monro,
The

M.A.,

for What

1880.

question
with those

to

conditions
ory

given
81. p. De of laws
a

determine

that either

is

true?]
the

Morgan
the is
Laws
an

says

(Fourth

Memoir

on

Syllogism,
is that is
a

5)

of

Thought:
inference laws. these
a

"Every
; every

transgression of
valid inference admit
laws

these
not

invalid

of transgression is

But

I cannot

everything which
valid these inference." three

not

transgressionof

these

Investigate the logicalrelations

between

assertions.

302 82. To ?

MISCELLANEOUS

QUESTIONS
do the

[CHAP.
of

what

type of assertion

premises

Darapti

belong

83. Give
and

the

converse,

inverse, contrapositive, obverse,


the
: following

reciprocal propositionsof

(1) All parallelogramshave (2)


If P

their

opposite angles equal.


R

is

greater
S.

than

Q,

then

will

be

greater

than

(3) Two
one

are triangles are

congruent

if the
to

three

sides

of the

respectively equal

the three sides of the

other.

84.

Why

have

some

mathematicians
to

been of
a

accustomed

to

say that it is necessary

prove

the

converse

mathematical

proposition? 85.
who
to

Where

exactly
with him his

lies the
on

error

of

the

Irishman, who
witnesses

being charged
had
seen

theft

the

evidence article in

of three

stealingthe

question,proposed
who had
not
seen

bring in
86.

defence

witnesses thirty

him

it ? stealing

Epimenides

says that is
a

every

statement

of

Cretan he
a

is

lie ; but
a

Epimenides
every

Cretan

; therefore

what

says lie.

is

lie ; therefore

statement

of

Cretan

is

not

[It]
87.
sure

If

in
the

saying that
statement

"few

strikes

are

I beneficial,"

feel
to

that

will be
and

misinterpreted by
the in
the
statement
"

those

whom
are

it is addressed,

that
not

no

strikes

beneficial," although
more

my

opinion literally true,


minds the pression im-

will

exactly
which
moral

convey
to

to

hearers'

I believe

be

true, ought I, having regard


the

to

the

obligation of speaking
or

truth,to

use

the

latter assertion 88.


too

the former

?
"

"I

will go

on," said King James,


ruined my

I have

been

only

indulgent. Indulgence

father."

xxix.]

AND

PROBLEMS.

303

Express clearlythe
utterance.

process
?
or

of

reasoning
?

involved

in this

Is it Induction

what

[M.]
the inductive in the

89.

What

is the and
the ?

relation, if any,
inductive methods

between

syllogism
Can

employed
be

physical sciences
90.

the

proposition, All
of

'

is all BJ
?

regarded
vol.

as

a singleact representing

thought

(See Mind,
given only
?

i.

p.

216.)
91. Are

[E.]
the
form

premises
sufficient
to

of

Darapti
the

in

numerical
92. verse, 93. Does

prove

conclusion
some

it follow
must

that, because
some verse
'

poetry is

not

in

there Take

be

which

is not

poetry ? [H.]
are

the

proposition, All

sciences it

and useful," what


terms

determine it leaves
'

preciselywhat
'

it affirms, what

denies, and
of the

doubtful, concerning the


and
'

relations

science
94.

useful

thing.'
many

Ascertain
are

precisely how description of


of

distinct conduct of

assertions the Thomas


sed maerens,

there

in

the

the

great
a

scholastic Becket continue 95. between had


se

logician, John
been

Salisbury, after
his side
:
"

murdered

by

Tacitus

subduxit. you
Some

Can
'

represent
Xs
are

equationally
some

the
'

contradiction is
one

not

Fs

'

and

There

only

and

that is the
of the the

only

Y'

96. Which
are

types of assertion
sense

involving three
additional

terms

complete,in

of

admitting no
without
X
can or

assertion
?
are

involving the
97. If Y Do

same

three

terms

self-contradiction

all
or

things are

either

Y, and
draw
?

all

things

either

Z, what
the

inference

you

98.

moods thirty-six

of Hamilton's table in

Syllogism with
Lessons,

quantified predicates (see


p.

Elementary
section

188; Thomson's

Laws

of Thought,
moods
?

103) comprise

all the

possible weakened

304
"

MISCELLANEOUS

QUESTIONS.
of
two must

[CH. xxix.

99.

Is the student

speaking,prepared logic, generally


and followingpropositions,
to

rapidlyto analyse the whether no or they


synonyms The (i)be

say

be

identical,if the

identityof

granted ?
a

suspicion of againstits more

nation
civilised and

is
as

easilyexcited, as against
its
more

well
like war-

neighbours,
removed. difficulty

such

suspicion

is

with

(2) When
its

we

see

nation
or

either backward
to

to

suspect

neighbour,
we

apt

be

satisfied

by explanations,
is
of

may
more

rely upon
civilised

it that the
nor

neighbour
warlike

neither

the

the

more

the two."

[DE
100.

MORGAN,

Third

Memoir,

1858,

p.

181.]
?

Is the

a following proposition or

definition

or

not

Is you

it

on

the your

matter
answer

the form

of

the

proposition that

found
"

LOGICA

EST

ARS

ARTIUM

ET

SCIENTIA

SCIENTIARUM."

THE

END.

LONDON:

K.

CLAY,

SONS,

AND

TAYLOR,

PRINTERS.

MACMILLANS

CLASS-BOOKS

OF

ELEMENTARY

SCIENCE.

POPULAR F.R.S.,

ASTRONOMY.
Astronomer

By Sir
Royal.
With
numerous

G.

B.

AIRY,

K.C.B.,
4^. 6"/.

Illustrations.

ELEMENTARY
Professor

LESSONS
HUXLEY,
4*. 6d.

IN
New
on,

PHYSIOLOGY.
Edition.
is.

By
numerous

F.R.S.

With

Illustrations.

(Questions

6d.)

LESSONS
Edition.

IN

LOGIC,
Professor
numerous

INDUCTIVE
STANLEY

AND

TIVE. DEDUCF.R.S.
New

By

JEVONS,

M.A.,

With

Illustrations.

35. (td.

LESSONS
Professor

IN
H. E.

ELEMENTARY ROSCOE,
6d.

CHEMISTRY.
F.R.S.
New

By
With
numerous

Edition.
to

Illustrations.

4^.

Problems i8mo.

adapted
2s.

the

same

by

Professor

THORPE.

With

Key.

OWENS
Professor

COLLEGE
CHEMISTRY.
ROSCOE.
zs.

JUNIOR
By
New F.

COURSE JONES.
With With
numerous

OF

TICAL PRACPreface

by

Edition.

Illustrations.

l8mo.

6d.

LESSONS
GEORGE

IN

ELEMENTARY
F.R.S.
With

ANATOMY.
numerous

By

ST.

MIVART,

Illustrations.

6s. 6d.

POLITICAL
Mrs.
2s.

ECONOMY
FAWCETT. New

FOR
Edition.
With

BEGINNERS.
numerous

By
Illustrations.

6d.

LESSONS OLIVER,

IN

ELEMENTARY
New

BOTANY.
Edition.
With
numerous

By Professor
Illustrations.

F.R.S.

MACMILLAN

AND

CO.,

LONDON.

MACMILLANS

GLASS-BOOKS

OF

ELEMENTARY

SCIENCE.

LESSONS
BALFOUR

IN

ELEMENTARY
STEWART,
4^. 6d.

PHYSICS.
F.R.S.
New Edition.

By
With

Professor
numerous

Illustrations.

ELEMENTARY

LESSONS LOCKYER,
$s. 6d. F.R.S.

IN
New
on,

ASTRONOMY.
Edition.
is.

By
With
numerous

J.

NORMAN

Illustrations.

(Questions
TREATISE

6d.)

AN

ELEMENTARY B.E., J. PERRY, Engineering, Yedo.


Exercises. 4^.

ON

STEAM.

By

Professor With

Engineering, Imperial College of Illustrations,Numerical Examples, and

of

6d.

NATURAL
I. TODHUNTER, and II. Fluid

PHILOSOPHY M.A.,
With and

FOR
F.R.S.
numerous

BEGINNERS.
I. The

By
3^.
6d. Part

Part

Properties of Solid

Bodies.

Illustrations. 3.*. 6d.

Sound,

Light,

Heat.

PHYSICAL
IN. 41. 6d.

GEOGRAPHY,
By

ELEMENTARY
With
numerous

LESSONS
Illustrations.

Professor GEIKIE, F.R.S. (Questions, is. 6d.)

SOUND,
W. H.

ELEMENTARY
STONE.
With

LESSONS
Illustrations.

ON.

By
3*. 6d.

Dr.

Fcap. 8vo.

QUESTIONS
and Exercises

ON
in

CHEMISTRY. Inorganic Organic Fcap. 8vo. 35.


and

Series of By

Problems
F,

Chemistry.

JONES,

F.R.S.E., THE
and

F.C.S.

ECONOMICS
M. P. MARSHALL,

OF

INDUSTRY.
zs.

By

A.

MARSHALL

6d.

SHORT

GEOGRAPHY
R. GREEN and
ALICE

OF
S.

THE
GREEN.

BRITISH
With

ISLANDS.

By J.

Maps.

3J. 6d.

MACMILLAN

AND

CO.,

LONDON.

"MANUALS
INORGANIC
and
2is.

FOR
CHEMISTRY.
Vol.
I. Two

STUDENTS.
By
NON-METALLIC

Professors

ROSCOE

SCHORLEMMER. Vol. II.

ELEMENTS,

METALS,

Parts,

iSs. each.

GEGENBAUR'S
Translation RAY

COMPARATIVE

ANATOMY. Revised,
8vo.
with
2is.

A
Professor

by

F.

J.

BELL.

Preface, by

LANKESTER.

Illustrated.

TEXT-BOOK M.D.,
A TREATISE
F.R.S. F.R.S.

OF

PHYSIOLOGY.
Illustrated.
8vo.
2is.

By

MICHAEL

FOSTER,

ON

EMBRYOLOGY.
8vo.
Vol.

By
I., l8s.

F.

M.

BALFOUR,

Illustrated.

(\ol.II.inthePrtss.)
CHEMISTRY.

TEXT-BOOK By
Vol. Professor

OF
ARTHUR

PHYSIOLOGICAL GAMGEE,
F.R.S.

Illustrated.

8vo.

I., i8j. PRACTICAL FOSTER,


8vo. 6s.

ELEMENTARY
MICHAEL Crown

PHYSIOLOGY.
F.R.S.,
and

By
B.A.

M.D.,

J.

N.

LANGLEY,

THE

STUDENT'S
ISLANDS. 8vo.
105.

FLORA

OF

THE

BRITISH
F.R.S.

By
6d.

SIR

J.

D.

HOOKER,

K.C.S.I.,

Globe

PHYSIOGRAPHY:
Nature, Crown 8vo.

An

Introduction
HUXLEY,
F.R.S.
Edition.

to

the
With

Study

of

By

Professor New and

Illustrations.

Cheaper

6s.

ELEMENTARY

BIOLOGY.
H. M.

By
MARTIN, M.B.,

Professor
D.Sc. Cr.

HUXLEY,
8vo. 6s.

F.R.S., assisted by

THE

PRINCIPLES Logic and LL.D., F.R.S.


Crown

OF
8vo.

SCIENCE. By
Professor 64.

A
W.

Treatise

on

Scientific Method.

STANLEY

JEVONS,

12s.

MANUAL FAWCETT,

OF
M.P.

POLITICAL
Crown

ECONOMY.
8vo.
12s.

By

Professor

6d.

MACMILLAN

AND

CO.,

LONDON.

BEDFORD

STREET,

COVENT

GARDEN, 1879.

LONDON,

December,

MACMILLAN
in

""

Co.'s

CATALOGUE
and

of

WORKS

MATHEMATICS PURE

PHYSICAL
APPLIED

SCIENCE;
MATICS; MATHE-

including
CHEMISTRY,
WORKS
and

and

PHYSICS,
in MENTAL

ASTRONOMY, BOTANY;
MORAL

GEOLOGY,
and

ZOOLOGY,
and

of

PHILOSOPHY

Allied

Subjects.

MATHEMATICS.

Airy."
ELEMENTARY

Works

by

Sir

G.

B.

AIRY,
OX

K.C.B.,
PARTIAL Use of

Astronomer

Royal
in the

:"

TREATISE

DIFFERENTIAL Students sities. Univer-

EQUATIONS.
With ON THE ERRORS OF 6.r. 6d. ALGEBRAICAL OF

Designed

for the

Diagrams.

New AND

Edition.

Crown

8vo.

5*. 6d.
OF TION COMBINACrown

NUMERICAL AND Second THE Edition.

THEORY

OBSERVATIONS

OBSERVATIONS.

8vo.

UNDULATORY
Students ON the SOUND in the AND

THEORY

OF New

OPTICS. Edition.

Designed
Crown

for the Use

of

University.
Elements

Svo.

6s. 6d. With

ATMOSPHERIC of Music. Second

VIBRATIONS.

Mathematical of the 8vo.

Students Crown A TREATISE Students

University.
ON MAGNETISM.

Designed Edition, revised

for

the

Use

of

and

enlarged.
Use of

9*.

Designed
Crown Svo. 9-r. 6"/.

for

the

in the

University.

Ball

(R. S., A.M.).


Course Ireland. Mathematics Ireland
' '

"

EXPERIMENTAL
at

MECHANICS.

A for

of

Lectures

delivered STAWELL Mechanics

the

Royal College
A.M.,

of Science of

By

ROBERT and

BALL,
in

Professor of

Applied
for

(Science
have
not

and

Art

Royal College Department). Royal Svo. of


of
the sort
a

the

Science

16.?. It dates eluci-

We

book any instructivelythe methods


met

with

in

English.
of
ail

teacher
it to

the
our

very

highest
"

rank.

We

most

cordiallyrecommend
A

nuders."

Mechanics'
5,000.
12.

Magazine.

79

SCIENTIFIC

CATALOGUE.

Bayma."

THE

ELEMENTS

OF

MOLECULAR Professor
ior.

NICS. MECHAoi

BAYMA, S.J., By JOSEPH Stonyhurst College. Demy 8vo. cloth. Boole.


"

Philosophy,

6d.

Works in ON Edited ON Volume.

by
the

G.

BoOLK,

D.C.L,

F.R.S.,
Ireland
:
"

Professor

of

Mathematics A TREATISE Edition. A TREATISE

Queen's
DIFFERENTIAL

University,

EQUATIONS.
Crown 8vo. cloth.

Third 14^.

by

I. TODHUNTER. DIFFERENTIAL

EQUATIONS.
I. TODHUNTER. Crown

mentary SuppleSvo.
cloth.

Edited

by

Ss. 6"/.

THE

CALCULUS
IOJ.

OF

FINITE Edition

DIFFERENCES.
revised.

Crown

Svo.

cloth.

6d.

New

Cheyne.
PLANETARY

"

AN

ELEMENTARY THEORY. With

TREATISE
a

ON of

THE Problems. Crown

Collection Second

ByC.
Svo.

H. cloth.

H.

CHEYNE,
6s. 6d.

M.A.,

F.R.A.S.

Edition.

Clifford.
to

"

THE of

ELEMENTS Motion and

OF Rest

DYNAMIC.
in Solid of and

An Fluid

Introduction Bodies.

the K.

study

By
and

W.

CLIFFORD,
at

F.R.S.,

Professor

Applied
Part

Mathematics I.
"

Mechanics Crown Svo.

University College, London.


7-r. 6d. INTRODUCTION
With
numerous

Kinematic.

Gumming.
GUMMING,
8s. 6d.

"

AN

TO

THE

THEORY
LINN/EUS

OF

ELECTRICITY.

M.A.,

Assistant

Master

at

By Examples. School. Rugby

Crown

Svo.

Cuthbertson.
BERTSON, School. M Extra

"

EUCLIDIAN
Mathematical
Svo.

GEOMETRY. Master of the

.A., Head

By F. City of

CUTHLondon

fcap.

4?. 6d.

Everett.
D.

"

UNITS

AND

PHYSICAL

CONSTANTS.

By
of Svo. Natural 4^.

J.
64.

EVERETT,

M.A.,

D.C.L.,

F.R.S.,

Professor

Philosophy, Queen's College,


Ferrers
and AN Tutor

Belfast.

Extra

fcap.

Worlcsby
of

the

Rev. and

N.M. Caius

Gonville

M. A., F.R.S., FERRERS, : College, Cambridge


"

Fellow

ELEMENTARY
the Method

TREATISE
of

ON

TRILINEAR and the

NATES, CO-ORDI-

Reciprocal Polars,
revised. AND Svo. Crown

Theory

of

Projectors.
SPHF.RTCAT
WITH THEM.

Third

Edition,

Svo.

6.r. 6d.

TTARMOXICS
Crown

SUBJECTS
7-r. 6d.

CONNECTED

MATHEMATICS.

Frost.

"

Works

by
THREE With

PERCIVAL

FROST,
Lecturer

M.A.,
of

late

Fellow

of

St.
"

John's College,
THE CIPIA. FIRST

Mathematical

King'sColl.Cambridge:
NEWTON'S
Also
a

SECTIONS and

OF

PRINCollection Methods. of

Notes

Illustrations.
as

Problems,
Third AN 8vo. SOLID of the

intended principally 8vo. cloth.

Examples
ON

of Newton's

Edition.

12s.

ELEMENTARY
I2j.

TREATISE

CURVE

TRACING.

GEOMETRY. Treatise

Being by
FROST and

New

Edition,

revised Vol.

and I.

enlarged,
8vo.
i6.r.

WOLSTENHOLME.

Godfray."
Lecturer A TREATISE Schools. AN
at

Works Pembroke ON 8vo.

by

HUGH

GODFRAY,
:
"

M.A.,

Mathematical

College, Cambridge
ASTRONOMY,
izs.

for

the

Use

of

Colleges
LUNAR the time

and

cloth,

6J. ON of the Problem 8vo. THE


to up cloth.

ELEMENTARY

TREATISE
a

THEORY,
Newton.

with Second

Brief

Sketch

of

Edition,

revised.

Crown

5^. 6d.

Green
LATE

(George)."
GEORGE

MATHEMATICAL

PAPERS Fellow of M. Gonville

OF and

THE Caius Fellow

GREEN,
Edited and

College, Cambridge.
and Tutor of Gonville

by

N.

FERRERS,
8vo.

M.A.,
15*.

Caius

College.

Hemming.
DIFFERENTIAL Use Fellow of

"

AN

ELEMENTARY
AND

TREATISE INTEGRAL CALCULUS. W.

ON

THE For the

of

Schools. Colleges and By G. St. John's College, Cambridge. and Additions. 8vo. cloth.

Second 9.?.

M.A., HEMMING, Edition, with

Corrections

Jackson.
Plane

"

GEOMETRICAL Treatise in which the

CONIC Conic

SECTIONS. Sections
are

An defined of
as

mentary Elethe

Sections STUART Crown

of

By J. College. Kelland
F. R.S.,

a Cone, and treated JACKSON, M.A., late

by

the Method of

Projections.
and Caius

Fellow

Gonville

8vo.

4^. 6d.

and
and

Tait.
With P. G.

"

AN

INTRODUCTION

TO

NIONS. QUATERM.A.,
of fs. 6d.

numerous

Examples.

By

P. in

KELLAND,
the

Mathematics

in the

M.A., University of Edinburgh.


TAIT,
Professors DRAW A STRAIGHT B. A.
A 2

department
8vo.

Crown

Kempe."
on

HOW

TO

LINE. Crown

Lecture is.6d.

Linkages.

By

A. B. KEMPE,

Illustrated.

8vo.

SCIENTIFIC

CATALOGUE.

Merriman. SQUARES.
Mechanical U.S.A.

"

ELEMENTS

OF

THE

METHOD Professor

OF of

LEAST Civil and

By
Crown A

MANSFIELD

MERRIMAN,

Engineering, Lehigh
8vo.

University, Bethlehem,

Penn.,

"js. 6d.
OF PROBLEMS With and
Answers.

Morgan."
IN

COLLECTION
MATHEMATICS.

AND

PLES EXAM-

By
Lecturer

H. of

A.

MORGAN,

M.A.,

Sadlerian Crown

Mathematical cloth.

Jesus

College, Cambridge.
Newton's
It is
a

8vo.

6s. 6d.

Principia.

"

4to. cloth.

31*.

6d.

of this complete edition of sufficient guarantee of the reliability been has that it printed for and under the care Principia and Thomson Professor Blackburn, oj of Professor Sir William Glasgow University.
Newton's
"

Parkinson. D.D.,

TREATISE
Fellow

ON and revised

OPTICS. Tutor and

F.R.S.,
Third 6d.

of

Cambridge.
cloth.
IDJ.

Edition,

SON, By S. PARKINJohn's College, 8vo. enlarged. Crown St.

Phear.

"

ELEMENTARY

HYDROSTATICS,

with and Cr.

Numerous
Tutor

Examples.
of Clare

ByJ. B. PHEAR, Coll. Cambridge.


ON Fellow 6s.

M.A.,
Fourth

Fellow Edition. DYNAMICS. of

late Assistant

Svo.

cloth. the

6d. 5-r.
Rev. G.

Pirrie.
Crown

"

LESSONS

RIGID and Tutor

By

PIRRIE,

M.A.,
Svo. AN AND

Queen's College, Cambridge.

Puckle.

"

ELEMENTARY ALGEBRAIC
Hints for their

TREATISE

ON

CONIC With
numerous

TIONS SEC-

GEOMETRY. Solution.
Crown

Examples
M.A.

and

By
Svo.

G.

HALE 7-r. 6d.

PUCKLE,

Fouth THE

Edition, enlarged.
THEORY
Fellow II. of
\2s.

Rayleigh.
F.R.S.,
Vol. 1.

"

OF

formerly
I2J.

6d. ; Vol. THE

RAYLEIGH, Svo. Trinity College, Cambridge. 6d. [Vol. HI. in preparation.


SOUND.

By

LORD

Reuleaux."
lines of
a

KINEMATICS
of Machines. edited

OF

MACHINERY. Professor F. REULEAUX. Professor

Out-

Theory
and

By
W.

Translated Civil With and

by

A.

B.

KENNEDY,

C.E.,

of

Mechanical
Illustrations.

450
"

Engineering, University College, 2os. Royal Svo. JOHN


of of St. London ON

London.

Routh.

Works and in

by
Assistant the

EDWARD Tutor

ROUTH,
Peter's
:
"

Fello%v Examiner AN THE

F.R.S., late College, Cambridge ; M.A.,


DYNAMICS
With
2U. numerous

University
TREATISE OF

ELEMENTARY
SYSTEM
Third

THE

OF

RIGID

BODIES.
Svo.

Examples.

Edition, enlarged.

SCIENTIFIC

CATALOGUE.

Todhunter
"

continued. OF THE and the MATHEMATICAL

HISTORY

THEORIES
of vols. the

OF the time of

ATTRACTION,
Newton
to

Figure
Two

Earth,
8vo.

from

that

of

Laplace.
TREATISE FUNCTIONS.

24^.

AN AND

ELEMENTARY

ON

LAPLACE'S,
Crown Svo.
lor.

LAME'S,
6d.

BESSEL'S

Wilson
W. and P.

(W.
WILSON,

P.).
"

TREATISE
Fellow of in St.

ON

DYNAMICS,

liy

M.A.,
of Mathematics

John's

College,

Cambridge,
Belfast.

Professor

Queen's

College,

Svo.

9-r.

6"/.

Wolstenholme.
"

MATHEMATICAL the First and Second Mathematical

PROBLEMS,
Divisions of the

on

jects Sub-

included of

in for the

Schedule

Subjects
and

Cambridge
by
sometime in the

Tripos

Examination. late Fellow and

Deviled of

arranged College,

JOSEPH
Fellow

WOLSTENHOLME,
of Indian 18.?. St.

Christ's
of

John's Engineering

College,

Professor
New

Mathematics

Royal
Svo.

College.

Edition,

greatly

enlarged.

Young.
"

SIMPLE

PRACTICAL

METHODS ON

OF

LATING CALCUAND in

STRAINS TRUSSES.
With
a

GIRDERS,

ARCHES,

Supplementary
W. of the

Essay
Associate of

on

Economy
of

sion suspen-

Bridges.
London,
Js. (xt.
and

By
Member

E.

YOUNG,
Institution

King's Engineers.

College,
Svo.

Civil

PHYSICAL

SCIENCE.

PHYSICAL

SCIENCE.

Airy
By

(G. B.)."
Sir G. B. 8vo.
A

POPULAR

ASTRONOMY.
Astronomer

With

Illustrations.
New Edition,

AIRY,
4-r. 6d. TREATISE

K.C.B.,

Royal.

fcap.
Balfbur.
OLOGY.

"

ON

COMPARATIVE

EMERY8vo.

By

F.

M.

BALFOUR,

F.R.S.

Illustrated.

{Shortly.
Bastian.
Professor "c.:" THE BEGINNINGS Modes Two "It of OF
and
"

Works of

CHARLTON by H. Pathological Anatomy

BASTIAN,
in

M.D.,

F.R.S.,

University College, London,


some

LIFE

Being
100

Account Lower

of the Naturt,

Origin,
book that

Transformations

of

Organisms.
Crown 8vo.

In

Volumes. is
a

With

upwards
cannot

of

Illustrations.
must

28.?.
to

be

ignored, and
A. R. Wallace OF

inevitably lead

renewed the EVOLUTION 6s.


"

discussions

and

repeated observations,and
"

through
Crown

these to

establishment AND

of

truth." THE

in Nature. LIFE. 8vo.

ORIGIN

6d.

Abounds science."-

information of -Daily News.

in

interest

to

the

student

of biological

Blake.
"The

"

ASTRONOMICAL

MYTHS.

Based With

on numerous

Flammarion't
tions. Illustra-

Heavens."
Crown

By John
8vo.
gs.

F.

BLAKE.

Blanford
H. F.

(H.
FOR

F.)."
THE

RUDIMENTS USE F.G.S. Terms OF With

OF INDIAN
numerous

PHYSICAL SCHOOLS. Illustrations New Edition.

GRAPHY GEO-

By
and Globe

BLANFORD,
of 6a. Technical

Glossary
8vo.
2s.

employed.

Blanford
ABYSSINIA.

(W.

T.).
By
W.

"

GEOLOGY T. BLANFORD.

AND 8vo. TREATISE

ZOOLOGY
2is.

OF

Bosanquet
INTERVALS
an

"

AN

ELEMENTARY AND TEMPERAMENT.

ON With
an

MUSICAL Account Collection


an

of of

Enharmonic

Harmonium South
to

exhibited

in

the

Loan

Scientific

Instruments,

Kensington, 1876
the Musical Fellow of

May,
Oxford.

exhibited Organ 1875. By R. H. 8vo. SEEING 6s. AND

; also of of Association

monic Enhar-

London,

Bosanquet,

St.

John's College,

Clifford.
K.

"

THINKING. With

By

the

late Professor 8vo.

W.

CLIFFORD,

F.R.S.

Diagrams.

Crown

6d. 3.5-. [Nature Series.

SCIENTIFIC

CATALOGUE.

Coal

ITS

HISTORY

AND

ITS and

USES.

By
8vo.

Professors of the

GREEN,
Yorkshire

MIALL,

THORPE,
Leeds.

RUCKER,
With

MARSHALL,

College,
' '

Illustrations.

12s.

6d.

It furnishes a very comprehensive treatise on the whole subject of Coal chemical, mechanical, and industrial from the geological, points of

vieiu, concluding liith


the
'

Coal

Question.'

""

chapter on Daily News.

the

important

topic known

as

Cooke
Ervine Third

(Josiah
Professor of

P.,

Jun.)."
and

FIRST

PRINCIPLES
P.

OF

CHEMICAL

PHILOSOPHY.

By
corrected.

JOSIAH
Crown

COOKE,
12s.

Jun.,

Chemistry
and

Mineralogy

in Harvard 8vo.

College.

Edition,

revised

Cooke
with Genera.
"

(M.
full

C.)."
M.C.

HANDBOOK
all the M.A.
as

OF

BRITISH

FUNGI,
of 24?.
on

descriptions of
By COOKE,
its

Species, and
Two standard years
to

Illustrations
crown

the

vols.

8vo.

Will

maintain which

place

the

English book,
come.'''
"

the

of subject

it treats,for many OF
1,200

Standard.

Crossley.
CATALOGUE

"

HANDBOOK OF

DOUBLE STARS FOR THE

STARS,
AND

WITH SIVE EXTENOF


TEURS. AMA-

DOUBLE

LISTS

OF

MEASURES

USE

and

J.

M.

By E. CROSSLEY, F.R. A. WILSON,


Works

F.R.A.S.,
S. With

J. GLEDHILL,
Illustrations.

F.R.A.S.,
8vo.
2\s.

Dawkins.
of CAVE-HUNTING the Woodcuts.
"

"

by
at
:

W.

BOYD

DAWKINS,
Manchester. Evidence With the

F.R.S.,
of Caves

"c.,

fessor Pro-

Geology Early

Owens Researches of

College,
on

ing respectPlate and

Inhabitants
2 is.

Europe.

Coloured

8vo.

The
use

mass

he

with the judicious of information he has brought together, has made of his materials, will be found to invest his book

with EARLY TERTIARY

much MAN

of

new

and

singular value."
AND With

"

Saturday
HIS PLACE 8vo.

Review^. IN THE

IN PERIOD.

BRITAIN,

Illustrations.

{Shortly.
The

Dawson
Scotia,
WILLIAM

(J. W.).
Organic
New

"

ACADIAN

GEOLOGY.
and Prince Mineral Edward

Geologic
of Nova

Structure,

Remains,
and

Resources Island.

Brunswick, M.A., DAWSON,


of M'Gill and

By

JOHN

Vice-Chancellor With with


a

LL.D., College
numerous

F.R.S.,
and

F.G.S., Principaland University, Montreal, "c.


Third

Geological Map 8vo. Supplement.


"

Illustrations.

Edition,
6d.

zis.

Supplement,
OTHER Lecturer
on

separately,2s.
By

Fiske.

DARWINISM; M.A., LL.D.,


Crown 8vo.

AND

ESSAYS.

JOHN

FISKE,

formerly

Philosophy

in Harvard

University.

7-r. 6d.

PHYSICAL

SCIENCE.

Fleischer. By
Dr. Edition

"

SYSTEM FLEISCHER.

OF

VOLUMETRIC
from with the Notes

ANALYSIS.
Second and German

E.

Translated

by

M.

M.

Pattison 8vo.

Muir,
7-r. "/.

Additions.

Illustrated.

Crown

Fluckiger
History
Great D. of

and
the Britain

Hanbury."
Principal Drugs and India. By
F.R.S. Second of

PIIARMACOGRAPHIA.
with Vegethlj Origin met F. A. FLUCKIGER, M.D., 8vo. Edition, revised. 2is,

A in and

HANBURY,
"

Forbes.
B.A.,
of

THE Professor

TRANSIT of Natural With

OF

VENUS. in the

By

GEORGE

FORBES,
sity Univer8vo. $s. 6d.

Philosophy

Andersonian Crown

Glasgow. and

numerous

Illustrations.

Foster

Balfour.
FOSTER,
I. Crown

"

ELEMENTS

OF andF. With M.

EMBRYOLOGY

By

MICHAEL of Part

M.D.,
8vo.

F.R.S.,
"js. 6d.

BALFOUR.
numerous

M.A.,
tions. Illustra-

Fellow

Trinity College, Cambridge.

Galton.

"

Works

by

FRANCIS
or

GALTON,
Methods Printed
:

F.R.S.

"

METEOROGRAPHICA, Illustrated by upwardsof


HEREDITARY GENIUS

600

of Mapping the Weather. LithographicDiagrams. 410. gs. into its Laws and sequences. Con-

An

Inquiry
most

Demy
The ENGLISH NURTURE.
"

8vo.
most

12s.

Times

calls it "a MEN OF 8vo.

able and

book" interesting NATURE AND

SCIENCE;
Ss. 6d.

THEIR

The

book

is

certainly one TEXT-BOOK,

of

very

great

interest."

"

Nature.

Gamgee."
OF MAL of THE

SYSTEMATIC CHEMISTRY

and

PRACTICAL,
OF THE
AiNI-

PHYSIOLOGICAL

BODY.

By

ARTHUR

GAMGEE,

M.D.,

F.R.S.,
With

Professor

Physiology

in Owens

College, Manchester.

Illustrations.

8vo.

[/"
"

tfie press.

Geikie.

Works

by

ARCHIBALD
of

GEIKIE,
and PHYSICAL 8vo.
4s.

LL.D.,
at

F.R.S.,
:
"

Murchison

Professor

Geology
IN

Mineralogy
6d.

Edinburgh
is.

ELEMENTARY
With
numerous

LESSONS Illustrations. OF 3?. 6d. OF OF FIELD

GEOGRAPHY.

-Fcap.

Questions,

6"/.

OUTLINES
8vo.

GEOLOGY.

With

Illustrations.

Crown

PRIMER
PRIMER

GEOLOGY.
PHYSICAL

Illustrated.
GEOGRAPHY.

i8mo.

is.

Illustrated.
BOOK ON Caius HEAT.

i8mo.

is.

Gordon.
H. Crown

"

AN

ELEMENTARY

GORDON,
8vo.

B.A.,
zs.

Gonville

and

By J. E. College, Cambridge.

io

SCIENTIFIC

CATALOGUE.

Gray.

"

STRUCTURAL

BOTANY

ON
ASA

THE With

BASIS

OF

MORPHOLOGY. 8vo.

By

Professor

GRAY.

Illustrations.

[In
"

the press. duction Intro-

Guillemin.
to

THE the

FORCES

OF of

NATURE Phenomena. French and

Popular
By
MRS.

Study
Translated

Physical
from the Additions

AMEIJKE

GUILLEMIN.
LOCKYER

by

NORMAN

LOCKYER,
Third
"

; and F.R.S.

Edited,

with

Illustrated

by

Coloured

Notes, by J. NORMAN cuts. Plates, and 455 Woodvo. 2is.

and and

cheaper
Editor

Edition. have

Royal
done

The justice to their task. has text all the force and flow of original writing combining faithfulnessto the author's meaning with purity and independence in regard to idiom ; while the historical precision and accuracy the "work pervading throughout, speak of the watchful editorial has been given to every scientific detail. supervision which Nothing cuts. well exceed the clearness and can delicacy oj the illustrative woodeither the be said have work to no Altogether, parallel, may in point of fulness or attraction,as a popular manual of physical

Translator

science."
THE

"

Saturday

Review. OF from PHYSICAL the French FORCES.

APPLICATIONS GUILLEMIN.
Edited With with

By
LOCKYER,

A. and

Translated
Notes and Plates 8vo.

by
N.

Mrs.

Additions and
numerous

by J.

LOCKYER,

F.R.S.

Coloured

Illustrations.

Cheaper

Edition.
Also in

Imperial cloth, Eighteen Monthly Parts, price 1878.


extra
"

gilt. 36^.
u.

each.

Part

I. in November,

book

width

of the heartily recommend, both on account lence soundness also because of the excelof its contents, and and external of its print, its illustrations, appearance""
we
can

which

and

Westminster

Review.
SCIENCE PAPERS DANIEL
:

Hanbury.
Memoir,

"

chiefly Pharmacological
F.R.S. Portrait

and with

Botanical.

B'y
8vo. THE and

HANBURY,
and

Edited,
C.

by J. INCE,
14*. THEORY

F.L.S.,

engraved by

H.

JEENS.
Henslow. THINGS,
"

OF of

EVOLUTION the

OF of

LIVING
to

Application
as

Principles
of the Rev.

Evolution and

Religion
of

considered the

Illustrative

Wisdom GEORGE

cence Benefi-

Almighty.
Crown 8 Sir
:"

By
vo.

the 6s. D.

HENSLOW,

M.A., Hooker. F.R.S.,


THE Second
"
"

F.L.S. Works

by
D.C.L.

J.

HOOKER,
BRITISH Globe manual

K.C.S.I.,

C.B.,

M.D.,

STUDENT'S Edition,

FLORA
revised and

OF

THE

ISLANDS.
8vo.
los.

improved.
accurate

6d. kind that

Certainly the fullestand most Hooker has yd appeared. Dr.

of

the

has

shown

his c/iaracteristic

industry

PHYSICAL H

SCIENCE.

\i

OOker
and

"

continued.

ability

in

the

care

and

skill
are

-which
to
a

he

has
extent

thrmun

into the

characters
are

of the plants.
admirable
"
"

These
their

great

and

really completeness.
OF BOTANY. and revised

for
Pall

combination Gazette.

original, and of clearness, brevity,


iSmo.
is.

Mall With

PRIMER

Illustrations.

New

Edition, Hooker
AND

corrected.

and
THE

Ball."
GREAT

JOURNAL
ATLAS.

OF

TOUR

IXMAROCCO

K.C.S.I., F.R.S., "c., and JOHN F.R.S. With BALL, Appendices, of the including a Sketch Geology of Marocco. By G. MAW, With 8vo. F.L.S., F.G.S. Map aad Illustrations. 2is. This the is, without doubt, one most valuable of interestingand books of travel published for many years." Spectator.
C.B.,
"
"

By

Sir

J. D.

HOOKER,

Huxley
HUXLEY,

and

Martin.
IN

"

COURSE

OF BIOLOGY.

PRACTICAL

STRUCTION INT. H.

ELEMENTARY Sec.

LL.D.,
D.

R.S.,

assisted

by

H.

N.

By MARTIN,
Crown

B.A.,
8vo.

M.B.,
6s.
"

Sc.,

Fellow

of Christ's

College, Cambridge.
book in
to

This of

is the

most

thoroughly
has
ever

valuable

teachers

and

students
"

biology

-which

appeared

the

English tongue"

London

Quarterly

Review.

Huxley
AND and

(Professor).
REVIEWS.

"

LAY T. H.

By
Edition.
on

SERMONS, ADDRESSES, HUXLEY, LL.D., F.R.S.


7.?.6d.
"

New

Cheaper

Crown the

8vo.

Fourteen
ness

following subjects: (i) On the Advisable(2) Emancipation Knoivledge : Liberal A White Black and to find : Education, and where (3) the Educational it: Value (5) On (4) Scientific Education: of the Sciences: On the Natural (6) History Study of Zoology: of Life:" (8) The ScientificAspects of (7) On the Physical Basis Positivism: poraneity (10) Geological Contem(9) On a Piece of Chalk: and Persistent of Life: Types (u) GeologicalReform: the (12) The Origin of Species: (13) Criticisms on "Origin of of Improving
Natural
"

Discourses

"

"

"

"

"

"

"

"

"

"

"

Species:" (14)
"

On

Descartes'

"Discourse

touching

the Method

of

using
ESSAYS

One's

Reason

rightlyand
FROM

of seekingScientific Truth."
"LAY Second 8vo. Nihilism.
2.

SELECTED AND

SERMONS,
Edition.
ios.

DRESSES, AD-

REVIEWS."
ADDRESSES.

Crown 6d. School On Medical 6.

8vo.

is.

CRITIQUES
Contents: "what
"

AND
I.
can

Administrative

The 3. of Coal.
Results

Boards: cation. EduCoral

they
4. Coral

do,
Yeast.

and

-what

they

may

do.

5-

On

Hie Formation the Methods and

On

and 8. On

Reefs.
Fixed

some

7. On Points

of Ethnology.
9.

in British

Ethnology.

Palaeontology

12

SCIENTIFIC

CATALOGUE.

Huxley
and
II.

(Professor)
the Mr. Doctrine Darwin's

"

continued.
10. 12.

of

Evolution. Critics.
on

Biogenesis and
The

Abiogenesis.
of
Animals.

13.

Bishop Berkeley
IN ELEMENTARY New

the

Genealogy Metaphysics of Sensation.


PHYSIOLOGY.

LESSONS

With 4?. 6d.

numerous

Illustrations.
"Pure and
any

Edition.
"

Fcap.
on

8vo.
"

gold throughout." most complete elementary language." Westminster


"

Guardian. treatise Review.


:

this

Unquestionably subjectthat we
Lecture
on

the

clearest

possess in

AMERICAN

ADDRESSES 8vo. 6*. 6"/. An Plates fs. 6d. would in be the


to
"

with

the

Study

of

Biology.
PHYSIOGRAPHY: Coloured 8vo.
"

Introduction

to

the

Study
New

of Nature. Edition.

With Crown

and

numerous

Woodcuts.

It

hardly possibleto place


hands make

more

useful
or one

or

suggestive
is

book

of

learners

and
a

teachers,

that
the

better science

calculated schools."

physiography Academy.

favourite

subject in

Jellet
THEORY Senior Irish

(John
OF Fellow

H.,
of

B.D.)."

TREATISE

ON H.

THE

FRICTION.

By
("d.

JOHN

JELLET,
of the

B.D.,

Trinity College,
8vo. Ss.

Dublin

; President

Royal

Academy.
"

Jones.
THE

Works in the OWENS

by

FRANCIS

JONES, F.R.S.E.,
School,
With With Manchester. COURSE

F.C.S.,
OF Professor

Chemical

Master

Grammar COLLEGE

JUNIOR
Preface Illustrations. A

TICAL PRACROSCOE.

CHEMISTRY. New Edition. ON in i8mo.

by
2s.

dd. of Problems 3-r. OF of Crown THE and

QUESTIONS
Exercises

CHEMISTRY. and

Series

Inorganic
GLAUCUS CHARLES

Organic OR,
THE

Chemistry.
WONDERS Canon Plates.

iSmo.

Kingsley.
SHORE. New

"

By Edition, with
"

KINGSLEY,
Coloured

Westminster. 8vo. 6s.

numerous

Landauer.
Authorised the Owens

BLOWPIPE

ANALYSIS.
TAYLOR

By
and

j.
W.

LANDAUER.
E.

English Edition, by JAMES With College, Manchester.

KAY,

of

Illustrations.

Extra

fcap.

8vo.

4-f 6d.
"

Langdon.
RAILWAY

THE

APPLICATION

OF E. With

ELECTRICITY
Member

TO
of the

WORKING. of

numerous Telegraph Society Extra fcap. 8vo. ^s. bd. -who in the telegraph service There is no officer the study of this book." Mining Journal.
"
"

By W. Engineers.

LANGDON,

Illustrations.

will

not

profitby

14

SCIENTIFIC

CATALOGUE.

Macmillan
FIRST and FORMS

(Rev.
OF

Hugh)"
VEGETATION.
Coloured 6s.

continued. Second

Edition,
numerous

corrected tions. Illustra-

enlarged, with
Globe 8vo.

Frontispiece and

The

first edition of this book was the name published under of from the Page of Nature; or, First Forms of Vegetation. Probably the best popular guide to the study of mosses, Its practical value lichens, and fun^i ever -written. a "zr help to
"Footnotes the student and collector
cannot

be

exaggerated."
"

Manchester

Examiner.

Mansfield
A THEORY'

(C. B.).
OF

"

Works A Chemical

by

the

late C.
on

B.

MANSFIELD Constitution Crown 8vo.

:"

SALTS.

Treatise

the

of 14^. its

Bipolar (two-membered)
AERIAL Solution. M. LUDLOW. NAVIGATION. Edited R.

Compounds.
Problem,
Crown With

The

with
a

Hints Preface ()d.

for

by
With

B.

MANSFIELD.

by J.

Illustrations.

8vo.

105.

Mayer.

"

SOUND

Series in the

of

Simple,
M.

Experiments
Students in the of
every

Phenomena A.

Stevens Crown

age. Institute 8vo.

By
of

expensive InEntertaining, and Sound, for the Use of Professor of Physics MAYER, of

Technology,

"c.

With

numerous

trations. Illus-

3^. 6d.

Mayer
the

and
and
use

Barnard.
Useful Students With of

"

LIGHT. in

A the

Series Phenomena M.

of

Experiments

taining, Simple, Enterof Light, for


MAYER

BARNARD.

of every By A. age. Svo. Illustrations. Crown IX COMPARATIVE


A

and

C.

2s.

6d. No.

Miall."
The

STUDIES

ANATOMY. Manual for Students. Svo. L. C.

i,

Skull

of

the

Crocodile.
of

By

L.
is.

C. 6d.

MIALL,
No. and

Professor

Biology
of the With

in Yorkshire Indian Plate-. OF

Anatomy 2, The F. GREENWOOD.


THE ROMANCE Fellow

College. Elephant. By
5.?.

MIALL

Miller."
MILLER,
Svo.

ASTRONOMY. Assistant Tutor and of

By
St.

R.

KALLEY lege, ColCrown

M.A.,

and

Peter's

Cambridge.
4^. 6d.

Second

Edition,

revised

enlarged.

Mivart
"c ON THE
notes

(St. George).
Lecturer
in

GENESIS
have been With work treated the in
at

Mi VART, F.R.S. by ST. GEORGE at St. Mary's Hospital: Comparative Anatomy Second SPECIES. OF Edition, to which
"

Works

"

added
numerous

in reference

and

reply to
Crown has

Darwin's Svo.

"

Descent gs.

of
"

Man."
In been
no

Illustrations.

the
once

English language
ivith liberal the and
same

this

broad

and
"

great controversy vigorous grasp of

facts,and

same

candid

temper."

Saturday

Review.

PHYSICAL

SCIENCE.

15

Mivart
THE 8vo.
"

(St. George)"
COMMON 3^. 6d.
is
an

continued. With Numerous

FROG.

Illustrations.

Crown

ft

throws Would of

Series.) It of the Frog, and something more. wide portions of animated valuable crosslightsover nature. that such works were more plentiful. Quarterly Journal
able
monogram
"
"

(Nature

Science.
"

Moseley.
the voyage

NOTES

BY
an

NATURALIST of various

ON

THE

LENGER," "CHALmade in the

being
of

account
"

observations the world

during
years

H.M.S.
H. of N.

Challenger"

round

1872" Plates,
"

76.
and
is

By
Staff

MOSELEY,
the

M.A..

F.R.S.,
With

Member

of

the

Scientific

"Challenger."
8vo.
2U.

Map,

Coloured

Woodcuts.

I"ook,descripsuggestive tive Mrsince travels, published of a Darwin's than forty more appeared, now Journal of Researches That be that it is to worthy placed alongside delightful ago. years and the record reflections impressions, specidations, of a master of

This

certainly the
na/uralis"s
'

most

interesting and
which

has

bten

'

mind,
"would

is, we
desire

do

not

for

his
"

doubt, the highest praise which do not hesitate book, and we


Nature. CHEMISTRY for the FOR

Mr.
to

Moscley
that
such

say

praise Muir.
M.
"

i-Sits desert.''

PRACTICAL

MEDICAL B.
is.

DENTS. STUCourse. 6d.


a

Specially arranged
M.
PATTISON

first M.

By

MUIR,
AND Laws of

F.R.S.E.

Fcap.

8vo.
:

Murphy.
Essays
M'~KPHY.

"

HABIT
on

INTELLIGENCE Life
and

Series

of

the Second With

Mind.

By
revised

Edition,

thoroughly
8vo. i6s.

and

JOSEPH JOHN written. mostly re-

Illustrations. WEEKLY Published

Nature."
SCIENCE.

ILLUSTRATED every

JOURNAL
Price 6d. Cases for

OF

Thursday.

Monthly
binding

Parts,
Vols.
"

2s. is.

and 6d. and

2s.

6d. ;

Half-yearly Volumes, Journal,


which

i$s.

This the
and

able

well-edited and

day promptly,
savants

"would
the

exaggerate
"

our

of promises to be of signal service to students can employ Scarcely any expressions that we and value the moral theological sense of of
posts up
Review.

the

science

work."
"

British POPULAR

Quarterly
U.S. of

Newcomb.
COMB,

ASTRONOMY. Naval the Stars.

LL.D.,

Professor Five

By SIMON With Observatory.


8vo. \%s.

NEW112

Maps Engravings and advanced reliable foundation for more "As a thoroughly affording is deserving Popular Astronomy reading, Professor Newcomfts Nature. of strong recommendation." of F.R.S., F.L.S., Professor Oliver OLIVER, \\nvks by DANIEL of the rium Herbaand Keeper Botany in University College, London,
'
'
"

and

Library

of

the

Royal Gardens.

Kew

"

SCIENTIFIC
continued. IN

CATALOGUE.

Oliver
LESSONS

"

ELEMENTARY
New INDIAN

BOTANY. Edition.

With 8vo. With

nearly
4^. 6d.
numerous

Two

Hundred FIRST

Illustrations.
BOOK OF Extra
a

Fcap.

BOTANY.
6s. 6d.

Illustrations. "ft contains


to

fcap. 8vo.
Botany,

-well-digested summary
Indian

of
out
"

all

essential

pertaining
test

wrought principles of scientific arrangement."


"

in accordance Allen's Indian The them.

knowledge ike with


Mail. Diseases

Pasteur.
of Beer

STUDIES
;

ON Causes
and

FERMENTATION.
Means or "Etudes of

their A

Preventing
sur

By

L.

PASTEUR.

Translation

la and

Biere,"
D. C.

With

Note-,
B.A.

Illustrations, "c.
8vo.
2U.
"

By

F.

FAULKNER

ROBB,

Pennington.
CAVES into Elden 8vo. OF

NOTES DERBYSHIRE.

ON

THE With

BARROWS
an

AND
account

BONE
a

of

Descent

Hole. 6s.

By

ROOKE

PENNINGTON,

B.A.,

LL.B.,

F.G.S.

Penrose
GRAPHICAL STARS ANY for the

(F. C.)"
BY GIVEN

ON

METHOD

OF

PREDICTING

BY OF FOR methods

CONSTRUCTION, THE AND MOON, PLACE. Together


Calculation of With

OCCULTATIONS SOLAR with


more

ECLIPSES

rigorous By
12s.

Accurate

Longitude.
"c.

F.

C.

PENROSE,

F.R.A.S.

Charts, Tables,

410.

Perry."

AN

ELEMENTARY

TREATISE Professor With of


numerous

ON

STEAM.

By
of amples, Ex-

JOHN Engineering,
and "Mr.

PERRY,

B.E.,
Yedo. Exercises. has in

Engineering. Imperial College


Woodcuts,
qs. 6d. little volume Numerical

i8mo.
this

brought togetheran and told, regarding steam of information, new its application, the least of its mails not being that it is suited to alike of the ty-o in the capacities engineeringscience or the better Iron. grade of artisan."
Perry
compact
immense
amount
"

Pickering."
By
Part
"

ELEMENTS

OF

PHYSICAL Professor Part of

MANIPULATION.

E.

C.

PICKERING,
of
los.

Thayer

Physics

in the

chusetts Massalos.

Institute

Technology.

I., medium
no
on

8vo.
doubt the

("d.

II.,

6d.
(

When iiiJcred "which

finished
the best
"

Physical Manipulation1 -will and most complete text-book


Nature. PAST AND

be

con-

subject of

it treats."
"

Prestwich.
An

THE

FUTURE

OF

GEOLOGY. F.R.

Inaugural Lecture, by J. PRESTWICH, 8vo. 2s. Professor of Geology, Oxford. Radcliffe."


B.

M.A.,

S., "c.,

PROTEUS

OR

UNITY
of 8vo.

IN "Vital

NATURE.
Motion
us a

By.
mode

C. of

RADCLIFFE,

M.D.,
Second

Author

Physical Motion.

Edition.

"js.6J.

PHYSICAL

SCIENCE.

Rendu."

THE M.
LE

THEORY CHANOINE of the

OF RENDU. Club.

THE

GLACIERS Translated

OF

SAVOY.

By

by
are

late President Memoir


KIN.

and Edited of

Alpine Supplementary
with Natural

To

which

WELLS, Q.C., added, the Original


A. TAIT and Rus-

Articles remarks in

by

Professors

Introductory
'js.6d.

by
the

GEORGE

FORBES,

B.A.,

Professor

Philosophy

Andersonian

University,

Glasgow.
ROSCOC.
"

8vo. Works in Owens IN

by

HENRY

E.

ROSCOE,
:
"

F.R.S.,

Professor

o.

Chemistry
LESSONS AND litho Earths. CHEMICAL THORPE.
"

College, Manchester
With

ELEMENTARY
numerous

CHEMISTRY,
Illustrations of the 4^. 6d.
to
2s.

INORGANIC
and and ChromoAlkaline Alkalis

ORGANIC. of the New Solar

Spectrum, Edition. Fcap.

and Svo.

We

PROBLEMS, adapted with Edition, Key. it the unhesitatingly pronounce


Fifth
on

the

above

by
our

Professor

best

of

all

elementary

treatises PRIMER OF

Chemistry."
"

Medical

Times.

CHEMISTRY.

Illustrated.
A

i8mo.

u.

.Roscoe
PROFESSORS

and

Schorlemmer.
CHEMISTRY. With and SCHORLEMMER. Elements. Svo.

"

TREATISE

ON

ORGANIC IN-

numerous

Illustrations.

By

ROSCOE
Non-metallic I. II.
as a

Vol. Vol. Vol.


"

I.,

The

Svo.

2is.

II., Part
II., Part

Metals.

i8s. iSs.

Metals.
treatise
on

Svo.

Regarded
no

the Non-metallic is

Elements,
most

there

can

be

doubt which

that
we

this volume
are

of
' '

in

possession"
"

incomparably the Spectator.

one satisfactory

It would

the merits too highly. All to praise the *uork difficult in the second. in the volume which are conspicuous we first the fads gained by modern is clear and scientific; The arrangement the and research selected;and judiciously are represented fairly Lancet. lucid. is singularly stylethroughout be noticed
"
"

Rumford
OF Notices Five

(Count)."
BENJAMIN
of his Svo. Vols.

THE

LIFE

AND COUNT

COMPLETE RUMFORD. ELLIS. With

WORKS With Portrait.

THOMPSON,

Daughter.
4/.
A

By
6d.

GEORGE

14^.

Schorlemmer."
THE

MANUAL

OF OR

THE ORGANIC in

CHEMISTRY
CHEMISTRY.

OF

CARBON
C.

COMPOUNDS

By
Owens "//

SCHORLEMMER,

F.R.S.,

Lecturer
IAS.

Organic Chemistry

in

College,
appears
to
us

Manchester.
to

Svo.

be

as

complete a
"

manual

could be at present produced, and carbon as Athenreum. student. " useful to the chemical
1!

of the melamorphoses of it must prove eminently

18

SCIENTIFIC

CATALOGUE.

Shann.

"

AV

ELEMENTARY TO STEAM With AND

TREATISE THE

ON STEAM Crown

HEAT,
ENGINE. 8vo.

IN

RELATION

By Smith.

G.

SHANN,
"

M.A.

Illustrations.

4^. 6d.

: An Exposition of the Nature, Review of the ciples PrinFerns, and which Genera and the of cation Classifiare founded, Systems upon of the principal Authors, with General new a Arrangement, A. SMITH. ex-Curator of the "c. L.S., J. By Royal Botanic With Plates Kew. W. H. FITCH, Thirty Lithographic Garden, by

HISTORIA
and

FILICUM

Number,

Organ ography

of

F.L.S.
"

Crown
one

vo.

12s.

6d.

No

anxious

to

work

up

thorough kncnvledgeof ferns


Chronicle.

can

ajford to do without
South
Vol.
I.
"

if."

"

Gardener's

Kensington
Containing
Professor G.

Science
Lectures

Lectures.

STOKES,
F.R.S.E.,
Crown

KENNEDY,
H. C.

FORBES,
S.

by Captain ABNEY, F.RS., Professor F. fessor F.R.S., ProJ. BRAMWELL, T. SORBY, F.R.S., J. BOTTOMLEY,
and Professor CAREY II.

H.
6s.

VINES,
Lectures

B.Sc.,

FOSTER.

8vo.

[Vol.

nearly ready.
P.R.S.,
W. Prof.

Vol.

II."

Containing
H.
Dr.

by

W.

SPOTTISWOODE,
T.

FORBES, F.R.S.,
Dr.

W.

CHISHOLM, Prof. SIEMENS,


BRUNTON,
"c. Crown

Prof.

F.

PIGOT,
Dr.

FROUDE,
Prof.

BARRETT,

SON, BURDEN-SANDER-

LAUDER

F.R.S.,
8vo. 6s. OF the

Prof.

McLEoo,

ROSCOE,

F.R.S.,
"

Spottiswoode.
SPOTTISWOODE,
Illustrations.
"

POLARIZATION President of

LIGHT. With

By
numerous

W.

Royal
8vo.

Society.
31-.6d.

The the

(Nature Series.) illustrations are exceedingly well adapted to assist in making "/4 text comprehensible." Athenaeum. clear, trustworthy
"

Second

Edition.

Cr.

manual.

"
"

Standard.
"

Stewart
of

(B.).
Natural
IN and

Works

by

BALFOUR

STEWART,
PHYSICS.

F.R.S.,jProfessoi
:
"

Philosophy

in Owens

College, Manchester
With of ("d. the of the

LESSONS
Nebulae.

ELEMENTARY
Chromolithos Edition. Times
calls

numerous

Illustrations
and The

Spectra
4^.

Sun, Stars,

New

Fcap.
this

8vo.

Educational

the beau-ideal

of
New

book, textscientific

clear, accurate,
PRIMER
OF PHYSICS. l8mo.
\s.

and

thorough."
With Illustrations.

Edition,

with

Questions.
Stewart and

Tait.
P. G.
one

"

THE
a

UNSEEN State. Sixth deserves is


a

UNIVERSE:

or,

Physical Speculations on
F.R.S.,and
"

Future M.A.

By

BALFOUR Crown

STEWART,
8vo.
6s.

TAIT,
-which
. .
.

Edition.

The

book

is

well It

the attention

of thoughtful and

religiousreaders.

sobrr inquiry,on perfectly


a

scientific

grounds,

into

the

of possibilities

future existence."

"

Guardian.

PHYSICAL

SCIENCE.

19

Stone.
W. With

"

ELEMENTARY H.

LESSONS Lecturer
on

ON
at

SOUND.
St. Thomas'

By

Dr.

STONE,

Physics
y.

Hospital.

Illustrations.
LECTURES

Fcap.
ON

8vo.

6d.

Tait."

SOME
P.

RECENT
G.

ADVANCES

IN of

PHYSICAL

Philosophy
revised the and British

By in the University enlarged, with the


Association. Crown

SCIENCE.

TAIT,
on

M.A.,

Professor

of

Edinburgh.
Force 9^.

Second delivered

edition,
before

Lecture

8vo. OF

Tanner."
HENRY

FIRST

PRINCIPLES

AGRICULTURE.

By
Science,
i8mo.

of F.C.S., Professor Examiner University College, Aberystwith,

TANNER,

Agricultural
in the of Science.

Principles of

Agriculture under
is.

the

Government

Department

Taylor.
on

"

SOUND the

AND

MUSIC

Non-Mathematical Sounds and

tise Trea-

Physical

Constitution

of Musical

Harmony,
HelmCol.

Acoustical Discoveries of including the Chief holtz. TAYLOR, M.A., late Fellow By SEDLEY 8s. 6d. 8vo. Large crown ledge, Cambridge.
"

Professor of

Trinity

In
so

no

treatise scientific previoi4s illustrated


n a

do

we

remember

so

exhaustive and

and

richly

description of forms
"

of

vibration

of

wave-mod

in

fluids" by

Musical WYVILLE SEA Cruises Summers


:

Standard.

Thomson.
THE DEPTHS Results

"

Works OF the

SIR

THOMSON,
An

K.C.B.,
of the

F.R.S. General and the

THE

Account SS.

of

Dredging
during
the

of

H.M. of

"Porcupine"
and

"Lightning"
scientific

1868-69

70, under

direction and Sir and

F.R.S.,

Illustrations

of Dr. Carpenter, F.R.S., J. Gwyn Jeffreys, With F.R.S. nearly 100 Wyville Thomson, Edition. 8 coloured and Plans. Second Maps 31*. 6d. The book the is
art

Royal
The

8vo.

cloth, gilt.
"

Athenaeum

says

full of interestingmutter,
of popular

and

exposition. It is is written of by a woodcuts both coloured and illustrated, possessing maps excellently in dredging interested become who have Those merit. already high make a point of reading this ivork ; those operations will of course and to the subject, to be pleasantly introduced wish rightly who the which ives from time to time from the news art to appreciate
master
'

Challengtr* should
VOYAGE
A

not

fail to
"

seek instruction

from it."
THE TIC. ATLANof H.M.S. of

THE

OF

THE

CHALLENGER.""
of the

Preliminary

account

Exploring Voyages
and the "

"Challenger," during
With of the
numerous

year Illustrations, Coloured

the

1873

early part
Charts, "

1876.

Maps

Portrait

Svo. 42^. Author, engraved.byC. II. JEENS. 2 Vols. Medium have the public should It is right that some Times The says : and of the general results of the expedition, account authoritative
"
"

SCIENTIFIC

CATALOGUE.

Thomson
that
as

"

continued. many should data as may of the ascertained their speedily Jind place in
can

be

fidence accepted"with conthe general body of

the public in. expedition to satisfy this respect. The paper, printing, and the especially numerous have We are illustrations, of the highest quality. rarely, if more seen specimens of wood engraving than abound beautiful ever, in this -work. Sir Wyville Thomson's style is particularly attractive ; he is easy and but vigorous and graceful, exceedingly happy in the choice of language, and throughout the work there are
. . . . . . . . .

one scientific knowledge. No accomplished scientific chief of

be

more

competent

than

the

tJie

touches

which

show

that

science

has

not

banished

sentiment

front,

his bo win."

Thudichum

and

Dupre.
AND

"

TREATISE
OF and

ON

THE

ORIGIN, NATURE, Being a Complete Manual W. THUDICHUM, M.D.,

VARIETIES

WINE.

of Viticulture and AUGUST

CEnology.
Ph.D.,
8vo.
cloth

By J.
Lecturer

L.
on

DUPRE,
Medium

Chemistry
"A treatise the

at

Westminster

Hospital.
its

gilt.25.?.

almost

unique for
the

vendor, or consumer have most complete we constituent principles of nearly


the
"

either to the wine-grower, usefulness wine. The analyses of wine are of exhibiting at a glance the yet seen, all the wines knoivn in this country.
"

Wine

Trade

Review.

Wallace
SELECTION.

(A. R.).

"

Works

by
THE of

ALFRED THEORY

RUSSEL OF New Ss. 6d.

WALLACE. NATURAL

CONTRIBUTIONS
A and

TO

Series

E^ays.
8vo.

Edition,

with

Corrections The

Additions.

Crown

fresh

"He has combined abundance an Saturday Review says: of and facts with a liveliness and sagacityof reasoning original which small scale." not oftendisplayedso effectively on so a are GEOGRAPHICAL
a

THE with

DISTRIBUTION

OF

ANIMALS,
as

of Living and Extinct Faunas study of the Relations Earth's the of the Past Surface. vols. 2 Elucidating Changes with and Illustrations numerous by Zwecker, 42.?. Maps, The says : whatever story, Mr. Wallace it. adornments Times
"
"

8vo.

Altogether it is a wonderful and fascinating be taken to theories founded objections upon may

has not attempted to add to its interest by any he has of style given a simple and char statement of ; and what he considers be to intrinsicallyinteresting facts, mate legitiinductions Naturalists from them. ought to be grateful to him toilsome a task. The work, indeed, so for having undertaken is a credit to all concerned the author, the publishers, the artist
" "

unfortunately now
but

no

more

by

no

ncans

feast, Mr.

of Statif.n-.r; v-af-fa^tmr.'''
"

the

attractive

illustrations

"

last

SCIENTIFIC

CATALOGUE.

SCIENCE

PRIMERS SCHOOLS.

FOR

ELEMENTARY

Under

the

joint Editorship
BALFOUR

of

Professors STEWART.

HUXLEY,

ROSCOE,

and

Introductory.

By

Professor

HUXLEY,
F.R.S.

F.R.S.
Professor
,

[Nearly ready.
of

Chemistry
in Owens l8mo.

"

By H. E. ROSCOE, College, Manchester.


New Edition. BALFOUR in Owens i8mo.
_

Chemistry

With

numerous

Illustrations.

is.

With

Questions. F.R.S.,
Edition.
With Professor With
ous numer-

Physics."
Natural

By

STEWART,
is.

of

Philosophy

College, Manchester.
New

Illustrations.

Questions. F.R.S.,

Physical
Murchison With i8mo.

Geography.
Professor of
numerous u.

By

ARCHIBALD and New

GEIKIE,
at

Geology

Mineralogy
Edition

Edinburgh.
Questions.

Illustrations.

with

Geology

"

By Professor
New Edition.

GEIKIE,
i8mo.

F.R.S.
cloth,
is.

With

numerous

trations. Illus-

Physiology
numerous

"

By

MICHAEL New

FOSTER,
Edition.

M.D.,
l8mo.

F.R.S.
is.

Wit

Illustrations.
"

Astronomy

By J.
New Sir

NORMAM Edition.

LOCKYER,
i8mo.
is.

F.R.S.

With

numerous

Illustrations.

Botany
numerous

"

By By
is.

J. D. HOOKER,
New STANLEY

K.C.S.I.,
Edition.

C.B.,

F.R.S.
is.

With

Illustrations. Professor

i8mo.

Logic

"

JEVONS,

F.R.S.

New

Edition.

i8mo.

Political
iSmo.

Economy
is.

"

By Professor
in

STANLEY

JEVONS,

F.R.S.

Others

preparation.

ELEMENTARY

SCIENCE By
the

CLASS-BOOKS.
ROYAL. POPULAR
G. B. TRONOMY. AS-

Astronomy
Astronomer

"

ASTRONOMER

With

Illustrations.
New Edition.

By

Sir

AIRY,

K.C.B.,

Royal.
"

i8mo.

4J. 6V/. IN of the NORMAN

Astronomy
With

ELEMENTARY

LESSONS
of the

ASTRONOMY.

Coloured
and New

Diagram
numerous

Spectra

Nebulae,
F.R.S.

Illustrations.

By J.
$s. ("d.

Sun, Stars, and LOCKYER,

Edition.

Fcap. 8vo.

SCIENCE

CLASS-BOOKS.

23

Elementary
QUESTIONS
IN FORBES ASTRONOMY.

Science
ON

Class-books"
ELEM the cloth Use of

continued. NTARY Schools.


is.

LOCKYER'S
For

LESSONS

By

JOHN

ROBERTSON.

i8mo,
IN

limp.

6d. PHYSIOLOGY.

Physiology
With
numerous

"

LESSONS

ELEMENTARY

Illustrations.

By
the

T.

H.

HUXLEY,
School of

F.R.S.,
Mines.

fessor ProNew

of Edition.

Natural

History
8vo. ON

in

Royal

Fcap.

^s. 6J.

QUESTIONS
SCHOOLS.

HUXLEY'S
T.

PHYSIOLOGY
M.D.

FOR
6d.

By
LESSONS

ALCOCK,
IN

i8mo.

is.

Botany

"

ELEMENTARY
of Hundred

BOTANY.

By
in

D.

OLIVER,
Edition.

F.R.S.,

College, London. Fcap.


"

With 8vo.

Professor F.L.S., nearly Two


4^.

Botany

University
New

Illustrations.

6d.

Chemistry
INORGANIC

LESSONS
AND of

IX ORGANIC.

ELEMENTARY

By
in

HENRY

F.R.S.,
With

Professor
numerous

Chemistry
and and Alkalies

Owens

CHEMISTRY, E. ROSCOE, College, Manchester.


of New the Solar

Illustrations of the

Chromo-Litho Alkaline Earths.

Spectrum, Fcap. 8vo.


A SERIES

and
4J.

Edition.

6d.
CHEMICAL

PROBLEMS, prepared with the THORPE, Ph.D., to above, by T. E. Special Reference of Chemistry in the Yorkshire Professor College of Science, Leeds. for the for the Government, preparation of Students Adapted With Preface Examinations. a by Science, and Society of Arts i8mo. with New Professor ROSCOE. 2s. Edition, Key.
OF

Practical
COURSE

Chemistry
OF PRACTICAL

THE

OWENS CHEMISTRY.

COLLEGE

JUNIOR
By
FRANCIS

JONES,
New

Manchester.

in the Master F.R.S.E., F.C.S., Chemical Professor With Preface ROSCOE, by i8mo.
2s.

Grammar and

School,

Illustrations.

Edition.
"

6d.

Chemistry.
Exercises

QUESTIONS
Inorganic
and F.C.S. i8mo.

ON.

Series

of

Problems

and

in

Organic
3*.

Chemistry.

By

F.

JONES,

F.R.S.E., Political
i8mo.

Economy
By
2s.

"

POLITICAL G.

ECONOMY
FAWCETT. New

FOR

GINNERS. BEEdition.

MILLICENT

6d.

Logic
of

ELEMENTARY
with

LESSONS

IN

LOGIC

Deductive

and

copious Questions and Examples, and a Vocabulary of Professor STANLEY JEVONS, M.A., By W. Logical Edition. New London. in University College, Political Economy Inductive,
Terms.

Fcap.

8vo.

y.

6d.

24

SCIENTIFIC

CA

TALOGUE.

Elementary

Science
LESSONS
IN

Class-books
ELEMENTARY Professor With of the of
numerous

"

continued.

Physics
BALFOUR

"

PHYSICS.
Natural

By
in and New

STEWART,

F.R.S.,

Philosophy
Illustrations

Owens Edition.

College, Fcap.
"

Manchester.

Chromo-Litho

of the

Spectra

Sun, Stars, and

Nebulae.

Svo.

4*. 6"/. IN ELEMENTARY Lecturer in ANATOMY.

Anatomy
ST.
at

LESSONS

GEORGE

MIVART,

F.R.S.,
With

Comparative

By Anatomy

St.

Mary's Hospital.
6s. 6d.
AN

upwards

of 400

Illustrations.

Fcap.

Svo.

Mechanics." W.KENNEDY,

ELEMENTARY

TREATISE.
of

C.E.
,

Professor With

Applied

Mechanics

College, London. Steam


With
"

Illustrations. TREATISE.

B. By A. University [hi preparation. in

AN of

ELEMENTARY

Professor

numerous

By JOHN PERRY, Engineering, Imperial College of Engineering, Yedo. and Numerical Woodcuts Examples and Exercises.

l8mo.

4-f. 6d.

Physical
PHYSICAL
son

Geography.
GEOGRAPHY.
of

"ELEMENTARY

LESSONS
A.

IN Murchinumerous

Professor

Illustrations.

Geology, Fcap. Svo.


ON
THE

"c.,

GEIKIE, By Edinburgh.
is.

F.R.S.,
With

4^. ("d. 6d.

QUESTIONS

SAME. OF

Geography
CLARKE, Natural
The

"

CLASS-BOOK

GEOGRAPHY.

By
6d.

C.

B.

M.A..

F.R.G.S.

Fcap.
-NATURAL

Svo.

zs.

Philosophy.
By
Light,
and

PHILOSOPHY

FOR Part I. Part

BEGINNERS.

I.

TODHUNTER,
and Fluid Bodies.

M.A.,
i8mo. 35. 6d.

F.R.S.

Properties of

Solid

$s. 6d.

II. Sound,

Heat.

iSmo.

Sound."
STONE.

AN With

ELEMENTARY

TREATISE.

By

Dr.

W.

H.

Illustrations.
Others in

iSino.

3-r. 6J.

Preparation.

MANUALS

FOR
Crown

STUDENTS.
Svo.

Dyer

and

Vines
"

THE

STRUCTURE

OF resisted

PLANTS.

By
SYDNEY

Professor

THISELTON

DYER,
and Lecturer

F.R.S.,

by

VINES,
With

B.Sc.,

Fellow

of Christ's

numerous

Illustrations.

College, Cambridge. [/" prefaratioti.

MANUALS

FOR

STUDENTS.

25

Manuals Fawcett
"

for

Students

"

continued.

MANUAL
M.P. 6d.

OF
New

POLITICAL

ECONOMY.
revised and

By

Professor Crown

FAWCETT,
8vo.
izs.

Edition,

enlarged.

Fleischer
"

A with M.

SYSTEM Notes M. and

OF

VOLUMETRIC
the Additions, from F.R.S.E. Mum,

ANALYSIS. second With German tions. Illustra-

Translated, Edition, by
Crown

PATTISON

8vo.

7-r. 6d. INTRODUCTION TO

Flower
Course

(W.
OF of

H.)
THE

"

AN

THE

OLOGY OSTE-

England
F.R.C.S.
Crown

of the Being the Substance Lectures the of at delivered Royal College Surgeons of in Professor W. H. F.R.S., 1870. FLOWER, By With New Illustrations. Edition, enlarged. numerous
icw.

MAMMALIA.

8vo.

6d.

Foster

and

Balfour
By
MICHAEL Part I.

THE

ELEMENTS

OF

OLOGY. EMBRYand

FOSTER,
crown

M.D.,
8vo.

F.R.S.,

F.

M.

BALFOUR, Foster and

M.A.

^s. 6J.

Langley
J. N.
THE

"

COURSE

OF
MICHAEL

ELEMENTARY

PRACTICAL

PHYSIOLOGY.
and

By
B.A. New

FOSTER,
Crown

M.D.,
Svo. 6s.

F.R.S., Hooker
ISLANDS.

LANGLEY,
STUDENT'S
Sir

Edition.

(Dr.)_
D.C.L.

FLORA Globe

OF

THE

BRITISH

By

M.D.,

New

J. D. HOOKER, Edition, revised.


An

K.C.S.I.,
8vo.

C.B., F.R.S.,
los.

6d.

Huxley
Nature.

"

PHYSIOGRAPHY.
Professor

Introduction

to

the

Study of
numerous

By Illustrations, and
7-r.6d.

HUXLEY,
Plates.

F.R.S.
New

With

Coloured

Edition.

Crown

Svo.

Huxley

and

Martin
"

COURSE

OF

PRACTICAL

STRUCTION INProfessor New

IN

ELEMENTARY assisted Crown

BIOLOGY.
H. Nv

By M.B.,

HUXLEY, Edition,

F.R.S.,
revised.

by
Svo.

MARTIN,

D.Sc.

6s. BIOLOGY. assisted


"

Huxley
II. With

and By

Parker
"

ELEMENTARY

PART PARKER.

Professor

HUXLEY,

F.R.S.,

Illustrations. SCIENCE.
Professor W.

by [In preparation.
A Treatise
on

Jevons."THE
Logic
LL.D.,
and

PRINCIPLES Scientific New


,

OF

Method. and

By
Revised

STANLEY Crown

JEVONS,
12s.

F.R.S.

Edition.

Svo.

6d.

26

SCIENTIFIC

CATALOGUE.

Manuals

for

Students
"

continued.

Oliver By

(Professor)
"

FIRST

BOOK

OF

INDIAN

BOTANY.

Professor

DANIEL and

OLIVER,

F.R.S.,
the

F.L.S., Gardens,
6s. 6d.

Keeper
Kew.

of

the With

Herbarium

Library

of Extra

Royal
8vo.

numerous

Illustrations.

fcap.

Parker
SKULL.

and

Bettany
"

THE PARKER
los.

MORPHOLOGY
and

OF

THE trated. Illus-

By
Crown

Professor

G.

T.

BETTANY.

8vo.

f"d.

Tait
"

AN

ELEMENTARY

TREATISE
E.

ON

HEAT.

By [In
the

ProPress.

fessorrTAlT,

F.R.S.

Illustrated.

Thomson
"

ZOOLOGY. F.R.S. Illustrated.

By

Sir

C.

WYVILLE

THOMSON, [In preparation.

Tylor
TYLOR,
F.R.S.

and M.A.,

Lankester
"

ANTHROPOLOGY.
and Professor

By
E.
RAY

E.

B.

F.R.S.,

LANKESTER, [In

M.A.,

Illustrated.

preparation.

Other

volumes

of

these

Manuals

will

follow.

MENTAL

AND

MORAL

PHILOSOPHY,

ETC.

27

WORKS

ON

MENTAL
AND ALLIED

AND

MORAL

PHILOSOPHY,

SUBJECTS.

Aristotle.
RHETORIC. M.

"

AN

INTRODUCTION
With

TO and 8vo.

ARISTOTLE'S

Analysis, Notes,

Appendices,
14$. THE

By

E.

COPE,

Trinity College, Cambridge.


ON With

ARISTOTLE ELENCHI.

FALLACIES;
a

OR,
and Notes

SOPHISTICI
EDWARD

Translation

by

POSTE,

M.A., Balfbur.
an

Fellow

of Oriel

College, Oxford.
OF

8vo.

8s. 6d.

"

A
on

DEFENCE the
\2s.

PHILOSOPHIC
of

DOUBT

being

Essay

Foundations

Belief.

By

A.

J.

BALFOUR,

M.P. "Mr. Pall


"

8vo.

Balfour1!
Mall able and

criticism

is

exceedinglybrilliant
to
one

and

suggestive."
"

Gazette.

An

refreshingcontribution
and

of the raging Birks.


FIRST Course Crown This

age,

deserves science

to

make

its mark
"

of the burning questions in the fierce battle now

oetween

and

theology."
T. R.

Athenaeum.

"

Works

by

the
:
"

Rev.

BIRKS,

Professor

of Moral

Philo-

sophy, Cambridge
PRINCIPLES of 8vo. work

OF delivered

MORAL in the

SCIENCE

or,

First

Lectures 8.r. (xt.

University

of

Cambridge.

all preliminary to the direct exposiof three topics tion the Certainty and Dignity Philosophy. These are of Moral Science, its Spiritual Geography, or relation to other of Moral main or thought, and its Formative Principles, of human subjects
treats
some

elementary

truths

on

-which

its

-whole

development

must

depend.
MODERN

Bentham,
MODERN
OF

UTILITARIANISM; and Mill, Examined


PHYSICAL

or,

The

Systems
Crown THE of

of 8vo.

Paley,
6s. 6d.

and

Compared.
AND Examination 6s.

FATALISM,
;

DOCTRINE Herbert

EVOLUTION First

including an 8vo. Principles. Crown


REVELATION;
Svo. 8s.

cer's Spen-

SUPERNATURAL Moral

or,

First

Principles

of

Theology.

28

SCIENTIFIC

CATALOGUE.

Boole.

"

AN

INVESTIGATION
ON WHICH ARE

OF

THE

LAWS

OF THE

THOUGHT,
MATHEMATICAL

FOUNDED
LOGIC AND Professor

THEORIES
in

OF

BABILITIES. PROof 14?,

Mathematics

GEORGE By BOOLE, LL.D., the Queen's University, Ireland, "c.


ON THE ARCHER HISTORY OF late

8vo.

Butler."
Moral

LECTURES

ANCIENT

PHILOSOPHY.

By
in

W.

BUTLER,

Professor

of

Philosophy

Author's

with MSS., Master M.A., in the

Greek

Edited from the University of Dublin. HEPWORTH SON, THOMPby WILLIAM of Trinity College, and of Regius Professor New and University of Cambridge. Cheaper Edition,

the

Notes,

revised

by
A

the

Editor.

8vo. ACCOUNT
an

12s.

Caird.
OF

"

CRITICAL With

OF

THE

PHILOSOPHY

KANT.
Professor

Historical

Introduction. in the

M.A.,
8vo.

of Moral

Philosophy

By E. CAIRD, University of Glasgow.

i8j.
"

Calderwood.
LL.D.,
:
"

Works
of

by
Moral

the

Rev.

HENRY in

Professor

Philosophy

the

CALDERWOOD, M.A., of burgh EdinUniversity


Man's

PHILOSOPHY

OF
of the

THE

INFINITE in

Treatise
to

on

Knowledge
and Dr.

Being, 8vo. Cheaper Edition. written book "A of great ability those who be easily understood by even British Review. discussions" Quarterly
Mansel.
....
"

Infinite

answer

Sir

W.

Hamilton

Js. 6d.
in
a

clear
not

stle,and
versed in

are

may such

HANDBOOK
Crown "ft

OF 6s.

MORAL

PHILOSOPHY.

Sixth

Edition.

8vo.

is,we feelconvinced, the best handbook

on

the its

intellectually subject,
author."
"

and "A in His


a

Standard. credit to infinite a useful work, going- over great deal of ground compact and and further study. manner facilitate adapted to suggest

morally,

and

does

book

will

be

an

assistance
"

University of Edinburgh.
THE
"

many Guardian. AND

to

students

outside

his

own

RELATIONS It should be

OF

MIND service
as

BRAIN.

Svo.

\2s.

criticism
"

a searching Westminster Review. of cerebral pyschology" is probably the best combination to be found Altogether his work in criticism the subject and at present on England of exposition The psychology" Academy. ofphysiological

of

real

clear

and exposition

"

"

Clifford."
W. K. FREDERICK

LECTURES

AND

ESSAYS.
Edited

By
by
LESLIE

the

late STEPHEN

Professor and Two

CLIFFORD,

F.R.S.
with

POLLOCK,
2

Introduction 25^.

by

F.

POLLOCK.

Portraits.

vols.

Svo.

30

SCIENTIFIC

CATALOGUE.

JC

VO

"

continued. OF New

THE

PRINCIPLES Scientific
Method.

SCIENCE.
and

Treatise

on

Logic

and

Cheaper
to

Edition,
any
true

revised.

Crown

8vo.
"

I2J.
one

6d.
in

No has

future
done in without

can

be said
-way

have

been

the

England
book."
THE
"

having
OF from
a

of carefully studied
SIMILARS,
Modification

logical and

knowledge of scientificmethod
Professor

what in

jfevoni
of

Spectator.
the True

SUBSTITUTION
Derived
2s.

Principle
Dictum.

Reasoning.
Fcap.
ELEMENTARY INDUCTIVE.
8vo.

of Aristotle's

6J. LESSONS With New AND DEDUCTIVE LOGIC, and Vocabulary Questions, Examples, IN

of

Logical
PRIMER

Terms. OF
"

Edition.
New

Fcap.
Edition.

8vo. iSmo.
from

3^. 6d.
u.

LOGIC.
THE which

Maccoll.
An NORMAN

GREEK
obtained

SCEPTICS,
the Hare Scholar Prize of

Pyrrho
the year

to

Sextus.

Essay

in

1868.

By
bridge. Cam-

MACCOLL,
Crown

B.A.,
3-r. 6d.

Downing

College,

8vo.

M'Cosh.
' '

"

Works New

by JAMES
Jersey,
U.S.

M'Cosn, skilful

LL.D.,
in that

President

of Princeton

College,
He

application of logicto in that inductive science of the human mind which is psychology', His whole is the fine side of English philosophy. as a philosophy
worthy of attention."
OF Tenth is
a
"

certainly s/icnvs himself

Revue

de

Deux

Mondes.

THE and
"

METHOD
Moral. work

THE Edition.

DIVINE 8vo.

GOVERNMENT,
icw.

Physical by
an

6d. similar
ones

This based

distinguishedfrom

other

its

being

upon

thorough study of physical

science, and

accurate

by its entering in a knowledge of its present condition, and than and its cussion manner more predecessors unfettered deeper upon the disthe and psychological,ethical, appropriate of theological tions. quesThe author keeps aloof at once from the a priori idealism and dreaminess and of German speculation since Schelling, from the and the onesidedness and narro-wness of empiricism positivism have which Dr. so prevailedin England." Ulrici, in "Zeitschrift
"

fur THE

Philosophic."
OF THE MIND. A New

INTUITIONS
cloth,
"

Edition.

8vo.

icxr.

6d.

The

and tuitional inuntiertaking to adjust the claims of the sensational and a. priorimethods, philosophies,and of thez. posteriori with is accomplished in this work a great amount of success." Review. "/ value it for its large acquaintance Westminster w/tich has led him with not to neglect the Philosophy, English
"

great
well
as

German

works.

I admire

the

moderation

and
"

clearness,

as

of comprehensiveness,

the authors

views."

Dr.

Dorner,

of

Berlin.

MENTAL

AND

MORAL

PHILOSOPHY,

E7T.

31

'

Sh

"

continued. OF
ol

AN

EXAMINATION"

MR.

J.

S.

MILL'S

PHILOSOPHY:
Second

Being

Defence
ios.

Fundamental

Truth.

edition,

with

additions. "Such
to
a

6J.

work

greatly needed
volume

do

it.

This

is but

done, and theaullior the man was not in merely important, reference to the
to

be

views

of Mr.

Mill,
and

of the

-whole he
so

present, British Review.


THE book
"

Continental,

school of writers, past and ably represents" Princeton


"

LAWS of

OF Formal

DISCURSIVE

THOUGHT
Crown 8vo.

Being

Text-

Logic. of
summarized it is the

5.?.
which it contains is Tery in the
a

The

amount

information
work
was on

threat ; and
which the
it

only
London

deals.
"

Never

such

very work

important subject with


so

much

needed

as

present day."

Quarterly

Review.
:

CHRISTIANITY the Times


on

AND Natural

POSITIVISM

Series

of

Lectures Crown

to

Theology

and

Apologetics.

8vo.

is. 6d. THE

SCOTTISH

PHILOSOPHY

FROM

HUTCHESON

TO 8vo. i6j.

HAMILTON,

Biographical, Critical,Expository.

Royal

Masson."
with
to

RECENT Criticisms
;

BRITISH

PHILOSOPHY Comments DAVID in the


on

Review Answer Professor

including
Hamilton.

some

Mr.

Mill's

Sir

William and

By
Literature Additional

MASSON,

M.A..
of

of Rhetoric Third
"

English
with
an

Edition,
can

University Crown Chapter.

Edinburgh.
6s

8vo.
an

We

nowhere
the
course

of
the past

which gh'es so point to a work of philosophicalspeculation in


or

clear

Britain

tion exposiduring
fluences in-

century,

which

indicates

so

instructivelythe
"

mutual

and ofphilosophic

thought." scientific

Fortnightly Review.

Maudsley.

"

Works in

by

H.

MAUDSLEY,

M.D.,

Professor

of Medical

Jurisprudence
THE

University College, London.


OF MIND and
;

PHYSIOLOGY

being

the

First

Part

of

Third "The

Edition,

Revised,
and

Enlarged,
of

Physiology
THE

Pathology
OF 8vo.
:

in great part " Crown Mind.

Re-written,
8vo.
lor.

of 6d.

PATHOLOGY

MIND.
i8j.

Revised, Enlarged,

and

in great

part Ke-written.
BODY
AND MIND

an

Inquiry
To

into

their
to

Connexion

and

Mutual
An

Influence,

specially
and Crown Revised 8vo.

with

reference

Mental
are

Disorders.

Enlarged Essays.

edition. 6s. 6d.

which

added, Psychological

32

SCIENTIFIC

CATALOGUE.

Maurice. M.A.,

"

Works
of

by

the

Rev.

FREDERICK in the

DEXISON

MAURICE,
bridge. Cam-

Professor

Moral 'Works

(For CATALOGUE.)
SOCIAL dd.
Whilst and

other

Philosophy University of Author, see THEOLOGICAL by the same

MORALITY.

University of
los.

Lectures delivered Twenty-one and Cheaper Edition. Crown Cambridge. New

in

the

8vo.

"

by the freedom from exclusiveness reading it we are charmed tJie loftiness the large cJiarity, of thought, the eagerprejudice, ness and whatever there is of real worth to appreciate recognize
in the
new

extant

-world, which

animates

it from

one

end

to

the other.

We

gain
and

perhaps, from
noble THE

thoughtsand new of viewing things, even more, ways the influenceof so being brought for a time under Athenaeum. spirituala mind."
"

CONSCIENCE
of The

Lectures New says: and

on

Cambridge.
Review is
a

Casuistry, delivered Crown Cheaper Edition.


rise

in the

versity Uni5-r.

8vo.

Saturday
all that is such AND

"We

from

them

with

detestation

of

there MORAL Ancient VoL


a

atid mean, and with a livingimpression that selfish as thing goodness afterall."
METAPHYSICAL from the First and
to

PHILOSOPHY.
the Thirteenth the

Vol. Centuries

I.
;

Philosophy
Fourteenth into Vols. the

II. the

Century
Nineteenth

French

Revolution,
New Edition

with and

glimpse
2

Century.

Preface.

8vo.

25.5-.
: or

Morgan.
Human

"

ANCIENT

SOCIETY

Researches

in
to

the

Lines

of

By

LEWIS

Progress, from Savagery, through Barbarism of the National Member H. MORGAN,


8vo.
THE

Civilisation.

Academy

of

Sciences.

i6s. SCIENTIFIC
BASES Author OF Habit and FAITH.

Murphy,
JOSEPH
8vo.
"

"

By

JOHN
14^. is not

MURPHY,
without

of

"

Intelligence."
continues be with the less

TJie book work

substantial the

value

the writer it may

of

the best

apologistsof
an

force and
tact; and
the

clearness, but still with


with
"

last century, commendable the

problem."

feeling for intelligent Academy.


"

persuasivenessand changed conditions of

Paradoxical
Crown

Philosophy.
8vo.

Sequel

to

"The

Unseen

verse." Uni-

"js.6d.
OF MATTER Author with New of AND
"

PlCton.
and Svo.

"

THE

MYSTERY

OTHER New Theories Crown

ESSAYS.
the

By J.
Old 6s.

ALLAXSOX

PICTOX,
issue

Faith."

Cheaper

Preface.

MENTAL

AND

MORAL

PHILOSOPHY,

ETC.

33

Picton

"

continued.
:"

CONTENTS
"

The

The
"

Antithesis

Mystery of of Faith

Matter and

"

TJie
"

Philosophy of Ignorance
The Essential Nature

Si^ht

of Religion

Christian

Pantheism.

Sidgwick."
SIDGWICK,
with A

THE

METHODS Praelector

OF
in Moral Second 14?.

ETHICS.
and Political

By

HENRY

M.A.,

Trinity College, Cambridge.

Philosophy in Edition, revised throughout


the

important
and

additions.
to

8vo.
First in the

SUPPLEMENT additions

the

Edition, containing all


Second. volume

important

alterations
and

8vo.

2s.

"

This

physicians Leaving to metarespectingthe any further may already over-discussed problem of the originof the moral faculty,he takes it for granted as readily as the geometrician takes space for he takes little But granted, or the physicist the existence of matter. else for granted, and the science of conduct, be definingethics as the various ethical systems that have been not carefully examines, but propounded by Aristotle and Aristotle s followers downwards, the principles upon which, so far as they confine themselves to tJie strict province of ethics, they are based.'"' Athenaeum.

excellent

very

-welcome

discussion

thai

be needed

'

'

"

Thornton."
SENSE WILLIAM

OLD-FASHIONED

ETHICS,
with
some

AND their

COMMON-

METAPHYSICS, THOMAS THORNTON,


IO.T.

of of

Applications. By
on

Author

"A

Treatise

Labour."

8vo.
The

("d. volume all aeals ivith

present

problems
The

-which
are

are

agitating the
Contents
:
"

minds

of

thoughtjulmen.
II.
as a

following
IV.

the

/. Ante-Utilitarianism. David Phase Hume

Pretensions. History'sScientific

III. Recent

Af eta physician. VI.

Huxleyism.

V.

of

Atheism. Scientific

Limits

of
ON

Demonstrable

Theism.

Thring

(E.,

M.

A.)."
New

THOUGHTS
M.A.

LIFE-SCIENCE. Head revised. Master Crown of

By EDWARD Uppingham
8vo. 7-r. 6d. THE and
reference

THRING,
School.

(Benjamin Place), Edition, enlarged and

Venn.

"

LOGIC Province
to

OF

CHANCE of

its

of the Theory logicalbearings, and

Essay on Probability,
Fellow and

An

the with

tions Founda-

especial
and of
written re-

its

applicationto
Second
ior.

Moral
Lecturer

Social

Science. and and

By JOHN
Caius

Gonville

M.A., VENN, College, Cambridge.


Crown 8vo.

Edition,

greatly enlarged.

6d.

"

One

of
or

the most

connected this p.
any

ject thoughtful and philosophicaltreatises on any subbeen with has produced in lo%ic and evidence which other country for years." Mill's Logic, vol. ii.' many
"

77.

Seventh

Edition.
C

NATURE

SERIES.

THE
By
8vo.

SPECTROSCOPE
J.
N. y.

AND
F.R.S.
With

ITS
Illustrations.

APPLICATIONS.
Second Edition.

LOCKYER,
6d.

Crown

THE
Crown

ORIGIN
By
Svo.
Sir 3*. 6d.

AND
JOHN
Second

METAMORPHOSES
LUBBOCK,
Edition.

OF
F.R.S.
With

SECTS. IN-

M.P.,

Illustrations.

THE
With

TRANSIT
Professor
numerous

OF
Natural

VENUS.
in the Svo.

By
Andersonian
y.

G.

FORBES,
University,

B.A.,
Glasgow

of

Philosophy
Crown

Illustrations.

6d.

THE
F.R.S.

COMMON
Illustrated.

FROG.
Crown
Svo. 3*.

By
6d.

ST.

GEORGE

MIVART,

POLARISATION
LL.D.,
Svo.
y.

OF
of
the

LIGHT.
Society.

By
Illustrated.

W.

SPOTTISWOODE,
Second Edition.

President
6d.

Royal

Crown

RELATION
Illustrated.

TO
Second

INSECTS.
Edition.

By
Crown

SiR Svo.

JOHN
4$.

LUBBOCK,
6d.

M.P.,

F.R.S.

THE
By
4*.

SCIENCE
H. kd. W.

OF

WEIGHING
Warden of

AND
the Standards.

MEASURING.
Illustrated.
Crown Svo.

CHISHOLM,

HOW

TO
Linkages.

DRAW
By
A. B.

A
KEMPE,

STRAIGHT
B.A. Illustrated.

LINE
Crown

Lecture
is.

on

Svo.

6d.

LIGHT

Series
in the M.
2S.

of
and

Simple, Entertaining
of

and
of

Useful
every

Experiments By ALFRED
Crown
Svo.

Phenomena

Light

for the

Use

of

Students
With

Age.

MAYER
f"d.

CHARLES

BARXARD.

Illustrations.

SOUND
every

Series
in M. With

of
the

Simple, Entertaining
of

and
of

pensive Inexo"
Institute

Experiments
Age. By A. of Technology, "c.

Phenomena

Sound,
of

fjr

the

Use

Students

MAYER,
numerous

Professor

Physics
Crown

in the Svo.

Stevens

Illustrations.

3.1. 6.^.

SEEING
F.R.S.

AND
With

THINKING.
Diagrams.
Crown
8vo. y.

By Prof.
6d.

W.

K.

CLIFFORD,

(Otters

to

follow.']

MACMILLAN

AND

CO.,

LONDON.

Published

every
2s.

Thursday,
6d.,

price 6d.

Monthly

Parts 15^.

2s.

and

Half-Yearly

Volumes,

NATURE:
AN ILLUSTRATED

JOURNAL

OF

SCIENCE.

NATURE the the


the

expounds
RESULTS
recent

in
OF

popular and
SCIENTIFIC

yet authentic

manner,

GRAND
most

RESEARCH,

discussing
out

scientific Science
upon

discoveries, and
civilisation and

pointing
progress, well
as

bearing

of
a

and
to
a

its claims

to

more

general recognition, as
educational

higher place
It

in the

system
on

of

the

country.
the and

contains of of

original articles
;

all

subjects within
the
nature

domain value

Science

Reviews Works

forth setting
;

recent
a

Scientific
of

Correspondence
and

Columns,

forming

medium among

Scientific
most

discussion

munication of intercomof

the

men distinguished

Science,

Serial
papers

Columns,

giving
in

the

gist

of

the

most

important
Home

appearing

Scientific
of

Journals, both

and

Foreign
and
In
course

; Transactions

the

principal Scientific
"c.

Societies

Academies Schools
of

of

the

World, Notes,
Science
paper

where

is
will

included be
most
over

in

the

regular
as

studies,this
is

acceptable,
the

it

tells

what without
amount

doing

in

Science

all

world, is

popular
it
a

lowering

the

standard is

of

Science, and
within
a

by

vast

of information
students
are

brought
to

small for

compass, what Science best

and

directed

the

best

sources

they

need.

The

various
are

questions
also

connected

with the

teaching in
of

schools

fullydiscussed,and

methods

teaching are

indicated.

You might also like